You are on page 1of 116

INSURANCE Page 1

ABOITIZ SHIPPING CORPORATION V NEW - Petitioner elevated the case to the Court of presumed to be negligent since it is tasked with the
INDIA ASSURANCE COMPANY, LTD. Appeals, which in turn, affirmed in toto the trial maintenance of its vessel. Though this duty can be
[CITATION] court’s decision. delegated, still, the shipowner must exercise close
QUISUMBING; May 2, 2006 - Petitioner moved for reconsideration but the same supervision over its men.
was denied. - In the present case, petitioner has the burden of
NATURE - Hence, this petition for review showing that it exercised extraordinary diligence in
Petition for review on certiorari Petitioner’s Claim the transport of the goods it had on board in order to
> Petitioner contends that respondent’s claim for invoke the limited liability doctrine. Differently put,
FACTS damages should only be against the insurance to limit its liability to the amount of the insurance
- Societe Francaise Des Colloides loaded a cargo of proceeds and limited to its pro-rata share in view of proceeds, petitioner has the burden of proving that
textiles and auxiliary chemicals from France on board the doctrine of limited liability the unseaworthiness of its vessel was not due to its
a vessel owned by Franco-Belgian Services, Inc. Respondent’s Comments fault or negligence.
- The cargo was consigned to General Textile, Inc., in > Respondent counters that the doctrine of real and - Considering the evidence presented and the
Manila and insured by respondent New India hypothecary nature of maritime law is not applicable circumstances obtaining in this case, we find that
Assurance Company, Ltd. in the present case because petitioner was found to petitioner failed to discharge this burden. Both the
- While in Hongkong, the cargo was transferred to have been negligent. Hence, according to trial and the appellate courts, in this case, found that
M/V P. Aboitiz for transshipment to Manila. respondent, petitioner should be held liable for the the sinking was not due to the typhoon but to its
- Before departing, the vessel was advised that it total value of the lost cargo unseaworthiness. Evidence on record showed that
was safe to travel to its destination, but while at sea, the weather was moderate when the vessel sank.
the vessel received a report of a typhoon moving ISSUE These factual findings of the Court of Appeals,
within its path. WON the limited liability doctrine applies in this case affirming those of the trial court are not to be
- To avoid the typhoon, the vessel changed its disturbed on appeal, but must be accorded great
course. However, it was still at the fringe of the HELD weight. These findings are conclusive not only on
typhoon when its hull leaked. NO the parties but on this Court as well.
- On October 31, 1980, the vessel sank, but the Ratio Where the shipowner fails to overcome the Disposition Petition is denied for lack of merit.
captain and his crew were saved. presumption of negligence, the doctrine of limited
- On November 3, 1980, the captain of M/V P. liability cannot be applied. PHILAMCARE HEALTH SYSTEMS, INC. V CA
Aboitiz filed his “Marine Protest”, stating that the (TRINOS)
wind force was at 10 to 15 knots at the time the ship Reasoning 379 SCRA 357
foundered and described the weather as “moderate - From the nature of their business and for reasons YNARES-SANTIAGO; March 18, 2002
breeze, small waves, becoming longer, fairly frequent of public policy, common carriers are bound to
white horse observe extraordinary diligence over the goods they NATURE
- Petitioner notified the consignee of the total loss of transport according to all the circumstances of each Petition for review of CA decision
the vessel and all of its cargoes. case. In the event of loss, destruction or
- General Textile, lodged a claim with respondent for deterioration of the insured goods, common carriers FACTS
the amount of its loss. are responsible, unless they can prove that the loss, - Ernani TRINOS, deceased husband of respondent
- Respondent paid General Textile and was destruction or deterioration was brought about by Julita, applied for a health care coverage with
subrogated to the rights of the latter. the causes specified in Article 17341 of the Civil Philamcare Health Systems, Inc. In the standard
- Respondent hired a surveyor to investigate, and the Code. In all other cases, common carriers are application form, he answered no to the question:
same concluded that the cause was the flooding of presumed to have been at fault or to have acted “Have you or any of your family members ever
the holds brought about by the vessel’s questionable negligently, unless they prove that they observed consulted or been treated for high blood pressure,
seaworthiness. extraordinary diligence. Moreover, where the vessel heart trouble, diabetes, cancer, liver disease, asthma
- Respondent filed a complaint for damages against is found unseaworthy, the shipowner is also or peptic ulcer? (If Yes, give details).”
petitioner Aboitiz, Franco-Belgian Services and the - The application was approved for period of one
latter’s local agent, F.E. Zuellig, Inc. (Zuellig) 1 year; upon termination, it was extended for another
Art. 1734. Common carriers are responsible for the loss, destruction,
- On November 20, 1989, the trial court ruled in 2 years. Amount of coverage was increased to a
or deterioration of the goods, unless the same is due to any of the
favor of respondent and held petitioner Aboitiz liable following causes only: maximum sum of P75T per disability.
for the total value of the lost cargo plus legal interest (1) Flood, storm, earthquake, lightning, or other natural disaster or - During this period, Ernani suffered a HEART
calamity;
- The complaint with respect to Franco and Zuellig (2) Act of the public enemy in war, whether international or civil; ATTACK and was confined at the Manila Medical
was dismissed (3) Act of omission of the shipper or owner of the goods; Center (MMC) for one month. While her husband was
(4) The character of the goods or defects in the packing or in the in the hospital, Julita tried to claim the
containers;
(5) Order or act of competent public authority. hospitalization benefits.
INSURANCE Page 2

- Petitioner treated the Health Care Agreement Ratio Every person has an insurable interest in the (b) Notice must be based on the occurrence after
(HCA) as void since there was a concealment life and health of himself2. The health care effective date of the policy of one or more of the
regarding Ernani’s medical history. Doctors at agreement was in the nature of non-life insurance, grounds mentioned;
the MMC allegedly discovered at the time of his which is primarily a contract of indemnity. Once the (c) Must be in writing, mailed or delivered to the
confinement, he was hypertensive, diabetic and member incurs hospital, medical or any other insured at the address shown in the policy;
asthmatic. Julita then paid the hospitalization expense arising from sickness, injury or other (d) Must state the grounds relied upon provided in
expenses herself, amounting to about P76T. stipulated contingent, the health care provider must Section 64 of the Insurance Code and upon request
- After her husband died, Julita instituted action for pay for the same to the extent agreed upon under of insured, to furnish facts on which cancellation is
damages against Philamcare and its Pres. After the contract. based.
trial, the lower court ruled in her favor and ordered Reasoning - These conditions have not been met. When the
Philamcare to reimburse medical and hospital - A contract of insurance3 is an agreement whereby terms of insurance contract contain limitations on
coverage amounting to P76T plus interest, until fully one undertakes for a consideration to indemnify liability, courts should construe them in such a way
paid; pay moral damages of P10T; pay exemplary another against loss, damage or liability arising from as to preclude insurer from non-compliance of
damages of P10T; atty’s fees of P20T. an unknown or contingent event. obligation. Being a contract of adhesion, terms of an
- CA affirmed the decision of the trial court but - An insurance contract exists where the following insurance contract are to be construed strictly
deleted all awards for damages and absolved elements concur: against the party which prepared it – the insurer.
petitioner Reverente. (a) The insured has an insurable interest; - Also, Philamcare had 12 months from the date of
Petitioner’s Claims (b) The insured is subject to a risk of loss by the issuance of the Agreement within which to contest
(1) Agreement grants “living benefits” such as happening of the peril; the membership of the patient if he had previous
medical check-ups and hospitalization which a (c) The insurer assumes the risk; ailment of asthma, and six months from the issuance
member may immediately enjoy so long as he is (d) Such assumption of risk is part of a general of the agreement if the patient was sick of diabetes
alive upon effectivity of the agreement until its scheme to distribute actual losses among a large or hypertension.
expiration. group of persons bearing a similar risk; and * The health care agreement is in the nature of a
(2) Only medical and hospitalization benefits are (e) In consideration of the insurer’s promise, the contract of indemnity. Hence, payment should be
given under the agreement without any insured pays a premium. made to the party who incurred the expenses. It is
indemnification, unlike in an insurance contract 2. NO clear that respondent paid all the hospital and
where the insured is indemnified for his loss. Ratio Where matters of opinion or judgment are medical bills; thus, she is entitled to reimbursement.
(3) HCAs are only for a period of one year; called for, answers made in good faith and without Disposition Petition DENIED.
therefore, incontestability clause does not apply, as it intent to deceive will not avoid a policy even though
requires effectivity period of at least 2 yrs. they are untrue; since in such case the insurer is not PINEDA V CA (INSULAR LIFE INSURANCE
(4) It is not an insurance company, governed by justified in relying upon such statement, but is COMPANY)
Insurance Commission, but a Health Maintenance obligated to make further inquiry. 226 SCRA 755
Organization under the authority of DOH. Reasoning DAVIDE; September 27, 1993
(5) Trinos concealed a material fact in his - The fraudulent intent on the part of the insured
application. must be established to warrant rescission of the NATURE
(6) Julita was not the legal wife since at the time insurance contract. The right to rescind should be Appeal by certiorari for review and set aside the
of their marriage, the deceased was previously exercised previous to the commencement of an Decision of the public respondent Court of Appeals
married to another woman who was still alive.* action on the contract. No rescission was made. and its Resolution denying the petitioners' motion for
Besides, the cancellation of health care agreements reconsideration
ISSUES as in insurance policies requires:
1. WON a health care agreement is an insurance (a) Prior notice of cancellation to insured; FACTS
contract (If so, “incontestability clause” under the - In 1983, Prime Marine Services, Inc. (PMSI)
Insurance Code is applicable) procured a group policy from Insular Life to provide
2. WON the HCA can be invalidated on the basis of 2 life insurance coverage to its sea-based employees
alleged concealment Sec.10. Every person has an insurable interest in the life and enrolled under the plan. During the effectivity of the
health:
(1) of himself, of his spouse and of his children;
policy, 6 covered employees perished at sea. They
HELD (2) of any person on whom he depends wholly or in part for education were survived by complainants-appellees, the
YES or support, or in whom he has a pecuniary interest; beneficiaries under the policy.
(3) of any person under a legal obligation to him for the payment of
money, respecting property or service, of which death or illness might - complainants-appellees sought to claim death
delay or prevent the performance; and benefits due them and approached Capt. Roberto
(4) of any person upon whose life any estate or interest vested in him
depends.
Nuval, President and GM of PMSI, then executed
3 special powers of attorney authorizing Capt. Nuval to
Section 2 (1) of the Insurance Code
INSURANCE Page 3

“follow up, ask, demand, collect and receive” for of Ayo and LOntok, even if no bond was posted as - the labor of the employees is the true source
their benefit indemnities of sums of money due required of the benefits, which are a form of additional
them…” compensation to them.
- Unknown to the complainants, PMSI filed with HELD - the employer is the agent of the insurer in
Insular Life claims for and in behalf of them through 1. YES performing the duties of administering group
Capt. Nuval, even using the 5 special powers of Ratio Third persons deal with agents at their peril insurance policies. It cannot be said that the
attorney that they executed as documents. Insular and are bound to inquire as to the extent of the employer acts entirely for its own benefit or for the
Life then released 6 checks, payable to the order of power of the agent with whom they contract. The benefit of its employees in undertaking
the complainant-appellees, to the treasurer of PMSI person dealing with an agent must also act administrative functions. While a reduced premium
(who happened to be Capt. Nuval’s son-in-law). with ordinary prudence and reasonable may result if the employer relieves the insurer of
Capt. Nuval then endorsed and deposited these diligence. Obviously, if he knows or has good these tasks, and this, of course, is advantageous to
checks (which were for the complainants) in his bank reason to believe that the agent is exceeding his both the employer and the employees, the insurer
account. authority, he cannot claim protection. So if the also enjoys significant advantages from the
- 3 years after, the complainants-appellees found out suggestions of probable limitations be of such a clear arrangement. The reduction in the premium which
that they were entitled, as beneficiaries, to life and reasonable quality, or if the character assumed results from employer-administration permits the
insular benefits under a group policy with by the agent is of such a suspicious or unreasonable insurer to realize a larger volume of sales, insurer to
respondent-appellant so they sought to recover nature, or if the authority which he seeks to exercise realize a larger volume of sales, and at the same
these benefits from Insular Life. Insular Life denied is of such an unusual or improbable character, as time the insurer's own administrative costs are
the claim, saying that their liability to complainants would suffice to put an ordinarily prudent man upon markedly reduced.
was already extinguished upon delivery to and his guard, the party dealing with him may not shut - the employee has no knowledge of or control over
receipt by PMSI of the 6 checks issued in the his eyes to the real state of the case, but should the employer's actions in handling the policy or its
complainants’ names. Complainants filed case with either refuse to deal with the agent at all, or should administration. An agency relationship is based upon
the Insurance Commission which decided in their ascertain from the principal the true condition of consent by one person that another shall act in his
favor. affairs. behalf and be subject to his control. It is clear from
- Insurance Commission held that the special powers Reasoning the evidence regarding procedural techniques here
of attorney executed by complainants in favor of the - The execution by the principals of special powers of that the insurer-employer relationship meets this
complainants do not contain in unequivocal and clear attorney, which clearly appeared to be in prepared agency test with regard to the administration of the
terms authority to Capt. Nuval to obtain, receive, forms and only had to be filled up with their names, policy, whereas that between the employer and its
receipt from respondent company insurance residences, dates of execution, dates of employees fails to reflect true agency. The insurer
proceeds arising from the death of the seaman- acknowledgement and others, excludes any intent to directs the performance of the employer's
insured; also, that Insular Life did not convincingly grant a general power of attorney or to constitute a administrative acts, and if these duties are not
refuted the claim of Mrs. Alarcon that neither she nor universal agency. Being special powers of attorney, undertaken properly the insurer is in a position to
her husband executed a special power of authority in they must be strictly construed. Insular Life knew exercise more constricted control over the
favor of Capt. Nuval (and therefore, the company that a power of attorney in favor of Capt. Nuval for employer's conduct.
should have not released the check to Capt. Nuval- the collection and receipt of such proceeds was a - ON GROUP INSURANCE: Group insurance is
PMSI); and that it did not observe Sec 180(3), as deviation from its practice with respect to group essentially a single insurance contract that
repealed by Art. 225 of the Family Code, when it policies (that the employer-policyholder is the agent provides coverage for many individuals. In its
released the benefits due to the minor children of of the insurer). original and most common form, group insurance
Ayo and Lontok, when the said complainants did not - The employer acts as a functionary in the collection provides life or health insurance coverage for
post a bond as required and payment of premiums and in performing related the employees of one employer. The coverage
- Insular Life appealed to the CA; CA modified the duties. Likewise falling within the ambit of terms for group insurance are usually stated in a
decision of the Insurance Commission, eliminating administration of a group policy is the disbursement master agreement or policy that is issued by the
the award to the Lontoks and Ayo of insurance payments by the employer to the insurer to a representative of the group or to an
employees. Most policies, such as the one in this administrator of the insurance program, such as an
ISSUES case, require an employee to pay a portion of the employer. Although the employer may be the titular
1. WON Insular Life should be liable to the premium, which the employer deducts from wages or named insured, the insurance is actually
complainants when they relied on the special powers while the remainder is paid by the employer. This is related to the life and health of the employee.
of attorney, which Capt. Nuval presented as known as a contributory plan as compared to a Indeed, the employee is in the position of a real
documents, when they released the checks to the non-contributory plan where the premiums are solely party to the master policy, and even in a non-
latter paid by the employer. contributory plan, the payment by the employer of
2. WON Insular Life should be liable to the the entire premium is a part of the total
complainants when they released the check in favor compensation paid for the services of the employee.
INSURANCE Page 4

2. YES CEBU SHIPYARD ENGINEERING WORKS, INC. V subject to the following overriding limitations and
Ratio Regardless of the value of the unemancipated WILLIAM LINES, INC. and PRUDENTIAL exceptions, namely:
common child's property, the father and mother ipso GUARANTEE and ASSURANCE COMPANY, INC. (a) The total liability of the Contractor to the
jure become the legal guardian of the child's [CITATION] Customer (over and above the liability to replace
property. However, if the market value of the PURISIMA; May 5, 1999 under Clause 10) or of any sub-contractor shall be
property or the annual income of the child exceeds limited in respect of any defect or event (and a series
P50,000,00, a bond has to be posted by the parents NATURE of accidents arising out of the same defect or event
concerned to guarantee the performance of the Petition for review on certiorari shall constitute one defect or event) to the sum of
obligations of a general guardian. Pesos Philippine Currency One Million only.
Reasoning FACTS x x x
- Sec 180, Insurance Code: 'In the absence of a - Cebu Shipyard and Engineering Works, Inc. 20. The insurance on the vessel should be
judicial guardian, the father, or in the latter's (CSEW) is engaged in the business of dry-docking maintained by the customer and/or owner of the
absence or incapacity, the mother of any minor, who and repairing of marine vessels while the Prudential vessel during the period the contract is in effect.”
is an insured or a beneficiary under a contract of life, Guarantee and Assurance, Inc. (Prudential) is in the - While the M/V Manila City was undergoing dry-
health or accident insurance, may exercise, in behalf non-life insurance business. docking and repairs within the premises of CSEW, the
of said minor, any right under the policy, without - William Lines, Inc. is in the shipping business. It master, officers and crew of M/V Manila City stayed
necessity of court authority or the giving of a bond was the owner of M/V Manila City, a luxury in the vessel, using their cabins as living quarters.
where the interest of the minor in the particular act passenger-cargo vessel, which caught fire and sank Other employees hired by William Lines to do repairs
involved does not exceed twenty thousand pesos " on Feb. 16, 1991. At the time of the unfortunate and maintenance work on the vessel were also
- …repealed by Art. 225, Family Code: "ART. 225. occurrence sued upon, subject vessel was insured present during the dry-docking.
The father and the mother shall jointly exercise legal with Prudential for P45M for hull and machinery. The - On February 16, 1991, after subject vessel was
guardianship over the property of their Hull Policy included an “Additional Perils transferred to the docking quay, it caught fire and
unemancipated common child without the necessity (INCHMAREE)” Clause covering loss of or damage to sank, resulting to its eventual total loss.
of a court appointment. In case of disagreement, the the vessel through the negligence of, among others, - On February 21, 1991, William Lines, Inc. filed a
father's decision shall prevail, unless there is judicial ship repairmen complaint for damages against CSEW, alleging that
order to the contrary. - Petitioner CSEW was also insured by Prudential for the fire which broke out in M/V Manila City was
Where the market value of the property or the third party liability under a Shiprepairer’s Legal caused by CSEW’s negligence and lack of care.
annual income of the child exceeds P50,000, the Liability Insurance Policy. The policy was for P10 - On July 15, 1991 was filed an Amended Complaint
parent concerned shall be required to furnish a bond million only, under the limited liability clause, to wit: impleading Prudential as co-plaintiff, after the latter
in such amount as the court may determine, but not - On Feb. 5, 1991, William Lines, Inc. brought its had paid William Lines, Inc. the value of the hull and
less than ten per centum (10%) of the value of the vessel, M/V Manila City, to the Cebu Shipyard in machinery insurance on the M/V Manila City. As a
property or annual income, to guarantee the Lapulapu City for annual dry-docking and repair. result of such payment Prudential was subrogated to
performance of the obligations prescribed for general - On Feb. 6, 1991, an arrival conference was held the claim of P45 million, representing the value of
guardians." between representatives of William Lines, Inc. and the said insurance it paid.
-"market value of the property or the annual CSEW to discuss the work to be undertaken on the On June 10, 1994, the trial court a quo came out
income of the child": the aggregate of the child's M/V Manila City. The contracts, denominated as Work with a judgment against CSEW:
property or annual income; if this exceeds Orders, were signed thereafter., with the following 1. To pay unto plaintiff Prudential Guarantee and
P50,000.00, a bond is required - There is no stipulations: Assurance, Inc., the subrogee, the amount of P45M,
evidence that the share of each of the minors in the “10. The Contractor shall replace at its own work and with interest at the legal rate until full payment is
proceeds of the group policy in question is the at its own cost any work or material which can be made; the amount of P56,715,000 representing loss
minor's only property. Without such evidence, it shown to be defective and which is communicated in of income of M/V MANILA CITY, with interest at the
would not be safe to conclude that, indeed, that is writing within one (1) month of redelivery of the legal rate until full payment is made;
his only property. vessel or if the vessel was not in the Contractor’s 2. To pay unto plaintiff, William Lines, Inc. the
Disposition the instant petition is GRANTED. The Possession, the withdrawal of the Contractor’s amount of P11M as payment, in addition to what it
Decision of 10 October 1991 and the Resolution of 19 workmen, or at its option to pay a sum equal to the received from the insurance company to fully cover
May 1992 of the public respondent in CA-G.R. SP No. cost of such replacement at its own works. These the injury or loss, in order to replace the M/V
22950 are SET ASIDE and the Decision of the conditions shall apply to any such replacements. MANILA CITY, with interest at the legal rate until full
Insurance Commission in IC Case No. RD-058 is 11. Save as provided in Clause 10, the Contractor payment is made; the sum of P927,039 for the loss
REINSTATED. Costs against the private respondent. shall not be under any liability to the Customer either of fuel and lub oil on board the vessel when she was
SO ORDERED. in contract or for delict or quasi-delict or otherwise completely gutted by fire at defendant, Cebu
except for negligence and such liability shall itself be Shipyard’s quay, with interest at the legal rate until
full payment is made; the sum of P3,054,677.95 as
INSURANCE Page 5

payment for the spare parts and materials used in - When M/V Manila City capsized, steel and angle - The that factual findings by the CA are conclusive
the M/V MANILA CITY during dry-docking with bars were noticed to have been newly welded along on the parties and are not reviewable by this Court.
interest at the legal rate until full payment is made; the port side of the hull of the vessel, at the level of They are entitled to great weight and respect, even
P500,000 in moral damages;the amount of P10Min the crew cabins. William Lines did not previously finality, especially when, as in this case, the CA
attorney’s fees; and to pay the costs of this suit. apply for a permit to do hotworks on the said portion affirmed the factual findings arrived at by the trial
- On September 3, 1997, the Court of Appeals of the ship as it should have done pursuant to its court. When supported by sufficient evidence,
affirmed the appealed decision of the trial court, work order with CSEW. findings of fact by the CA affirming those of the trial
ordering CSEW to pay Prudential, the subrogee, the Prudential’s version court, are not to be disturbed on appeal. The
sum of P45 Million, with interest at the legal rate > At around 7AM of Feb. 16, 1991, the Chief Mate of rationale behind this doctrine is that review of the
until full payment is made. M/V Manila City was inspecting the various works findings of fact of the CA is not a function that the
CSEW’s version: being done by CSEW on the vessel, when he saw Supreme Court normally undertakes.
On Feb. 13, 1991, the CSEW completed the that some workers of CSEW were cropping out steel - The CA and the Cebu RTC are agreed that the fire
drydocking of M/V Manila City at its grave dock. It plates on Tank Top No. 12 using acetylene, oxygen which caused the total loss of subject M/V Manila
was then transferred to the docking quay of CSEW and welding torch. He also observed that the rubber City was due to the negligence of the employees and
where the remaining repair to be done was the insulation wire coming out of the air-conditioning unit workers of CSEW. Both courts found that the M/V
replating of the top of Water Ballast Tank No. 12 was already burning, prompting him to scold the Manila City was under the custody and control of
which was subcontracted by CSEW to JNB General workers. petitioner CSEW, when the ill-fated vessel caught
Services. Tank Top No. 12 was at the rear section of > At 2:45 PM of the same day, witnesses saw smoke fire. The decisions of both the lower court and the CA
the vessel, on level with the flooring of the crew coming from Tank No. 12. The vessel’s reeferman set forth clearly the evidence sustaining their finding
cabins located on the vessel’s second deck. reported such occurence to the Chief Mate who of actionable negligence on the part of CSEW. This
At around 7AM of Feb. 16, 1991, the JNB workers immediately assembled the crew members to put factual finding is accorded great weight and is
trimmed and cleaned the tank top framing which out the fire. When it was too hot for them to stay on conclusive on the parties. The court discerns no basis
involved minor hotworks (welding/cutting works). board and seeing that the fire cannot be controlled, for disturbing such finding firmly anchored on
The said work was completed at about 10AM. The the vessel’s crew were forced to withdraw from enough evidence.
JNB workers then proceeded to rig the steel plates, CSEW’s docking quay. - Furthermore, in petitions for review on certiorari,
after which they had their lunch break. The rigging - In the morning of Feb. 17, 1991, M/V Manila City only questions of law may be put into issue.
was resumed at 1PM sank. As the vessel was insured with Prudential Questions of fact cannot be entertained. The finding
While in the process of rigging the second steel Guarantee, William Lines filed a claim for of negligence by the CA is a question which this
plate, the JNB workers noticed smoke coming from constructive total loss, and after a thorough Court cannot look into as it would entail going into
the passageway along the crew cabins. When one of investigation of the surrounding circumstances of the factual matters on which the finding of negligence
the workers, Mr. Casas, proceeded to the tragedy, Prudential found the said insurance claim to was based. Such an approach cannot be allowed by
passageway to ascertain the origin of the smoke, he be meritorious and issued a check in favor of William this Court in the absence of clear showing that the
noticed that smoke was gathering on the ceiling of Lines in the amount of P45 million pesos case falls under any of the exceptions to the well-
the passageway but did not see any fire as the crew representing the total value of M/V Manila City’s hull established principle.
cabins on either side of the passageway were locked. and machinery insurance. The finding by the trial court and the Court of
He immediately sought out the proprietor of JNB, Mr. Appeals that M/V Manila City caught fire and sank by
Buenavista, and the Safety Officer of CSEW, Mr. ISSUES reason of the negligence of the workers of CSEW,
Aves, who sounded the fire alarm. CSEW’s fire 1. WON CSEW had “management and supervisory when the said vessel was under the exclusive
brigade immediately responded as well as the other control“ of the m/v manila city at the time the fire custody and control of CSEW is accordingly upheld.
fire fighting units in Metro Cebu. However, there broke out 2. YES
were no WLI representative, officer or crew to guide 2. WON the doctrine of res ipsa loquitur applies - For the doctrine of res ipsa loquitur to apply to a
the firemen inside the vessel. against the crew given situation, the following conditions must concur:
- Despite the combined efforts of the firemen of the 3. WON CSEW’S expert evidence is admissible or of (1) the accident was of a kind which does not
Lapulapu City Fire Dept., Mandaue Fire Dept., probative value ordinarily occur unless someone is negligent; and (2)
Cordova Fire Dept. Emergency Rescue Unit 4. WON Prudential has the right of subrogation that the instrumentality or agency which caused the
Foundation, and fire brigade of CSEW, the fire was against its own insured THE CONTRACTUAL 5. 5. 5. injury was under the exclusive control of the person
not controlled until 2AM of the following day. 5. WON the provisions limiting csew’s liability for charged with negligence.
- On the early morning of Feb. 17, 1991, gusty winds negligence to a maximum of p1 million are valid The facts and evidence on record reveal the
rekindled the flames on the vessel and fire again concurrence of said conditions in the case under
broke out. Then the huge amounts of water pumped HELD scrutiny. First, the fire that occurred and consumed
into the vessel, coupled with the strong current, 1. YES M/V Manila City would not have happened in the
caused the vessel to tilt until it capsized and sank ordinary course of things if reasonable care and
INSURANCE Page 6

diligence had been exercised. In other words, some - According to petitioner, under the aforecited clause,
negligence must have occurred. Second, the agency William Lines, Inc., agreed to assume the risk of loss
charged with negligence, as found by the trial court of the vessel while under drydock or repair and to
and the CA and as shown by the records, is the 4. YES such extent, it is benefited and effectively
herein petitioner, CSEW, which had control over - Petitioner contends that Prudential is not entitled to constituted as a co-assured under the policy.
subject vessel when it was docked for annual repairs. be subrogated to the rights of William Lines, Inc., - This theory of petitioner is devoid of sustainable
So also, as found by the RTC, “other responsible theorizing that (1) the fire which gutted M/V Manila merit. Clause 20 of the Work Order in question is
causes, including the conduct of the plaintiff, and City was an excluded risk and (2) it is a co-assured clear in the sense that it requires William Lines to
third persons, are sufficiently eliminated by the under the Marine Hull Insurance Policy. maintain insurance on the vessel during the period of
evidence.” - It is petitioner’s submission that the loss of M/V dry-docking or repair. Concededly, such a stipulation
What is more, in the present case the trial court Manila City or damage thereto is expressly excluded works to the benefit of CSEW as the shiprepairer.
found direct evidence to prove that the workers from the coverage of the insurance because the However, the fact that CSEW benefits from the said
and/or employees of CSEW were remiss in their duty same resulted from “want of due diligence by the stipulation does not automatically make it as a co-
of exercising due diligence in the care of subject Assured, Owners or Managers” which is not included assured of William Lines. The intention of the parties
vessel. The direct evidence substantiates the in the risks insured against. Again, this theory of to make each other a co-assured under an insurance
conclusion that CSEW was really negligent. Thus, petitioner is bereft of any factual or legal basis. It policy is to be gleaned principally from the insurance
even without applying the doctrine of res ipsa proceeds from a wrong premise that the fire which contract or policy itself and not from any other
loquitur, in light of the direct evidence on record, the gutted subject vessel was caused by the negligence contract or agreement because the insurance policy
ineluctable conclusion is that CSEW was negligent of the employees of William Lines, Inc. To repeat, denominates the assured and the beneficiaries of the
and consequently liable for damages to the the issue of who between the parties was negligent insurance. The hull and machinery insurance
respondent, William Lines, Inc. has already been resolved against CSEW. Upon proof procured by William Lines, Inc. from Prudential
3. NO of payment by Prudential to William Lines, Inc., the named only “William Lines, Inc.” as the assured.
- Petitioner maintains that the CA erred in former was subrogated to the right of the latter to There was no manifestation of any intention of
disregarding the testimonies of the fire experts, indemnification from CSEW. As aptly ruled by the William Lines, Inc. to constitute CSEW as a co-
Messrs. David Grey and Gregory Michael Southeard, Court of Appeals, the law on the matter is succinct assured under subject policy. It is axiomatic that
who testified on the probable origin of the fire in M/V and clear, to wit: when the terms of a contract are clear its stipulations
Manila City. Petitioner avers that since the said fire - Art. 2207. If the plaintiff’s property has been control.i] Thus, when the insurance policy involved
experts were one in their opinion that the fire did not insured, and he has received indemnity from the named only William Lines, Inc. as the assured
originate in the area of Tank Top No. 12 where the insurance company for the injury or loss arising out thereunder, the claim of CSEW that it is a co-assured
JNB workers were doing hotworks but on the crew of the wrong or breach of contract complained of, is unfounded.
accommodation cabins on the portside No. 2 deck, the insurance company shall be subrogated to the - Then too, in the Additional Perils Clause of the
the RTC and the CA should have given weight to such rights of the insured against the wrongdoer or the same Marine Insurance Policy, it is provided that:
finding based on the testimonies of fire experts; person who has violated the contract. If the amount Subject to the conditions of this Policy, this
petitioner argues. paid by the insurance company does not fully cover insurance also covers loss of or damage to vessel
But courts are not bound by the testimonies of the injury or loss, the aggrieved party shall be directly caused by the following:
expert witnesses. Although they may have probative entitled to recover the deficiency from the person xxx
value, reception in evidence of expert testimonies is causing the loss or injury. Negligence of Charterers and/or Repairers, provided
within the discretion of the court, under Section 49, - Thus, when Prudential, after due verification of the such Charterers and/or Repairers are not an
Rule 130 of the Revised Rules of Court. It is never merit and validity of the insurance claim of William Assured hereunder.
mandatory for judges to give substantial weight to Lines, Inc., paid the latter the total amount covered - As correctly pointed out by respondent Prudential,
expert testimonies. If from the facts and evidence on by its insurance policy, it was subrogated to the right if CSEW were deemed a co-assured under the policy,
record, a conclusion is readily ascertainable, there is of the latter to recover the insured loss from the it would nullify any claim of William Lines, Inc. from
no need for the judge to resort to expert opinion liable party, CSEW. Prudential for any loss or damage caused by the
evidence. In the case under consideration, the - Petitioner theorizes further that there can be no negligence of CSEW. Certainly, no shipowner would
testimonies of the fire experts were not the only right of subrogation as it is deemed a co-assured agree to make a shiprepairer a co-assured under
available evidence on the probable cause and origin under the subject insurance policy. To buttress its such insurance policy; otherwise, any claim for loss
of the fire. There were witnesses who were actually stance that it is a co-assured, petitioner placed or damage under the policy would be invalidated.
on board the vessel when the fire occurred. Between reliance on Clause 20 of the Work Order which Such result could not have been intended by William
the testimonies of the fire experts who merely based states: Lines, Inc.
their findings and opinions on interviews and the 20. The insurance on the vessel should be 5. NO
testimonies of those present during the fire, the maintained by the customer and/or owner of the - Although in this jurisdiction, contracts of adhesion
latter are of more probative value. vessel during the period the contract is in effect. have been consistently upheld as valid per se; as
INSURANCE Page 7

binding as an ordinary contract, the Court recognizes partnership in the City of Lucena. Under the business Reliance Insurance purveyed the same message as
instances when reliance on such contracts cannot be name of New Life Enterprises, the partnership well as Equitable Insurance Corporation.
favored especially where the facts and circumstances engaged in the sale of construction materials at its - The said policy in question follows:
warrant that subject stipulations be disregarded. place of business, a two storey building situated at "The insured shall give notice to the Company of any
Thus, in ruling on the validity and applicability of the Iyam, Lucena City. The facts show that Julian Sy insurance or insurances already effected, or which
stipulation limiting the liability of CSEW for insured the stocks in trade of New Life Enterprises may subsequently be effected, covering any of the
negligence to P1M only, the facts and circumstances with Western Guaranty Corporation, Reliance Surety property or properties consisting of stocks in trade,
vis-a-vis the nature of the provision sought to be and Insurance Co. Inc., and Equitable Insurance goods in process and/or inventories only hereby
enforced should be considered, bearing in mind the Corporation. insured, and unless such notice be given and the
principles of equity and fair play. - On May 15, 1981, Western Guaranty Corporation particulars of such insurance or insurances be stated
- It is worthy to note that M/V Manila City was issued Fire Insurance Policy No. 37201 in the amount therein or endorsed on this policy pursuant to
insured with Prudential for P45M. To determine the of P350,000.00. This policy was renewed on May 13, Section 50 of the Insurance Code, by or on behalf of
validity and sustainability of the claim of William 1982. the Company before the occurrence of any loss or
Lines, Inc., for a total loss, Prudential conducted its - On July 30, 1981, Reliance Surety and Insurance damage, all benefits under this policy shall be
own inquiry. Upon thorough investigation by its hull Co., Inc. issued Fire Insurance Policy No. 69135 in deemed forfeited, provided however, that this
surveyor, M/V Manila City was found to be beyond the amount of P300,000.00 (Renewed under Renewal condition shall not apply when the total insurance or
economical salvage and repair. The evaluation of the Certificate No. 41997). An additional insurance was insurances in force at the time of loss or damage is
average adjuster also reported a constructive total issued by the same company on November 12, 1981 not more than P200,000.00."
loss. The said claim of William Lines, Inc., was then under Fire Insurance Policy No. 71547 in the amount Petitioner’s comments
found to be valid and compensable such that of P700,000.00. > Petitioners contend that they are not to be blamed
Prudential paid the latter the total value of its - On February 8, 1982, Equitable Insurance for the omissions, alleging that insurance agent Leon
insurance claim. Furthermore, it was ascertained that Corporation issued Fire Insurance Policy No. 39328 in Alvarez (for Western) and Yap Kam Chuan (for
the replacement cost of the vessel (the price of a the amount of P200,000.00. Reliance and Equitable) knew about the existence of
vessel similar to M/V Manila City), amounts to P55M. - Thus when the building occupied by the New Life the additional insurance coverage and that they were
- Considering the aforestated circumstances, let Enterprises was gutted by fire at about 2:00 o'clock not informed about the requirement that such other
alone the fact that negligence on the part of in the morning of October 19, 1982, the stocks in or additional insurance should be stated in the policy,
petitioner has been sufficiently proven, it would trade inside said building were insured against fire in as they have not even read said policies.
indeed be unfair and inequitable to limit the liability the total amount of P1,550,000.00. According to the
of petitioner to One Million Pesos only. As aptly held certification issued by the Headquarters, Philippine ISSUE
by the trial court, “it is rather unconscionable if not Constabulary/Integrated National Police, Camp WON New Life Enterprises’ claim for payment be
overstrained.” To allow CSEW to limit its liability to Crame, the cause of fire was electrical in nature. denied
P1M notwithstanding the fact that the total loss According to the plaintiffs, the building and the
suffered by the assured and paid for by Prudential stocks inside were burned. After the fire, Julian Sy HELD
amounted to P45M would sanction the exercise of a went to the agent of Reliance Insurance whom he YES
degree of diligence short of what is ordinarily asked to accompany him to the office of the Ratio Furthermore, when the words and language of
required because, then, it would not be difficult for company so that he can file his claim. He averred documents are clear and plain or readily
petitioner to escape liability by the simple expedient that in support of his claim, he submitted the fire understandable by an ordinary reader thereof, there
of paying an amount very much lower than the clearance, the insurance policies and inventory of is absolutely no room for interpretation or
actual damage or loss suffered by William Lines, Inc. stocks. construction anymore. Courts are not allowed to
Disposition Petition is DENIED. Resolution of the He further testified that the three insurance make contracts for the parties; rather, they will
CA is AFFIRMED. companies are sister companies, and as a matter of intervene only when the terms of the policy are
fact when he was following-up his claim with ambiguous, equivocal, or uncertain. The parties
NEW LIFE ENTERPRISES V CA Equitable Insurance, the Claims Manager told him to must abide by the terms of the contract because
207 SCRA 669 go first to Reliance Insurance and if said company such terms constitute the measure of the insurer's
REGALADO; March 31, 1992 agrees to pay, they would also pay. The same liability and compliance therewith is a condition
treatment was given him by the other insurance precedent to the insured's right of recovery from the
NATURE companies. Ultimately, the three insurance insurer.
Appeal by certiorari companies denied plaintiffs' claim for payment. - While it is a cardinal principle of insurance law that
Respondent’s comments a policy or contract of insurance is to be construed
FACTS > Western Guaranty Corporation through Claims liberally in favor of the insured and strictly against
- The antecedents of this case show that Julian Sy Manager Bernard S. Razon told the plaintiff that his the insurer company, yet contracts of insurance, like
and Jose Sy Bang have formed a business claim 'is denied for breach of policy conditions.' other contracts, are to be construed according to the
INSURANCE Page 8

sense and meaning of the terms which the parties at a high speed, as a result of which, the plaintiff lost ISSUE
themselves have used. If such terms are clear and balance and fell from the bus. As plaintiff clung WON the CA erred in the interpretation of the
unambiguous, they must be taken and understood in instinctively to the handle bar, he was dragged by insurance contract on the limit of the insurer’s
their plain, ordinary and popular sense. Moreover, the bus along the asphalted road. The bus driver, Gil liability
obligations arising from contracts have the force of Agpalo, abruptly stopped the bus. Then fled from the
law between the contracting parties and should be scene, leaving the bus and the injured plaintiff HELD
complied with in good faith. behind. YES
Reasoning - The plaintiff was brought to the Manila Sanitarium - The insurance policy clearly placed the maximum
a. The terms of the contract are clear and and Hospital where the doctors performed 2 major limit of the petitioner's liability for damages arising
unambiguous. The insured is specifically required to surgical operations on plaintiffs right leg. from death or bodily injury at P12,000.00 per
disclose to the insurer any other insurance and its - Plaintiff was confined at the hospital for (40) days, passenger and its maximum liability per accident at
particulars which he may have effected on the same from June 10, 1984 to August 26, 1984. Medical (P50,000.00. Since only one passenger was injured
subject matter. The knowledge of such insurance by expenses totaled the amount of P69,444.41. in the accident, the insurer's liability for the damages
the insurer's agents, even assuming the acquisition Plaintiff’s medical expenses were advanced by his suffered by said passenger is pegged to the amount
thereof by the former, is not the "notice" that would employer Maglines but he was required to reimburse of P12,000.00 only.
stop the insurers from denying the claim. Besides, Maglines on a staggered basis by way of salary - The limit of P50,000.00 per accident means that
the so-called theory of imputed knowledge, that is, deductions. After his release from the hospital, he the insurer's maximum liability for any single
knowledge of the agent is knowledge of the principal, returned to the hospital for further treatment and accident will not exceed P50,000.00 regardless of the
aside from being of dubious applicability here has checkup. The injuries had left plaintiff with a huge number of passengers killed or injured therein.
likewise been roundly refuted by respondent court scar on his right leg. Also, the plaintiff incurred lost The bus company may not recover from the
whose factual findings we find acceptable. earning by way of unearned salaries amounting to insurance company more than P12,000.00 per
b. Petitioners should be aware of the fact that a P7,500.00 due to said physical injuries and the passenger killed or injured, or (P50,000.00) per
party is not relieved of the duty to exercise the consequent hospital confinement. accident even if under the judgment of the court, the
ordinary care and prudence that would be exacted in - Plaintiff filed on June 26, 1985 the complaint erring bus operator will have to pay more than
relation to other contracts. The conformity of the against DMTC and its driver. Agpalo was later P12,000.00 to each injured passenger. The trial
insured to the terms of the policy is implied from his dropped as a party defendant because he could not court's interpretation of the insurance contract was
failure to express any disagreement with what is be served with summons. Upon filing its answer, the correct interpretation.
provided for. defendant DMTC filed a thirdparty complaint against Disposition petition for review is GRANTED. The
First Quezon City Insurance Co., Inc. September 17, decision promulgated by the CA, ordering the third
FIRST QUEZON CITY INSURANCE CO. v. CA (DE 1985, third-party defendant filed its answer to the party defendent, First Quezon City Insurance Co.,
DIOS MARIKINA TRANSPORT CO) third-party complaint. Inc. to indemnify theI private respondent, (DMTC),
218 SCRA 526 - TC held DMTC complaint dismissed for lack of merit the sum of P50,000.00 for the damages of the
GRINO-AQUINO; February 28, 1993 and as regards the third-party complaint First passenger, Jose V. Del Rosario, is hereby modified by
Quezon City Insurance Co., Inc. was to indemnify reducing the award to 12,000.00 only. Costs against
NATURE third-party plaintiff DMTC in the sum of P12,000.00 the private respondent De Dios Marikina
PETITION for review of the decision of the Court of with interest. There being no satisfactory warrant the Transportation Co., Inc.
Appeals. FQCIC seeks to limit to P12000, the amount court dismissed the rest of the claims in the
specified in the insurance contract, it’s liability to complaint and third-party complaint. TY V FIRST NATIONAL SURETY
indemnify the respomdemt DMTC, for the damages - The bus company appealed to the CA, which 1 SCRA 1324
suffered by a passenger, who accidentally fell off the modified the dispositive as regards the third-party LABRADOR; April 29, 1961
bug. complaint, that the third-party defendant First
Quezon City Insurance Co., Inc. be ordered to FACTS
FACTS indemnify third-party plaintiff DMTC the SUM of - At different times within a period of two months
- After sending off certain seamen at the departure P50,000.00 with legal interest. Insurance company prior to 24 December 1953, Diosdado C. Ty,
area of MIA, Jose V. del Rosario proceeded to the filed a MFR which was denied. employed as operator mechanic foreman in the
public utility bus stop. While at the bus stop, the Hence, this petition for review, assailing the Broadway Cotton Factory insured himself in 18 local
plaintiff saw a DMTC bus. While moving at a crawling appellate courts' interpretation of the provision of the insurance companies, among which being the 8
pace, it was taking several passengers, all of whom insurance contract on the limit of the insurer's above-named defendants, which issued to him
managed to board the bus while it was already at the liability. personal accident policies. Plaintiff’s beneficiary was
bus stop; plaintiff was the last one to board the bus. his employer, Broadway Cotton Factory, which paid
While the plaintiff was still on the bus with his hand the insurance premiums. On 24 December 1953, a
on the bus door, the slowly moving bus sped forward fire broke out which totally destroyed the Broadway
INSURANCE Page 9

Cotton Factory. Fighting his way out of the factory, - The Supreme Court affirmed the appealed decision, alongside the water hole. The car was towed and
plaintiff was injured on the left hand by a heavy with costs against the plaintiff-appellant. repaired by Morosi Motors at a total cost of P302.27.
object. He was brought to the Manila Central - When the repairs on the car had already been
University hospital, and after receiving first-aid, he MISAMIS LUMBER V CAPITAL INSURANCE made, Misamis made a report of the accident to
went to the National Orthopedic Hospital for 17 SCRA 288 Capital Insurance.
treatment of his injuries (fractures in index, middle, REYES; May 20, 1966 - Since Capital refused to pay for the total cost of to
fourth, and fifth fingers of left hand). From 26 wage and repairs, suit was filed in the municipal
December 1953 to 8 February 1954, he underwent NATURE court originally.
medical treatment in the hospital. The above- Direct appeal on a point of law from the judgment of - The defendant-appellant admits liability in the
described physical injuries have caused temporary the Court of First Instance of Manila amount of P150, but not for any excess thereof. The
total disability of plaintiff’s left hand. Plaintiff filed the lower court did not exonerate the said appellant for
corresponding notice of accident and notice of claim FACTS the excess because the company's absolution would
with all of the above-named defendants to recover - Misamis Lumber Corporation, under its former render the insurance contract one-sided and that the
indemnity. Defendants rejected plaintiff’s claim for name, Lanao Timber Mills, Inc., insured its Ford said insurer had not shown that the cost of repairs in
indemnity for the reason that there being no Falcon motor car for the amount of P14,000 with the sum of P302.27 is unreasonable, excessive or
severance of amputation of the left hand, the Capital Insurance & Surety Company, Inc. The padded, nor had it shown that it could have
disability suffered by him was not covered by his pertinent provisions of the policy provided, as undertaken the repairs itself at less expense.
policy. follows:
- Plaintiff sued the defendants in the Municipality 1. The Company will subject to the Limits of ISSUE
Court of this City, which dismissed his complaints. Liability indemnify the Insured against loss or WON Capital Insurance can be made to pay more
Thereafter, the plaintiff appealed to the Court of First damage to the Motor Vehicle and its accessories than P150
Instance Manila, presided by Judge Gregorio S. and spare parts whilst thereon.
Narvasa, which absolved the defendants from the 2. (a) by accidental collision or overturning or HELD
complaints. Hence, the appeal. collision or overturning consequent when NO
mechanical breakdown or consequent upon wear - The insurance policy stipulated in paragraph 4 that
ISSUE and tear. if the insured authorizes the repair the liability of the
WON Diosdado Ty is entitled to indemnity under the 3. At its option, the Company may pay in cash the insurer, per its sub-paragraph (a), is limited to
insurance policy for the disability of his left hand amount of the loss or damage or may repair, P150.00. The literal meaning of this stipulation must
reinstate or replace the Motor Vehicle or any part control, it being the actual contract, expressly and
HELD thereof or its accessories or spare parts. The plainly provided for in the policy.
- The agreement contained in the insurance policies liability of the Company shall not exceed the value - Recourse to legal hermeneutics is not called for
is the law between the parties. As the terms of the of the parts lost or damaged and the reasonable because paragraph 4 of the policy is clear and
policies are clear, express and specific that only cost of fitting such parts or the value of the Motor specific and leaves no room for interpretation.
amputation of the left hand should be considered as Vehicle at the time of the loss or damage - The option to undertake the repairs is accorded to
a loss thereof, an interpretation that would include whichever is the loss. The Insured's estimate of the insurance company per paragraph 2. The said
the mere fracture or other temporary disability not value stated in the schedule shall be the maximum company was deprived of the option because the
covered by the policies would certainly be amount payable by the Company in respect of any insured took it upon itself to have the repairs made,
unwarranted. In the case at bar, due to the clarity of claim for loss or damage. and only notified the insurer when the repairs were
the stipulation, distinction between “temporary 4. The Insured may authorize the repair of the done. As a consequence, paragraph 4, which limits
disability” and “total disability” need not be made in Motor Vehicle necessitated by damage for which the company's liability to P150.00, applies.
relation to one’s occupation means that the condition the Company may be liable under this policy - The insurance contract may be rather onerous
of the insurance is such that common prudence provided that: ("one-sided", as the lower court put it), but that in
requires him to desist from transacting his business (a) the estimated cost of such repair does not itself does not justify the abrogation of its express
or renders him incapable of working. While the Court exceed the authorized Repair Limit. terms, terms which the insured accepted or adhered
sympathizes with the plaintiff or his employer, for (b) a detailed estimate of the cost is forwarded to and which is the law between the contracting
whose benefit the policies were issued, it can not go to the Company without delay and providing also parties.
beyond the clear and express conditions of the that the authorized repair limit is P150.00. - To require the insurer to prove that the cost of the
insurance policies, all of which define partial disability - One night, the insured car, while traveling along in repairs ordered by the insured is unreasonable, when
as loss of either hand by a amputation through the Aurora Boulevard, passed over a water hole which the insurer was not given an opportunity to inspect
bones of the wrist.” There was no such amputation in the driver did not see because an oncoming car did and assess the damage before the repairs were
the case at bar. not dim its light. The crankcase and flywheel housing made, is contrary to elementary justice and equity.
of the car broke when it hit a hollow block lying
INSURANCE Page 10

SUN INSURANCE OFFICE LTD. V CA (TAN) and popular sense. The rejection letter of February - Demands were made by the Producers upon the
195 SCRA 193 29, 1984 was clear and plain. The Court noted that Fortune to pay the amount of the loss of
PARAS; March 13, 1991 the one year period is likewise in accord with Section P725,000.00 but the latter refused to pay as the loss
23 of the Insurance Code which states that any is excluded from the coverage of the insurance policy
NATURE condition which limits the time for commencing an specifically under "General Exceptions"
Petition for certiorari to review the decision of the CA action to a period of less than one year when the > The company shall not be liable under this
cause of action accrues is void. The right of action, policy in respect of x x x (b) any loss caused by
FACTS according to the SC, accrues at the time that the any dishonest, fraudulent or criminal act of the
- Private respondent Emilio Tan took from the claim is rejected at the first instance. A request for insured or any officer, employee, partner,
petitioner a Peso 300,000 property insurance policy reconsideration of the denial cannot suspend the director, trustee or authorized representative of
to cover his interest in the electrical insurance store running of the prescriptive period. The Court noted the Insured whether acting alone or in
of his brother housed in a building in Iloilo City on that the rationale for the one year period is to ensure conjunction with others.
August 15, 1983. Four days after the issuance of the that the evidence as to the origin and cause of the - Fortune opposes the contention of Producers that
policy, the building including the insured store destruction have not yet disappeared. Atiga and Magalong are not its "officer, employee, x x
burned. 2. NO x trustee or authorized representative x x x at the
- On August 20, 1983, Tan filed his claim for fire loss. - The Court clarified its ruling in Eagle Star Insurance time of the robbery
Sun Insurance, on February 29, 1984, wrote the Co. vs Chia Yu where it ruled that “the cause of - Trial Court
private respondent denying the claim. On April 3, action in an insurance contract does not accrue until > On being “EMPLOYEES”
1984, private respondent wrote another letter to the the Insured’s claim is finally rejected by the Insurer” Magalong and Atiga were not employees or
insurance company requesting reconsideration of the by stating the use of the word “finally” cannot be representatives of Producers as their services as
denial. Tan’s lawyer wrote another letter to the construed to mean the rejection of a petition for armored car driver and as security guard having
insurance company inquiring about the April 3 letter reconsideration. What the court referred to in effect been merely offered by PRC Management and by
which sought for a reconsideration of the denial. In is the rejection in the first instance as claimed by Unicorn Security and which latter firms assigned
its reply to the lawyer’s letter, Sun Insurance Sun Insurance them to plaintiff. The wages and salaries of both
reiterated its denial of the claim and enclosed therein Disposition The decision of the CA is reversed and Magalong and Atiga are presumably paid by their
copies of the two previous denials dated February set aside. The case is dismissed respective firms, which alone wields the power to
29, 1984 and May 17, 1985. dismiss them
- On November 20, 1985, Tan filed a civil case with FORTUNE INSURANCE AND SURETY CO. INC.V > On being “AUTHORIZED REPRESENTATIVE”
the RTC. Petition filed a motion to dismiss on the CA (PRODUCERS BANK OF THE PHILIPPINES) They were merely an assigned armored car driver
alleged ground that the action has already prescribed 244 SCRA 308 and security guard for the money transfer. It was
based on Condition 27 of the Insurance Policy which DAVIDE; May 23, 1995 teller Maribeth Alampay who had "custody" of the
stated that the window to file the appropriate action P725,000.00 cash being transferred along a specified
with either the Insurance Commission or in any court NATURE money route
of competent jurisdiction is twelve months from the Petition for Review on certiorari of CA decision - Court of Appeals
rejection of the claim. RTC denied the motion and the > affirmed in toto
subsequent motion for reconsideration. The CA FACTS > A policy or contract of insurance is to be construed
likewise denied the petition of Sun Insurance. - Producers Bank of the Philippines filed a complaint liberally in favor of the insured and strictly against
against Fortune Insurance and Surety Co., Inc. for the insurance company (New Life Enterprises vs.
ISSUE recovery of P725,000.00 under the policy issued by Court of Appeals; Sun Insurance Office, Ltd. vs.
1. WON the court the filing of a motion for Fortune. The sum was allegedly lost on June 29, Court of Appeals). Contracts of insurance, like other
reconsideration interrupts the 12 months prescription 1987 during a robbery of Producer's armored vehicle contracts, are to be construed according to the sense
period to contest the denial of the insurance claim while it was in transit to transfer the money from its and meaning of the terms which the parties
2. WON the rejection of the claim shall be deemed Pasay City Branch to its head office in Makati under themselves have used. If such terms are clear and
final only if it contains words to the effect that the the custody of its teller, Maribeth Alampay. The unambiguous, they must be taken and understood in
denial is final armored car was driven by Benjamin Magalong Y de their plain, ordinary and popular sense (New Life
Vera, escorted by Security Guard Saturnino Atiga Y Enterprises Case; Sun Insurance Office).
Rosete. Driver Magalong was assigned by PRC > The language used by Fortune in the policy is
HELD Management Systems. plain, ordinary and simple. No other interpretation is
1. NO - After an investigation by the Pasay police, driver necessary. The word "employee" should be taken to
- The SC held that Condition 27 of the Insurance Magalong and guard Atiga were charged, together mean in the ordinary sense. The Labor Code is a
policy is very clear and free from any doubt or with Batigue , Aquino and John Doe, with violation of special law specifically dealing with/and specifically
ambiguity. It has to be taken in its plain, ordinary, P.D. 532 (Anti-Highway Robbery Law) designed to protect labor and therefore its definition
INSURANCE Page 11

as to employer-employee relationships insofar as the placed by such persons are performing activities NO
application/enforcement of said Code is concerned which are directly related to the principal business Ratio A contract of insurance is a contract of
must necessarily be inapplicable to an insurance of such employer. In such cases, the person or adhesion, thus any ambiguity therein should be
contract. Had it intended to apply the Labor Code in intermediary shall be considered merely as an resolved against the insurer, or it should be
defining what the word "employee" refers to, it must/ agent of the employer who shall be responsible to construed liberally in favor of the insured and strictly
should have so stated expressly in the insurance the workers in the same manner and extent as if against the insurer. Limitations of liability should be
policy. Said driver and security guard cannot be the latter were directly employed by him. regarded with extreme jealousy and must be
considered as employees of Producers bank because > International Timber Corp. vs. NLRC - a "labor- construed in such a way as to preclude the insurer
it has no power to hire or to dismiss said driver and only" contractor is equivalent to a finding that there from non-compliance with its obligation. It goes
security guard under the contracts except only to ask is an employer-employee relationship between the without saying then that if the terms of the contract
for their replacements from the contractors. owner of the project and the employee of the "labor- are clear and unambiguous, there is no room for
- Fortune’s Contention only" contractor construction and such terms cannot be enlarged or
> when Producers commissioned a guard and a - Producer’s Contention diminished by judicial construction.
driver to transfer its funds from one branch to > Magalong and Atiga were not its employees since it - An insurance contract is a contract of indemnity
another, they effectively and necessarily became its had nothing to do with their selection and upon the terms and conditions specified therein. It is
authorized representatives in the care and custody of engagement, the payment of their wages, their settled that the terms of the policy constitute the
the money. Assuming that they could not be dismissal, and the control of their conduct. measure of the insurer's liability. In the absence of
considered authorized representatives, they were, > International Timber Corp. is not applicable to all statutory prohibition to the contrary, insurance
nevertheless, employees of Producers. It asserts that cases but only when it becomes necessary to prevent companies have the same rights as individuals to
the existence of an employer-employee relationship any violation or circumvention of the Labor Code, a limit their liability and to impose whatever conditions
"is determined by law and being such, it cannot be social legislation whose provisions may set aside they deem best upon their obligations not
the subject of agreement." Thus, if there was in contracts entered into by parties in order to give inconsistent with public policy.
reality an employer-employee relationship between protection to the working man. Reasoning
Producers, on the one hand, and Magalong and > American President Lines vs. Clave should be - It should be noted that the insurance policy entered
Atiga, on the other, the provisions in the contracts of applied which stated into by the parties is a theft or robbery insurance
Producers with PRC Management System for In determining the existence of employer- policy which is a form of casualty insurance. Section
Magalong and with Unicorn Security Services for employee relationship, the following elements are 174 of the Insurance Code provides:
Atiga which state that Producers is not their generally considered, namely: (1) the selection Sec. 174. Casualty insurance is insurance covering
employer and that it is absolved from any liability as and engagement of the employee; (2) the loss or liability arising from accident or mishap,
an employer, would not obliterate the relationship. payment of wages; (3) the power of dismissal; and excluding certain types of loss which by law or
> an employer-employee relationship depends upon (4) the power to control the employee's conduct. custom are considered as failing exclusively within
four standards: - Since under Producers' contract with PRC the scope of insurance such as fire or marine. It
(1) the manner of selection and engagement of the Management Systems it is the latter which assigned includes, but is not limited to, employer's liability
putative employee Magalong as the driver of Producers' armored car insurance, public liability insurance, motor vehicle
(2) the mode of payment of wages and was responsible for his faithful discharge of his liability insurance, plate glass insurance, burglary
(3) the presence or absence of a power to dismiss duties and responsibilities, and since Producers paid and theft insurance, personal accident and health
and the monthly compensation of P1,400.00 per driver to insurance as written by non-life insurance
(4) the presence and absence of a power to control PRC Management Systems and not to Magalong, it is companies, and other substantially similar kinds of
the putative employee's conduct. clear that Magalong was not Producers' employee. As insurance. (italics supplied)
> Of the four, the right-of-control test has been held to Atiga, Producers relies on the provision of its - Except with respect to compulsory motor vehicle
to be the decisive factor. It asserts that the power of contract with Unicorn Security Services which liability insurance, the Insurance Code contains no
control over Magalong and Atiga was vested in and provides that the guards of the latter "are in no other provisions applicable to casualty insurance or
exercised by Producers. Fortune further insists that sense employees of the CLIENT." to robbery insurance in particular. These contracts
PRC Management System and Unicorn Security are, therefore, governed by the general provisions
Services are but "labor-only" contractors under ISSUE applicable to all types of insurance. Outside of these,
Article 106 of the Labor Code which provides: WON Fortune Insurance and Surety Co. Inc. is liable the rights and obligations of the parties must be
Art. 106. Contractor or subcontractor. - There is under the Money, Security, and Payroll Robbery determined by the terms of their contract, taking into
"labor-only" contracting where the person policy it issued to Producers Bank of the Philippines consideration its purpose and always in accordance
supplying workers to an employer does not have or WON recovery is precluded under the general with the general principles of insurance law.
substantial capital or investment in the form of exceptions clause of the policy - With the foregoing principles in mind, it may now
tools, equipment, machineries, work premises, be asked whether Magalong and Atiga qualify as
among others, and the workers recruited and HELD employees or authorized representatives has been
INSURANCE Page 12

aptly observed that in burglary, robbery, and theft three acted as agents of Producers. A Ratio As the insurance contract is the law between
insurance, "the opportunity to defraud the insurer - "representative" is defined as one who represents or the parties, Verendia is deemed to have forfeited his
the moral hazard - is so great that insurers have stands in the place of another; one who represents right to claim by the misrepresentation he made.
found it necessary to fill up their policies with others or another in a special capacity, as an agent, Reasoning
countless restrictions, many designed to reduce this and is interchangeable with "agent." - the court reviewed the factual findings of the courts
hazard. Seldom does the insurer assume the risk of Disposition instant petition is hereby GRANTED. CA below, since it appears that there was a
all losses due to the hazards insured against." decision and RTC Makati decision are REVERSED and misapprehension of the facts by the CA.
Persons frequently excluded under such provisions SET ASIDE. Civil Case is DISMISSED. - Verendia is found to have concocted the lease
are those in the insured's service and employment. contract to deflect responsibility for the fire towards
The purpose of the exception is to guard against VERENDIA V CA (FIDELITY & SURETY CO. OF an alleged lessee, even making it appear that the
liability should the theft be committed by one having THE PHILS) alleged lessee had disappeared, inflated the value of
unrestricted access to the property. In such cases, 217 SCRA 417 the property, and insured same property with two
the terms specifying the excluded classes are to be MELO; January 22, 1993 other companies.
given their meaning as understood in common - An insurance contract is the law between the
speech. The terms "service" and "employment" are NATURE parties, its terms and conditions constitute the
generally associated with the idea of selection, Petition to review decision of the CA measure of the insurer’s liability and compliance
control, and compensation. therewith is a condition precedent to the insured’s
- There is marked disagreement between the parties FACTS right to recovery from the insurer.
on the correct meaning of the terms "employee" - Fidelity Co. issued a Fire Insurance Policy covering - As it is also a contract of adhesion, an insurance
and "authorized representatives." Verendia’s residential building in the amount of contract should be liberally construed in favor of the
It is clear to us that insofar as Fortune is concerned, P385k. Verendia also insured the same building with insured and strictly against the insurer company
it was its intention to exclude and exempt from two other companies (Country Bankers Insurance for which usually prepares it.
protection and coverage losses arising from P56k, and Development Insurance for P400k). - Considering, however, the fact that Verendia used a
dishonest, fraudulent, or criminal acts of persons - While all 3 policies were in force, the insured false lease contract to support his claim, the terms of
granted or having unrestricted access to Producers' property was completely destroyed by fire. Verendia the policy should be strictly construed against the
money or payroll. When it used then the term filed a claim against Fidelity, but the latter refused insured. Verendia failed to live by the terms of the
"employee," it must have had in mind any person payment, thus a complaint was filed in the RTC. policy, specifically Section 13 thereof which is
who qualifies as such as generally and universally Fidelity’s reason for refusal: the policy was avoided expressed in terms that are clear and unambiguous,
understood, or jurisprudentially established in the by reason of over-insurance, and that Verendia that all benefits under the policy shall be forfeited “If
light of the four standards in the determination of the maliciously represented that the building was under the claim be in any respect fraudulent, or if any false
employer-employee relationship or as statutorily lease to a Roberto Garcia, when it was actually a declaration be made or used in support thereof, or if
declared even in a limited sense as in the case of Marcelo Garcia who was the lessee. any fraudulent means or devises are used by the
Article 106 of the Labor Code which considers the - RTC: policy was violated by Verendia when it failed Insured or anyone acting in his behalf to obtain any
employees under a "labor-only" contract as to inform Fidelity of his other insurance coverages, benefit under the policy”. Verendia, having presented
employees of the party employing them and not of thus no need to pay. a false declaration to support his claim for benefits in
the party who supplied them to the employer. - CA: reversed decision the form of a fraudulent lease contract, he forfeited
- But even granting for the sake of argument that all benefits therein by virtue of Section 13 of the
these contracts were not "labor-only" contracts, and ISSUE policy in the absence of proof that Fidelity waived
PRC Management Systems and Unicorn Security (There is a procedural issue involved here, but is such provision. Worse yet, by presenting a false
Services were truly independent contractors, we are irrelevant to our discussion. It concerns the filing of a lease contract, Verendia reprehensibly disregarded
satisfied that Magalong and Atiga were, in respect of motion for extension of time to file a motion for the principle that insurance contracts are uberrimae
the transfer of Producer's money from its Pasay City reconsideration, where the court said that although it fidae and demand the most abundant good faith.
branch to its head office in Makati, its "authorized now prohibits filing of such motion, the instant Disposition Decision of CA reversed, and that of
representatives" who served as such with its teller motion was filed before the effectivity of this rule, RTC is reinstated.
Maribeth Alampay. Howsoever viewed, Producers thus allowing the adjudication of the case) FIELDMEN'S INSURANCE CO. INC V VDA. DE
entrusted the three with the specific duty to safely WON Fidelity was liable to pay Verendia considering SONGCO
transfer the money to its head office, with Alampay the circumstances 25 SCRA 20
to be responsible for its custody in transit; Magalong FERNANDO; 1968
to drive the armored vehicle which would carry the HELD
money; and Atiga to provide the needed security for 1. NO FACTS
the money, the vehicle, and his two other - An insurance firm, petitioner Fieldmen's Insurance
companions. In short, for these particular tasks, the Co., Inc., was not allowed to escape liability under a
INSURANCE Page 13

common carrier insurance policy on the pretext that Sambat, because our vehicle is an "owner" private which it knew all along when not once but twice its
what was insured, not once but twice, was a private vehicle and not for passengers,' to which agent agent, no doubt without any objection in its part,
vehicle and not a common carrier, the policy being Sambat replied: 'whether our vehicle was an "owner" exerted the utmost pressure on the insured, a man
issued upon the insistence of its agent who type or for passengers it could be insured because of scant education, to enter into such a contract.
discounted fears of the insured that his privately their company is not owned by the Government and - Nor is there any merit to the second alleged error
owned vehicle might not fall within its terms, the the Government has nothing to do with their of respondent Court that no legal liability was
insured moreover being "a man of scant education," company. So they could do what they please incurred under the policy by petitioner. Why liability
finishing only the first grade. So it was held in a whenever they believe a vehicle is insurable' ... In under the terms of the policy 5 was inescapable was
decision of the lower court thereafter affirmed by spite of the fact that the present case was filed and set forth in the decision of respondent Court of
respondent Court of Appeals. Petitioner in seeking tried in the CFI of Pampanga, the defendant Appeals. Thus: "Since some of the conditions
the review of the above decision of respondent Court company did not even care to rebut Amor Songco's contained in the policy issued by the defendant-
of Appeals cannot be so sanguine as to entertain the testimony by calling on the witness-stand agent appellant were impossible to comply with under the
belief that a different outcome could be expected. To Benjamin Sambat, its Pampanga Field existing conditions at the time and 'inconsistent with
be more explicit, we sustain the Court of Appeals. Representative." 2 the known facts,' the insurer 'is estopped from
- The facts as found by respondent Court of Appeals, - The plaintiffs in the lower court, likewise asserting breach of such conditions.' From this
binding upon us, follow: "This is a peculiar case. respondents here, were the surviving widow and jurisprudence, we find no valid reason to deviate and
Federico Songco of Floridablanca, Pampanga, a man children of the deceased Federico Songco as well as consequently hold that the decision appealed from
of scant education being only a first grader ..., owned the injured passenger Jose Manuel. On the above should be affirmed. The injured parties, to wit, Carlos
a private jeepney with Plate No. 41-289 for the year facts they prevailed, as had been mentioned, in the Songco, Angelito Songco and Jose Manuel, for whose
1960. On September 15, 1960, as such private lower court and in the respondent Court of hospital and medical expenses the defendant
vehicle owner, he was induced by Fieldmen's Appeals.1awphîl.nèt company was being made liable, were passengers of
Insurance Company Pampanga agent Benjamin - The basis for the favorable judgment is the doctrine the jeepney at the time of the occurrence, and
Sambat to apply for a Common Carrier's Liability announced in Qua Chee Gan v. Law Union and Rock Rodolfo Songco, for whose burial expenses the
Insurance Policy covering his motor vehicle ... Upon Insurance Co., Ltd., 3 with Justice J. B. L. Reyes defendant company was also being made liable was
paying an annual premium of P16.50, defendant speaking for the Court. It is now beyond question the driver of the vehicle in question. Except for the
Fieldmen's Insurance Company, Inc. issued on that where inequitable conduct is shown by an fact, that they were not fare paying passengers, their
September 19, 1960, Common Carriers Accident insurance firm, it is "estopped from enforcing status as beneficiaries under the policy is recognized
Insurance Policy No. 45-HO- 4254 ... the duration of forfeitures in its favor, in order to forestall fraud or therein." 6
which will be for one (1) year, effective September imposition on the insured." 4 - Even if it be assumed that there was an ambiguity,
15, 1960 to September 15, 1961. On September 22, - As much, if not much more so than the Qua Chee an excerpt from the Qua Chee Gan decision would
1961, the defendant company, upon payment of the Gan decision, this is a case where the doctrine of reveal anew the weakness of petitioner's contention.
corresponding premium, renewed the policy by estoppel undeniably calls for application. After Thus: "Moreover, taking into account the well known
extending the coverage from October 15, 1961 to petitioner Fieldmen's Insurance Co., Inc. had led the rule that ambiguities or obscurities must be strictly
October 15, 1962. This time Federico Songco's insured Federico Songco to believe that he could interpreted against the party that caused them, the
private jeepney carried Plate No. J-68136- qualify under the common carrier liability insurance 'memo of warranty' invoked by appellant bars the
Pampanga-1961. ... On October 29, 1961, during the policy, and to enter into contract of insurance paying latter from questioning the existence of the
effectivity of the renewed policy, the insured vehicle the premiums due, it could not, thereafter, in any appliances called for in the insured premises, since
while being driven by Rodolfo Songco, a duly licensed litigation arising out of such representation, be its initial expression, 'the undernoted appliances for
driver and son of Federico (the vehicle owner) permitted to change its stand to the detriment of the the extinction of fire being kept on the premises
collided with a car in the municipality of Calumpit, heirs of the insured. As estoppel is primarily based insured hereby, ... it is hereby warranted ...,' admits
province of Bulacan, as a result of which mishap on the doctrine of good faith and the avoidance of of interpretation as an admission of the existence of
Federico Songco (father) and Rodolfo Songco (son) harm that will befall the innocent party due to its such appliances which appellant cannot now
died, Carlos Songco (another son), the latter's wife, injurious reliance, the failure to apply it in this case contradict, should the parol evidence rule apply." 7
Angelita Songco, and a family friend by the name of would result in a gross travesty of justice. - To the same effect is the following citation from the
Jose Manuel sustained physical injuries of varying - That is all that needs be said insofar as the first same leading case: "This rigid application of the rule
degree." 1 alleged error of respondent Court of Appeals is on ambiguities has become necessary in view of
- It was further shown according to the decision of concerned, petitioner being adamant in its far-from- current business practices. The courts cannot ignore
respondent Court of Appeals: "Amor Songco, 42- reasonable plea that estoppel could not be invoked that nowadays monopolies, cartels and concentration
year-old son of deceased Federico Songco, testifying by the heirs of the insured as a bar to the alleged of capital, endowed with overwhelming economic
as witness, declared that when insurance agent breach of warranty and condition in the policy. lt power, manage to impose upon parties dealing with
Benjamin Sambat was inducing his father to insure would now rely on the fact that the insured owned a them cunningly prepared 'agreements' that the
his vehicle, he butted in saying: 'That cannot be, Mr. private vehicle, not a common carrier, something weaker party may not change one whit, his
INSURANCE Page 14

participation in the 'agreement' being reduced to the - TKC Marketing Corp. was the owner/consignee of acquisition value brought about by a voluntary
alternative to 'take it or leave it' labelled since some 3,189.171 metric tons of soya bean meal sacrifice sale and not by arrest, detention or seizure
Raymond Saleilles 'contracts by adherence' (contrats which was loaded on board the ship MV Al Kaziemah of the ship.
d'adhesion), in contrast to those entered into by for carriage from the port of Rio del Grande, Brazil, - its act of rejecting the claim was a result of its
parties bargaining on an equal footing, such to the port of Manila. Said cargo was insured against honest belief that the arrest of the vessel was not a
contracts (of which policies of insurance and the risk of loss by petitioner Malayan Insurance compensable risk under the policies issued
international bills of lading are prime examples) Corporation for which it issued two (2) Marine Cargo Respondents Comments
obviously call for greater strictness and vigilance on Policies. - petitioner, being the sole author of the policies,
the part of courts of justice with a view to protecting - While the vessel was docked in Durban, South "arrests" should be strictly interpreted against it
the weaker party from abuses and imposition, and Africa the civil authorities arrested and detained it because the rule is that any ambiguity is to be taken
prevent their becoming traps for the unwary (New because of a lawsuit on a question of ownership and contra proferentum. Risk policies should be
Civil Code. Article 24; Sent. of Supreme Court of possession. TKC Marketing notified Malayan of the construed reasonably and in a manner as to make
Spain, 13 Dec. 1934, 27 February 1942)." 8 arrest of the vessel and made a formal claim for the effective the intentions and expectations of the
- The last error assigned which would find fault with dollar equivalent on the policies (US$916,886.66) for parties.
the decision of respondent Court of Appeals insofar non-delivery of the cargo. It likewise sought the - the policies clearly stipulate that they cover the
as it affirmed the lower court award for exemplary assistance of Malayan on what to do with the cargo. risks of non-delivery of an entire package and that it
damages as well as attorney's fees is, on its face, of - Malayan replied that the arrest of the vessel by civil was petitioner itself that invited and granted the
no persuasive force at all. authority was not a peril covered by the policies. TKC extensions and collected premiums thereon.
- The conclusion that inescapably emerges from the advised Malayan that it might tranship the cargo and
above is the correctness of the decision of requested an extension of the insurance coverage ISSUES
respondent Court of Appeals sought to be reviewed. until actual transhipment, which extension was 1. WON the arrest of the vessel was a risk covered
For, to borrow once again from the language of the approved upon payment of additional premium. The under the subject insurance policies
Qua Chee Gan opinion: "The contract of insurance is insurance coverage was extended under the same 2. WON insurance policies should be strictly
one of perfect good faith (uberima fides) not for the terms and conditions embodied in the original construed against the insurer
insured alone,but equally so for the insurer; in fact, policies while in the process of making arrangements
it is more so for the latter, since its dominant for the transhipment of the cargo from Durban to HELD
bargaining position carries with it stricter Manila. However the cargo was sold in Durban, 1.YES
responsibility." 9 South Africa, for US$154.40 per metric ton or a total - With the incorporation of subsection 1.1 of Section
- This is merely to stress that while the morality of of P10,304,231.75 due to its perishable nature which 1 of the Institute War Clauses, "arrest" caused by
the business world is not the morality of institutions could no longer stand a voyage of twenty days to ordinary judicial process is deemed included among
of rectitude like the pulpit and the academe, it Manila and another twenty days for the discharge the covered risks. This interpretation becomes
cannot descend so low as to be another name for thereof. It reduced its claim to US$448,806.09 (or its inevitable when subsection 1.1 of Section 1 of the
guile or deception. Moreover, should it happen thus, peso equivalent of P9,879,928.89 at the exchange Institute War Clauses provided that "this insurance
no court of justice should allow itself to lend its rate of P22.0138 per $1.00) representing its loss covers the risks excluded from the Standard Form of
approval and support.1awphîl.nèt after the proceeds of the sale were deducted from English Marine Policy by the clause 'Warranted free of
- We have no choice but to recognize the monetary the original claim.Malayan maintained its position capture, seizure, arrest, etc. x x x'" or the F.C. & S.
responsibility of petitioner Fieldmen's Insurance Co., that the arrest of the vessel by civil authorities on a Clause. Jurisprudentially, "arrests" caused by
Inc. It did not succeed in its persistent effort to avoid question of ownership was an excepted risk under ordinary judicial process is also a risk excluded from
complying with its obligation in the lower court and the marine insurance policies. the Standard Form of English Marine Policy by the
the Court of Appeals. Much less should it find any Petitioners Claim F.C. & S. Clause.
receptivity from us for its unwarranted and - an arrest by civil authority is not compensable - Petitioner cannot adopt the argument that the
unjustified plea to escape from its liability. since the term "arrest" refers to "political or "arrest" caused by ordinary judicial process is not
executive acts" and does not include a loss caused included in the covered risk simply because the F.C.
MALAYAN INSURANCE CORP. V CA (TKC by riot or by ordinary judicial process as in this case & S. Clause under the Institute War Clauses can only
MARKETING CORP.) - the deletion of the Free from Capture or Seizure be operative in case of hostilities or warlike
270 SCRA 242 Clause would leave the assured covered solely for operations on account of its heading "Institute War
ROMERO; March 20, 1997 the perils specified by the wording of the policy itself Clauses."
- the rationale for the exclusion of an arrest pursuant 2. YES
NATURE to judicial authorities is to eliminate collusion Ratio Insurance Policies should be construed
Petition for review on certiorari between unscrupulous assured and civil authorities. liberally in favor of the insured and strictly against
- any loss which private respondent may have the insurer.
FACTS incurred was in the nature and form of unrecovered Reasoning
INSURANCE Page 15

- An insurance contract should be so interpreted as disregarded the stop signal given by a traffic WON the Schedule of indemnities as stated in the
to carry out the purpose for which the parties policeman to allow pedestrians to cross the road. insurance policy should be construed strictly to
entered into the contract which is, to insure against Priscilla was thrown to the ground, hitting her exclude all others not explicitly stated therein
risks of loss or damage to the goods. Such forehead. She was treated at the Protacio Emergency
interpretation should result from the natural and Hospital and later on hospitalized at the San Juan De
reasonable meaning of language in the policy. Where Dios Hospital. Her face was permanently disfigured,
restrictive provisions are open to two interpretations, causing her serious anxiety and moral distress. HELD
that which is most favorable to the insured is - Respondent bus company was insured with NO
adopted. petitioner Western Guaranty Corporation ("Western") Ratio An insurance policy being in the nature of an
Indemnity and liability insurance policies are under its Master Policy which enumerated specific adhesion contract is to be strictly construed against
construed in accordance with the general rule of liabilities of the insurance company and ended with a the insurer and liberally in favor of the insured.
resolving any ambiguity therein in favor of the clause to clarify the limitations of the amount which Reasoning
insured, where the contract or policy is prepared by could be granted as indemnity. - Firstly, the Schedule of Indemnities does not
the insurer. A contract of insurance, being a contract - Respondent Priscilla Rodriguez filed a complaint for purport to restrict the kinds of damages that may be
of adhesion, par excellence, any ambiguity therein damages before the Regional Trial Court of Makati awarded against Western once liability has arisen.
should be resolved against the insurer.Limitations of against De Dios Transportation Co. and Walter A. Section 1, quoted above, does refer to certain "Limits
liability should be regarded with extreme jealousy Saga. Respondent De Dios Transportation Co., in of Liability" which in the case of the third party
and must be construed in such a way as to preclude turn, filed a third-party complaint against its liability section of the Master Policy, is apparently
the insurer from noncompliance with its obligations insurance carrier, petitioner Western. P50,000.00 per person per accident. Within this
- It must be borne in mind that such contracts are - On 6 August 1985, the trial court rendered a over-all quantitative limit, all kinds of damages
invariably prepared by the companies and must be decision in favor of respondent Priscilla E. Rodriguez, allowable by law "actual or compensatory
accepted by the insured in the form in which they are - On appeal, the Court of Appeals affirmed in toto damages"; "moral damages"; "nominal damages";
written. Any construction of a marine policy the decision of the trial court. Petitioner moved for "temperate or moderate damages"; "liquidated
rendering it void should be avoided. Such policies the reconsideration of the appellate court's decision. damages"; and "exemplary damages" may be
will, therefore, be construed strictly against the In a Resolution dated 10 January 1990, the Court of awarded by a competent court against the insurer
company in order to avoid a forfeiture, unless no Appeals denied the motion for reconsideration for once liability is shown to have arisen, and the
other result is possible from the language used. lack of merit. Petitioner Western is now before us on essential requisites or conditions for grant of each
- If a marine insurance company desires to limit or a Petition for Review alleging that the Court of species of damages are present. It appears to us
restrict the operation of the general provisions of its Appeals erred in holding petitioner liable to pay self-evident that the Schedule of Indemnities was not
contract by special proviso, exception, or exemption, beyond the limits set forth in the Schedule intended to be an enumeration, much less a closed
it should express such limitation in clear and Indemnities and in finding Western liable for loss of enumeration, of the specific kinds of damages which
unmistakable language. earnings, moral damages and attorney's fees. may be awarded under the Master Policy Western
Be that as it may, exceptions to the general coverage Succinctly stated, it is petitioner Western's position has issued.
are construed most strongly against the company. that it cannot be held liable for loss of earnings, - Secondly, the reading urged by Western of the
Even an express exception in a policy is to be moral damages and attorney's fees because these Schedule of Indemnities comes too close to working
construed against the underwriters by whom the items are not among those included in the Schedule fraud upon both the insured and the third party
policy is framed, and for whose benefit the exception Indemnities set forth in the insurance policy. beneficiary of Section 1, quoted above. For Western's
is introduced. - Petitioner Western in effect contends before this reading would drastically and without warning limit
Court, as it did before the Court of Appeals, that the otherwise unlimited (save for the over-all
WESTERN GUARANTY CORPORATION V CA because the Schedule of Indemnities limits the quantitative limit of liability of P50,000.00 per person
(RODRIGUEZ, and DE DIOS TRANSPORTATION amount payable for certain kinds of expenses per accident) and comprehensive scope of liability
CO) "hospital room", "surgical expenses", "an assumed by the insurer Western under Section 1:
187 SCRA 652 aesthesiologists' fee", "operating room" and "medical "all sums necessary to discharge liability of the
FELICIANO; July 20, 1990 expenses" that Schedule should be read as insured in respect of [bodily injury to a third party]".
excluding liability for any other type of expense or This result which is not essentially different from
FACTS damage or loss even though actually sustained or taking away with the left hand what had been given
- At around 4:30 in the afternoon of 27 March 1982, incurred by the third party victim. We are not with the right hand we must avoid as obviously
while crossing Airport Road on a pedestrian lane on persuaded by Western's contention. repugnant to public policy. If what Western now
her way to work, respondent Priscilla E. Rodriguez urges is what Western intended to achieve by its
was struck by a De Dios passenger bus owned by ISSUE Schedule of Indemnities, it was incumbent upon
respondent De Dios Transportation Co., Inc., then Western to use language far more specific and
driven by one Walter Saga y Aspero. The bus driver precise than that used in fact by Western, so that the
INSURANCE Page 16

insured, and potential purchasers of its Master Policy, payment, claiming violation of warranties and 1. WON the defendant-appellant can claim the
and the Office of the Insurance Commissioner, may conditions, filing of fraudulent claims, and that the policies it had issued as void ab initio
be properly informed and act accordingly. fire had been deliberately caused by the insured or 2. WON the insured violated the "Hemp Warranty"
- Petitioner Western would have us construe the by other persons in connivance with him. provisions of Policy No. 2637165 against the storage
Schedule of Indemnities as comprising contractual - Que Chee Gan, with his brother, Qua Chee Pao, and of gasoline
limitations of liability which, as already noted, is some employees of his, were indicted and tried in 3. WON the insured connived at the loss and
comprehensively defined in Section 1 "Liability to 1940 for the crime of arson, it being claimed that fraudulently inflated the quantity of the insured stock
the Public" of the Master Policy. It is well-settled, they had set fire to the destroyed warehouses to in the burnt bodegas
however, that contractual limitations of liability found collect the insurance. They were, however, acquitted
in insurance contracts should be regarded by courts by the trial court. HELD
with a jaundiced eye and extreme care and should - the civil suit to collect the insurance money 1. NO
be so construed as to preclude the insurer from proceeded to its trial with the CFI holding that: Ratio It is usually held that where the insurer, at the
evading compliance with its just obligations. judgment is rendered for the plaintiff and against the time of the issuance of a policy of insurance, has
- Finally, an insurance contract is a contract of defendant condemning the latter to pay the former knowledge of existing facts which, if insisted on,
adhesion. The rule is well entrenched in our — (a) Under the first cause of action, the sum of would invalidate the contract from its very inception,
jurisprudence that the terms of such contract are to P146,394.48; (b) Under the second cause of action, such knowledge constitutes a waiver of conditions in
be construed strictly against the party which the sum of P150,000; (c) Under the third cause of the contract inconsistent with the facts, and the
prepared the contract, which in this case happens to action, the sum of P5,000; (d) Under the fourth insurer is stopped thereafter from asserting the
be petitioner Western. cause of action, the sum of P15,000; and (e) Under breach of such conditions. The law is charitable
the fifth cause of action, the sum of P40,000; all of enough to assume, in the absence of any showing to
QUA CHEE GAN V LAW UNION AND ROCK which shall bear interest at the rate of 8% per the contrary, that an insurance company intends to
INSURANCE CO., LTD. annum in accordance with Section 91 (b) of the executed a valid contract in return for the premium
96 PHIL 85 Insurance Act from September 26, 1940, until each received; and when the policy contains a condition
REYES; December 17, 1955 is paid, with costs against the defendant. which renders it voidable at its inception, and this
- In its first assignment of error, the insurance result is known to the insurer, it will be presumed to
NATURE company alleges that the trial Court should have held have intended to waive the conditions and to execute
An appeal by defendant insurance company from the that the policies were avoided for breach of warranty, a binding contract, rather than to have deceived the
decision of CFI in favor of the plaintiff specifically the one appearing on a rider pasted (with insured into thinking he is insured when in fact he is
other similar riders) on the face of the policies.4 not, and to have taken his money without
FACTS - It is argued that since the bodegas insured had an consideration.
- before the last war, plaintiff-appellee owned 4 external wall perimeter of 500 meters or 1,640 feet, Reasoning
warehouses or bodegas in Tabaco, Albay, used for the appellee should have 11 fire hydrants in the - The appellant is barred estoppel to claim violation
the storage of stocks of copra and of hemp, baled compound, and that he actually had only 2, with a of the so-called fire hydrants warranty, for the reason
and loose, in which the appellee dealt extensively. further pair nearby, belonging to the municipality of that knowing fully all that the number of hydrants
They had been, with their contents, insured with the Tabaco. demanded therein never existed from the very
defendant Company since 1937, and the loose made beginning, the appellant neverthless issued the
payable to the Philippine National Bank as mortgage ISSUES policies in question subject to such warranty, and
of the hemp and crops, to the extent of its interest. received the corresponding premiums. The insurance
- Fire of undetermined origin that broke out in the company was aware, even before the policies were
early morning of July 21, 1940, and lasted almost 4 issued, that in the premises insured there were only
Memo. of Warranty. — The undernoted Appliances for the extinction
one week, gutted and completely destroyed Bodegas two fire hydrants installed by Qua Chee Gan and two
of fire being kept on the premises insured hereby, and it being declared
Nos. 1, 2 and 4, with the merchandise stored and understood that there is an ample and constant water supply with others nearby, owned by the municipality of Tabaco,
therein. Plaintiff-appellee informed the insurer by sufficient pressure available at all seasons for the same, it is hereby contrary to the requirements of the warranty in
warranted that the said appliances shall be maintained in efficient
telegram on the same date; and on the next day, the working order during the currency of this policy, by reason whereof a
question
fire adjusters engaged by appellant insurance discount of 2 1/2 per cent is allowed on the premium chargeable under - The plain, human justice of this doctrine is
company arrived and proceeded to examine and this policy. perfectly apparent. To allow a company to
Hydrants in the compound, not less in number than one for each 150
photograph the premises, pored over the books of feet of external wall measurement of building, protected, with not less accept one's money for a policy of insurance
the insured and conducted an extensive than 100 feet of hose piping and nozzles for every two hydrants kept which it then knows to be void and of no effect,
under cover in convenient places, the hydrants being supplied with
investigation. The plaintiff having submitted the though it knows as it must, that the assured
water pressure by a pumping engine, or from some other source,
corresponding fire claims, totalling P398,562.81 (but capable of discharging at the rate of not less than 200 gallons of water believes it to be valid and binding, is so
reduced to the full amount of the insurance, per minute into the upper story of the highest building protected, and a contrary to the dictates of honesty and fair
trained brigade of not less than 20 men to work the same.'
P370,000), the Insurance Company resisted dealing, and so closely related to positive fraud,
INSURANCE Page 17

as to the abhorrent to fair-minded men. It held strictly against the insurer and liberally in favor alter this finding. The insured gave the insurance
would be to allow the company to treat the of the insured, specially to avoid a forfeiture. examiner all the date he asked for, and the examiner
policy as valid long enough to get the premium Insurance is, in its nature, complex and difficult even kept and photographed some of the examined
on it, and leave it at liberty to repudiate it the for the layman to understand. Policies are books in his possession. What does appear to have
next moment. This cannot be deemed to be the prepared by experts who know and can been rejected by the insured was the demand that
real intention of the parties. To hold that a anticipate the hearing and possible he should submit "a list of all books, vouchers,
literal construction of the policy expressed the complications of every contingency. So long as receipts and other records", but the refusal of the
true intention of the company would be to insurance companies insist upon the use of insured in this instance was well justified, since the
indict it, for fraudulent purposes and designs ambiguous, intricate and technical provisions, demand for a list of all the vouchers (which were not
which we cannot believe it to be guilty of. which conceal rather than frankly disclose, in use by the insured) and receipts was positively
- The appellant company so worded the policies that their own intentions, the courts must, in unreasonable, considering that such listing was
while exacting the greater number of fire hydrants fairness to those who purchase insurance, superfluous because the insurer was not denied
and appliances, it kept the premium discount at the construe every ambiguity in favor of the access to the records, that the volume of Qua Chee
minimum of 2 1/2%, thereby giving the insurance insured. An insurer should not be allowed, by Gan's business ran into millions, and that the
company a double benefit. Such abnormal treatment the use of obscure phrases and exceptions, to demand was made just after the fire when
of the insured strongly points at an abuse of the defeat the very purpose for which the policy everything was in turmoil. That the representatives
insurance company's selection of the words and was procured. of the insurance company were able to secure all the
terms of the contract, over which it had absolute Reasoning date they needed is proved by the fact that the
control. - Appellee admitted that there were 36 cans of adjuster Alexander Stewart was able to prepare his
- Receipt of Premiums or Assessments after Cause gasoline in the building designed. It However, own balance sheet that did not differ from that
for Forfeiture Other than Nonpayment. — It is a well gasoline is not specifically mentioned among the submitted by the insured except for the valuation of
settled rule of law that an insurer which with prohibited articles listed in the so-called "hemp the merchandise, as expressly found by the Court in
knowledge of facts entitling it to treat a policy as no warranty." The cause relied upon by the insurer the criminal case for arson.
longer in force, receives and accepts a premium on speaks of "oils (animal and/or vegetable and/or 3. NO
the policy, estopped to take advantage of the mineral and/or their liquid products having a flash Ratio Both defenses are predicted on the
forfeiture. It cannot treat the policy as void for the point below 300 Fahrenheit)", and is decidedly assumption that the insured was in financial
purpose of defense to an action to recover for a loss ambiguous and uncertain; for in ordinary parlance, difficulties and set the fire to defraud the insurance
thereafter occurring and at the same time treat it as "Oils" mean "lubricants" and not gasoline or company, presumably in order to pay off the
valid for the purpose of earning and collecting further kerosene. And how many insured, it may well be Philippine National Bank, to which most of the
premiums. wondered, are in a position to understand or insured hemp and copra was pledged. Both defenses
- Moreover, taking into account the well known rule determine "flash point below 300 Fahrenheit. are fatally undermined by the established fact that,
that ambiguities or obscurities must be strictly - If the company intended to rely upon a condition of notwithstanding the insurer's refusal to pay the value
interpreted against the party that caused them, the that character, it ought to have been plainly of the policies the extensive resources of the insured
"memo of warranty" invoked by appellant bars the expressed in the policy. enabled him to pay off the National Bank in a short
latter from questioning the existence of the - The contract of insurance is one of perfect good time; and if he was able to do so, no motive appears
appliances called for in the insured premises faith not for the insured alone, but equally so for the for attempt to defraud the insurer. While the
On the alleged violations of the plaintiff The insurer; in fact, it is mere so for the latter, since its acquittal of the insured in the arson case is not res
alleged violation of the warranty of 100 feet of fire dominant bargaining position carries with it stricter judicata on the present civil action, the insurer's
hose for every two hydrants, must be equally responsibility. evidence, to judge from the decision in the criminal
rejected, since the appellant's argument thereon is - Another point that is in favor of the insured is that case, is practically identical in both cases and must
based on the assumption that the insured was bound the gasoline kept in Bodega No. 2 was only incidental lead to the same result, since the proof to establish
to maintain no less than eleven hydrants, which to his business, being no more than a customary 2 the defense of connivance at the fire in order to
requirement appellant is estopped from enforcing. day's supply for the five or six motor vehicles used defraud the insurer "cannot be materially less
- As to maintenance of a trained fire brigade of 20 for transporting of the stored merchandise. "It is well convincing than that required in order to convict the
men, the record is preponderant that the same was settled that the keeping of inflammable oils on the insured of the crime of arson."
organized, and drilled, from time to give, although premises though prohibited by the policy does not - As to the defense that the burned bodegas could
not maintained as a permanently separate unit, void it if such keeping is incidental to the business." not possibly have contained the quantities of copra
which the warranty did not require. On the submission of books, voucbers, etc. The and hemp stated in the fire claims, the insurer's case
2. NO charge that the insured failed or refused to submit to rests almost exclusively on the estimates, inferences
Ratio Here, again, by reason of the exclusive control the examiners of the insurer the books, vouchers, and conclusions of its adjuster investigator who
of the insurance company over the terms and etc. demanded by them was found unsubstantiated examined the premises during and after the fire. His
phraseology of the contract, the ambiguity must be by the trial Court, and no reason has been shown to testimony, however, was based on inferences from
INSURANCE Page 18

the photographs and traces found after the fire, and Section 5. Injury sustained by a stroke of
must yield to the contradictory testimony of those lightning or by a cyclone P3000 HELD
who actually saw the contents of the bodegas shortly xxxx xxxx xxxx - All the parties agree that indemnity has to be paid,
before the fire, while inspecting them for the Part VI. Exceptions but the conflict centers on how much it should be.
mortgagee Bank. This policy shall not cover disappearance - Where there is ambiguity with respect to the terms
Disposition We find no reversible error in the of the Insured nor shall it cover Death, and conditions of the policy, the same will be
judgment appealed from, wherefore the same is Disability, Hospital fees, or Loss of time, resolved against the one responsible thereof.
hereby affirmed. caused to the insured: Generally, the insured has little, if any, participation
x x x (h) By drowning except as a in the preparation of the policy, together with the
DEL ROSARIO V EQUITABLE INSURANCE & consequence of the wrecking or drafting of its terms and conditions. The
CASUALTY CO., INC disablement in the Philippine waters of a interpretation of obscure stipulations in a contract
8 SCRA 343 passenger steam or motor vessel in should not favor the party who caused the obscurity.
PAREDES; June 29, 1963 which the Insured is traveling as a - SC agreed with the ruling of the lower court:
farepaying passenger; x x x x x x death by drowning is a ground for recovery
NATURE - A rider to the Policy contained the following; apart from the bodily injury because death by
Appeal from judgment of CFI Rizal IV. DROWNING bodily injury is covered by Part I of the policy
It is hereby declared and agreed that while death by drowning is covered by Part VI
FACTS exemption clause Letter (h) in PART VI of thereof. But while the policy mentions specific
- Francisco del Rosario was insured by Equitable the policy is hereby waived by the amounts that may be recovered for death for
Insurance and Casualty Co. Inc under Personal company, and to form a part of the bodily injury, yet, there is not specific amount
Accident Policy no. 7136. The Company bound itself provision covered by the policy. mentioned in the policy for death thru drowning
to pay P1000 to P3000 as indemnity for the death - Feb 24, 1957, Francisco del Rosario while on board although the latter is, under Part VI of the policy,
of the insured. the motor launch ISLAMA, with his beneficiary to the a ground for recovery thereunder. Since the
- Under the policy: policy, Remedios Jayme, were forced to jump off said defendant has bound itself to pay P1000 to
Part I. Indemnity for Death launch on account of fire which broke out on said P3000 as indemnity for the death of the insured
If the insured sustains any bodily injury vessel, resulting in the death by drowning of the but the policy does not positively state any
which is effected solely through violent, insured and his beneficiary. definite amount that may be recovered in case of
external, visible and accidental means, - Simeon del Rosario, the insured’s father, filed a death by drowning, there is an ambiguity in this
and which shall result, independently of claim for payment with the company. The company respect in the policy, which ambiguity must be
all other causes and within sixty days paid him P1000 pursuant to section 1 Part I of the interpreted in favor of the insured and strictly
from the occurrence thereof, in the Death policy. against the insurer so as to allow a greater
of the Insured, the Company shall pay the - On the same date, Atty. Francisco wrote to the indemnity. x x x plaintiff is therefore entitled to
amount set opposite such injury: company acknowledging receipt by his client of the recover P3000.
P1000 but informing said company that said amount Disposition Judgment appealed from is affirmed.
Section 1. Injury sustained other than was not the correct one. He claimed that the amount
those specified below unless excepted payable should be P1500 under the provision of GEAGONIA v. CA (COUNTRY BANKERS
hereinafter P1000 Section 2 Part I, based on the rule of pari materia. INSURANCE)
Section 2. Injury sustained by the - The company referred the matter to the Insurance 8 SCRA 343
wrecking or disablement of a railroad Coomissioner, who was of the opinion that the DAVIDE; February 6 1995
passenger car or street railway car in or liability of the company was only P1000. thus the
on which the Insured is traveling as a company refused to pay more that P1000. Atty. FACTS
farepaying passenger P1500 Francisco wrote a subsequent letter to company -Geagonia is the owner of Norman's Mart located in
Section 3. Injury sustained by the burning asking for p3000, which the company refused to pay. the public market of San Francisco, Agusan del Sur.
of a church, theatre, public library or - A complaint for recovery of the balance of P2000 On 22 Dec 1989, he obtained from the private
municipal administration building while was instituted with the CFI Rizal, praying for a respondent fire insurance policy for P100,000.00.
the Insured is therein at the further sum of P10000 as attorney’s fees, expenses The period of the policy was from 22 Dec 1989 to 22
commencement of the fire P2000 of litigation and costs. Dec 1990 and covered the ff: "Stock-in-trade
Section 4. Injury sustained by the - CFI ruled in favor of petitioner, ordering the consisting principally of dry goods such as RTW's for
wrecking or disablement of a regular company to pay P2000 to del Rosario. men and women wear and other usual to assured's
passenger elevator car in which the business.
Insured is being conveyed as a passenger ISSUE -The policy contained the following condition:
(Elevator in mines exluded) P2500 How much should the indemnity be
INSURANCE Page 19

"3. The insured shall give notice to the Company of on the stocks. These findings were based on the whole or a portion of the objects thereby assured
any insurance or insurances already effected, or petitioner's testimony that he came to know of the must be declared by the insured in writing and he
which may subsequently be effected, covering any of PFIC policies only when he filed his claim with the must cause the company to add or insert it in the
the property or properties consisting of stocks in private respondent and that Cebu Tesing Textile policy, without which such policy shall be null and
trade, goods in process and/or inventories only obtained them and paid for their premiums w/o void, and the insured will not be entitled to
hereby insured, and unless notice be given and the informing him. The Insurance Commission then indemnity in case of loss," Condition 3 in the
particulars of such insurance or insurances be stated ordered the respondent company to pay complainant private respondent's policy No. F-14622 does
therein or endorsed in this policy pursuant to Section the sum of P100,000.00 with legal interest from the not absolutely declare void any violation
50 of the Insurance Code, by or on behalf of the time the complaint was filed until fully satisfied plus thereof. It expressly provides that the
Company before the occurrence of any loss or the amount of P10,000.00 as attorney's fees. condition "shall not apply when the total
damage, all benefits under this policy shall be -CA reversed the decision of the Insurance insurance or insurances in force at the time of
deemed forfeited, provided however, that this Commission because it found that the petitioner the loss or damage is not more than
condition shall not apply when the total insurance or knew of the existence of the two other policies issued P200,000.00."
insurances in force at the time of the loss or damage by the PFIC - Interpretation: It is a cardinal rule on insurance
is not more than P200,000.00." that a policy or insurance contract is to be
-On 27 May 1990, fire of accidental origin broke out ISSUES interpreted liberally in favor of the insured and
at around 7:30 p.m. at the public market of San 1. WON the petitioner had prior knowledge of the strictly against the company, the reason being,
Francisco, Agusan del Sur. The petitioner's insured two insurance policies issued by the PFIC when he undoubtedly, to afford the greatest protection which
stocks-in-trade were completely destroyed prompting obtained the fire insurance policy from the private the insured was endeavoring to secure when he
him to file w/ the private respondent a claim under respondent, thereby, for not disclosing such fact, applied for insurance. It is also a cardinal principle of
the policy. On 28 Dec 1990, the private respondent violating Condition 3 of the policy law that forfeitures are not favored and that any
denied the claim because it found that at the time of 2. if he had, WON he is precluded from recovering construction which would result in the forfeiture of
the loss the petitioner's stocks-in-trade were likewise therefrom the policy benefits for the person claiming
covered by two fire insurance policies for thereunder, will be avoided, if it is possible to
P100,000.00 each, issued by the Cebu Branch of the HELD construe the policy in a manner which would permit
Philippines First Insurance Co., Inc. (PFIC). 1. YES recovery, as, for example, by finding a waiver for
-The basis of the private respondent's denial was the - We agree w/ the CA that the petitioner knew of the such forfeiture. Stated differently, provisions,
petitioner's alleged violation of Condition 3 of the prior policies issued by the PFIC. His letter of 18 conditions or exceptions in policies which tend
policy. January 1991 to the private respondent conclusively to work a forfeiture of insurance policies should
- Geagonia then filed a complaint against the private proves this knowledge. His testimony to the contrary be construed most strictly against those for
respondent w/ the Insurance Commission for the before the Insurance Commissioner and which the whose benefits they are inserted, and most
recovery of P100,000.00 under fire insurance policy, latter relied upon cannot prevail over a written favorably toward those against whom they are
for attorney's fees, and costs of litigation. He claims admission made ante litem motam. It was, indeed, intended to operate. The reason for this is that,
that the time he obtained the private respondent's incredible that he did not know about the prior except for riders which may later be inserted, the
fire insurance policy he knew that the two policies policies since these policies were not new or original. insured sees the contract already in its final form and
issued by the PFIC were already in existence; 2. NO has had no voice in the selection or arrangement of
however, he had no knowledge of the provision in the - It must, however, be underscored that unlike the the words employed therein. On the other hand, the
private respondent's policy requiring him to inform it "other insurance" clauses involved in General language of the contract was carefully chosen and
of the prior policies; this requirement was not Insurance and Surety Corp. vs. Ng Hua or in deliberated upon by experts and legal advisers who
mentioned to him by the private respondent's agent; Pioneer Insurance & Surety Corp. vs. Yap, which had acted exclusively in the interest of the insurers
and had it been so mentioned, he would not have read: and the technical language employed therein is
withheld such information. He further asserted that "The insured shall give notice to the company of any rarely understood by ordinary laymen.
the total of the amounts claimed under the three insurance or insurances already effected, or which - With these principles in mind, we are of the
policies was below the actual value of his stocks at may subsequently be effected covering any of the opinion that Condition 3 of the subject policy is
the time of loss, w/c was P1M. property hereby insured, and unless such notice be not totally free from ambiguity and must be
- The Insurance Commission found that the given and the particulars of such insurance or meticulously analyzed. Such analysis leads us
petitioner did not violate Condition 3 as he had no insurances be stated in or endorsed on this Policy by to conclude that (a) the prohibition applies only
knowledge of the existence of the two fire insurance or on behalf of the Company before the occurrence to double insurance, and (b) the nullity of the
policies obtained from the PFIC; that it was Cebu of any loss or damage, all benefits under this Policy policy shall only be to the extent exceeding
Tesing Textiles w/c procured the PFIC policies w/o shall be forfeited." or in the 1930 case of Santa Ana P200,000.00 of the total policies obtained.
informing him or securing his consent; and that Cebu vs. Commercial Union Assurance Co. which - Furthermore, by stating within Condition 3 itself
Tesing Textile, as his creditor, had insurable interest provided "that any outstanding insurance upon the that such condition shall not apply if the total
INSURANCE Page 20

insurance in force at the time of loss does not exceed Petitioners’ Claim his miscalculations. They were intended to provide
P200,000.00, the private respondent was amenable - Sun Insurance cites one of the four exceptions in for contingencies.
to assume a co-insurer's liability up to a loss not the contract of insurance which includes bodily injury - Lim was unquestionably negligent but it should not
exceeding P200,000.00. What it had in mind was to consequent upon the insured person attempting to prevent his widow from recovering from the
discourage over-insurance. Indeed, the rationale commit suicide or willfully exposing himself to insurance policy he obtained precisely against
behind the incorporation of "other insurance" clause needless peril in an attempt to save a human life. accident.
in fire policies is to prevent over-insurance and thus - There mere act of pointing the gun to his temple - Insurance contracts are, as a rule, supposed to be
avert the perpetration of fraud. When a property showed that Felix willfully exposed himself to danger interpreted liberally in favor of the assured.
owner obtains insurance policies from two or more because a gun should always be handled with 2. NO, the claim for damages should not be granted
insurers in a total amount that exceeds the caution. for being unjust.
property's value, the insured may have an Respondents’ Comments Ratio A person may be made liable to the payment
inducement to destroy the property for the purpose - Felix believed the gun to be safe because he had of moral damages if his act is wrongful. The adverse
of collecting the insurance. The public as well as the removed the magazine. result of an action does not per se make the act
insurer is interested in preventing a situation in - He repeatedly assured his secretary that the gun wrongful and subject the act or to the payment of
which a fire would be profitable to the insured. was not loaded. moral damages.
Disposition Petition granted. The decision of the Reasoning
Court of Appeals in CA-G.R. SP No. 31916 is SET ISSUES - Petitioner was acting in good faith when it resisted
ASIDE and the decision of the Insurance Commission 1. WON Felix Lim’s death was an accident, thus the private respondent’s claim on the ground that the
in Case No. 3340 is REINSTATED. making his widow Nerissa liable to claim the accident death of the insured was covered by the exception.
insurance - The issue was debatable and was clearly not raised
SUN INSURANCE OFFICE, LTD. V CA (LIM) 2. WON the award of damages to Nerissa Lim was only for the purpose of evading a legitimate
211 SCRA 554 justified obligation.
CRUZ; July 17, 1992
HELD RIZAL SURETY & INSURANCE COMPANY V CA
NATURE 1. YES, Felix Lim’s death was an accident. (TRANSWORLD KNITTING MILLS, INC.)
Petition for review from the decision of the Court of Ratio There is no accident when a deliberate act is 336 SCRA 12
Appeals performed unless some additional, unexpected, PURISIMA; July 18, 2000
independent and unforeseen happening occurs which
FACTS produces or brings bout their injury or death. NATURE
- Felix Lim was issued a Personal Accident Policy Reasoning Petition for Review on Certiorari under Rule 45 of the
insurance with petitioner company with a face value - An accident has been defined to be that which Rules of Court
of P200,000. His beneficiary was his wife Nerissa. happens by chance or fortuitously without intention
- October 6, 1982 – Felix accidentally shot himself in or design and which is unexpected, unusual and FACTS
the head with his own gun.  unforeseen. It an event that takes pace without - Rizal Surety & Insurance Company (Rizal
- He was playing with the handgun after he had one’s foresight or expectastion – an event that Insurance) issued Fire Insurance Policy No. 45727 in
removed the gun’s magazine (kasi naman…). proceeds from an unknown cause or is an unusual favor of Transworld Knitting Mills, Inc. (Transworld).
- He pointed the gun at his secretary and only effect of a known case and therefore not expected. - Pertinent portions of subject policy on the buildings
witness Pilar Nalagon as a joke and assured her It happens without any human agency, an event insured, and location thereof, read:
that the gun was not loaded (are you sure…). which, under the circumstances, is unusual to and "‘On stocks of finished and/or unfinished
- He then put the gun to his temple and fired it not expected by the person to whom it happens. products, raw materials and supplies of every
(haaay, sabi ko na nga ba). - The firing of the gun was deemed to be the kind and description, the properties of the
- Both parties are in agreement that there was no unexpected and independent and unforeseen Insureds and/or held by them in trust, on
suicide. occurrence that led to the insured person’s death. commission or on joint account with others
- Nerissa claimed as Felix’s beneficiary but Sun - There was no willful exposure to needless peril for and/or for which they (sic) responsible in case
Insurance would not grant her claim, saying that her the part of Felix. Suicide and exposure to needless of loss whilst contained and/or stored during
husband’s death was not an accident. peril are similar in the sense that both signify the currency of this Policy in the premises
- Nerissa sued Sun Insurance and won the case. Sun disregard for one’s life. Suicide imparts a positive occupied by them forming part of the buildings
Insurance was ordered to pay her P200,000 act of ending one’s life whereas the latter indicates situate (sic) within own Compound at
representing the face value of the claim along with recklessness that is almost suicidal in intent. MAGDALO STREET, BARRIO UGONG, PASIG,
moral, exemplary and compensatory damages and - Accident insurance policies were never meant to METRO MANILA, PHILIPPINES, BLOCK NO.
attorney’s fees. The decision was affirmed by the reward the insured for his tendency to show off or for 601.’
CA.
INSURANCE Page 21

x appellant the amount of P1,818,604.19 while the - The Court is mindful of the well-entrenched
xx............ Rizal Surety has to pay the plaintiff-appellant doctrine that factual findings by the Court of Appeals
...xxx....... P470,328.67. are conclusive on the parties and not reviewable by
........xxx - New India appealed to the Court theorizing inter this Court, and the same carry even more weight
‘Said building of four-span lofty one storey in alia that the private respondent could not be when the Court of Appeals has affirmed the findings
height with mezzanine portions is constructed compensated for the loss of the fun and of fact arrived at by the lower court.
of reinforced concrete and hollow blocks amusement machines and spare parts stored at - In the case under consideration, both the trial court
and/or concrete under galvanized iron roof the two-storey building because it (Transworld) and the Court of Appeals found that the so called
and occupied as hosiery mills, garment and had no insurable interest in said goods or items. "annex " was not an annex building but an integral
lingerie factory, transistor-stereo assembly - The Court denied the appeal with finality. and inseparable part of the four-span building
plant, offices, warehouse and caretaker's - Petitioner Rizal Insurance and private respondent described in the policy and consequently, the
quarters. Transworld, interposed a Motion for machines and spare parts stored therein were
'Bounds in front partly by one-storey concrete Reconsideration before the Court of Appeals, which covered by the fire insurance in dispute.
building under galvanized iron roof occupied reconsidered its decision of July 15, 1993, as - Verily, the two-storey building involved, a
as canteen and guardhouse, partly by building regards the imposition of interest. permanent structure which adjoins and
of two and partly one storey constructed of - Undaunted, petitioner Rizal Surety & Insurance intercommunicates with the "first right span of the
concrete below, timber above undergalvanized Company found its way to the Court. lofty storey building", formed part thereof, and
iron roof occupied as garage and quarters and meets the requisites for compensability under the
partly by open space and/or tracking/ ISSUE fire insurance policy sued upon.
packing, beyond which is the aforementioned WON the fire insurance policy litigated upon - So also, considering that the two-storey building
Magdalo Street; on its right and left by protected only the contents of the main building aforementioned was already existing when subject
driveway, thence open spaces, and at the rear (four-span), and did not include those stored in the fire insurance policy contract was entered into,
by open spaces.'" two-storey annex building petitioner should have specifically excluded the said
- The same pieces of property insured with the two-storey building from the coverage of the fire
petitioner were also insured with New India HELD insurance if minded to exclude the same but if did
Assurance Company, Ltd., (New India). NO not, and instead, went on to provide that such fire
- Fire broke out in the compound of Transworld, - Resolution of the issue posited hinges on the proper insurance policy covers the products, raw materials
razing the middle portion of its four-span building interpretation of the stipulation in subject fire and supplies stored within the premises of
and partly gutting the left and right sections insurance policy regarding its coverage, which reads: respondent Transworld which was an integral part of
thereof. A two-storey building (behind said four- "xxx contained and/or stored during the currency of the four-span building occupied by Transworld,
span building) where fun and amusement this Policy in the premises occupied by them forming knowing fully well the existence of such building
machines and spare parts were stored, was also part of the buildings situate (sic) within own adjoining and intercommunicating with the right
destroyed by the fire. Compound xxx" section of the four-span building.
- Transworld filed its insurance claims with Rizal - It can be gleaned unerringly that the fire insurance - Indeed, the stipulation as to the coverage of the
Surety & Insurance Company and New India policy in question did not limit its coverage to what fire insurance policy under controversy has created a
Assurance Company but to no avail. were stored in the four-span building. As opined by doubt regarding the portions of the building insured
- Private respondent brought against the said the trial court of origin, two requirements must thereby. Article 1377 of the New Civil Code provides:
insurance companies an action for collection of concur in order that the said fun and amusement "Art.1377. The interpretation of obscure words
sum of money and damages. machines and spare parts would be deemed or stipulations in a contract shall not favor the
- Petitioner Rizal Insurance countered that its fire protected by the fire insurance policy under scrutiny, party who caused the obscurity"
insurance policy sued upon covered only the to wit: - Conformably, it stands to reason that the doubt
contents of the four-span building, which was "First, said properties must be contained and/or should be resolved against the petitioner, Rizal
partly burned, and not the damage caused by the stored in the areas occupied by Transworld and Surety Insurance Company, whose lawyer or
fire on the two-storey annex building. second, said areas must form part of the building managers drafted the fire insurance policy contract
- The trial court dismissed the case as against The described in the policy xxx" under scrutiny. Citing the aforecited provision of law
New India Assurance Co., Ltd. but ordered - Said building of four-span lofty one storey in height in point, the Court in Landicho vs. Government
defendant Rizal Surety And Insurance Company to with mezzanine portions is constructed of reinforced Service Insurance System, ruled:
pay Transwrold (sic) Knitting Mills, Inc. concrete and hollow blocks and/or concrete under "This is particularly true as regards insurance
- Both the petitioner, Rizal Insurance Company, galvanized iron roof and occupied as hosiery mills, policies, in respect of which it is settled that the
and private respondent, Transworld Knitting Mills, garment and lingerie factory, transistor-stereo 'terms in an insurance policy, which are
Inc., went to the Court of Appeals, which required assembly plant, offices, ware house and caretaker's ambiguous, equivocal, or uncertain x x x are to
New India Assurance Company to pay plaintiff- quarter. be construed strictly and most strongly against
INSURANCE Page 22

the insurer, and liberally in favor of the insured fine, the previous judgment is conclusive in the or negligence of a party other than the assured, then
so as to effect the dominant purpose of second case, only as those matters actually and the insurer, upon payment to the assured, will be
indemnity or payment to the insured, especially directly controverted and determined and not as to subrogated to the rights of the assured to recover
where forfeiture is involved' and the reason for matters merely involved therein." from the wrongdoer to the extent that the insurer
this is that the 'insured usually has no voice in Disposition Decision, and the Resolution of the CA has been obligated to pay.
the selection or arrangement of the words WERE AFFIRMED in toto. No pronouncement as to - General Rule: Payment by the insurer to the
employed and that the language of the contract costs. assured operates as an equitable assignment to the
is selected with great care and deliberation by former of all remedies which the latter may have
experts and legal advisers employed by, and PAN MALAYAN INSURANCE CORPORATION vs. against the third party whose negligence or wrongful
acting exclusively in the interest of, the COURT OF APPEALS (ERLINDA FABIE & HER act caused the loss. The right of subrogation is not
insurance company.' " UNKNOWN DRIVER) dependent upon, nor does it grow out of, any privity
- Equally relevant is the following disquisition of the 184 SCRA 55; G.R. No. 81026 of contract or upon written assignment of claim. It
Court in Fieldmen's Insurance Company, Inc. vs. CORTES; April 3, 1990 accrues simply upon payment of the insurance claim
Vda. De Songco, to wit: by the insurer.
"'This rigid application of the rule on ambiguities FACTS - Exceptions:
has become necessary in view of current - December 10, 1985: PANMALAY filed a complaint a.) if the assured by his own act releases the
business practices. The courts cannot ignore for damages with the RTC of Makati against private wrongdoer or third party liable for the loss or
that nowadays monopolies, cartels and respondents Erlinda Fabie and her driver. PANMALAY damage, from liability
concentration of capital, endowed with averred the following: that it insured a Mitsubishi b.) where the insurer pays the assured the value of
overwhelming economic power, manage to Colt Lancer car registered in the name of Canlubang the lost goods without notifying the carrier who has
impose upon parties dealing with them Automotive Resources Corporation [CANLUBANG]; in good faith settled the assured's claim for loss
cunningly prepared 'agreements' that the that on May 26, 1985, due to the "carelessness, c.) where the insurer pays the assured for a loss
weaker party may not change one whit, his recklessness, and imprudence" of the unknown which is not a risk covered by the policy, thereby
participation in the 'agreement' being reduced to driver of a pick-up, the insured car was hit and effecting "voluntary payment"
the alternative to 'take it or leave it' labelled suffered damages in the amount of P42,052.00; that - None of the exceptions are availing in the present
since Raymond Saleilles 'contracts by PANMALAY defrayed the cost of repair of the insured case.
adherence' (contrats [sic] d'adhesion), in car and, therefore, was subrogated to the rights of - When PANMALAY utilized the phrase "own damage"
contrast to these entered into by parties CANLUBANG against the driver of the pick-up and his — a phrase which, incidentally, is not found in the
bargaining on an equal footing, such contracts employer, Erlinda Fabie; and that, despite repeated insurance policy — to define the basis for its
(of which policies of insurance and international demands, defendants, failed and refused to pay the settlement of CANLUBANG's claim under the policy, it
bills of lading are prime example) obviously call claim of PANMALAY. PANMALAY clarified that the simply meant that it had assumed to reimburse the
for greater strictness and vigilance on the part damage caused to the insured car was settled under costs for repairing the damage to the insured
of courts of justice with a view to protecting the the "own damage", coverage of the insurance policy. vehicle.
weaker party from abuses and imposition, and - Private respondents filed a Motion to Dismiss - It is a basic rule in the interpretation of contracts
prevent their becoming traps for the unwary.'" alleging that PANMALAY had no cause of action that the terms of a contract are to be construed
- The issue of whether or not Transworld has an against them. RTC dismissed PANMALAY's complaint according to the sense and meaning of the terms
insurable interest in the fun and amusement for no cause of action and denied PANMALAY's which the parties thereto have used. In the case of
machines and spare parts, which entitles it to be motion for reconsideration. CA affirmed. Hence, this property insurance policies, the evident intention of
indemnified for the loss thereof, had been settled in petition for review. the contracting parties, i.e., the insurer and the
G.R. No. L-111118, entitled New India Assurance assured, determine the import of the various terms
Company, Ltd., vs. Court of Appeals, where the ISSUE and provisions embodied in the policy. It is only
appeal of New India from the decision of the Court of WON the insurer PANMALAY may institute an action when the terms of the policy are ambiguous,
Appeals under review, was denied with finality by to recover the amount it had paid its assured in equivocal or uncertain, such that the parties
this Court on February 2, 1994. settlement of an insurance claim against private themselves disagree about the meaning of particular
- The rule on conclusiveness of judgment, which respondents as the parties allegedly responsible for provisions, that the courts will intervene. In such an
obtains under the premises, precludes the relitigation the damage caused to the insured vehicle event, the policy will be construed by the courts
of a particular fact or issue in another action between liberally in favor of the assured and strictly against
the same parties based on a different claim or cause HELD the insurer.
of action. "xxx the judgment in the prior action YES - Considering that the very parties to the policy were
operates as estoppel only as to those matters in - Article 2207 of the Civil Code is founded on the not shown to be in disagreement regarding the
issue or points controverted, upon the determination well-settled principle of subrogation. If the insured meaning and coverage of Section III-1, specifically
of which the finding or judgment was rendered. In property is destroyed or damaged through the fault sub-paragraph (a) thereof, it was improper for the
INSURANCE Page 23

appellate court to indulge in contract construction, to 1991 to 1992.The new oil mill was insured under - As may be gleaned from the testimony of the
apply the ejusdem generis rule, and to ascribe Policy No. 306-7432321-9 for the same term. Official petitioner’s employee, the source of the discrepancy
meaning contrary to the clear intention and receipts indicating payment for the full amount of the happened during the preparation of the written
understanding of these parties. premium were issued by the petitioner's agent. contract.
- Although the terms "accident" or "accidental" as - Policy description: - Respondent is not estopped from claiming that the
used in insurance contracts have not acquired a Front: by a driveway thence at 18 meters distance policy description is wrong. Evidence on record
technical meaning, the Court has on several by Bldg. No. 2. reveals that respondent’s operating manager, Mr.
occasions defined these terms to mean that which Right: by an open space thence by Bldg. No. 4. Edison Tantuco, notified the petitioner’s agent with
takes place "without one's foresight or expectation, Left: Adjoining thence an imperfect wall by Bldg. whom respondent negotiated for the contract about
an event that proceeds from an unknown cause, or is No. 4. the inaccurate description in the policy. However, Mr.
an unusual effect of a known cause and, therefore, Rear: by an open space thence at 8 meters Borja assured Mr. Tantuco that the use of the
not expected." The concept "accident" is not distance.” adjective new will distinguish the insured property.
necessarily synonymous with the concept of "no - A fire that broke out in the early morning of - Regarding policy requirements that fire
fault". It may be utilized simply to distinguish September 30,1991 gutted and consumed the new extinguishment appliances should be available and in
intentional or malicious acts from negligent or oil mill. Petitioner rejected respondent’s claim for good working condition, warranty did not require
careless acts of man. the insurance proceeds on the ground that no policy respondent to provide for all the fire extinguishing
- Obiter Dicta: Even if under the above was issued by it covering the burned oil mill. It appliances enumerated therein. Neither did it
circumstances PANMALAY could not be deemed stated that the description of the insured require that the appliances are restricted to those
subrogated to the rights of its assured under Article establishment referred to another building. mentioned in the warranty. (Within the vicinity of the
2207 of the Civil Code, PANMALAY would still have a Petitioner’s Claim new oil mill can be found the following devices:
cause of action against private respondents. The The policies referred to the old mill, as stated in the numerous portable fire extinguishers, two fire hoses,
insurer who may have no rights of subrogation due description contained in the policy. fire hydrant, and an emergency fire engine.)
to "voluntary" payment may nevertheless recover - The object of the court in construing a contract is
from the third party responsible for the damage to ISSUE to ascertain the intent of the parties to the contract
the insured property under Article 1236 of the Civil WON new oil mill is insured by fire insurance policy and to enforce the agreement which the parties have
Code. entered into. In determining what the parties
Disposition Petition is GRANTED. Petitioner's HELD intended, the courts will read and construe the policy
complaint for damages against private respondents is YES, new oil mill is insured. as a whole and if possible, give effect to all the parts
REINSTATED. Case remanded to the lower court for Ratio In construing the words used descriptive of a of the contract.
trial on the merits. building insured, the greatest liberality is shown by Disposition Petition is dismissed.
the courts in giving effect to the insurance. In view
AMERICAN HOME ASSURANCE COMPANY V of the custom of insurance agents to examine PERLA COMPANIA DE SEGUROS, INC. v, CA
TANTUCO ENTERPRISES buildings before writing policies upon them, and (MILAGROS CAYAS)
366 SCRA 740 since a mistake as to the identity and character of 185 SCRA 741
PUNO; October 8, 2001 the building is extremely unlikely, the courts are FERNAN; May 28, 1990
inclined to consider that the policy of insurance
NATURE covers any building which the parties manifestly NATURE
Petition for Review on Certiorari assailing the intended to insure, however inaccurate the Petition for review on certiorari of a decision of the
Decision of the Court of Appeals. description may be. Court of Appeals
Reasoning
FACTS - The parties manifestly intended to insure the new FACTS
- Respondent Tantuco Enterprises, Inc. is engaged in oil mill. “On machineries and equipment with - Private respondent Milagros Cayas was the
the coconut oil milling and refining industry. It owns complete accessories usual to a coconut oil mill registered owner of a Mazda bus, insured with Perla
two oil mills. Both are located at its factory including stocks of copra, copra cake and copra mills Compania de Seguros, Inc. (PCSI) under a policy
compound at Iyam, Lucena City. Respondent whilst contained in the new oil mill building, situate issued on February 3, 1978.
commenced its business operations with only one oil (sic) at UNNO. ALONG NATIONAL HIGH WAY, BO. - On December 17, 1978, the bus figured in an
mill. In 1988, it started operating its second oil mill IYAM, LUCENA CITY UNBLOCKED.” accident in Naic, Cavite injuring several of its
( the new oil mill). - If the parties really intended to protect the first oil passengers.
- The two oil mills were separately covered by fire mill, then there is no need to specify it as new. It - One of them, 19-year old Edgardo Perea, sued
insurance policies issued by petitioner American would be absurd to assume that respondent would Milagros Cayas for damages in the CFI of Cavite,
Home Assurance Co. The first oil mill was insured protect its first oil mill for different amounts and while three others agreed to a settlement of
Policy No. 306-7432324-3 for the period March 1, leave uncovered its second one. P4,000.00 each.
INSURANCE Page 24

- After trial, the court rendered a decision in favor of the measure of the insurer's liability and compliance may be, whenever the essential requisites for their
Perea, ordering Cayas to compensate him, with an therewith is a condition precedent to the insured's validity are present.
award of exemplary and moral damages, as well as right of recovery from the insurer. - In Pacific Oxygen & Acetylene Co. vs. Central
attorney’s fees. ( P32,000 total) - In the case at bar, the insurance policy clearly and Bank," it was stated that the first and fundamental
- On November 11, 1981, Milagros Cayas filed a categorically placed petitioner's liability for all duty of the courts is the application of the law
complaint for a sum of money and damages against damages arising out of death or bodily injury according to its express terms, interpretation being
PCSI in the Court of First Instance of Cavite. sustained by one person as a result of any one called for only when such literal application is
- In view of Milagros Cayas' failure to prosecute the accident at P12,000.00. impossible.
case, the court motu propio ordered its dismissal - Said amount complied with the minimum fixed by - We observe that although Milagros Cayas was able
without prejudice. the law then prevailing, Section 377 of Presidential to prove a total loss of only P44,000.00, petitioner
- Alleging that she had not received a copy of the Decree No. 612, which provided that the liability of was made liable for the amount of P50,000.00, the
answer to the complaint, and that "out of land transportation vehicle operators for bodily maximum liability per accident stipulated in the
sportsmanship", she did not file a motion to hold injuries sustained by a passenger arising out of the policy. This is patent error. An insurance indemnity,
PCSI in default, Milagros Cayas moved for the use of their vehicles shall not be less than P12,000. being merely an assistance or restitution insofar as
reconsideration of the dismissal order. Said motion - In other words, under the law, the minimum can be fairly ascertained, cannot be availed of by any
for reconsideration was acted upon favorably by the liability is P12,000 per passenger. Petitioner's liability accident victim or claimant as an instrument of
court. under the insurance contract not being less than enrichment by reason of an accident.
- About two months later, Milagros Cayas filed a P12,000.00, and therefore not contrary to law, Disposition Petition granted. The decision of the
motion to declare PCSI in default for its failure to file morals, good customs, public order or public policy, Court of Appeals is modified in that petitioner shall
an answer. said stipulation must be upheld as effective, valid pay Milagros Cayas the amount of Twelve Thousand
- The motion was granted and plaintiff was allowed and binding as between the parties. Pesos (P12,000. 00) plus legal interest from the
to adduce evidence ex-parte. - In like manner, we rule as valid and binding upon promulgation of the decision of the lower court until
- On July 13, 1982, the court rendered judgment by private respondent the condition requiring her to it is fully paid and attorney's fees in the amount of
default ordering PCSI to pay Milagros Cayas P50,000 secure the written permission of petitioner before P5,000.00.
as compensation for the injured passengers, P5,000 effecting any payment in settlement of any claim
as moral damages and P5,000 as attorney's fees. against her. POLTAN v. BPI & JOHN DOE
- Said decision was set aside after the PCSI filed a - There is nothing unreasonable, arbitrary or G.R. No. 164307
motion therefor. Trial of the case ensued. objectionable in this stipulation as would warrant its CHICO-NAZARIO; March 5, 2007
- In due course, the court promulgated a decision nullification. The same was obviously designed to
ordering defendant Perla Compania de Seguros, Inc. safeguard the insurer's interest against collusion NATURE
to pay plaintiff Milagros Cayas the sum of P50,000.00 between the insured and the claimants. Petition for review of CA decision
under its maximum liability as provided for in the - It being specifically required that petitioner's
insurance policy; and the sum of P5,000.00 as written consent be first secured before any payment FACTS
reasonable attorney's fee in settlement of any claim could be made, private - Petitioners POLTAN obtained a loan evidenced by a
- PCSI appealed to the Court of Appeals, which respondent is precluded from seeking reimbursement promissory note from the MANTRADE Dev’t Corp.
affirmed in toto the lower court's decision. of the payments made to the three other passangers This was secured by a chattel mortgage over a 1-unit
- Its motion for reconsideration having been denied in view of her failure to comply with the condition Nissan Sentra vehicle.
by said appellate court, PCSI filed this petition contained in the insurance policy. - With notice to petitioners, MANTRADE assigned to
- Clearly, the fundamental principle that contracts BPI, by way of a Deed of Assignment, all its rights,
ISSUE are respected as the law between the contracting title and interest to the promissory note and chattel
WON, as maintained by petitioner, its liability is parties finds application in the present case. mortgage.
limited only to the payment made by private - It was error on the part of the trial and appellate - Petitioners defaulted and so BPI demanded the
respondent to Perea and only up to the amount of courts to have disregarded the stipulations of the whole balance of P286.5T including accrued interest,
P12,000.00 parties and to have substituted their own or to return to BPI the possession of the motor
interpretation of the insurance policy. vehicle for foreclosure. It is specifically provided in
HELD - In Phil. American General Insurance Co., Inc vs. the promissory note and chattel mortgage that
YES Mutuc, we ruled that contracts which are the private failure to pay any installment when due shall make
- The insurance policy involved explicitly limits laws of the contracting parties should be fulfilled subsequent installments and the entire balance of
petitioner's liability to P12,000.00 per person and to according to the literal sense of their stipulations, if the obligation due and payable.
P50,000.00 per accident. their terms are clear and leave no room for doubt as - After they refused to do so, BPI then filed
- In Stokes vs. Malayan Insurance Co., Inc., the to the intention of the contracting parties, for complaint.
Court held that the terms of the contract constitute contracts are obligatory, no matter what form they
INSURANCE Page 25

- Petitioners claimed that BPI required them to benefit of BPI, this will result in partial or full covering any loss other than a willful and fraudulent
obtain a motor vehicle insurance policy from FGU satisfaction of the obligation only if the insurer pays act of the insured. 7 This is pursuant to the very
Insurance Corporation (FGU Insurance). This is a the mortgagee, BPI, or if the insurance proceeds purpose of an "all risks" insurance to give protection
sister company of BPI. They had been paying the were paid to BPI. In this case, upon the loss of the to the insured in those cases where difficulties of
monthly installments on the vehicle until it figured in vehicle due to total wreck, the petitioners filed a logical explanation or some mystery surround the
an accident where it became a total wreck. Under the claim under the insurance policy, collected and loss or damage to property.
terms of the insurance policy from FGU Insurance, received the proceeds thereof, but did not settle their - Generally, the burden of proof is upon the insured
the vehicle had to be replaced or its value paid to obligation with BPI which remained outstanding to show that a loss arose from a covered peril, but
them. Due to the failure and refusal of FGU despite the loss of the vehicle. under an "all risks" policy the burden is not on the
Insurance to replace the vehicle or pay its value, Disposition CA decision AFFIRMED with the insured to prove the precise cause of loss or damage
they stopped payment of the monthly installments. modification that the interest rate be reduced to 12% for which it seeks compensation. The insured under
- RTC ordered POLTANS to pay BPI the said amount. per annum from 24 May 1994 until fully paid, and an "all risks insurance policy" has the initial burden
- CA reversed and remanded case to RTC for trial on the award of attorney’s fees be reduced to P50T. of proving that the cargo was in good condition when
the merits. the policy attached and that the cargo was damaged
- RTC again ruled in favor of BPI. CA affirmed. FILIPINO MERCHANTS INS. v. CA (CHOA TIEK when unloaded from the vessel; thereafter, the
SENG) burden then shifts to the insurer to show the
ISSUES 179 SCRA 638 exception to the coverage. As we held in Paris-Manila
1. WON contracts presented in evidence by BPI were REGALADO; November 28, 1989 Perfumery Co. vs. Phoenix Assurance Co., Ltd. the
unjust and unacceptable contracts of adhesion basic rule is that the insurance company has the
2. WON the terms and conditions of the NATURE burden of proving that the loss is caused by the risk
comprehensive car insurance policy issued by FGU Review of the decision of the CA excepted and for want of such proof, the company is
should be deemed as having automatically operated liable. In the present case, there being no showing
in favor of BPI as the assured mortgagee, and if so, FACTS that the loss was caused by any of the excepted
it should be deemed as resulting in the - Plaintiff insured said shipment with defendant perils, the insurer is liable under the policy.
extinguishment of petitioner’s obligation insurance company under said cargo for the goods 2. YES
described as 600 metric tons of fishmeal in new - Section 13 of the Insurance Code defines insurable
HELD gunny bags of 90 kilos each from Bangkok, Thailand interest in property as every interest in property,
1. NO to Manila against all risks under warehouse to whether real or personal, or any relation thereto, or
Ratio A contract of adhesion is one in which one of warehouse terms. liability in respect thereof, of such nature that a
the parties imposes a ready-made form of contract, - Some of the goods arrived in bad condition. Plaintiff contemplated peril might directly damnify the
which the other party may accept or reject, but made a claim against Filipino Merchants Insurance insured. In principle, anyone has an insurable
which the latter cannot modify. It is just as binding Company. The latter refused to pay. Plaintiff brought interest in property who derives a benefit from its
as ordinary contracts. an action against them. The defendant insurance existence or would suffer loss from its destruction
Reasoning company presented a third party complaint against whether he has or has not any title in, or lien upon
- Petitioners failed to show that they were under the vessel and the arrastre contractor. or possession of the property y. 16 Insurable interest
duress or forced to sign the loan documents. The - Judgment was rendered against the insurance in property may consist in (a) an existing interest;
natural presumption is that one does not sign a company. On the third party complaint, the third (b) an inchoate interest founded on an existing
document without first informing himself of its party defendants were ordered to pay the third party interest; or (c) an expectancy, coupled with an
contents and consequences. plaintiffs. The CA affirmed, but modified the same existing interest in that out of which the expectancy
- Contracts of adhesion are not entirely prohibited with regard to the adjudication of the third-party arises.
even as the courts remain careful in scrutinizing the complaint - Respondent’s interest over the goods is based on
factual circumstances underlying each case to the perfected contract of sale. The perfected contract
determine the respective claims of contending ISSUES of sale between him and the shipper of the goods
parties on their efficacy. 1. WON some fortuity, casualty or accidental cause is operates to vest in him an equitable title even before
2. NO needed to be proved despite the “all risks” policy (as delivery or before be performed the conditions of the
Reasoning asserted by the insurance company) sale.
- Petitioners failed to show any provision in the 2. WON the respondent has an insurable interest - Further, Article 1523 of the Civil Code provides that
insurance policy or mortgage contract providing that where, in pursuance of a contract of sale, the seller
the loss of the mortgaged vehicle extinguishes their HELD is authorized or required to send the goods to the
principal obligation to BPI. 1. NO buyer, delivery of the goods to a carrier, whether
- While it is true that the proceeds from the - The very nature of the term "all risks" must be named by the buyer or not, for, the purpose of
insurance policy over the mortgaged chattel is for the given a broad and comprehensive meaning as transmission to the buyer is deemed to be a delivery
INSURANCE Page 26

of the goods to the buyer, the exceptions to said rule - Petitioner is a customer and dealer of the products Article 1504 (1) of the Civil Code, to the general rule
not obtaining in the present case. The Court has of IMC and LSPI. On February 25, 1991, the Gaisano that if the thing is lost by a fortuitous event, the risk
heretofore ruled that the delivery of the goods on Superstore Complex in Cagayan de Oro City, owned is borne by the owner of the thing at the time the
board the carrying vessels partake of the nature of by petitioner, was consumed by fire. Included in the loss under the principle of res perit domino; that
actual delivery since, from that time, the foreign items lost or destroyed in the fire were stocks of petitioner’s obligation to IMC and LSPI is not the
buyers assumed the risks of loss of the goods and ready-made clothing materials sold and delivered by delivery of the lost goods but the payment of its
paid the insurance premium covering them IMC and LSPI. On February 4, 1992, respondent unpaid account and as such the obligation to pay is
- Moreover, the issue of lack of insurable interest was filed a complaint for damages against petitioner. It not extinguished, even if the fire is considered a
not raised in petitioner’s answer. alleges that IMC and LSPI filed with respondent their fortuitous event; that by subrogation, the insurer has
Disposition Petition denied claims under their respective fire insurance policies the right to go against petitioner; that, being a fire
with book debt endorsements; that as of February insurance with book debt endorsements, what was
GAISANO CAGAYAN v. INSURANCE Co. OF 25, 1991, the unpaid accounts of petitioner on the insured was the vendor’s interest as a creditor.
NORTH AMERICA sale and delivery of ready-made clothing materials Petitioner filed a motion for reconsideration but it
490 SCRA 296 with IMC was P2,119,205.00 while with LSPI it was was denied by the CA in its Resolution dated April
Austria-Martinez; June 8, 2006 P535,613.00; that respondent paid the claims of IMC 11, 2001.
and LSPI and, by virtue thereof, respondent was
NATURE subrogated to their rights against petitioner; that ISSUES
Petition for review on certiorari of the Decision of the respondent made several demands for payment upon 1. WON the CA erred in construing a fire insurance
Court of Appeals petitioner but these went unheeded. In its Answer policy on book debts as one covering the unpaid
with Counter Claim dated July 4, 1995, petitioner accounts of IMC and LSPI since such insurance
FACTS contends that it could not be held liable because the applies to loss of the ready-made clothing materials
- Intercapitol Marketing Corporation (IMC) is the property covered by the insurance policies were sold and delivered to petitioner.
maker of Wrangler Blue Jeans. Levi Strauss (Phils.) destroyed due to fortuities event or force majeure; 2. WON IMC bears the risk of loss because it
Inc. (LSPI) is the local distributor of products bearing that respondent’s right of subrogation has no basis expressly reserved ownership of the goods by
trademarks owned by Levi Strauss & Co.. IMC and inasmuch as there was no breach of contract stipulating in the sales invoices that “[i]t is further
LSPI separately obtained from respondent fire committed by it since the loss was due to fire which agreed that merely for purpose of securing the
insurance policies with book debt endorsements. it could not prevent or foresee; that IMC and LSPI payment of the purchase price the above described
The insurance policies provide for coverage on “book never communicated to it that they insured their merchandise remains the property of the vendor
debts in connection with ready-made clothing properties; that it never consented to paying the until the purchase price thereof is fully paid.”
materials which have been sold or delivered to claim of the insured. 3. WON the petitioner liable for the unpaid accounts
various customers and dealers of the Insured - At the pre-trial conference the parties failed to
anywhere in the Philippines.” The policies defined arrive at an amicable settlement. Thus, trial on the HELD
book debts as the “unpaid account still appearing in merits ensued. On August 31, 1998, the RTC 1. NO
the Book of Account of the Insured 45 days after the rendered its decision dismissing respondent’s - It is well-settled that when the words of a contract
time of the loss covered under this Policy.” The complaint. It held that the fire was purely accidental; are plain and readily understood, there is no room
policies also provide for the following conditions: that the cause of the fire was not attributable to the for construction. In this case, the questioned
1. Warranted that the Company shall not negligence of the petitioner; that it has not been insurance policies provide coverage for “book debts
be liable for any unpaid account in established that petitioner is the debtor of IMC and in connection with ready-made clothing materials
respect of the merchandise sold and LSPI; that since the sales invoices state that “it is which have been sold or delivered to various
delivered by the Insured which are further agreed that merely for purpose of securing customers and dealers of the Insured anywhere in
outstanding at the date of loss for a the payment of purchase price, the above-described the Philippines.”; and defined book debts as the
period in excess of six (6) months from merchandise remains the property of the vendor “unpaid account still appearing in the Book of
the date of the covering invoice or until the purchase price is fully paid”, IMC and LSPI Account of the Insured 45 days after the time of the
actual delivery of the merchandise retained ownership of the delivered goods and must loss covered under this Policy.” Nowhere is it
whichever shall first occur. bear the loss. Dissatisfied, petitioner appealed to the provided in the questioned insurance policies that the
2. Warranted that the Insured shall submit to the CA. On October 11, 2000, the CA rendered its subject of the insurance is the goods sold and
Company within twelve (12) days after the close of decision setting aside the decision of the RTC. The delivered to the customers and dealers of the
every calendar month all amount shown in their CA held that the sales invoices are proofs of sale, insured.
books of accounts as unpaid and thus become being detailed statements of the nature, quantity and - Indeed, when the terms of the agreement are clear
receivable item from their customers and dealers. x cost of the thing sold; that loss of the goods in the and explicit that they do not justify an attempt to
xx fire must be borne by petitioner since the proviso read into it any alleged intention of the parties, the
contained in the sales invoices is an exception under terms are to be understood literally just as they
INSURANCE Page 27

appear on the face of the contract. Thus, what were with reference to the property that he would be liable - Thus, whether fire is a fortuitous event or petitioner
insured against were the accounts of IMC and LSPI to loss should it be injured or destroyed by the peril was negligent are matters immaterial to this case.
with petitioner which remained unpaid 45 days after against which it is insured. Anyone has an insurable What is relevant here is whether it has been
the loss through fire, and not the loss or destruction interest in property who derives a benefit from its established that petitioner has outstanding accounts
of the goods delivered. existence or would suffer loss from its destruction. with IMC and LSPI.
2. NO Indeed, a vendor or seller retains an insurable - With respect to IMC, the respondent has
- The present case clearly falls under paragraph (1), interest in the property sold so long as he has any adequately established its claim. Petitioner has an
Article 1504 of the Civil Code: interest therein, in other words, so long as he would outstanding account with IMC in the amount of
ART. 1504. Unless otherwise agreed, the goods suffer by its destruction, as where he has a vendor’s P2,119,205.00, check voucher evidencing payment
remain at the seller’s risk until the ownership lien. In this case, the insurable interest of IMC and to IMC, subrogation receipt executed by IMC in favor
therein is transferred to the buyer, but when the LSPI pertain to the unpaid accounts appearing in of respondent upon receipt of the insurance
ownership therein is transferred to the buyer the their Books of Account 45 days after the time of the proceeds. All these documents have been properly
goods are at the buyer’s risk whether actual loss covered by the policies. identified, presented and marked as exhibits in court.
delivery has been made or not, except that: 3. YES The subrogation receipt, by itself, is sufficient to
(1) Where delivery of the goods has been made to - Petitioner’s argument that it is not liable because establish not only the relationship of respondent as
the buyer or to a bailee for the buyer, in pursuance the fire is a fortuitous event under Article 1174 of the insurer and IMC as the insured, but also the amount
of the contract and the ownership in the goods Civil Code is misplaced. As held earlier, petitioner paid to settle the insurance claim. The right of
has been retained by the seller merely to bears the loss under Article 1504 (1) of the Civil subrogation accrues simply upon payment by the
secure performance by the buyer of his Code. insurance company of the insurance claim.
obligations under the contract, the goods are - Moreover, it must be stressed that the insurance in Respondent’s action against petitioner is squarely
at the buyer’s risk from the time of such this case is not for loss of goods by fire but for sanctioned by Article 2207 of the Civil Code which
delivery; (Emphasis supplied) petitioner’s accounts with IMC and LSPI that provides:
- Thus, when the seller retains ownership only to remained unpaid 45 days after the fire. Accordingly, Art. 2207. If the plaintiff’s property has been
insure that the buyer will pay its debt, the risk of loss petitioner’s obligation is for the payment of money. insured, and he has received indemnity from the
is borne by the buyer. Accordingly, petitioner bears Where the obligation consists in the payment of insurance company for the injury or loss arising
the risk of loss of the goods delivered. money, the failure of the debtor to make the out of the wrong or breach of contract complained
- IMC and LSPI did not lose complete interest over payment even by reason of a fortuitous event shall of, the insurance company shall be subrogated to
the goods. They have an insurable interest until full not relieve him of his liability. The rationale for this is the rights of the insured against the wrongdoer or
payment of the value of the delivered goods. Unlike that the rule that an obligor should be held exempt the person who has violated the contract. x x x
the civil law concept of res perit domino, where from liability when the loss occurs thru a fortuitous - Petitioner failed to refute respondent’s evidence.
ownership is the basis for consideration of who bears event only holds true when the obligation consists in - As to LSPI, respondent failed to present sufficient
the risk of loss, in property insurance, one’s the delivery of a determinate thing and there is no evidence to prove its cause of action. No evidentiary
interest is not determined by concept of title, stipulation holding him liable even in case of weight can be given to Exhibit “F Levi Strauss”, a
but whether insured has substantial economic fortuitous event. It does not apply when the letter dated April 23, 1991 from petitioner’s General
interest in the property. obligation is pecuniary in nature. Manager, Stephen S. Gaisano, Jr., since it is not an
- Section 13 of our Insurance Code defines insurable - Under Article 1263 of the Civil Code, “[i]n an admission of petitioner’s unpaid account with LSPI.
interest as “every interest in property, whether real obligation to deliver a generic thing, the loss or It only confirms the loss of Levi’s products in the
or personal, or any relation thereto, or liability in destruction of anything of the same kind does not amount of P535,613.00 in the fire that razed
respect thereof, of such nature that a contemplated extinguish the obligation.” If the obligation is generic petitioner’s building on February 25, 1991.
peril might directly damnify the insured.” in the sense that the object thereof is designated - Moreover, there is no proof of full settlement of the
Parenthetically, under Section 14 of the same Code, merely by its class or genus without any particular insurance claim of LSPI; no subrogation receipt was
an insurable interest in property may consist in: (a) designation or physical segregation from all others of offered in evidence. Thus, there is no evidence that
an existing interest; (b) an inchoate interest founded the same class, the loss or destruction of anything of respondent has been subrogated to any right which
on existing interest; or (c) an expectancy, coupled the same kind even without the debtor’s fault and LSPI may have against petitioner. Failure to
with an existing interest in that out of which the before he has incurred in delay will not have the substantiate the claim of subrogation is fatal to
expectancy arises. effect of extinguishing the obligation. This rule is petitioner’s case for recovery of the amount of
- Therefore, an insurable interest in property does based on the principle that the genus of a thing can P535,613.00.
not necessarily imply a property interest in, or a lien never perish. Genus nunquan perit. An obligation Disposition Petition is partly GRANTED. The
upon, or possession of, the subject matter of the to pay money is generic; therefore, it is not assailed Decision dated October 11, 2000 and
insurance, and neither the title nor a beneficial excused by fortuitous loss of any specific Resolution dated April 11, 2001 of the Court of
interest is requisite to the existence of such an property of the debtor. Appeals in CA-G.R. CV No. 61848 are AFFIRMED
interest, it is sufficient that the insured is so situated with the MODIFICATION that the order to pay the
INSURANCE Page 28

amount of P535,613.00 to respondent is DELETED WON petitioner Tai Tong has insurable interest in the Petition for review on certiorari
for lack of factual basis. said policy
FACTS
TAI TONG CHUACHE & CO v. INSURANCE HELD - Primitivo Perez has been insured with the BF
COMMISSION and TRAVELLERS MULTI- YES Lifeman Insurance Corporation (BF hereafter) since
INDEMNITY CORPORATION - First, respondent insurance commission based its 1980 for Php20,000.
158 SCRA 366 findings on mere inference. Respondent Insurance Sometime in 1987, Rodolfo Lalog (agent of BF)
GANCAYCO; February 29, 1988 Commission absolved respondent insurance company convinced him to apply for additional insurance
from liability on the basis of the certification issued coverage of Php50,000.
NATURE by the then CFI, that in a certain civil action against Perez accomplished the application form and passed
Petition for review on certiorari of the decision of the the Palomos, Arsenio Lopez Chua stands as the the required medical examination. He also paid
Insurance Commission complainant and not Tai Tong Chuache. From said Php2,075 premium) to Lalog.
evidence respondent commission inferred that the - On November 25, 1987, Perez died while riding a
FACTS credit extended by herein petitioner to the Palomos banca which capsized during a storm. During this
- Complainants Palomo acquired a parcel of land and secured by the insured property must have been time his application papers for the additional
a building located in Davao City. They assumed the paid. Such is a glaring error which this Court cannot insurance coverage was still with the Gumaca,
mortgage of the building in favor of SSS, which sanction. Quezon
building was insured with respondent SSS Accredited - Second, it has been held in a long line of cases that office of BF.
Group of Insurers for P25K. when the creditor is in possession of the document of - Without knowing that Perez died on November 25,
- On April 19, 1975, Azucena Palomo obtained a credit, he need not prove non-payment for it is 1987, BF approved Perez's application and issued the
P100K loan from Tai Tong Chuache Inc. (TTCC) and presumed. The validity of the insurance policy taken corresponding policy for the Php50,000 on December
executed a mortgage over the land and the building b petitioner was not assailed by private respondent. 2, 1987.
in favor of Tai Tong Chuache & Co. as security of Moreover, petitioner's claim that the loan extended to - Virginia Perez (wife of the deceased) claimed the
payment .On April 25, 1975, Arsenio Chua, the Palomos has not yet been paid was corroborated benefits under the insurance policies of the
representative of TTCC insured the latter's interest by Azucena Palomo who testified that they are still deceased, but she was only able to receive
with Travellers Multi-Indemnity Corporation indebted to herein petitioner. So at the time of the Php40,000 under the first insurance policy.
(Travellers) for P100K (P70K for bldg and P30K for fire, petitioner as mortgagee still had insurable BF refused to pay the proceeds amounting to
the contents thereof) interest therein. Php150,000 under the additional policy coverage of
- On June 11, 1975, Pedro Palomo secured a Fire - And third, petitioner's declaration that Arsenio Php50,000 because they maintain that such policy
Insurance Policy, covering the building for P50K with Lopez Chua acts as the managing partner of the had not been perfected.
respondent Zenith Insurance Corporation (ZIC). partnership was corroborated by respondent - On September 21, 1990, BF filed a complaint
Another Fire Insurance Policy was later procured insurance company. Thus Chua as the managing against Mrs. Perez seeking recission and declaration
from respondent Philippine British Assurance partner of the partnership may execute all acts of of nullity of the insurance contract in question. Mrs.
Company (PBAC), covering the same building for administration including the right to sue debtors of Perez filed a conterclaim
P50K and contents thereof for P70K. On July 31, the partnership in case of their failure to pay their for the collection of Php150,000 plus damages.
1975, the building and the contents were totally obligations when it became due and demandable. Or
razed by fire. at the least, Chua being a partner of petitioner Tai ISSUE
- Based on the computation of the loss, including the Tong Chuache & Company is an agent of the WON there was a consummated contract of
Travellers, respondents, ZIC, PBAC, and SSS paid partnership. Being an agent, it is understood that he insurance between Perez and BF
their corresponding shares of the loss. Complainants acted for and in behalf of the firm.
were paid the following: P41,546.79 by PBAC, Disposition Appealed decision SET ASIDE and HELD
P11,877.14 by ZIC, and P5,936.57 by SSS. Demand ANOTHER judgment is rendered order private NO
was made from respondent Travellers for its share in respondent Travellers to pay petitioner the face value - An essential requisite of a valid contract is consent.
the loss but was refused. Hence, complainants of Fire Insurance Policy in the amount of P100K. Consent must be manifested by the meeting of the
demanded from the other 3 respondents the balance Costs against said private respondent. offer and the acceptance upon the thing and the
of each share in the loss based on the computation cause which are to constitute the contract.
excluding Travellers Multi-Indemnity in the amount of - The offer must be certain and the acceptance
P30,894.31 (P5,732.79-ZIC: P22,294.62, PBAC: and PEREZ v. CA (BF LIFEMAN INSURANCE CORP.) absolute. When Perez filed the application, it was
P2,866.90, SSS) but was refused, hence, this action. 323 SCRA 613 subject to the acceptance of BF. The perfection was
YNARES-SANTIAGO; January 28, 2000 also further conditioned upon (1) the issuance of the
ISSUE policy,
NATURE
INSURANCE Page 29

(2) the payment of the premium, and (3) the remained at home. On January 15, he called a
delivery to and acceptance by the applicant in good physician who found that he was suffering from HELD
health. acute nephritis and uremia and on January 19, 1933, YES
- The delivery and acceptance by the applicant was a he died. - There is one line of cases which holds that the
suspensive condition which was not fulfilled - On January 18, 1933, the agent, in accordance with stipulation contained in paragraph 3 is in the nature
inasmuch as the applicant was already dead at the his agreement with the insured, delivered the policy of a condition precedent, that is to say, that there
time the policy was issued. The non-fulfillment of the to Felicidad Estrada upon her payment of the balance can be no valid delivery to the insured unless he is in
condition resulted of the first year's annual premium. The agent asked good health at the time; that this condition
in the non-perfection of the contract. Felicidad Estrada if her nephew was in good health precedent goes to the very essence of the contract
- An application for insurance is merely an offer and she replied that she believed so because she had and cannot be waived by the agent making delivery
which requires the overt act of the insurer for it to no information that he was sick and he thereupon of the policy
ripen to a contract. Delay in acting on the application delivered to her the policy. - On the other hand, a number of American decisions
does not constitute acceptance even though the - On January 20, 1933, the agent learned of the hold that an agent to whom a life insurance policy
insured has forwarded death of Arturo Sindayen and called on Felicidad similar to the one here involved was sent with
his first premium with his application. Delay, in this Estrada and asked her to return the policy. But he instructions to deliver it to the insured has authority
case, does not constitute gross negligence because did not return or offer to return the premium paid. to bind the company by making such delivery,
the application was granted within the normal Felicidad Estrada on his aforesaid statement gave although the insured was not in good health at the
processing time. him the policy. time of delivery, on the theory that the delivery of
Disposition Decision of CA affirmed in so far as it - On February 4, 1933 Insular Life obtained from the the policy being the final act to the consummation of
declared the insurance policy for Php50,000 issued beneficiary, Sindayen’s wife, her signature to a legal the contract, the condition as to the insurer's good
by BF null and void (no recission because it document entitled "ACCORD, SATISFACTION AND health was waived by the company.
presupposes a valid contract) RELEASE" whereby in consideration of the sum of - we are inclined to the view that it is more
P40.06 paid to her by a check of the company, she consonant with the well known practice of life
VDA. DE SINDAYEN v. INSULAR "assigns, releases and forever discharges said Isular insurance companies and the evidence in the present
62 Phil 51 Life Assurance Co., Ltd., its successors and assigns, case to rest our decision on the proposition that
BUTTE; September 4, 1935 of all claims, obligation in or indebtedness. The said Mendoza was authorized by the company to make
check for P40.06 was never cashed but returned to the delivery of the policy when he received the
FACTS the company and appears in the record of this case payment of the first premium and he was satisfied
- Arturo Sindayen, up to the time of his death on as Exhibit D. Thereupon this action was brought to that the insured was in good health. As was well said
January 19, 1933, was employed as a linotype enforce payment of the policy. in the case of MeLaurin vs. Mutual Life Insurance Co.
operator in the Bureau of Printing at Manila and had By the terms of the policy, an annual premium of “It is plain, therefore, that upon the facts it is not
been such for eleven years prior thereto. While there P40.06 is due on the first day of December of each necessarily a case of waiver or of estoppel, but a
he made a written application on December 26, year, the first premium already paid by the insured case where the local agents, in the exercise of the
1932, to the defendant Insular Life Assurance Co., covering the period from December 1, 1932. It is to powers lodged in them, accepted the premium and
Ltd., through its agent, Cristobal Mendoza, for a December 1, 1933. It is to be noted that the policy delivered the policy. That act binds their principal,
policy of insurance on his life in the sum of P1,000 was not issued and the company assumed no actual the defendant.”
and he paid to the agent P15 cash as part of the first risk prior to January 11, 1933.The application which - Mendoza was duly licensed by the Insurance
premium. It was agreed with the agent that the the insured signed in Camiling, Tarlac, on December Commissioner to act as the agent of the defendant
policy, when and if issued, should be delivered to his 26, 1932, contained among others the following insurance company. The well known custom of the
aunt. Felicidad Estrada, with whom Sindayen left the provisions: insurance business and the evidence in this case
sum of P26.06 to complete the payment of the first “3 That the said policy shall not take effect until the prove that Mendoza was not regarded by the
annual premium of P40.06. first premium has been paid and the policy has been company as a mere conduit or automaton for the
- On January 1, 1933, Sindayen, who was then delivered to and accepted by me, while I am in good performance of the physical act of placing the policy
twenty-nine years of age, was examined by the health.” in the hands of the insured
company's doctor who made a favorable report, to -Main defense of the company in this case, namely, - Granted that Mendoza's decision that the condition
the company. On January 11, 1933, The company that the said policy never took effect because of had been met by the insured and that it was proper
accepted the risk and issued policy No. 47710 dated paragraph 3 of the application above quoted, for at to make a delivery of the policy to him is just as
back to December 1, 1932, and mailed the same to the time of its delivery by the agent as aforesaid the binding on the company as if the decision had been
its agent, Cristobal Mendoza, in Camiling, Tarlac, for insured was not in good health made by its board of directors. Granted that
delivery to the insured. -On January 11, 1933, Mendoza made a mistake of judgement because he
Sindayen was at work in the Bureau of Printing. On ISSUE acted on insufficient evidence as to the state of
January 12, he complained of a severe headache and WON the insurance policy is valid health of the insured. But it is not charged that the
INSURANCE Page 30

mistake was induced by any misconduct or omission could have been no waiver. In view of the facts local manager, Mr. White, testified to having received
of duty of the insured. established and admitted, there is no doubt, as to the cablegram accepting the application of Mr. Herrer
- It is the interest not only the applicant but of all the existence of the fraud. -Estrada, as a from the home office on November 26, 1917. He said
insurance companies as well that there should be representative of the insured was not only bound to that on the same day he signed a letter notifying Mr.
some act which gives the applicant the definite give a truthful information on the state of health of Herrer of this acceptance. The witness further said
assurance that the contract has been consummated. the insured, but it was her duty to find out it his true that letters, after being signed, were sent to the
This sense of security and of peace of mind that state of health in order to give true and correct chief clerk and placed on the mailing desk for
one's defendants are provided for without risk either information. When she gave Mendoza an incorrect transmission. Mr. Tuason, who was the chief clerk on
of loss or of litigation is the bedrock of life insurance. information tending to create the impression that the November 26, 1917, was not called as a witness.
When the policy is issued and delivered, in the insured was well when in fact he was seriously ill, - For the defense, attorney Manuel Torres testified to
absence of fraud or other grounds for rescission, it is there is no doubt that she committed fraud and having prepared the will of Joaquin Ma. Herrer. That
plainly not within the intention of the parties that imparted a deceitful information to the defendant on this occasion, Mr. Herrer mentioned his
there should be any questions held in abeyance or agent application for a life annuity, and that he said that
reserved for future determination that leave the very the only document relating to the transaction in his
existence of the contract in suspense and doubt. ENRIQUEZ v. SUN LIFE OF CANADA possession was the provisional receipt. Rafael
- It is therefore in the public interest, for the public is 41 PHIL 269 Enriquez, the administrator of the estate, testified
profoundly and generally interested in life insurance, MALCOLM; November 29, 1920 that he had gone through the effects of the deceased
as well as in the interest of the insurance companies and had found no letter of notification from the
themselves by giving certainly and security to their NATURE insurance company to Mr. Herrer.
policies, that we are constrained to hold, as we, do, Appeal from judgment of trial court denying
that the delivery of the policy to the insured by an plaintiff’s (administrator of the estate of the late ISSUE
agent of the company who is authorized to make Joaquin Ma. Herrer) action to recover from the WON there exists a contract for life annuity between
delivery or without delivery is the final act which defendant life insurance company the sum of pesos Herrer and defendant
binds the company (and the insured as well) in the 6,000 paid by the deceased for a life annuity.
absence of fraud or other legal ground for rescission HELD
- The company therefore having decided that all the FACTS NO
conditions precedent to the taking effect of the policy - On September 24, 1917, Joaquin Herrer made Ratio The law applicable to the case is found to be
had been complied with and having accepted the application to the Sun Life Assurance Company of the second paragraph of article 1262 of the Civil
premium and delivered the policy thereafter to the Canada through its office in Manila for a life annuity. Code providing that an acceptance made by letter
insured, the company is now estopped to assert that Two days later he paid the sum of P6,000 to the shall not bind the person making the offer except
it never intended that the policy should take effect. manager of the company's Manila office and was from the time it came to his knowledge.
given a receipt. Reasoning
SEPARATE OPINION - The application was immediately forwarded to the - Until quite recently, all of the provisions concerning
head office of the company at Montreal, Canada. On life insurance in the Philippines were found in the
IMPERIAL [dissent] November 26, 1917, the head office gave notice of Code of Commerce and the Civil Code. After July 1,
- "A local agent of an insurance company, whose only acceptance by cable to Manila. (Whether on the 1915, there was, however, in force the Insurance
power is to solicit applications for insurance, and same day the cable was received, notice was sent by Act. No. 2427. Chapter IV of this Act concerns life
forward them to the company for approval, when, if the Manila office of Herrera that the application had and health insurance. The Act expressly repealed
approved to the insured, has no power to waive any been accepted, is a disputed point, which will be Title VIII of Book II and Section III of Title III of
of the provision of the policy so delivered." discussed later.) On December 4, 1917, the policy Book III of the code of Commerce. The law of
- It is clear, therefore, that the delivery of the policy was issued at Montreal. On December 18, 1917, insurance is consequently now found in the
by Mendoza does not bind the defendant, nor is the attorney Aurelio A. Torres wrote to the Manila office Insurance Act and the Civil Code.
defendant estopped from alleging its defense, for the of the company stating that Herrer desired to - While, as just noticed, the Insurance Act deals with
simple reason that Mendoza was not an agent with withdraw his application. The following day the local life insurance, it is silent as to the methods to be
authority to issue policies or to accept risks in the office replied to Mr. Torres, stating that the policy followed in order that there may be a contract of
name of his principle. had been issued, and called attention to the insurance. On the other hand, the Civil Code, in
-There is another ground upon which the majority notification of November 26, 1917. This letter was article 1802, not only describes a contact of life
opinion is based, namely, that the defendant waived received by Mr. Torres on the morning of December annuity markedly similar to the one we are
the defense it now invokes, by reason of the delivery 21, 1917. Mr. Herrer died on December 20, 1917. considering, but in two other articles, gives strong
of the policy by its invokes, by reason of the delivery - The chief clerk of the Manila office of Sun Life clues as to the proper disposition of the case. For
of the policy by its agent. It is admitted that if the testified that he prepared the letter and handed it to instance, article 16 of the Civil Code provides that
delivery of the policy was due to fraud, legally there the local manager, Mr. E. E. White, for signature. The "In matters which are governed by special laws, any
INSURANCE Page 31

deficiency of the latter shall be supplied by the center island in the road and collided with their car. Ratio Act No. 2427: an insurance policy would be
provisions of this Code." On the supposition, Private respondent Maharlika was eventually valid and binding notwithstanding the non-payment
therefore, which is incontestable, that the special law impleaded as a defendant in this case, with an of the premium if there was a clear agreement to
on the subject of insurance is deficient in enunciating allegation that the taxicab involved was insured grant to the insured credit extension. Such
the principles governing acceptance, the subject- against third party liability for P20,000.00 with agreement may be express or implied.
matter of the Civil code, if there be any, would be private respondent at the time of the accident Reasoning
controlling. In the Civil Code is found article 1262 - Maharlika claimed there was no cause of action - Petitioners maintain that in spite of their late
providing that "Consent is shown by the concurrence against it because at the time of the accident, the payment, the policy is binding because there was an
of offer and acceptance with respect to the thing and alleged insurance policy was not in force due to the implied agreement to grant a credit extension so as
the consideration which are to constitute the non-payment of the premium thereon. Also, even if to make the policy effective. To them, the
contract. An acceptance made by letter shall not bind the cab had been insured, the complaint would be subsequent acceptance of the premium and delivery
the person making the offer except from the time it premature since the policy provides that the insurer of the policy estops the respondent company from
came to his knowledge. would be liable only when the insured becomes asserting that the policy is ineffective.
- According to the provisional receipt, three things legally liable. The court however sees no proof of any such implied
had to be accomplished by the insurance company - Trial court ruled in favor of the plaintiff, holding the agreement. The purported nexus between the
before there was a contract: (1) There had to be a defendants liable for repair of the car, medical delivery of the policy and the grant of credit
medical examination of the applicant; (2) there had expenses, etc. BUT Maharlike was exonerated on the extension is too tenuous to support the conclusion
to be approval of the application by the head office of gnd that the policy was not in force. for which petitioners contend.
the company; and (3) this approval had in some way - Petitioners elevated this case to this court, faulting  Parenthetically mention: in the present law,
to be communicated by the company to the the respondent judge for considering the defense of Section 77 of the Insurance Code of 1978 has
applicant. The further admitted facts are that the late payment of premium when “the same was deleted the clause "unless there is clear agreement
head office in Montreal did accept the application, did waived at the pre-trial”, hence the evidence of late to grant the insured credit extension of the premium
cable the Manila office to that effect, did actually payment should be disregarded supposedly because due" which was then involved in this controversy.
issue the policy and did, through its agent in Manila, the private respondent had admitted that such fact Disposition Fnding no reversible error, the judgment
actually write the letter of notification and place it in was not in issue. appealed from is hereby AFFIRMED.
the usual channels for transmission to the addressee. - (More pertinent to this class: ) petitioners assert
- The contract for a life annuity in the case at bar that the private respondent had agreed to grant the
was not perfected because it has not been proved then prospective insured a credit extension of the
satisfactorily that the acceptance of the application premium due.
ever came to the knowledge of the applicant. - This controversy arose under the old insurance law,
Disposition Judgment is reversed, and the plaintiff Act No. 2427. TIBAY v. CA (FORTUNE LIFE & GENERAL
shall have and recover from the defendant the sum - The accident occurred in 1973. The complaint was INSURANCE)
of P6,000 with legal interest from November 20, filed on July 20, 1974.  both before the 257 SCRA 126
1918, until paid, without special finding as to costs in effectivity of Presidential Decree no. 612, the BELLOSILLO; May 24, 1996
either instance. subsequent insurance law which repealed its
predecessor FACTS
- The former insurance law, which applies to the case - On 22 January 1987 Fortune Life and General
here, provided: An insurer is entitled to the payment Insurance Co., Inc. (FORTUNE) issued Fire Insurance
of premium as soon as the thing insured is exposed Policy No. 136171 in favor of Violeta R. Tibay and/or
VELASCO and ACOSTA v. APOSTOL and to the peril insured against, unless there is clear Nicolas Roraldo on their two-storey residential
MAHARLIKA INSURANCE CO., INC. agreement to grant the insured credit extension of building located at 5855 Zobel Street, Makati City,
173 SCRA 228 the premium due. No policy issued by an insurance together with all their personal effects therein. The
REGALADO.; May 9, 1989 company is valid and binding unless and until the insurance was for P600,000 covering the period from
premium thereof has been paid. 23 January 1987 to 23 January 1988. On 23 January
NATURE 1987, of the total premium of P2,983.50, Violeta
Petition for review on certiorari ISSUE Tibay only paid P600 thus leaving a considerable
WON the insurance policy would be valid and binding balance unpaid.
FACTS notwithstanding the non-payment of the premium - On 8 March 1987 the insured building was
- Petitioners were plaintiffs in a civil case of which completely destroyed by fire. Two days later, Violeta
public respondent Hon. Apostol was the judge. HELD Tibay paid the balance of the premium. On the same
- The case was an offshoot of an incident: plaintiffs NO day, she filed with FORTUNE a claim on the fire
were riding in their car, when a taxicab crossed a insurance policy. Her claim was accordingly referred
INSURANCE Page 32

to its adjuster, Goodwill Adjustment Services, Inc. - Indeed, and far more importantly, the cardinal
(GASI), which immediately wrote Violeta requesting polestar in the construction of an insurance contract NATURE
her to furnish it with the necessary documents for is the intention of the parties as expressed in the Appeal from decision of the CA
the investigation and processing of her claim. policy. Courts have no other function but to enforce
Petitioner forthwith complied. On 28 March 1987 she the same. The rule that contracts of insurance will be FACTS
signed a nonwaiver agreement with GASI to the construed in favor of the insured and most strongly - American Home Assurance Co. (AHAC),
effect that any action taken by the companies shall against the insurer should not be permitted to have represented by American International Underwriters
not be, or be claimed to be, an admission of liability. the effect of making a plain agreement ambiguous (Phils.), Inc., issued in favor of petitioner Makati
- FORTUNE denied the claim of Violeta for violation of and then construe it in favor of the insured. Verily, it Tuscany Condominium Corporation an insurance
Policy Condition No. 2♪ and of Sec. 77 of the is elemental law that the payment of premium is policy on the latter's building and premises, for the
Insurance Code. Efforts to settle the case before the requisite to keep the policy of insurance in force. If period 1 March 1982 to1 March 1983. The premium
Insurance Commission proved futile. On 3 March the premium is not paid in the manner prescribed in was paid on installments all of which were accepted
1988 Violeta and the other petitioners sued FORTUNE the policy as intended by the parties the policy is by AHAC.
for damages in the amount of P600,000 representing ineffective. Partial payment even when accepted as a - A second policy was issued to renew the first one,
the total coverage of the fire insurance policy plus partial payment will not keep the policy alive even this time covering the period 1 March 1983 to 1
12% interest per annum, P100,000 moral damages, for such fractional part of the year as the part March 1984. This was also pain in installment basis.
and attorney's fees equivalent to 20% of the total payment bears to the whole payment. - A third policy was again issued for the period 1
claim. The trial court ruled for petitioners. CA Disposition Petition is DENIED. Decision of the CA March 1984 to 1 March 1985. For this, petitioner
reversed. is AFFIRMED. made two installment payments, both accepted by
AHAC. Thereafter, petitioner refused to pay the
ISSUE balance of the premium. AHAC filed an action to
WON a fire insurance policy is valid, binding and recover the unpaid balance of P314,103.05.
enforceable upon mere partial payment of premium SEPARATE OPINION - Petitioner explained that it discontinued the
payment of premiums because the policy did not
HELD VITUG [dissent] contain a credit clause in its favor and the receipts
NO - The law neither requires, nor measures the for the installment payments covering the policy for
Ratio Where the insurer and the insured expressly strength of the vinculum juris by, any specific 1984-85, as well as the two (2) previous policies,
stipulated that the policy is not in force until the amount of premium payment. It should thus be stated the following reservations:
premium has been fully paid the payment of partial enough that payment on the premium, partly or in 2. Acceptance of this payment shall not waive
premium by the assured in this particular instance full, is made by the insured which the insurer any of the company rights to deny liability on any
should not be considered the payment required by accepts. In fine, it is either that a juridical tie exists claim under the policy arising before such
the law and the stipulation of the parties. Rather, it (by such payment) or that it is not extant at all (by payments or after the expiration of the credit
must be taken in the concept of a deposit to be held an absence thereof). Once the juridical relation clause of the policy; and
in trust by the insurer until such time that the full comes into being, the full efficacy, not merely pro 3. Subject to no loss prior to premium payment.
amount has been tendered and duly receipted for. tanto, of the insurance contract naturally follows. If there be any loss such is not covered.
Reasoning Verily, not only is there an insurance perfected but - Petitioner further claimed that the policy was never
- As expressly agreed upon in the contract, full also a partially performed contract. In case of loss, binding and valid, and no risk attached to the policy.
payment must be made before the risk occurs for the recovery on the basis of the full contract value, less It then pleaded a counterclaim for P152k for the
policy to be considered effective and in force. Thus, the unpaid premium can accordingly be had; premiums already paid for 1984-85, and in its
no vinculum juris whereby the insurer bound itself to conversely, if no loss occurs, the insurer can demand answer with amended counterclaim, sought the
indemnify the assured according to law ever resulted the payment of the unpaid balance of the premium. refund of P924,206.10 representing the premium
from the fractional payment of premium. The The insured, on the one hand, cannot avoid the payments for 1982-85.
insurance contract itself expressly provided that the obligation of paying the balance of the premium - Trial court dismissed the complaint and the
policy would be effective only when the premium was while the insurer, upon the other hand, cannot treat counterclaim upon the following findings: (1)
paid in full. It would have been altogether different the contract as valid only for the purpose of payment of the premiums of the three policies were
were it not so stipulated. Ergo, petitioners had collecting premiums and as invalid for the purpose of made during the term of said policies, hence, it could
absolute freedom of choice whether or not to be indemnity. not be said, inspite of the reservations, that no risk
insured by FORTUNE under the terms of its policy attached under the policies; (2) as regards the
and they freely opted to adhere thereto. MAKATI TUSCANY v. CA ( AMERICAN HOME unpaid premiums, in view of the reservation in the
ASSURANCE CO.) receipts ordinarily issued by AHAC on premium

This policy including any renewal thereof and/or any endorsement 215 SCRA 462 payments the only plausible conclusion is that AHAC
thereon is not in force until the premium has been fully paid to and BELLOSILLO; November 6, 1992 has no right to demand their payment after the lapse
duly receipted by the Company in the manner provided herein.
INSURANCE Page 33

of the term of said policy on March 1, 1985. binding and effective notwithstanding the staggered Appellant argues that Chua was not its broker, but it
Therefore, Tuscany was justified in refusing to pay payment of the premiums. Acceptance of payments was found that Chua was authorized by South Sea to
the same. speaks loudly of the insurer's intention to honor the receive the premium on its behalf.
- CA modified the decision by ordering Tuscany to policies it issued to petitioner.
pay the balance of the premiums due on the third - Section 78 of the Insurance Code in effect allows
policy plus legal interest until fully paid, and waiver by the insurer of the condition of prepayment AREOLA v. CA (PRUDENTIAL GUARANTEE AND
affirming the denial of the counterclaim. by making an acknowledgment in the insurance ASSURANCE, INC.)
Petitioner’s Claims policy of receipt of premium as conclusive evidence 236 SCRA 643
Petitioner argues that where the premiums is not of payment so far as to make the policy binding ROMERO; September 22, 1994
actually paid in full, the policy would only be effective despite the fact that premium is actually unpaid.
if there is an acknowledgment in the policy of the Section 77 merely precludes the parties from NATURE CERTIORARI
receipt of premium pursuant to Sec. 78 of the stipulating that the policy is valid even if premiums
Insurance Code. The absence of an express are not paid, but does not expressly prohibit an FACTS
acknowledgment in the policies of such receipt of the agreement granting credit extension, and such an - June 29, 1985- 7 months after the issuance of
corresponding premium payments, and petitioner's agreement is not contrary to morals, good customs, Santos Areola's Personal Accident Insurance Policy
failure to pay said premiums on or before the public order or public policy. No. PA-20015 (covering a period of one year),
effective dates of said policies rendered them invalid. - At the very least, both parties should be deemed in Prudential unilaterally cancelled the same since
Petitioner thus concludes that there cannot be a estoppel to question the arrangement they have company records revealed that Areola failed to pay
perfected contract of insurance upon mere partial voluntarily accepted. his premiums.
payment of the premiums because under Sec. 77 of Disposition Judgment affirmed. Costs against o Under the terms of the statement of account
the Insurance Code, no contract of insurance is valid petitioner. issued by Prudential, Areola was supposed to
and binding unless the premium thereof has been pay the total amount of P1,609.65 which
paid, notwithstanding any agreement to the contrary. SOUTH SEA SURETY AND INSURANCE v. CA included the premium of P1,470.00,
(VALENZUELA HARDWOOD) documentary stamp of P110.25 and 2%
ISSUE 244 SCRA 744 premium tax of P29.40.
WON payment by installment of the premiums due VITUG; June 2, 1995 o The statement of account stated that it must
on an insurance policy invalidates the contract of not be considered a receipt as an official
insurance NATURE receipt will be issued upon payment of the
Petition for review on certiorari account. And if payment was made to a
HELD representative, the client must demand for a
Ratio Where the risk is entire and the contract is FACTS Provisional Receipt and if Official Receipts
indivisible, the insured is not entitled to a refund of - Hardwood entered into agreement with Seven Bros aren’t received within 7 days, Prudential
the premiums paid if the insurer was exposed to the Shipping, where latter undertook to load the former’s should be notified. If payment is made to their
risk insured for any period, however brief or logs on vessel. Hardwood insured the logs with office, clients should demand for an OR.
momentary. South Sea Surety which issued Marine Cargo - August 3, 1985- Prudential offered to reinstate
Reasoning Insurance Policy. The vessel sank Jan 25, 1984. same policy it had previously cancelled and even
- The obligation to pay premiums when due is - Hardwood filed claim with South Sea and Seven proposed to extend its lifetime to December 17,
ordinarily as indivisible obligation to pay the entire Bros. Trial Court favored Hardwood. CA decided 1985, upon a finding that the cancellation was
premium. Here, the parties herein agreed to make against South Sea, but absolved Seven Bros. South erroneous and that the premiums were paid in full by
the premiums payable in installments, and there is Sea filed this instant petition. Areola but were not remitted by Teofilo M. Malapit,
no pretense that the parties never envisioned to Prudential's branch manager.
make the insurance contract binding between them. ISSUES Petitioners’ Claims
And the insured never informed the insurer that it WON the insurance contract was already in effect - The fraudulent act of in misappropriating Areola’s
was terminating the policy because the terms were when the vessel sank premium payments is the proximate cause of the
unacceptable. cancellation of the insurance policy.
- There is nothing in Section 77 which suggests that HELD - Areola theorized that Malapit's act of signing and
the parties may not agree to allow payment of the YES even sending the notice of cancellation himself,
premiums in installment, or to consider the contract - It is already in effect because Hardwood has notwithstanding his personal knowledge of
as valid and binding upon payment of the first already paid the insurance premium. petitioner-insured's full payment of premiums,
premium. It delivered the check to Victorio Chua before the further reinforces the allegation of bad faith.
- The records clearly show that petitioner and private vessel sank, but Victorio Chua was only to deliver the - Such fraudulent act committed by Malapit is
respondent intended subject insurance policies to be check to South Sea five days after the vessel sank. attributable to Prudential.
INSURANCE Page 34

- Malapit's actuations are therefore not separate and As for any obligation wherein the agent has - Moreover, no actual or substantial damage or injury
distinct from that of Prudential’s. It must, therefore, exceeded his power, the principal is not bound was inflicted on petitioner Areola at the time the
bear the consequences of the erroneous cancellation except when he ratifies it expressly or tacitly. insurance policy was cancelled.
of subject insurance policy caused by the non- - Malapit's failure to remit the premiums he received - Nominal damages are "recoverable where a legal
remittance by its own employee of the premiums cannot constitute a defense for private respondent right is technically violated and must be vindicated
paid. insurance company; no exoneration from liability against an invasion that has produced no actual
- Subsequent reinstatement could not possibly could result therefrom. present loss of any kind, or where there has been a
absolve respondent insurance company from liability, - Prudential’s earlier act of reinstating the insurance breach of contract and no substantial injury or actual
there being an obvious breach of contract. After all policy can not obliterate the injury inflicted on damages whatsoever have been or can be shown.
damage had already been inflicted on him and no petitioner-insured. Disposition Petition for review on certiorari is
amount of rectification could remedy the same. - Respondent company should be reminded that a hereby GRANTED. RTC’ s DECISION is REINSTATED.
Respondent’s Argument contract of insurance creates reciprocal obligations
- Prudential argues that where reinstatement, the for both insurer and insured. UCPB GENERAL INSURANCE CO., INC. v.
equitable relief sought by Areola was granted at an - Reciprocal obligations are those which arise from MASAGANA TELAMART, INC.
opportune moment, i.e. prior to the filing of the the same cause and in which each party is both a 308 SCRA 259
complaint, Areola is left without a cause of action on debtor and a creditor of the other, such that the PARDO; June 15, 1999
which to predicate his claim for damages. obligation of one is dependent upon the obligation of
- Reinstatement effectively restored Areola to all his the other. NATURE
rights under the policy. - Under the circumstances of instant case, the Petition for review on certiorari of a decision of the
relationship as creditor and debtor between the Court of Appeals.
ISSUES parties arose from a common cause: i.e., by reason
1. WON the erroneous act of canceling subject of their agreement to enter into a contract of FACTS
insurance policy entitle petitioner-insured to payment insurance under whose terms, Prudential promised to - On April 15, 1991, petitioner issued five (5)
of damages extend protection to Areola against the risk insured insurance policies covering respondent's various
2. WON the subsequent act of reinstating the for a consideration in the form of premiums to be property described therein against fire, for the period
wrongfully cancelled insurance policy obliterate paid by the latter. from May 22, 1991 to May 22, 1992.
whatever liability for damages Prudential has - Under the law governing reciprocal obligations, - In March 1992, petitioner evaluated the policies
particularly the second paragraph of Article 1191, the and decided not to renew them upon expiration of
HELD injured party, Areola in this case, is given a choice their terms on May 22, 1992. Petitioner advised
1. YES between fulfillment or rescission of the obligation in respondent's broker, Zuellig Insurance Brokers, Inc.
2. NO case one of the obligors, such as respondent of its intention not to renew the policies.
Reasoning insurance company, fails to comply with what is - On April 6, 1992, petitioner gave written notice to
- Malapit's fraudulent act of misappropriating the incumbent upon him. respondent of the non-renewal of the policies at the
premiums paid by petitioner-insured is beyond doubt - However, said article entitles the injured party to address stated in the policies.
directly imputable to Prudential. payment of damages, regardless of whether he - On June 13, 1992, fire razed respondent's property
- A corporation, such as respondent insurance demands fulfillment or rescission of the obligation. covered by three of the insurance policies petitioner
company, acts solely thru its employees. The latter’s - Untenable then is reinstatement insurance issued.
acts are considered as its own for which it can be company's argument, namely, that reinstatement - On July 13, 1992, respondent presented to
held to account. being equivalent to fulfillment of its obligation, petitioner's cashier at its head office five (5)
- The facts are clear as to the relationship between divests petitioner-insured of a rightful claim for manager's checks in the total amount of
private respondent insurance company and Malapit. payment of damages. Such a claim finds no support P225,753.95, representing premium for the renewal
His act of receiving the premiums collected is well in our laws on obligations and contracts. of the policies from May 22, 1992 to May 22, 1993.
within the province of his authority as manager. DAMAGES: No notice of loss was filed by respondent under the
Thus, his receipt of said premiums is receipt by - The nature of damages to be awarded, however, policies prior to July 14, 1992.
private respondent insurance company who, by would be in the form of nominal damages - On July 14, 1992, respondent filed with petitioner
provision of law, particularly under Article 1910 of - Although the erroneous cancellation of the its formal claim for indemnification of the insured
the Civil Code, is bound by the acts of its agent. insurance policy constituted a breach of contract, property razed by fire. On the same day, petitioner
- Article 1910 thus reads: Prudential within a reasonable time took steps to returned to respondent the five manager's checks
Art. 1910. The principal must comply with all the rectify the wrong committed by reinstating the that it tendered, and at the same time rejected
obligations which the agent may have contracted insurance policy of petitioner. respondent's claim for the reasons (a) that the
within the scope of his authority. policies had expired and were not renewed, and (b)
INSURANCE Page 35

that the fire occurred on June 13, 1992, before renewal of its policies, and that the acceptance of the this issue in the negative in view of Section 77 of the
respondent's tender of premium payment. late premium payment suggested an understanding Insurance Code and its decisions in Valenzuela v.
- On July, 21, 1992, respondent filed with the that payment could be made later. Hence, this Court of Appeals; South Sea Surety and Insurance
Regional Trial Court, Branch 58, Makati City, a civil appeal. Co., Inc. v. Court of Appeals; and Tibay v. Court of
complaint against petitioner for recovery, of Appeals. Accordingly, it reversed and set aside the
P18.645,000.00, representing the face value of the ISSUE decision of the Court of Appeals.
policies covering respondent's insured property razed WON the fire insurance policies issued by petitioner - Respondent seasonably filed a motion for the
by fire, and for attorney's fees. to the respondent covering the period May 22, 1991 reconsideration of the adverse verdict. It alleges in
- On October 23, 1992, after its motion to dismiss to May 22, 1992, had expired on the latter date or the motion that the SC had made in the decision its
had been denied, petitioner filed an answer to the had been extended or renewed by an implied credit own findings of facts, which are not in accord with
complaint. It alleged that the complaint "fails to state arrangement though actual payment of premium was those of the trial court and the Court of Appeals.
a cause of action"; that petitioner was not liable to tendered on a later date after the occurrence of the The courts below correctly found that no notice of
-respondent for insurance proceeds under the risk (fire) insured against non-renewal was made within 45 days before 22 May
policies because at the time of the loss of 1992, or before the expiration date of the fire
respondent's property due to fire, the policies had HELD insurance policies. Thus, the policies in question
long expired and were not renewed. NO were renewed by operation of law and were effective
After due trial, on March 10, 1993, the Regional Trial - An insurance policy, other than life, issued and valid on 30 June 1992 when the fire occurred,
Court, Branch 58, Makati, rendered decision, the originally or on renewal, is not valid and binding until since the premiums were paid within the 60- to 90-
dispositive portion of which reads: actual payment of the premium. Any agreement to day credit term.
"WHEREFORE, premises considered, judgment is the contrary is void. The parties may not agree - Respondent likewise disagrees with its ruling that
hereby rendered in favor of the plaintiff and against expressly or impliedly on the extension of credit or parties may neither agree expressly or impliedly on
the defendant, as follows. time to pay the premium and consider the policy the extension of credit or time to pay the premium
"(1) Authorizing and allowing the plaintiff to binding before actual payment. nor consider a policy binding before actual payment.
consign/deposit with this Court the sum of Disposition Judgment reversed and set aside It urges the Court to take judicial notice of the fact
P225,753.95 (refused by the defendant) as full that despite the express provision of Section 77 of
payment of the corresponding premiums for the the Insurance Code, extension of credit terms in
replacement-renewal policies for Exhibits A, B, C, D premium payment has been the prevalent practice in
and E; "(2) Declaring plaintiff to have fully complied the insurance industry. Most insurance companies,
with its obligation to pay the premium thereby including Petitioner, extend credit terms because
rendering the replacement-renewal policy of Exhibits Section 77 of the Insurance Code is not a prohibitive
A, B, C, D and E effective and binding for the injunction but is merely designed for the protection
duration May 22, 1992 until May 22, 1993; and, of the parties to an insurance contract. The Code
ordering defendant to deliver forthwith to plaintiff the itself, in Section 78, authorizes the validity of a policy
said replacement-renewal policies; "(3) Declaring notwithstanding non-payment of premiums.
Exhibits A & B, in force from August 22, 1991 up to UCPB GENERAL INSURANCE CO., INC. v. - Respondent also asserts that the principle of
August 23, 1992 and August 9, 1991 to August 9, MASAGANA TELAMART, INC. (EN BANC) estoppel applies to Petitioner. Despite its awareness
1992, respectively; and "(4) Ordering the defendant 356 SCRA 307 of Section 77 Petitioner persuaded and induced
to pay plaintiff the sums of. (a) P18,645,000.00 DAVIDE; April 4, 2001 Respondent to believe that payment of premium on
representing the latter's claim for indemnity under the 60- to 90-day credit term was perfectly alright;
Exhibits A, B & C and/or its replacement-renewal NATURE in fact it accepted payments within 60 to 90 days
policies; (b) 25% of the total amount due as and for Motion for reconsideration of the decision of the after the due dates. By extending credit and
attorney's fees; (c) P25,000.00 as necessary Supreme Court. habitually accepting payments 60 to 90 days from
litigation expenses; and, (d) the costs of suit. the effective dates of the policies, it has implicitly
“xxx ” FACTS agreed to modify the tenor of the insurance policy
- In due time, petitioner appealed to the Court of - In its decision of 15 June 1999, the SC defined the and in effect waived the provision therein that it
Appeals (CA). The CA promulgated its decision main issue to be “whether the fire insurance policies would pay only for the loss or damage in case the
affirming that of the Regional Trial Court with the issued by petitioner to the respondent covering the same occurred after payment of the premium.
modification that item No. 3 of the dispositive portion period from May 22, 1991 to May 22, 1992 had been - Petitioner filed an opposition to the Respondent’s
was deleted, and the award of attorney's fees was extended or renewed by an implied credit motion for reconsideration. It argues that both the
reduced to 10% of the total amount due. arrangement though actual payment of premium was trial court and the Court of Appeals overlooked the
It held that following previous practice, respondent tendered on a later date and after the occurrence of fact that on 6 April 1992 Petitioner sent by ordinary
was allowed a 60- to 90-day credit term for the the (fire) risk insured against.” The Court resolved mail to Respondent a notice of non-renewal and sent
INSURANCE Page 36

by personal delivery a copy thereof to Respondent’s approved on 21 June 1963, which read: recovery on the policy would not be permitted
broker, Zuellig. Both courts likewise ignored the fact SEC. 72. An insurer is entitled to payment of against Petitioner, which had consistently granted a
that Respondent was fully aware of the notice of premium as soon as the thing insured is exposed 60- to 90-day credit term for the payment of
non-renewal. A reading of Section 66 of the to the peril insured against, unless there is clear premiums despite its full awareness of Section 77.
Insurance Code readily shows that in order for an agreement to grant the insured credit extension of Estoppel bars it from taking refuge under said
insured to be entitled to a renewal of a non-life the premium due. No policy issued by an Section since Respondent relied in good faith on such
policy, payment of the premium due on the effective insurance company is valid and binding unless and practice. Estoppel then is the fifth exception to
date of renewal should first be made. Respondent’s until the premium thereof has been paid. Section 77.
argument that Section 77 is not a prohibitive (Underscoring supplied) Disposition Judgment reconsidered and set aside,
provision finds no authoritative support. - It can be seen at once that Section 77 does not that of the Court of Appeals affirmed in toto.
- The following facts, as found by the trial court and restate the portion of Section 72 expressly
the Court of Appeals, are indeed duly established: permitting an agreement to extend the period to pay SEPARATE OPINION
1. For years, Petitioner had been issuing fire the premium. But there are exceptions to Section
policies to the Respondent, and these policies 77. VITUG
were annually renewed. The first exception is provided by Section 77 itself, - An essential characteristic of an insurance is its
2. Petitioner had been granting Respondent a and that is, in case of a life or industrial life policy being synallagmatic, a highly reciprocal contract
60- to 90-day credit term within which to pay whenever the grace period provision applies. where the rights and obligations of the parties
the premiums on the renewed policies. The second is that covered by Section 78 of the correlate and mutually correspond.
3. There was no valid notice of non-renewal of Insurance Code, which provides: - By weight of authority, estoppel cannot create a
the policies in question, as there is no proof at SEC. 78. Any acknowledgment in a policy or contract of insurance, neither can it be successfully
all that the notice sent by ordinary mail was contract of insurance of the receipt of premium is invoked to create a primary liability, nor can it give
received by Respondent, and the copy thereof conclusive evidence of its payment, so far as to validity to what the law so procribes as a matter of
allegedly sent to Zuellig was ever transmitted make the policy binding, notwithstanding any public policy.
to Respondent. stipulation therein that it shall not be binding until
4. The premiums for the policies in question in premium is actually paid. PARDO [dissent]
the aggregate amount of P225,753.95 were - A third exception was laid down in Makati Tuscany - An assured’s failure to give notice of the fire
paid by Respondent within the 60- to 90-day Condominium Corporation vs. Court of Appeals, immediately upon its occurrence blatantly showed
credit term and were duly accepted and wherein we ruled that Section 77 may not apply if the fraudulent character of its claims. Respondent is
received by Petitioner’s cashier. the parties have agreed to the payment in required by law and by express terms of the policy to
installments of the premium and partial payment has give immediate written notice of loss. This must be
ISSUE been made at the time of loss. Tuscany has complied with in the utmost good faith.
WON Sec. 77 of the Insurance Code of 1978 must be provided a fourth exception to Section 77, namely, - Assuming arguendo that the 60- to 90-day credit
strictly applied to Petitioner’s advantage despite its that the insurer may grant credit extension for the has been agreed between the parties, respondent
practice of granting a 60- to 90-day credit term for payment of the premium. This simply means that if could not still invoke estoppel to back up its claim.
the payment of premiums the insurer has granted the insured a credit term for Estoppel cannot give validity to an act that is
HELD the payment of the premium and loss occurs before prohibited by law or against public policy. The actual
NO the expiration of the term, recovery on the policy payment of premiums is a condition precedent to the
- Section 77 of the Insurance Code of 1978 provides: should be allowed even though the premium is paid validity of an insurance contract other than life
SEC. 77. An insurer is entitled to payment of the after the loss but within the credit term. insurance policy. Any agreement to the contrary is
premium as soon as the thing insured is exposed Moreover, there is nothing in Section 77 which void as against law and public policy.
to the peril insured against. Notwithstanding any prohibits the parties in an insurance contract to
agreement to the contrary, no policy or contract of provide a credit term within which to pay the ACME SHOE RUBBER & PLASTIC CORP. v. CA
insurance issued by an insurance company is valid premiums. That agreement is not against the law, (DOMESTIC INSURANCE COMPANY OF THE
and binding unless and until the premium thereof morals, good customs, public order or public policy. PHILS.)
has been paid, except in the case of a life or an The agreement binds the parties. Article 1306 of the 134 SCRA 155
industrial life policy whenever the grace period Civil Code provides: MELENCIO-HERRERA; January 17, 1985.
provision applies. ART. 1306. The contracting parties may establish
- This Section is a reproduction of Section 77 of P.D. such stipulations clauses, terms and conditions as NATURE
No. 612 (The Insurance Code) promulgated on 18 they may deem convenient, provided they are not Petition for Review on Certiorari of the Decision of
December 1974. In turn, this Section has its source contrary to law, morals, good customs, public the then Court of Appeals (CA-G. R. No. 58917-R),
in Section 72 of Act No. 2427 otherwise known as order, or public policy. denying recovery on an insurance policy, thereby
the Insurance Act as amended by R.A. No. 3540, - Finally, it would be unjust and inequitable if reversing the judgment of the Court of First Instance
INSURANCE Page 37

of Rizal, Branch XII, at Caloocan City, which had CA reversed TC and dismissed the suit on the ground Disposition The judgment under review is hereby
allowed such recovery. that, as of the moment of loss, ACME's properties affirmed. Without pronouncement as to costs.
were not insured and the INSURER could not be held
FACTS liable for any indemnity as a result of the loss. PEDRO ARCE v. THE CAPITAL INSURANCE &
- ACME Shoe Rubber and Plastic Corporation (ACME) SURETY CO., INC.
had been insuring yearly against fire its building, ISSUE 11 SCRA 63
machines and general merchandise with Domestic WON the premium payment for 1964-1965 was paid ABAD SANTOS; September 30, 1982.
Insurance Company (INSURER) since 1946. On May
14, 1962, ACME continued to insure its properties HELD NATURE
with INSURER in the amount of P200,000 for the NO Appeal from CFI decision on question of law.
period May 15, 1962 up to May 15, 1963. - Not having paid the 1964-1965 premium within the
- On May 14, 1963, INSURER issued Renewal Receipt extension granted, and pursuant to R.A. No. 3540, FACTS
to cover the period May 15, 1963 to May 15, 1964. the policy was automatically cancelled and there was - Arce (INSURED) owned a residential house which
- On January 8, 1964, ACME paid P3,331.26 as no insurance coverage to speak of as of the date of was insured with the appellant COMPANY since 1961.
premium. The INSURER applied the payment as the fire on October 13, 1964. In November 1965, the COMPANY sent to the
renewal premium for the period of May 15, 1963 to - The pertinent provision of Republic Act No. 3540 INSURED a Renewal Certificate to cover the period
May 15, 1964. reads: from December 5, 1965 to December 5,1966, and
- On May 15, 1964, INSURER issued a Renewal "Sec. 72. An insurer is entitled to payment of the requested payment of the corresponding premium.
Receipt for the period of May 15, 1964 to May 15, premium as soon as the thing insured is exposed Anticipating that the premium could not be paid on
1965 (for renewal premium of P3,331.26 yet to be to the peril insured against, unless there is clear time, the INSURED asked for an extension which was
paid) with a stamped note that says that the agreement to grant the insured credit extension of granted by the COMPANY. After the lapse of the
insurance will be deemed valid and binding only the premium due. No policy issued by an requested extension, INSURED still failed to pay the
when the premium and documentary stamps have insurance company is valid and binding unless and premium. Thereafter, the house of the INSURED was
actually been paid in full and duly acknowledged in until the premium thereof has been paid." totally destroyed by fire. Upon INSURED's
an official receipt. ACME was given 90 days to pay - RA 3540 was approved on June 20, 2963 and was presentation of claim for indemnity, he was told that
otherwise the policy would automatically become put into effect on Oct 1, 1963. It could not be applied no indemnity was due because the premium was not
void and ineffective. (ACME should pay short period retroactively to the renewal of the policy for the paid. The INSURED sued the COMPANY for
premium for 90 days before the period expires. If 1963-1964 period because said policy was renewed indemnity.
they are able to pay the whole amount before the on May 14, 1963. (Laws have no retroactive effect - The trial court held the COMPANY liable to
90-day period, the automatic termination won’t apply unless the contrary is provided.) Therefore, the Jan indemnify the INSURED on the ground that since the
anymore). 8, 1964 payment was properly applied to the 1963- COMPANY could have demanded payment of the
- On May 26, 1964, ACME, through its President, 1964 premium. The Trial Court's opinion that there premium, mutuality of obligation required that it
signed a promissory note saying that they promise to was a clear agreement to grant ACME credit should be liable on the policy.
pay the premium and documentary stamps and extension for 1964-1965 is negated by ACME's
agreed to the automatic cancellation penalty for not Promissory Note binding itself to pay within ninety ISSUE
complying. days from the effective date of this policy, 15th May, WON the COMPANY can be held liable on its policy
- On October 13, 1964, ACME’s properties were 1964. The credit extension was granted for 90 days
completely destroyed by fire. ACME filed insurance only. (So wala na by August 16, 1964.) HELD
claim but the INSURER disclaimed liability on the - If ACME was granted credit extensions in the past, NO.
ground that as of the date of loss, the properties the promissory note it signed did away with such - The Court commiserates with the INSURED. They
burned were not covered by insurance. credit arrangement. Also, before RA 3540, the are well aware that many insurance companies have
- ACME claims that the January 8, 1964 payment Renewal Receipts issued by INSURER did not contain fallen into the condemnable practice of collecting
was for the period 1964-1965 and that INSURER had the auto-cancellation after 90 days note. By 1964, premiums promptly but resort to all kinds of excuses
no right to apply it to the period 1963-1964 because however, the situation had changed by the passage to deny or delay payment of just claims. Unhappily
under RA 3540, the policy was void and INSURER of the RA: no policy could be valid and binding the instant case is one where the insurer has the law
could have validly disclaimed liability for loss had one unless and until the premium thereof had been on its side.
occurred then. paid. - Sec. 72 of the Insurance Act, as amended by R.A.
- TC found INSURER liable for P200k and opined that - What became automatically cancelled by R.A. No. No. 3540 reads:
there was a clear intention on the INSURER's part to 3540 was the 1964-1965 policy for ACME's failure to "SEC. 72. An insurer is entitled to payment of
grant ACME a credit extension for the payment of the pay the premium within the 90-day extension premium as soon as the thing insured is exposed
premium due; and that to allow the INSURER to granted, and in accordance with the express terms of to the perils insured against, unless there is clear
apply the premium ACME paid on January 8, 1964. the Promissory Note that it had signed. agreement to grant credit extension for the
INSURANCE Page 38

premium due. No policy issued by an insurance corresponding insurance premium. On January 8, held for an unreasonable time before presenting it
company is valid and binding unless and until the 1961, in partial payment of the insurance premium, for payment, the insurer may be held estopped from
premium thereof has been paid." Plastic Era delivered to Capital Insurance, a check for claiming a forfeiture if the check is dishonored.
- It is obvious from both the Insurance Act, as the amount of P1,000.00 postdated January 16, Disposition The decision of the CA is AFFIRMED in
amended, and the stipulation of the parties that time 1961. However, Capital Insurance tried to deposit the toto.
is of the essence in respect of the payment of the check only on February 20, 1961 and the same was
insurance premium so that if it is not paid the dishonored by the bank for lack of funds. MALAYAN INSURANCE CO., INC. v. ARNALDO
contract does not take effect unless there is still - Two days after the insurance premium became due, and PINCA
another stipulation to the contrary. In the instant at about 4:00 to 5:00 o'clock in the morning, the 154 SCRA 672
case, the INSURED was given a grace period to pay property insured by Plastic Era was destroyed by fire. CRUZ; October 12, 1987
the premium but the period having expired with no In less than a month Plastic Era demanded from
payment made, he cannot insist that the COMPANY is Capital Insurance the payment of the sum of FACTS
nonetheless obligated to him. P100,000.00 as indemnity for the loss of the insured - On June 7, 1981, Malayan Insurance Co. (MICO),
- Prior to the amendment (italicized portion above), property under Policy No. 22760 but the latter issued fire insurance for the amount of P14,000 on
an insurance contract was effective even if the refused for the reason that, among others, Plastic the property of private respondent, Pinca, effective
premium had not been paid so that an insurer was Era failed to pay the insurance premium. July 1981-1982. MICO later allegedly cancelled the
obligated to pay indemnity in case of loss and policy for non-payment of the premium and sent a
correlatively he had also the right to sue for payment ISSUES notice to Pinca. On Dec. 24 Adora, an agent of MICO,
of the premium. But the amendment to Sec. 72 has 1. WON a contract of insurance has been duly received Pinca’s payment, which was remitted to
radically changed the legal regime in that unless the perfected between petitioner and respondent MICO. On Jan. 18, 1982, Pinca’s property was
premium is paid there is no insurance. 2. WON the dishonored check constituted payment completely burned. On Feb. 5, MICO returned Pinca’s
Disposition The decision of the court a quo is payment to Adora on the ground that her policy had
reversed; the appellee's complaint is dismissed. No HELD been cancelled; the latter refused to accept it. Her
special pronouncement as to costs. 1. YES demand for payment having been rejected by MICO,
- Irrelevant facts: The premium costs P38.10. After - Tender of draft or check in order to effect payment Pinca went to the Insurance Commission. Public
the fire, the COMPANY issued a check for P300 to that would extinguish the debtor's liability should be respondent Arnaldo, the Insurance Commissioner,
Arce as donation. Arce accepted the check, but still actually cashed. If the delivery of the check of Plastic sustained Pinca, hence this petition from MICO.
sued the company. Era to Capital Insurance were to be viewed in the Records show MICO received Arnaldo’s decision on
CAPITAL INC. v. PLASTIC ERA CO. light of the foregoing, no payment of the premium April 10; MICO filed a MFR on April 25 which was
65 SCRA 134 had been effected. Significantly, Capital Insurance denied on June 4; MICO received notice of this denial
MARTIN; July 18, 1975 accepted the promise of Plastic Era to pay the on June 14; instant petition was filed on July 2.
insurance premium within 30 days from the effective
NATURE date of policy. By so doing, it has implicitly agreed to ISSUES
Petition for review of a decision of the CA affirming modify the tenor of the insurance policy and in Procedural
the decision of the CFI of Manila effect, waived the provision therein that it would only 1. WON the petition should be dismissed for late
pay for the loss or damage in case the same occurs filing
FACTS after the payment of the premium. Considering that Substantive
- On December 17, 1960, petitioner Capital the insurance policy is silent as to the mode of 2. WON there was a valid insurance contract at the
Insurance & Surety Co., Inc. delivered to the payment, Capital Insurance is deemed to have time of the loss
respondent Plastic Era Manufacturing Co., Inc., its accepted the promissory note in payment of the 3. WON Adora was authorized to receive such
open Fire Policy No. 22760 wherein the former premium. This rendered the policy immediately payment
undertook to insure the latter's building, equipments, operative on the date it was delivered. 4. WON an adjuster is indispensable in the
raw materials, products and accessories located at 2. YES valuation of the loss
Sheridan Street, Mandaluyong, Rizal. The policy - Although the check was due for payment on
expressly provides that if the property insured would January 16, 1961 and Plastic Era had sufficient funds HELD
be destroyed or damaged by fire after the payment to cover it as of January 19, 1961, Capital Insurance Procedural
of the premiums, at anytime between the 15th day decided to hold the same for thirty-five (35) days 1. YES
of December 1960 and one o'clock in the afternoon before presenting it for payment. Having held the - Petitioner invokes Sec 416 of the Insurance Code
of the 15th day of December 1961, the insurance check for such an unreasonable period of time, which grants it 30 days from notice of the Insurance
company shall make good all such loss or damage in Capital Insurance was estopped from claiming a Commission within which to appeal by certiorari with
an amount not exceeding P100,000.00. When the forfeiture of its policy for non-payment even if the the Court. MICO filed its MFR on April 25, 15 days
policy was delivered, Plastic Era failed to pay the check had been dishonored later. Where the check is after the notice; the reglementary period began to
INSURANCE Page 39

run again after June 13. Since the petition was filed made on the original transaction and validly received - From January 1, 1942 to December 31, 1946,
only on July 2, it was tardy by 4 days. Alternatively it by Adora, who was not informed of the alleged Plaintiff head office at Toronto applied the provisions
invokes Rule 45 of the Rules of Court for certiorari cancellation and thus saw no reason to reject the of the automatic premium loan clauses upon the
but the petition still exceeds the 15 day limit from payment. nonpayment of the corresponding premiums by the
the June 13 notice. 3. YES people who subscribed to the insurance. The net
-Respondents, on the other hand, invoke Sec. 39 of - Sec. 306 of the Insurance Code provides that any amount of premiums advanced (by the company) or
B.P. 129 which pegs the period for appeal from insurance company that delivers a policy to its agent loaned (to the insured) as payment for the premium
decisions of any court in all cases at 15 days from is deemed to have authorized such agent to receive due totaled P1,069,254.98.
the notice of the decision appealed from. Since the payment of premium on its behalf. It is a well-known - Meer, the Collector of the National Internal Revenue
MFR was filed only 15 days after receiving notice of principle under the law of agency that payment to an assessed the net amount of premium at P17,917.12
the decision, it was already 18 days late by July 2. authorized agent is equivalent to payment to the pursuant to SEC.255, National Internal Revenue
So whichever is applied, the petition is still late. principal himself. MICO’s acknowledgement of Adora Code6
Substantive as its agent thus defeats its contention that he was - Company protested the assessment, but paid the
2. YES not authorized to receive payments on its behalf. taxes anyway. Then they filed a complaint to recover
- A valid cancellation requires the following 4. NO money paid under protest for taxes
conditions based on Sections 64-65 of the Code: - In absence of fraud, the amount of the loss may be - CFI: Dissmiss complaint
prior notice which must be based on the occurrence determined on the basis of such proof offered by the - PLAINTIFF’s MAIN CONTENTION: when it made
of one or more of the grounds mentioned in Sec 64 insured. Here. The certification of the Integrated premium loans or premium advances by virtue of the
(in this case, non-payment of premium), after the National Police as the extent of the loss should non-forfeiture clauses, it did not collect premiums
effective date of the policy; the notice must be suffice. within the meaning of the above sections of the law,
written and mailed to the address on the policy; it Disposition petition is DENIED and therefore it is not amenable to the tax therein
must state the ground(s) for cancellation and the provided.
insurer must furnish details upon the request of the MANUFACTURERS LIFE INSURANCE CO. v. MEER
insured. 89 PHIL 351 ISSUES
- It is undisputed that payment of premium was BENGZON, June 29, 1951 1. WON premium advances made by plaintiff-
made. Petitioner relies heavily on Sec 77 of the appellant under the automatic premium loan clause
Insurance Code to contest this, the said provision NATURE of its policies are premiums collected' by the
requiring payment of premium as soon as the thing APPEAL from a judgment of the Court of First Company subject to tax
is exposed to the peril insured against and that the Instance of Manila 2. WON, in the application of the automatic premium
policy is invalid without it. However, this is not loan clause of plaintiff-appellant's policies, there is
applicable in the instant case as payment was FACTS 'payment in money, notes, credits, or any substitutes
eventually made. It is to be noted that the premium (this is a tax case. What’s really important here is for money
invoice was stamped “Payment Received”, indicating the definition of CASH SURRENDER VALUE). 3. WON the collection of the alleged deficiency
an understanding between the parties that payment - Manufacturers Life Insurance Company is a duly premium taxes constitutes double taxation
could be made later. This is furthered by the fact that organized corporation which has its head office at 4. WON the making of premium advances, granting
Adora had earlier told her to call him anytime she Toronto. It is duly registered and licensed to engage for the sake of argument that it amounted to
was ready with her payment. The Court also finds it in life insurance business in the Philippines, and, collection of premiums, were done in Toronto,
strange that MICO only sought to return Pinca’s Jan. maintains a branch office in Manila. It was engaged Canada
15 payment only on Feb. 5, long after her house had in such business in the Philippines for more than five
burned down—this makes petitioner’s motives highly years before and including the year 1941. But due to 'When the premium falls due and is not paid in cash within the month's grace, if the Cash Value of this policy and of any
suspect. the exigencies of the war It closed the branch office bonus additions and dividends left on accumulation (after deducting any accumulated indebtedness) be less than the

- MICO claims to have sent a notice to Pinca, who at Manila during 1942 up to September 1945. premium then due, the Company will, without further requests, continue this insurance in force for a period * * *.

'10. Cash and Paid-Up Insurance Values.-At the end of the third policy year or thereafter, upon the legal surrender of this
flatly denied receiving one. Pinca did not have to - Plaintiff issued a number of life insurance policies in Policy to the Company while there is no default in premium payments or within two months after the due date of the

prove this since the strict language of Sec 64 the Philippines containing stipulations referred to as premium in default, the Company will (1) grant a cash value as specified in Column (A) increased by the cash value of any

requires that MICO ensure the cancellation was NONFORFEITURE CLAUSES5 bonus additions and dividends left on accumulation, which have been alloted to this Policy, less all indebtedness to the

Company on this Polley an the date of ouch surrender, or (2) endorse this Policy as a Non-Participating Paid-up Polley for the
actually sent to and received by the insured. amount as specified In Column (B) of the Table of Guaranteed Values * * *.

- MICO also suggests that Pinca knew the policy had 5"'8. Automatic Premium Loan.-This Policy shall not lapse for non-payment of any premium after it has been three full years '11. Extended Insurance-After the premiums for three or more full years have been paid hereunder in cash, if any

been cancelled and was paying the premium in order


subsequent premium is not paid when due, and there is no indebtedness to the Company on the written request of the
in force, it, at the due date of such premium, the Cash Value of this Policy and of any bonus additions and dividends left on insured * * *."

to renew the policy. A close study of the transcripts accumulation (after deducting any indebtedness to the company and the interest accrued thereon) shall exceed the amount
6"SEC. 255. Taxes on insurance premiums.-There shall be collected from every person, company, or corporation (except
show, however, that Pinca only meant to renew the of said premium. In which event the company will, without further request, treat the premium then due as paid, and the

amount of such premium, with interest from its actual due date at six per cent per annum, compounded yearly, and one per purely cooperative companies or associations) doing insurance business of any sort in the Philippines a tax of one per
policy had it been cancelled but not if it was still in cent, compounded yearly, for expenses, shall be a first lien on this Policy in the Company's favour in priority to the claim of centum of the total premiums collected * * * whether such permiums are paid in money, notes, credits, or any substitute for

effect—it was conditional. Payment was thus legally any assignee or any other person. The accumulated lien may at any time, while the Policy is in force, be paid in whole or in money but premiums refunded within six months after payment on account of rejection of risk or returned for other reason

part. to person insured shall not be included in the taxable receipts * * *."
INSURANCE Page 40

5. WON the fact that plaintiff-appellant was not doing he surrenders it and releases his claims upon it. The
business in the Philippines during the period from more premiums the insured has paid the greater will
January 1, 1942 to September 30, 1945, inclusive, be the surrender value; but the surrender value is
exempts it from payment of premium taxes always a lesser sum than the total amount of
corresponding to said period premiums paid." (Cyclopedia Law Dictionary 3d. ed.
1077.) The cash value or cash surrender value is ANDRES v. CROWN LIFE INSURANCE
therefore an amount which the insurance company 102 Phil. 919
HELD holds In trust for the insured to be delivered to him REYES, J.B.L., Jan.28, 1958
NOTE (example given by the plaintiff): upon demand. It is therefore a liability of the
"Suppose that 'A', 30 years of age, secures a 20-year company to the insured. Now then, when the NATURE
endowment policy for P5,000 from plaintiff-appellant company's credit for advances is paid out of the cash Appeal from judgment of CFI
Company and pays an annual premium of P250. 'A' value or cash surrender value, that value and the
pays the first ten yearly premiums amounting to company's liability is thereby diminished pro tanto. FACTS
P2,500 and on this amount plaintiff-appellant pays 2. YES - Feb. 13, 1950: For the sum of P5,000, defendant-
the corresponding taxes under section 255 of the - the insurer agreed to consider the premium paid on appellee Crown Life issued an insurance policy in the
National Internal Revenue Code. Suppose also that the strength of the automatic loan. The premium was name of plaintiff-appellant Rufino and his wife, with
the cash value of said policy after the payment of the therefore paid by means of a "note" or "credit" or the stipulation that the premiums are to be paid
10th annual premium amounts to P1,000." When on "other substitute for money" and the tax is due semi-annually.
the eleventh year the annual premium fell due and because section 255 above quoted levies taxes - The premiums for the 1st and 2nd semester of the 1st
the insured remitted no money within the mouth according to the total premiums collected by the year, in the amount of P165.15 were paid by Rufino
grace, the insurer treated the premium then over insurer "whether such premiums are paid in money, but the premium for the third semester, in the same
due as paid from the cash value, the amount being notes, credits or any substitute for money. amount, was not paid.
a loan to the policyholder1 who could discharge it 3. NO - Jan. 6, 1951, Crown Life, through its branch
at any time with interest at 6 per cent. The insurance - No constitutional prohibition against double secretary, wrote to Mr. and Mrs. Andres advising
contract, therefore, continued in force for the taxation. them that their insurance policy lapsed on Dec. 26,
eleventh year. 4. NO 1950 and the amount of P165.15 was overdue,
1. YES - The loans are made to policyholders in the giving them 60 days from the date of lapse to file an
- Based on the example given by the plaintiff, the Philippines, who in turn pay therewith the premium application for reinstatement. Crown Life later sent
insurer collected the amount of P250 as the annual to the insurer thru the Manila branch. Approval of another letter telling the spouses Andres that their
premium for the eleventh year on the said policy appellant's position will enable foreign insurers to insurance policy was no longer in force.
when it loaned to “A” the sum of P250. The insurer evade the tax by contriving to require that premium - Feb. 1951: Plaintiff and his wife executed a
“became a creditor” of the loan, but not of the payments shall be made at their head offices. What Statement of Health and application for
premium that had already been paid (advanced by is important, the law does not contemplate reinstatement of the aforesaid policy.
the insurer). The insurer is entitled to collect interest premiums collected in the Philippines. It is enough - Feb. 20, 1951: Plaintiff wrote a letter to the
on the loan, not on the premium. "A" paid the that the insurer is doing insurance business in the defendant, enclosed with a money order for P100.
premium for the eleventh year; but in turn he Philippines, irrespective of the place of its Upon acceptance, defendant advised Rufino that its
became a debtor of the company for the sum of organization or establishment. main office had approved the application and that
P250. This debt he could repay either by later 5. NO the reinstatement of the lapsed policy was subject to
remitting the money to the insurer or by letting the - Although during those years the appellant was not the payment of the remaining premium balance of
cash value compensate for it. The debt may also be open for new business because its branch office was P65.15.
deducted from the amount of the policy should "A" closed, still it was practically and legally, operating in - May 3, 1951: Severa Andres died of dystocia,
die thereafter during the continuance of the policy. this country by collecting premiums on its contracted pelvis.
- ON ARGUMENT THAT THE ASSETS OF THE INSURER REMAINED THE outstanding policies, incurring the risks and/or - May 5, 1951: Plaintiff sent a letter enclosed with a
SAME AFTER THE APPLICATION OF THE AUTOMATIC PREMIUM LOAN CLAUSE: enjoying the benefits consequent thereto, without money order in the amount of P65, for the remaining
there was an increase in assets in the form of having previously taken any steps indicating balance due.
CREDIT for the advances made (in the example, the withdrawal in good faith from this field of economic - May 15, 1951: Defendant sent a letter with official
P250 for the 11th year). activity. receipt of the P165.15 paid by Rufino as well as a
- ON ARGUMENT THAT IF THE CREDIT IS PAID OUT OF THE CASH Disposition finding no prejudicial error in the Certificate of Reinstatement.
SURRENDER VALUE, THERE WERE NO NEW FUNDS ADDED TO THE appealed decision, we hereby affirm it with costs. - June 7, 1951: Rufino presented a death claim as
COMPANY'S ASSETS”: Cash surrender value "as applied survivor-beneficiary of his deceased wife. Payment
to a life insurance policy, is the amount of money the was denied by the defendant.
company agrees to pay to the holder of the policy if
INSURANCE Page 41

- April 1952: Rufino filed a complaint in CFI against presented such other evidence of insurability at the Insurance Company, Inc. (Philamgen for short) since
Crown Life for the recovery of the amount of P5,000 date of application for reinstatement; (E) there 1965. As such, he was authorized to solicit and sell
as the face value of a joint 20-year endowment should be no change which has taken place in such in behalf of Philamgen all kinds of non-life insurance,
insurance policy issued by defendant in favor of good health and insurability subsequent to the date and in consideration of services rendered was
plaintiff and his wife, on Feb. 13, 1950. In its answer, of such application and before the policy is entitled to receive the full agent's commission of
Crown Life disclaimed liability and set forth the reinstated; and (F) all overdue premiums and other 32.5% from Philamgen under the scheduled
special defense that the aforementioned policy had indebtedness in respect of the policy, together with commission rates.
already lapsed. interest at 6%, compounded annually, should first be - From 1973 to 1975, Valenzuela solicited marine
- Aug. 5, 1954: CFI rendered a decision absolving paid. insurance from one of his clients, the Delta Motors,
the defendant company from any liability on the - The plaintiff did not comply with the last condition; Inc. (Division of Electronics Airconditioning and
ground that the policy had lapsed and it was not for he only paid P100 before his wife’s death; and Refrigeration) in the amount of P4.4 Million from
reinstated at the time of the plaintiff’s wife’s death. despite the Company’s reminders, he only remitted which he was entitled to a commission of 32%.
Plaintiff later appealed to the CA but the same was the balance of P65.15 two days after his wife died. However, Valenzuela did not receive his full
certified by the CA to the SC for having no question On the face of such facts, the Company had the right commission which amounted to P1.6 Million from the
of fact. to treat the contract as lapsed and refuse payment of P4.4 Million insurance coverage of the Delta Motors.
the policy. During the period 1976 to 1978, premium payments
ISSUE - Rufino contends that the condition regarding amounting to P1,946,886.00 were paid directly to
WON the insurance policy, which has been in a state payment of the premium was waived by the Philamgen and Valenzuela's commission to which he
of lapse before May 3, 1951, has been validly and insurance Company through its letters, wherein it is entitled amounted to P632,737.00.
completely reinstated after said date (Was there a made statements such as: “If you are unable to pay - In 1977, Philamgen started to become interested in
perfected contract of reinstatement after the policy the full amount immediately, send as large amount and expressed its intent to share in the commission
lapsed due to non-payment of premiums?) as possible and advise us how soon you expect to be due Valenzuela on a fifty-fifty basis. Valenzuela
able to pay the balance; we will work out an refused.
HELD adjustment most beneficial to you.” The Court found - Because of the refusal of Valenzuela, Philamgen
NO the statements to be too vague and indefinite to and its officers took drastic action against
Ratio The stipulation in a life insurance policy giving indicate an intention on the insurer’s part to waive Valenzuela. They: (a) reversed the commission due
the insured the privilege to reinstate it upon written the full payment as prerequisite to the reinstatement him by not crediting in his account the commission
application does not give the insured absolute right of the lapsed policy. The Court reiterated the rule earned from the Delta Motors, Inc. insurance ; (b)
to such reinstatement by the mere filing of an that a waiver must be clear and positive, the intent placed agency transactions on a cash-and-carry
application. The Company has the right to deny the to waive shown clearly and convincingly. On the basis; (c) threatened the cancellation of policies
reinstatement if it is not satisfied as to the other hand, It found subsequent letters sent by issued by his agency; and (d) started to leak out
insurability of the insured and if the latter does not defendant indicating that they insisted on full news that Valenzuela has a substantial account with
pay all overdue premiums and all other indebtedness payment of the premium before the policy was Philamgen. All of these acts resulted in the decline of
to the Company. After the death of the insured the reinstated and that defendant did not consider partial his business as insurance agent.
insurance Company cannot be compelled to entertain payment as sufficient consideration for the - Then on December 27, 1978, Philamgen terminated
an application for reinstatement of the policy reinstatement. Plaintiff-Appellant’s failure to remit the General Agency Agreement of Valenzuela.
because the conditions precedent to reinstatement the balance before the death of his wife operated to - Lower court: the termination of Valenzuela as
can no longer be determined and satisfied. deprive him of any right to waive the policy and General Agent was improper because the record will
Reasoning recover the face value thereof. show the principal cause of the termination of the
- The stipulations of facts render it undisputable that Disposition Judgment appealed from is affirmed. plaintiff as General Agent of defendant Philamgen
the original policy lapsed for non-payment of was his refusal to share his Delta commission.
premiums on Dec. 26, 1950, upon expiration of the VALENZUELA v. CA (PHILIPPINE AMERICAN - CA: In any event the principal's power to revoke an
31-day grace period. GENERAL INSURANCE COMPANY, INC.) agency at will is so pervasive, that the Supreme
- As found by the lower court, the conditions set 191 SCRA 1 Court has consistently held that termination may be
forth in the policy for reinstatement as provided in GUTIERREZ; October 19, 1990 effected even if the principal acts in bad faith,
the contract itself are the following: (A) application subject only to the principal's liability for damages.
shall be made within 3 years from the date of lapse; NATURE (CA ordered Valenzuela to pay Philamgen the amount
(B) there should be a production of evidence of the Petition for review of the decision of theca. of One Million Nine Hundred Thirty-Two Thousand
good health of the insured; (C) if the rate of Five Hundred Thirty-Two and 17/100 Pesos
premium depends upon the age of the Beneficiary, FACTS (P1,932,532.17) with legal interest)
there should likewise be a production of evidence of - Petitioner Arturo P. Valenzuela is a General Agent of
his or her good health; (D) there should be private respondent Philippine American General ISSUES
INSURANCE Page 42

1. WON whether or not Philamgen and/or its officers but also for the fact that by the very acts of the Appeal from an order of the CFI of Zamboanga
can be held liable for damages due to the respondents, he was made liable to Philamgen in the sustaining a demurrer to plaintiff's complaint upon
termination of the General Agency Agreement it event the insured fail to pay the premiums due. They the ground that it fails to state a cause of action.
entered into with the petitioners are estopped by their own positive averments and
2. WON petitioners are liable to Philamgen for the claims for damages. FACTS
unpaid and uncollected premiums - "The principal may not defeat the agent's right to - On July 6, 1917, Luis Lim of Zamboanga made
indemnification by a termination of the contract of application to the Sun Life Assurance Company of
HELD agency (Erskine v. Chevrolet Motors Co. 185 NC 479, Canada for a policy of insurance on his life in the
1. YES 117 SE 706, 32 ALR 196). sum of P5,000. In his application Lim designated his
- If a principal acts in bad faith and with abuse of - For the pivotal factor rendering Philamgen and the wife, Pilar de Lim, the plaintiff herein, as the
right in terminating the agency, then he is liable in other private respondents liable in damages is that beneficiary. The first premium of P433 was paid by
damages. the termination by them of the General Agency Lim, and upon such payment the company issued
- There is an exception to the principle that an Agreement was tainted with bad faith. This is in what was called a ''provisional policy." Luis Lim died
agency is revocable at will and that is when the accordance with the precepts in Human Relations on August 23, 1917, after the issuance of the
agency has been given not only for the interest of enshrined in our Civil Code. provisional policy but before approval of the
the principal but for the interest of third persons or 2. NO. The respondent court erred in holding application by the home office of the insurance
for the mutual interest of the principal and the agent. Valenzuela liable. There was no factual and legal company. Pilar de Lim brought an action to recover
In these cases, it is evident that the agency ceases basis for the award. Under Section 77 of the from the Sun Life sum of P5,000, the amount named
to be freely revocable by the sole will of the principal Insurance Code, the remedy for the non-payment in the provisional policy.
(PROCEDURAL: Where the findings of the Court of of premiums is to put an end to and render the - The "provisional policy" reads: "Received (subject
Appeals and the trial court are contrary to each insurance policy not binding - "Sec. 77 . . . to the following stipulations and agreements) the
other, this Court may scrutinize the evidence on [N]otwithstanding any agreement to the contrary, no sum of P433, being the amount of the first year's
record policy or contract of insurance is valid and binding premium for a Life Assurance Policy on the life of Mr.
- After a painstaking review of the entire records of unless and until the premiums thereof have been Luis D. Lim of Zamboanga for P5,000, for which an
the case and the findings of facts of both the court a paid except in the case of a life or industrial life application dated the 6th day of July, 1917, has been
quo and respondent appellate court, the Court policy whenever the grace period provision applies made to the Sun Life Assurance Company of Canada.
affirmed the trial court’s findings.) (P.D. 612, as amended otherwise known as the - The above-mentioned life is to be assured in
- The principal cause of the termination of Valenzuela Insurance Code of 1974) accordance with the terms and conditions contained
as General Agent of Philamgen arose from his refusal - This is buttressed by Section 776 of the or inserted by the Company in the policy which may
to share his Delta commission. The records sustain Insurance Code (Presidential Decree No. 612, be granted by it in this particular case for four
the conclusions of the trial court on the apparent bad promulgated on December 18, 1974), which now months only from the date of the application,
faith of the private respondents in terminating the provides that no contract of Insurance by an provided that the Company shall confirm this
General Agency Agreement of petitioners. insurance company is valid and binding unless and agreement by issuing a policy on said application
- It is also evident from the records that the agency until the premium thereof has been paid, when the same shall be submitted to the Head Office
involving petitioner and private respondent is one notwithstanding any agreement to the contrary." in Montreal. Should the Company not issue such a
"coupled with an interest," and, therefore, should not Disposition Petition is GRANTED. CA decision SET policy, then this agreement shall be null and void ab
be freely revocable at the unilateral will of the latter. ASIDE. The decision of the TC REINSTATED with the initio, and the Company shall be held not to have
- The private respondents by the simple expedient of MODIFICATIONS. And that the contractual been on the risk at all, but in such case the amount
terminating the General Agency Agreement relationship between Arturo P. Valenzuela and herein acknowledged shall be returned.
appropriated the entire insurance business of Philippine American General Insurance Company
Valenzuela. With the termination of the General shall be deemed terminated upon the satisfaction of ISSUE
Agency Agreement, Valenzuela would no longer be the judgment as modified. WON the contract of insurance between Luis Lim and
entitled to commission on the renewal of insurance Sun Life Assurance Company of Canada was
policies of clients sourced from his agency. Worse, CHAPTER V – THE POLICY, PARTIES THERETO, perfected
despite the termination of the agency, Philamgen & RIGHTS THEREON
continued to hold Valenzuela jointly and severally HELD
liable with the insured for unpaid premiums. Under DE LIM v. SUN LIFE ASSURANCE COMPANY OF NO.
these circumstances, it is clear that Valenzuela had CANADA - The document it is to be a provisional policy "for
an interest in the continuation of the agency when it 41 PHIL 263 four months only from the date of this application."
was unceremoniously terminated not only because of MALCOLM; November 29, 1920 Immediately following the words fixing the four
the commissions he should continue to receive from months period comes the word "provided" which has
the insurance business he has solicited and procured NATURE the meaning of "if." Otherwise stated, the policy for
INSURANCE Page 43

four months is expressly made subject to the agreed to insure the lives of eligible housing loan Ratio Insured, being the person with whom the
affirmative condition that the company shall confirm mortgagors of DBP. contract was made, is primarily the proper person to
this agreement by issuing a policy on said application - In Nov. 1983, Dr. Wilfredo Leuterio, a physician and bring suit. Subject to some exceptions, insured may
when the same shall be submitted to the head office a housing debtor of DBP applied for membership in thus sue, although the policy is taken wholly or in
in Montreal. To re-enforce the same there follows the the group life insurance plan. In an application form, part for the benefit of another person named or
negative condition - "Should the company not issue Dr. Leuterio answered Qs concerning his health unnamed, and although it is expressly made payable
such a policy, then this agreement shall be null and condition as follows: to another as his interest may appear or otherwise.
void ab initio, and the company shall be held not to Q: Have you ever had, or consulted, a physician for Although a policy issued to a mortgagor is taken out
have been on the risks." Certainly language could a heart condition, high blood pressure, cancer, for the benefit of the mortgagee and is made payable
hardly be used which would more clearly stipulate diabetes, lung, kidney or stomach disorder or any to him, yet the mortgagor may sue thereon in his
that the agreement should not go into effect until the other physical impairment? No. own name, especially where the mortgagee's interest
home office of the company should confirm it by Q: Are you now, to the best of your knowledge, in is less than the full amount recoverable under the
issuing a policy. As we read and understand the so- good health? Yes. policy. (See Sec. 8, Insurance Code)
called provisional policy, it amounts to nothing but - Grepalife issued an insurance coverage of Dr. Reasoning
an acknowledgment on behalf of the company, that Leuterio, to the extent of his DBP mortgage [a] The insured private respondent did not cede to
it has received from the person named therein the indebtedness of P86,200.00. In Aug. 1984, Dr. the mortgagee all his rights or interests in the
sum of money agreed upon as the first year's Leuterio died due to "massive cerebral hemorrhage." insurance, the policy stating that: “In the event of
premium upon a policy to be issued upon the DBP submitted a death claim to Grepalife. Grepalife the debtor's death before his indebtedness with the
application, if the application is accepted by the denied the claim because Dr. Leuterio was not Creditor (DBP) shall have been fully paid, an amount
company. physically healthy when he applied for an insurance. to pay the outstanding indebtedness shall first be
- It is of course a primary rule that a contract of Grepalife insisted that Dr. Leuterio did not disclose he paid to the creditor and the balance of sum assured,
insurance, like other contracts, must be assented to had been suffering from hypertension, which caused if there is any, shall then be paid to the
by both parties either in person or by their agents. his death. Allegedly, such non-disclosure constituted beneficiary/ies designated by the debtor.” When DBP
So long as an application for insurance has not been concealment that justified the denial of the claim. submitted the insurance claim against Grepalife, the
either accepted or rejected, it is merely an offer or - Herein respondent Medarda Leuterio, widow, filed a latter denied payment thereof, interposing the
proposal to make a contract. The contract, to be complaint with RTC against Grepalife for "Specific defense of concealment committed by the insured.
binding from the date of the application must have Performance with Damages." Dr. Mejia, who issued Thereafter, DBP collected the debt from the
been a completed contract, one that leaves nothing the death certificate, testified that Dr. Leuterio mortgagor and took the necessary action of
to be done, nothing to be completed, nothing to be complained of headaches presumably due to high foreclosure on the residential lot of private
passed upon, or determined, before it shall take blood pressure. The inference was not conclusive respondent.
effect. There can be no contract of insurance unless because Dr. Leuterio was not autopsied, hence, other [b] Since a policy of insurance upon life or health
the minds of the parties have met in agreement. Our causes were not ruled out. may pass by transfer, will or succession to any
view is, that a contract of insurance was not here - RTC ruled in favor of respondent widow and against person, whether he has an insurable interest or not,
consummated by the parties. Grepalife. CA sustained the RTC decision. Hence, the and such person may recover it whatever the insured
- The trial court committed no error in sustaining the present petition. might have recovered, the widow of the decedent Dr.
demurrer and dismissing the case. It is to be noted, Leuterio may file the suit against the insurer,
however that counsel for appellee admits the liability Grepalife.
of the company for the return of the first premium to 2. NO
the estate of the deceased. ISSUES Ratio The fraudulent intent on the part of the
1. WON CA erred in holding petitioner liable to DBP insured must be established to entitle the insurer to
GREAT PACIFIC LIFE v. CA (LEUTERIO) as beneficiary in a group life insurance contract from rescind the contract. Misrepresentation as a defense
316 SCRA 677 a complaint filed by the widow of the of the insurer to avoid liability is an affirmative
QUISUMBING; October 13, 1999 decedent/mortgagor defense and the duty to establish such defense by
2. WON CA erred in not finding that Dr. Leuterio satisfactory and convincing evidence rests upon the
NATURE concealed that he had hypertension, which would insurer. In the case at bar, the petitioner failed to
Petition for Review of CA decision vitiate the insurance contract clearly and satisfactorily establish its defense, and is
3. WON CA erred in holding Grepalife liable for therefore liable to pay the proceeds of the insurance.
FACTS P86,200.00 without proof of the actual outstanding Reasoning
- A contract of group life insurance was executed mortgage payable by the mortgagor to DBP [a] The insured, Dr. Leuterio, had answered in his
between petitioner Great Pacific Life Assurance insurance application that he was in good health and
Corporation (hereinafter Grepalife) and Development HELD that he had not consulted a doctor or any of the
Bank of the Philippines (hereinafter DBP). Grepalife 1. NO enumerated ailments, including hypertension; when
INSURANCE Page 44

he died the attending physician had certified in the FACTS premiums are collected by private respondent on all
death certificate that the former died of cerebral - March 19, 1963 - the plaintiff secured temporary its cover notes
hemorrhage, probably secondary to hypertension. insurance from the defendant for its exportation of 2. WON the court of appeals erred in holding that
From this report, petitioner Grepalife refused to pay 1,250,000 board feet of Philippine Lauan and Apitong private respondent was released from liability under
the insurance claim. It alleged that the insured had logs to be shipped from the Diapitan Bay, Quezon to the cover note due to unreasonable delay in giving
concealed the fact that he had hypertension. Okinawa and Tokyo, Japan. The defendant issued on notice of loss because the court disregarded the
[b] Contrary to Grepalife’s allegations, there was no said date Cover Note No. 1010, insuring the said proven fact that private respondent did not promptly
sufficient proof that the insured had suffered from cargo of the plaintiff "Subject to the Terms and and specifically object to the claim on the ground of
hypertension. Aside from the statement of the Conditions of the WORKMEN'S INSURANCE delay in giving notice of loss and, consequently,
insured's widow who was not even sure if the COMPANY, INC. printed Marine Policy form as filed objections on that ground are waived under section
medicines taken by Dr. Leuterio were for with and approved by the Office of the Insurance 84 of the insurance act
hypertension, the appellant had not proven nor Commissioner.
produced any witness who could attest to Dr. - April 2, 1963 - The two (2) regular marine cargo HELD
Leuterio's medical history. policies were issued by the defendant in favor of the 1. NO
[c] Grepalife had failed to establish that there was plaintiff. The total cargo insured under the two Ratio Cover note is issued with a consideration
concealment made by the insured, hence, it cannot marine policies accordingly consisted of 1,395 logs, when, by express stipulation, the cover note is made
refuse payment of the claim. or the equivalent of 1,195,498 bd. ft. subject to the terms and conditions of the marine
3. NO - After the issuance of cover note but before the policies, and the payment of premiums is one of the
- Considering the supervening event that DBP issuance of the two marine policies some of the logs terms of the policies.
foreclosed in 1995 their residential lot, in satisfaction intended to be exported were lost during loading Reasoning
of mortgagor's outstanding loan, the insurance operations in the Diapitan Bay due to bad weather. a. the cover note in question is subject to the terms
proceeds shall inure to the benefit of the heirs of the - April 4, 1963 - The plaintiff informed the defendant and conditions of the marine policies
deceased person or his beneficiaries. Equity dictates about the loss of 'approximately 32 pieces of logs' b. Nature of the Cover Note: The fact that no
that DBP should not unjustly enrich itself at the during loading through a letter. separate premium was paid on the Cover Note before
expense of another. Hence, it cannot collect the - The plaintiff subsequently submitted a 'Claim the loss insured against occurred, does not militate
insurance proceeds, after it already foreclosed on the Statement' demanding payment of the loss under the against the validity of petitioner's contention, for no
mortgage. The proceeds now rightly belong to Dr. second marine cargo policy. such premium could have been paid, since by the
Leuterio's heirs represented by his widow, herein - July 17, 1963 - the defendant requested the First nature of the Cover Note, it did not contain, as all
private respondent. Philippine Adjustment Corporation to inspect the loss Cover Notes do not contain particulars of the
- The Court ruled this issue based on the clear and assess the damage. shipment that would serve as basis for the
provisions of the policy. The mortgagor paid the - August 23, 1963 - the adjuster reported that 'the computation of the premiums. As a logical
premium according to the coverage of his insurance, loss of 30 pieces of logs is not covered by the two consequence, no separate premiums are intended or
which states that: "The policy states that upon policies inasmuch as said policies covered the actual required to be paid on a Cover Note.
receipt of due proof of the Debtor's death during the number of logs loaded on board. But it is covered by c. The petitioner paid in full all the premiums as
terms of this insurance, a death benefit in the Cover Note. called for by the statement issued by private
amount of P86,200.00 shall be paid… In the event of - On January 13, 1964 - the defendant wrote the respondent after the issuance of the two regular
the debtor's death before his indebtedness with the plaintiff denying the latter's claim, on the ground marine insurance policies, thereby leaving no
creditor shall have been fully paid, an amount to pay that defendant's investigation revealed that the account unpaid by petitioner due on the insurance
the outstanding indebtedness shall first be paid to entire shipment of logs covered by the two marines coverage, which must be deemed to include the
the Creditor and the balance of the Sum Assured, if policies were received in good order at their point of Cover Note. If the Note is to be treated as a separate
there is any shall then be paid to the beneficiary/ies destination. It was further stated that the said loss policy instead of integrating it to the regular policies
designated by the debtor." From this, it is clear that may not be considered as covered under cover note subsequently issued, the purpose and function of the
Grepalife is liable and that Dr. Leuterio’s heirs must because the said note had become 'null and void by Cover Note would be set at naught or rendered
get the proceeds. virtue of the issuance of two marine policies. meaningless, for it is in a real sense a contract, not a
Disposition Petition DENIED. CA Decision AFFIRMED - The CFI of Manila ruled in favour of the petitioner. mere application for insurance which is a mere offer.
with modification. - The Court of Appeals reversed the decision of the Had all the logs been lost during the loading
CFI. operations, but after the issuance of the Cover Note,
PACIFIC TIMBER EXPORT CORPORATION v. CA liability on the note would have already arisen even
(WORKMEN’S INSURANCE CO) ISSUES before payment of premium. This is how the cover
112 SCRA 199 1. WON the cover note is null and void for lack of note as a "binder" should legally operate; otherwise,
DE CASTRO; February 25, 1982 valuable consideration because no separate it would serve no practical purpose in the realm of
commerce, and is supported by the doctrine that
INSURANCE Page 45

where a policy is delivered without requiring or partial, it is understood that the amount of the
payment of the premium, the presumption is that a HELD loss shall be subject to appraisal and the liability of
credit was intended and policy is valid. 1. NO the company, if established, shall be limited to the
2. NO - Summons was served through its vice-president. actual loss, subject to the applicable terms,
- The private respondent company never raised this There were even several extensions to the original conditions, warranties and clauses of this Policy,
ground in the proceedings. It must be because it did period to answer. As a consequence, the TC, on and in no case shall exceed the amount of the
not find any delay, as this Court fails to find a real motion of the private respondent filed declared the policy."
and substantial sign thereof. But even on the petitioner in default. This was done almost one - As defined in the aforestated provision, which is
assumption that there was delay, this Court is month later. Even so, the petitioner made no move now Sec60 of the Insurance Code, "an open policy is
satisfied and convinced that as expressly provided by at all for two months thereafter. It was only more one in which the value of the thing insured is not
law, waiver can successfully be raised against private than one month after the judgment of default was agreed upon but is left to be ascertained in case of
respondent. Thus Section 84 of the Insurance Act rendered by the TC that it filed a motion to lift the loss.".
provides: order of default and vacate the judgment by default. - The actual loss has been ascertained in this case
"Section 84. - Delay in the presentation to an - There is a pattern of inexcusable neglect. and the Court will respect such factual determination
insurer of notice or proof of loss is waived if 2. The policy is an open policy which means that the in the absence of proof that it was arrived at
caused by any act of his or if he omits to take actual loss, as determined, will represent the total arbitrarily. There is no such showing. Hence, applying
objection promptly and specifically upon that indemnity due the insured from the insurer except the open policy clause as expressly agreed upon by
ground." only that the total indemnity shall not exceed the the parties in their contract, we hold that the private
- From what has been said, We find duly face value of the policy. respondent is entitled to the payment of indemnity
substantiated petitioner's assignments of error. - The petitioner argues that since at the time of the under the said contract in the total amount of
Disposition The appealed decision is set aside and fire the building insured was worth P5,800,000.00, P508,867.00.
the decision of the Court of First Instance is the private respondent should be considered its own - The refusal of its vice-president to receive the
reinstated in toto with the affirmance of this Court. insurer for the difference between that amount and private respondent's complaint, as reported in the
the face value of the policy and should share pro rata sheriff's return, was the first indication of the
DEVELOPMENT INSURANCE v. IAC (PHIL UNION in the loss sustained. Accordingly, the private petitioner's intention to prolong this case and
REALTY DEVELOPMENT CORP) respondent is entitled to an indemnity of only postpone the discharge of its obligation to the private
143 SCRA 62 P67,629.31, the rest of the loss to be shouldered by respondent under this agreement. That intention was
CRUZ; July 16, 1986 it alone. The petitioner cites Condition 17 of the revealed further in its subsequent acts ---- or
policy, which provides: inaction ---- which indeed enabled it to avoid
FACTS "If the property hereby insured shall, at the payment for more than five years from the filing of
- A fire occurred in the building of the private breaking out of any fire, be collectively of greater the claim against it in 1980.
respondent and it sued for recovery of damages from value than the sum insured thereon then the Disposition The appealed decision is affirmed in
the petitioner on the basis of an insurance contract insured shall be considered as being his own full, with costs against the petitioner.
between them. The petitioner allegedly failed to insurer for the difference, and shall bear a ratable
answer on time and was declared in default by TC. A proportion of the loss accordingly. Every item, if HARDING v. COMMERCIAL UNION ASSURANCE
judgment of default was rendered on the strength of more than one, of the policy shall be separately 38 PHIL 464
the evidence submitted ex parte by the private subject to this condition." FISHER; August 10, 1918
respondent, which was allowed full recovery of its - However, there is no evidence on record that the
claimed damages. building was worth P5,800,000.00 at the time of the FACTS
- On learning of this decision, the petitioner moved loss. On the contrary, the building was insured at - Mrs. Harding was the owner of a Studebaker
to lift the order of default, invoking excusable P2,500,000.00, and this must be considered, by automobile; in consideration of the payment to the
neglect, and to vacate the judgment by default. Its agreement, the actual value of the property insured defendant of the premium of P150, by said plaintiff,
motion was denied. on the day the fire occurred. This valuation becomes Mrs. Henry E. Harding, with the consent of her
- On appeal, IAC affirmed the TC decision in toto. even more believable if it is remembered that at the husband, the defendant by its duly authorized agent,
time the building was burned it was still under Smith, Bell & Company (limited), made its policy of
ISSUE construction and not yet completed. insurance in writing upon said automobile was set
1. WON default of petitioner is based on excusable - The Court notes that the policy in this case is an forth in said policy to be P3,000 that the value of
neglect open policy and is subject to the express condition said automobile was set forth in said policy to be
2. What is the amount of indemnity due to the that: P3,000; that on March 24, 1916, said automobile
private respondent under its insurance contract? "Open Policy. was totally destroyed by fire; that the loss thereby to
WON CFI was correct in interpreting the contract This is an open policy as defined in Sec57 of the plaintiffs was the sum of P3,000.
Insurance Act. In the event of loss, whether total
INSURANCE Page 46

- The defendant’s version is that by request of Mrs. defendant charged the said plaintiff Mrs. Henry E. HELD
Harding, it issued the policy of insurance on an Harding as premium on said policy the sum of P150, 1. NO
automobile alleged by the said plaintiff to be her or 5 per cent of the then estimated value of P3,000. - Article 1334 of the Civil Code which provides that
property. It was made by means of a proposal in - The "Schedule" in said policy of insurance describes "All gifts between spouses during the marriage shall
writing signed and delivered by said plaintiff to the the automobile here in question, and provides in part be void. Moderate gifts which the spouses bestow on
defendant, guaranteeing the truth of the statements of follows: each other on festive days of the family are not
contained therein which said proposal is referred to "That during the period above set forth and during included in this rule."
in the said policy of insurance made a part thereof; any period for which the company may agree to - Even assuming that defendant might have invoked
that certain of the statements and representations renew this policy the company will subject to the article 1334 as a defense, the burden would be upon
contained in said proposal and warranted by said exception and conditions contained herein or it to show that the gift in question does not fall
plaintiff to be true, to wit: (a) the price paid by the endorsed hereon indemnify the insured against loss within the exception therein established. We cannot
proposer for the said automobile; (b) the value of of or damage to any motor car described in the say, as a matter of law, that the gift of an automobile
said automobile at the time of the execution and schedule hereto (including accessories) by whatever by a husband to his wife is not a moderate one.
delivery of the said proposal and (c) the ownership of cause such loss or damage may be occasioned and Whether it is or is not would depend upon the
said automobile, were false and known to be false by will further indemnify the insured up to the value of circumstances of the parties, as to which nothing is
the said plaintiff at the time of signing and delivering the car or P3,000 whichever is the greater against disclosed by the record.
the said proposal and were made for the purpose of any claim at common law made by any person (not - We are of the opinion that it would be unfair to hold
misleading and deceiving the defendant, and being a person in the said motor car nor in the the policy void simply because the outlay
inducing the defendant, relying upon the warranties, insured's service) for loss of life or for accidental represented by the automobile was made by the
statements, and representations contained in the bodily injury or damage to property caused by the plaintiff's husband and not by his wife, to whom he
said proposal and believing the same to be true, said motor car including law costs payable in had given the automobile. It cannot be assumed that
issued the said policy of insurance. connection with such claim when incurred with the defendant should not have issued the policy unless it
- The evidence shows that Hermanos, the Manila consent of the company." were strictly true that the price representing the cost
agents for the Studebaker automobile, sold the - On March 24, 1916, the said automobile was totally of the machine had been paid by the insured and by
automobile to Canson for P3,200 (testimony of Mr. destroyed by fire, and that the iron and steel no other person ? that it would no event insure an
Diehl); who sold the said automobile to Henry portions of said automobile which did not burn were automobile acquired by gift, inheritance, exchange,
Harding for the sum of P1,500. Harding sold the said taken into the possession of the defendant by and or any other title not requiring the owner to make a
automobile to J. Brannigan for the sum of P2,000 through its agent Smith, Bell & Company (limited), specific cash outlay for its acquisition.
who sold the said automobile Henry Harding for the and sold by it for a small sum, which had never been 2. NO
sum of P2,800; Henry Harding gave the said tendered to the plaintiff prior to the trial of this case, - It has not been shown by the evidence that the
automobile to his wife as a present; that said but in open court during the trial the sum of P10 as statement was false; on the contrary we believe that
automobile was repaired and repainted at the Luneta the proceeds of such sale was tendered to plaintiff it shows that the automobile had in fact cost more
Garage at a cost of some P900; that while the said and refused. than the amount mentioned. The court below found,
automobile was at the Luneta Garage; the latter - Trial judge decided that there was no proof of fraud and the evidence shows, that the automobile was
solicited of Mrs. Harding the insurance of said on the part of plaintiff in her statement of the value bought by plaintiff's husband a few weeks before the
automobile by the Company; that a proposal was of the automobile, or with respect to its ownership; issuance of the policy in question for the sum of
filled out by the said agent and signed by the plaintiff that she had an insurable interest therein; and that P2,800, and that between that time and the issuance
Mrs. Henry E. Harding, and in said proposal under defendant, having agreed to the estimated value, of the policy some P900 was spent upon it in repairs
the heading "Price paid by proposer," is the amount P3,000, and having insured the automobile for that and repainting.
of "3,500" and under another heading "Present amount, upon the basis of which the premium was - The witness Server, an expert automobile
value" is the amount of "3,000". paid, is bound by it and must pay the loss in mechanic, testified that the automobile was
- After the said proposal was made a representative accordance with the stipulated insured value. practically as good as new at the time the insurance
of the Manila agent of defendant went to the Luneta was effected. The form of proposal upon which the
Garage and examined said automobile and Mr. ISSUE policy was issued does not call for a statement
Server, the General Manager of the Luneta Garage, 1. WON Mrs. Harding was not the owner of the regarding the value of the automobile at the time of
an experienced automobile mechanic, testified that automobile at the time of the issuance of the policy, its acquisition by the applicant for the insurance, but
at the time this automobile was insured it was worth and, therefore, had no insurable interest in it merely a statement of its cost. The amount stated
about P3,000, and the defendant, by and through its 2. WON the statement regarding the cost of the was less than the actual outlay which the automobile
said agent Smith, Bell & Company (limited), automobile was a warranty, that the statement was represented to Mr. Harding, including repairs, when
thereafter issued a policy of insurance upon proposal false, and that, therefore, the policy never attached the insurance policy was issued.
in which policy the said automobile was described as to the risk - The court below found and the evidence shows,
of the "present value" of P3,000 and the said without dispute, that the proposal upon which the
INSURANCE Page 47

policy in question was issued was made out by to fully pay its accounts, Steamship Mutual refused of the Insurance Code enumerates the coverage of
defendant's agent by whom the insurance was to renew the coverage. marine insurance.
solicited, and that appellee simply signed the same. - Steamship Mutual thereafter filed a case against - Relatedly, a mutual insurance company is a
It also appears that an examiner employed by the White Gold for collection of sum of money to recover cooperative enterprise where the members are both
defendant made an inspection of the automobile the latter’s unpaid balance. White Gold on the other the insurer and insured. In it, the members all
before the acceptance of the risk, and that the sum hand, filed a complaint before the Insurance contribute, by a system of premiums or
after this examination. The trial court found that Mrs. Commission claiming that Steamship Mutual violated assessments, to the creation of a fund from which all
Harding, in fixing the value of the automobile at Sections 186 and 187 of the Insurance Code, while losses and liabilities are paid, and where the profits
P3,000, acted upon information given her by her Pioneer violated Sections 299, 300 and 301 in are divided among themselves, in proportion to their
husband and by Mr. Server, the manager of the relation to Sections 302 and 303, thereof. interest. Additionally, mutual insurance associations,
Luneta Garage. She merely repeated the information - The Insurance Commission dismissed the or clubs, provide three types of coverage, namely,
which had been given her by her husband, and at the complaint. It said that there was no need for protection and indemnity, war risks, and defense
same time disclosed to defendant's agent the source Steamship Mutual to secure a license because it was costs.
of her information. There is no evidence to sustain not engaged in the insurance business. It explained - A P & I Club is “a form of insurance against third
the contention that this communication was made in that Steamship Mutual was a Protection and party liability, where the third party is anyone other
bad faith. We do not think that the facts stated in Indemnity Club (P & I Club). Likewise, Pioneer need than the P & I Club and the members.” By definition
the proposal can be held as a warranty of the not obtain another license as insurance agent and/or then, Steamship Mutual as a P & I Club is a mutual
insured, even if it should have been shown that they a broker for Steamship Mutual because Steamship insurance association engaged in the marine
were incorrect in the absence of proof of willful Mutual was not engaged in the insurance business. insurance business.
misstatement. Under such circumstance, the Moreover, Pioneer was already licensed, hence, a - The records reveal Steamship Mutual is doing
proposal is to be regarded as the act of the insurer separate license solely as agent/broker of Steamship business in the country albeit without the requisite
and not of the insured. Mutual was already superfluous. certificate of authority mandated by Section 187 of
Disposition Plaintiff was the owner of the - The Court of Appeals affirmed the decision of the the Insurance Code. It maintains a resident agent in
automobile in question and had an insurable interest Insurance Commissioner. In its decision, the the Philippines to solicit insurance and to collect
therein; that there was no fraud on her part in appellate court distinguished between P & I Clubs payments in its behalf. We note that Steamship
procuring the insurance; that the valuation of the vis-à-vis conventional insurance. The appellate court Mutual even renewed its P & I Club cover until it was
automobile, for the purposes of the insurance, is also held that Pioneer merely acted as a collection cancelled due to non-payment of the calls. Thus, to
binding upon the defendant corporation, and that the agent of Steamship Mutual. continue doing business here, Steamship Mutual or
judgment of the court below is, therefore, correct through its agent Pioneer, must secure a license from
and must be affirmed, with interest, the costs of this ISSUES the Insurance Commission.
appeal to be paid by the appellant. 1. WON Steamship Mutual, a P & I Club, is engaged - Since a contract of insurance involves public
in the insurance business in the Philippines interest, regulation by the State is necessary. Thus,
WHITE GOLD MARINE SERVICES v. PIONEER 2. WON Pioneer needs a license as an insurance no insurer or insurance company is allowed to
INSURANCE agent/broker for Steamship Mutual engage in the insurance business without a license or
464 SCRA 448 a certificate of authority from the Insurance
QUISUMBING; July 28, 2005 HELD Commission.
1. YES 2. YES
NATURE - The test to determine if a contract is an insurance - SEC. 299 . . .
This petition for review assails the Decision of the contract or not, depends on the nature of the - No person shall act as an insurance agent or as an
Court of Appeals, affirming the Decision of the promise, the act required to be performed, and the insurance broker in the solicitation or procurement of
Insurance Commission. Both decisions held that exact nature of the agreement in the light of the applications for insurance, or receive for services in
there was no violation of the Insurance Code and the occurrence, contingency, or circumstances under obtaining insurance, any commission or other
respondents do not need license as insurer and which the performance becomes requisite. It is not compensation from any insurance company doing
insurance agent/broker. by what it is called. Basically, an insurance contract business in the Philippines or any agent thereof,
is a contract of indemnity. In it, one undertakes for without first procuring a license so to act from the
FACTS a consideration to indemnify another against loss, Commissioner, which must be renewed annually on
- White Gold procured a protection and indemnity damage or liability arising from an unknown or the first day of January, or within six months
coverage for its vessels from Steamship Mutual contingent event. thereafter.
through Pioneer Insurance. Subsequently, White - In particular, a marine insurance undertakes to Disposition The petition is PARTIALLY GRANTED.
Gold was issued a Certificate of Entry and indemnify the assured against marine losses, such as The Decision dated July 30, 2002 of the Court of
Acceptance. Pioneer also issued receipts evidencing the losses incident to a marine adventure. Section 99 Appeals affirming the Decision dated May 3, 2000 of
payments for the coverage. When White Gold failed the Insurance Commission is hereby REVERSED AND
INSURANCE Page 48

SET ASIDE. The Steamship Mutual Underwriting - February 27, 1942 or during the Japanese military
Association (Bermuda) Ltd., and Pioneer Insurance HELD occupation - building and insured merchandise were
and Surety Corporation are ORDERED to obtain 1. NO burned. In due time the Huenefeld Co submitted to
licenses and to secure proper authorizations to do - Pandiman is not an insurance agent as defined by the Filipinas Cia its claim under the policy. The
business as insurer and insurance agent, Section 3007 of the Insurance Code. In this case, salvage goods were sold at public auction and, after
respectively. The petitioner’s prayer for the there was no showing that Pndiman in fact deducting their value, the total loss suffered by the
revocation of Pioneer’s Certificate of Authority and negotiated the insurance contract between Sun respondent was fixed at P92,650.
removal of its directors and officers, is DENIED. Richie Five and the insurer OMMIAL. Even, if - Filipinas Cia refused to pay the claim on the ground
Pandiman were an agent, payment for claims arising that the policy in favor of the respondent had ceased
PANDIMAN v. MARINE MANNING MNGT CORP. from peril insured against, to which the insurer is to be in force on the date the United States declared
460 SCRA 418 liable, is definitely not one of the liabilities of an war against Germany, the respondent Corporation
GARCIA; June 21, 2005 insurance agent. Thus, there is no legal basis (though organized under and by virtue of the laws of
whatsoever for holding petitioner solidarily liable with the Philippines) being controlled by the German
NATURE insurer OMMIAL for the widow’s claim for death subjects and the Filipinas Cia being a company under
Petition for certiorari to review CA decision benefits. Also, Pandiman is not a party to the American jurisdiction when said policy was issued on
insurance contract and hence under Article 1311 of October 1, 1941. Filipinas Cia, however, in pursuance
FACTS the Civil Code, it is not liable for the obligation of the order of the Director of Bureau of Financing,
- Benito Singhid was hired as chief cook on board the arising out of the insurance contract. Philippine Executive Commission, dated April 9,
vessel MV Sun Richie Five for a term of one year by 2. NO 1943, paid to the Huenefeld Co the sum of P92,650
Fullwin Maritime Limited through its Philippine agent, - Fullwin, as Benito’s principal employer is liable on April 19, 1943.
Marine Manning and Management Corporation. While under the employment contract. Marine is also - August 6, 1946 – action filed in CFI Manila to
the said vessel was on its way to Shanghai from Ho bound by its undertaking pursuant to the Rules and recover from the Huenefeld Co the sum of P92,650
Chih Minh City, Benito suffered a heart attack and Regulations Governing Overseas Employment that “it above mentioned. The theory of the Filipinas Cia is
subsequently died on June 24, 1997. shall assume joint and solidary liability with the that the insured merchandise were burned up after
- Apparently, the vessel and the crew were insured employer for all the claims and liabilities which may the policy issued in 1941 in favor of Huenefeld Co
with Ocean Marine Mutual Insurance Association arise in connection with the implementation of the has ceased to be effective because of the outbreak of
Limited (OMMIAL), a Protective and Indemnity Club contract, including but not limited to the payment of the war between the United States and Germany on
of which Sun Richie Five Bulkers S.A. is a member. wages, heath and disability compensation and December 10, 1941, and that the payment made by
Pandiman Philippines, the petitioner, is the local repatriation”. In other words, both Fullwin and the Filipinas Cia to Huenefeld Co during the Japanese
correspondent of OMMIAL. Marine should be held liable for whatever death military occupation was under pressure.
- Benito’s widow, Rosita, filed a claim for death benefits the widow of Benito may be entitled to. - CFI: dismissed the action without pronouncement
benefits with Marine which referred her to Pandiman. Disposition The petition is granted and the CA as to costs.
After her submission of the required documentation, decision is reversed and set aside. - CA: CFI judgment affirmed, with costs. The case is
Pandiman recommended payment of the death now before us on appeal by certiorari from the
benefits amounting to $79,000. However, payment FILIPINAS COMPANIA DE SEGUROS V decision of the Court of Appeals.
has not been made. CHRISTERN, HUENEFELD AND CO INC
- Rosita filed a complaint with the Labor Arbiter 89 PHIL 54 ISSUE
naming Marine, Pandiman, OMMIAL, and Fullwin as PARAS; May 25, 1951 WON the policy in question became null and void
respondents. The Arbiter ordered all the upon the declaration of war between United States
respondents, except Pandiman, to jointly and FACTS and Germany
severally pay the widow the death benefits plus legal - October 1, 1941 - Christern Huenefeld, & Co., Inc.,
fees. The NLRC, on appeal by Marine, limited the after payment of corresponding premium, obtained HELD
liable parties to Pandiman and OMMIAL but from the Filipinas Cia. de Seguros a fire policy in the YES
maintained the money award. The CA sustained the sum of P1000,000, covering merchandise contained Ratio The Philippine Insurance Law (Act No. 2427,
decision of the NLRC. Hence this appeal. in No. 711 Roman Street, Binondo Manila. as amended,) in section 8, provides that "anyone
except a public enemy may be insured." It stands to
reason that an insurance policy ceases to be
ISSUE 7
Section 300. Any person who for compensation solicits or obtains on
allowable as soon as an insured becomes a public
1. WON Pandiman may be held liable for the death behalf of any insurance company transmits for a person other than enemy.
benefits himself an application for a policy or contract of insurance to or from > Effect of war, generally. - All intercourse
such company or offers or assumes to act in the negotiating of such
2. WON Marine and its foreign principal, Fullwin, insurance shall be an insurance agent within the intent of this section
between citizens of belligerent powers which is
should be absolved from the death claim liabilities and shall thereby become liable to all the duties, requirements, inconsistent with a state of war is prohibited by
liabilities, and penalties to which an insurance agent is subject.
INSURANCE Page 49

the law of nations. Such prohibition includes all and American cases relied upon by the Court of
negotiations, commerce, or trading with the Appeals have lost their force in view of the latest NATURE
enemy; all acts which will increase, or tend to decision of the Supreme Court of the United States in Appeal from judgment of RTC.
increase, its income or resources; all acts of Clark vs. Uebersee Finanz Korporation, decided on
voluntary submission to it; or receiving its December 8, 1947, in which the controls test has FACTS
protection; also all acts concerning the been adopted. In "Enemy Corporation" by Martin - Buenaventura Ebrado obtained a whole-life
transmission of money or goods; and all contracts Domke, a paper presented to the Second insurance policy from Insular, for P5,882.00 with a
relating thereto are thereby nullified. It further International Conference of the Legal Profession held rider for accidental death benefits for the same
prohibits insurance upon trade with or by the at the Hague (Netherlands) in August. 1948 also amount. He designated Carponia Ebrado as the
enemy, upon the life or lives of aliens engaged in discussed this dilemma revocable beneficiary, referring to her as the wife.
service with the enemy; this for the reason that > In Clark vs. Uebersee Finanz Korporation, A. G., - Afterwards, he died as a result of an accident when
the subjects of one country cannot be permitted to dealing with a Swiss corporation allegedly he was hit by a falling branch of a tree. Carponia
lend their assistance to protect by insurance the controlled by German interest, the Court: "The filed a claim for the proceeds as the designated
commerce or property of belligerent, alien property of all foreign interest was placed within beneficiary in the policy, although she admits that
subjects, or to do anything detrimental too their the reach of the vesting power (of the Alien she and Buenaventura were merely living as husband
country's interest. The purpose of war is to cripple Property Custodian) not to appropriate friendly or and wife without the benefit of marriage. The legal
the power and exhaust the resources of the neutral assets but to reach enemy interest which wife, Pascuala Vda De Ebrado, also filed her claim as
enemy, and it is inconsistent that one country masqueraded under those innocent fronts. . . . The the widow of the deceased.
should destroy its enemy's property and repay in power of seizure and vesting was extended to all - Insular then filed an interpleader in court (CFI
insurance the value of what has been so property of any foreign country or national so that Rizal) to determine to whom the proceeds should be
destroyed, or that it should in such manner no innocent appearing device could become a paid. CFI declared that Carponia was disqualified
increase the resources of the enemy, or render it Trojan horse." from becoming beneficiary of the insured and
aid, and the commencement of war determines, - The respondent having become an enemy directing the Insular to pay the proceeds to the
for like reasons, all trading intercourse with the corporation on December 10, 1941, the insurance estate of Buenaventura.
enemy, which prior thereto may have been lawful. policy issued in its favor on October 1, 1941, by the
All individuals therefore, who compose the petitioner (a Philippine corporation) had ceased to be ISSUE
belligerent powers, exist, as to each other, in a valid and enforcible, and since the insured goods 1. WON a common-law wife named as beneficiary in
state of utter exclusion, and are public enemies. (6 were burned after December 10, 1941, and during the insurance policy of a legally married man claim
Couch, Cyc. of Ins. Law, pp. 5352-5353.) the war, the respondent was not entitled to any the proceeds of the same
> In the case of an ordinary fire policy, which indemnity under said policy from the petitioner.
grants insurance only from year, or for some other However, elementary rules of justice (in the absence HELD
specified term it is plain that when the parties of specific provision in the Insurance Law) require 1. NO
become alien enemies, the contractual tie is that the premium paid by the respondent for the Ratio The prohibition that husband and wife cannot
broken and the contractual rights of the parties, so period covered by its policy from December 11, donate to each other applies to common-law
far as not vested. lost. (Vance, the Law on 1941, should be returned by the petitioner. relationships. As the appointment of a beneficiary in
Insurance, Sec. 44, p. 112.) Disposition the appealed decision is hereby insurance may be considered a donation, one cannot
Reasoning reversed and the respondent corporation is ordered name as beneficiary his common-law wife.
- The Court of Appeals overruled the contention of to pay to the petitioner the sum of P77,208.33, Reasoning
the petitioner that the respondent corporation Philippine currency, less the amount of the premium, - It is quite unfortunate that the Insurance Code
became an enemy when the United States declared in Philippine currency, that should be returned by the does not contain any specific provision grossly
war against Germany, relying on English and petitioner for the unexpired term of the policy in resolutory of the prime question at hand.
American cases which held that a corporation is a question, beginning December 11, 1941. - Rather, general rules of civil law should be applied
citizen of the country or state by and under the laws to resolve the issue. Art.2011, CC states: “The
of which it was created or organized. It rejected the contract of insurance is governed by special laws.
theory that nationality of private corporation is Matters not expressly provided for in such special
determined by the character or citizenship of its laws shall be regulated by this Code.” Thus, when not
controlling stockholders. otherwise specifically provided for by the Insurance
- There is no question that majority of the Law, the contract of life insurance is governed by the
stockholders of the respondent corporation were general rules of the civil law regulating contracts.
German subjects. Therefore, Huenefeld Co became INSULAR LIFE ASSURANCE CO. v. EBRADO - Also, Art.2012 “any person who is forbidden from
an enemy corporation upon the outbreak of the war 80 SCRA 181 receiving any donation under Article 739 cannot be
between the United States and Germany. The English MARTIN; October 28, 1977 named beneficiary of a life insurance policy by the
INSURANCE Page 50

person who cannot make a donation to him.” the first with herein respondent Rosario Diaz, WON GSIS was correct in awarding half of the
Common-law spouses are, definitely, barred from solemnized in the parish church of San Nicolas de retirement benefit of the deceased to Rosario Diaz,
receiving donations from each other. Tolentino, Surigao, Surigao, on July 15, 1937, out of the first wife, notwithstanding the fact that the
- Art.739, CC: The following donations shall be void: which marriage were born two children, namely, Jose petitioners were named as beneficiaries of the life
1. Those made between persons who were guilty of Consuegra, Jr. and Pedro Consuegra, but both insurance
adultery or concubinage at the time of donation; predeceased their father; and the second, which was
- In essence, a life insurance policy is no different contracted in good faith while the first marriage was HELD
from a civil donation insofar as the beneficiary is subsisting, with herein petitioner Basilia Berdin, on YES
concerned. Both are founded upon the same May 1, 1957 in the same parish and municipality, out - The GSIS offers two separate and distinct systems
consideration: liberality. A beneficiary is like a donee, of which marriage were born seven children, namely, of benefits to its members, one is the life insurance
because from the premiums of the policy which the Juliana, Pacita, Maria Lourdes, Jose, Rodrigo, Lenida and the other is the retirement insurance. These two
insured pays out of liberality, the beneficiary will and Luz, all surnamed Consuegra. distinct systems of benefits are paid out from two
receive the proceeds or profits of said insurance. As - Being a member of the Government Service distinct and separate funds that are maintained by
a consequence, the proscription in Art.739 CC should Insurance System (GSIS, for short) when Consuegra the GSIS. Thus, it doesn’t necessarily mean that the
equally operate in life insurance contracts. The died on September 26, 1965, the proceeds of his life beneficiaries in the life insurance are also the
mandate of Art.2012 cannot be laid aside: any insurance under policy No. 601801 were paid by the beneficiaries in the retirement insurance.
person who cannot receive a donation cannot be GSIS to petitioner Basilia Berdin and her children - Consuegra started in the government service
named as beneficiary in the life insurance policy of who were the beneficiaries named in the policy. sometime during the early part of 1943, or before
the person who cannot make the donation. - However, Consuegra did not designate any 1943. In 1943 Com. Act 186 was not yet amended,
- Policy considerations and dictates of morality rightly beneficiary who would receive the retirement and the only benefits then provided for in said Com.
justify the institution of a barrier between common- insurance benefits due to him. Respondent Rosario Act 186 were those that proceed from a life
law spouses in regard to property relations since Diaz, the widow by the first marriage, filed a claim insurance. Upon entering the government service
such relationship ultimately encroaches upon the with the GSIS asking that the retirement insurance Consuegra became a compulsory member of the
nuptial and filial rights of the legitimate family. There benefits be paid to her as the only legal heir of GSIS, being automatically insured on his life,
is every reason to hold that the bar in donations Consuegra, considering that the deceased did not pursuant to the provisions of Com. Act 186 which
between legitimate spouses and those between designate any beneficiary with respect to his was in force at the time. During 1943 the operation
illegitimate ones should be enforced in life insurance retirement insurance benefits. Petitioner Basilia of the Government Service Insurance System was
policies since the same are based on similar Berdin and her children, likewise, filed a similar claim suspended because of the war, and the operation
consideration. with the GSIS, asserting that being the beneficiaries was resumed sometime in 1946. When Consuegra
- So long as marriage remains the threshold of named in the life insurance policy of Consuegra, they designated his beneficiaries in his life insurance he
family laws, reason and morality dictate that the are the only ones entitled to receive the retirement could not have intended those beneficiaries of his life
impediments imposed upon married couple should insurance benefits due the deceased Consuegra. insurance as also the beneficiaries of his retirement
likewise be imposed upon extra-marital relationship. Resolving the conflicting claims, the GSIS ruled that insurance because the provisions on retirement
If legitimate relationship is circumscribed by these the legal heirs of the late Jose Consuegra were insurance under the GSIS came about only when
legal disabilities, with more reason should an illicit Rosario Diaz, his widow by his first marriage who is Com. Act 186 was amended by Rep. Act 660 on June
relationship be restricted by these disabilities. entitled to one-half, or 8/16, of the retirement 16, 1951. Hence, it cannot be said that because
Disposition Decision AFFIRMED. insurance benefits, on the one hand; and Basilia herein appellants were designated beneficiaries in
CONSUEGRA v. GSIS Berdin, his widow by the second marriage and their Consuegra's life insurance they automatically
37 SCRA 315 seven children, on the other hand, who are entitled became the beneficiaries also of his retirement
ZALDIVAR; January 30, 1971 to the remaining one-half, or 8/16, each of them to insurance.
receive an equal share of 1/16. - The provisions of subsection (b) of Section 11 of
NATURE - Dissatisfied with the foregoing ruling and Commonwealth Act 186, as amended by Rep. Act
Appeal from the decision of the Court of First apportionment made by the GSIS, Basilia Berdin and 660, clearly indicate that there is need for the
Instance of Surigao del Norte awarding the 8/16 part her children filed on October 10, 1966 a petition for employee to file an application for retirement
of the proceeds of the deceased Consuegra’s mandamus with preliminary injunction in the Court of insurance benefits when he becomes a member of
retirement benefits to Rosario Diaz. First Instance of Surigao. the GSIS, and he should state in his application the
- The CFI of Surigao ruled in favor of respondent beneficiary of his retirement insurance. Hence, the
FACTS Rosario Diaz and upheld the ruling of GSIS in all beneficiary named in the life insurance does not
- The late Jose Consuegra, at the time of his death, aspect. Thus, Basilia Berdin and her children automatically become the beneficiary in the
was employed as a shop foreman of the office of the appealed said decision to the Supreme Court. retirement insurance unless the same beneficiary in
District Engineer in the province of Surigao del Norte. the life insurance is so designated in the application
In his lifetime, Consuegra contracted two marriages, ISSUE for retirement insurance.
INSURANCE Page 51

- In the case of the proceeds of a life insurance, the entitled to receive the death benefits payable for the ART. 2012. Any person who is forbidden from
same are paid to whoever is named the beneficiary death of Petronilo Davac. receiving any donation under Article 739 cannot be
in the life insurance policy. As in the case of a life named beneficiary of a life insurance policy by the
insurance provided for in the Insurance Act, the ISSUES person who cannot make any donation to him
beneficiary in a life insurance under the GSIS may 1. WON the Social Security Commission Candelaria according to said article.
not necessarily be an heir of the insured. The Davac is entitled to receive the death benefits ART. 739. The following donations shall be void:
insured in a life insurance may designate any 2. WON a beneficiary under the Social Security (1) Those made between persons who were guilty
person as beneficiary unless disqualified to be System partakes of the nature of a beneficiary in a of adultery or concubinage at the time of the
so under the provisions of the Civil Code. And in life insurance policy and, therefore the designation donation; (the court did not decide whether this
the absence of any beneficiary named in the life made in the person DAVAC as bigamous wife is null partakes the nature of a life insurance policy)
insurance policy, the proceeds of the insurance will and void, because it contravenes the provisions of 3. NO
go to the estate of the insured. the Civil Code - The benefit receivable under the Act is in the
- On the other hand, the beneficiary of the 3. WON the benefits accruing from membership with nature of a special privilege or an arrangement
retirement insurance can only claim the proceeds of SSS forms part of the conjugal property thus the secured by the law pursuant to the policy of the
the retirement insurance if the employee dies before resolution deprives the lawful wife of her share in the State to provide social security to the workingmen.
retirement. If the employee failed or overlooked to conjugal property as well as of her own and her The amounts that may thus be received cannot be
state the beneficiary of his retirement insurance, the child's legitime in the inheritance considered as property earned by the member during
retirement benefits will accrue to his estate and will his lifetime. His contribution to the fund constitutes
be given to his legal heirs in accordance with law, as HELD only an insignificant portion thereof. Then, the
in the case of a life insurance if no beneficiary is 1. YES benefits are specifically declared not transferable,
named in the insurance policy. - Section 13, RA1161 provides that the beneficiary and exempted from tax, legal processes, and lien.
Disposition Petition Denied. It is Our view, "as recorded" by the employee's employer is the one Furthermore, in the settlement of claims thereunder,
therefore, that the respondent GSIS had correctly entitled to the death benefits. the procedure to be observed is governed not by the
acted when it ruled that the proceeds of the - Section 13, Republic Act No. 1161, as amended by general provisions of law, but by rules and
retirement insurance of the late Jose Consuegra Republic Act No. 1792, in force at the time of regulations promulgated by the Commission. Thus, if
should be divided equally between his first living wife Petronilo Davac's death provides: Upon the covered the money is payable to the estate of a deceased
Rosario Diaz, on the one hand, and his second wife employee's death or total and permanent disability member, it is the Commission, not the probate or
Basilia Berdin and his children by her. under such conditions as the Commission may regular court that determines the person or persons
define, before becoming eligible for retirement and if to whom it is payable.
SSS v. DAVAC either such death or disability is not compensable - They are disbursed from a public special fund
17 SCRA 863 under the Workmen's Compensation Act, he or. in created by Congress.The sources of this special fund
BARRERA: July 30, 1966 case of his death, his beneficiaries, as recorded by are the covered employee's contribution (equal to 2-
his employer shall be entitled to the following 1/2 per cent of the employee's monthly
NATURE benefit: compensation) ; the employer's 'Contribution
APPEAL from a resolution Of the Social Security - In Tecson vs. Social Security System. Section 13 (equivalent to 3-1/2 per cent of the monthly
Commission. was construed:"it may be true that the purpose of compensation of the covered employee) ;and the
the coverage under the Social Security System is Government contribution which consists in yearly
FACTS protection of the employee as well as of his family, appropriation of public funds to assure the
- Petronilo Davac, became a member of the Social but this purpose or intention of the law cannot be maintenance of an adequate working balance of the
Security System (SSS for short) on September 1, enforced to the extent of contradicting the very funds of the System. Additionally, Section 21 of the
1957. In the Member's Record he designated provisions of said law contained in Section 13, Social Security Actprovides that the benefits
respondent, Candelaria Davac as his beneficiary and thereof” prescribed in this Act shall not be diminished and the
indicated his relationship to her as that of "wife". - When the provisions of a law are clear and explicit, Government of the Republic of the Philippines
- He died on April 5, 1959. It appears that the the courts can do nothing but apply its clear and accepts general responsibility for the solvency of the
deceased contracted two marriages, the first, with explicit provisions (Velasco vs. Lopez) System.
Lourdes Tuplano on August 29, 1946, who bore him 2. NO - The benefits under the Social Security Act are not
a child, Romeo Davac, and the second, with - The disqualification mentioned in Article 739 is not intended by the lawmaking body to form part of the
Candelaria Davac on January 18, 1949, with whom applicable to herein appellee Candelaria Davac estate of the covered –members.
he had a minor daughter, Elizabeth Davac. Both filed because she was not guilty of concubinage, there - Social Security Act is not a law of succession.
their claims for death benefit with the SSS. being no proof that she had knowledge of the Disposition Resolution of the Social Security
- Social Security Commission issued the resolution previous marriage of her husband Petronilo. Commission appealed is affirmed
declaring respondent Candelaria Davac as the person
INSURANCE Page 52

FRANCISCO DEL VAL v. ANDRES DEL VAL He, therefore, asks that he be declared the owner of "The amounts which the underwriter must deliver
29 PHIL 534 the real estate redeemed by the payment of the to the person insured, in fulfillment of the
MORELAND; February 16, 1915 P18,365.20, the owner of the remaining P21,634.80, contract, shall be the property of the latter, even
the balance of the insurance policy, and that the against the claims of the legitimate heirs or
NATURE plaintiffs account for the use and occupation of the creditors of any kind whatsoever of the person
Appeal from a judgment of the Court of First premises so redeemed since the date of the who effected the insurance in favor of the former."
Instance of the city of Manila dismissing the redemption. 2. NO
complaint with costs. - The trial court refused to give relief to either party - The contract of life insurance is a special contract
and dismissed the action. In this appeal, it is claimed and the destination of the proceeds thereof is
FACTS by the attorney for the plaintiffs that insurance determined by special laws which deal exclusively
- Plaintiffs and defendant are brothers and sisters; provisions in the Code of Commerce are with that subject. The Civil Code has no provisions
that they are the only heirs at law and next of kin of subordinated to the provisions of the Civil Code as which relate directly and specifically to life-insurance
Gregorio Nacianceno del Val, who died in Manila on found in article 1035. This article reads: contracts or to the destination of life insurance
August 4, 1910, intestate "An heir by force of law surviving with others of the proceeds. That subject is regulated exclusively by
- During the lifetime of the deceased he took out same character to a succession must bring into the the Code of Commerce which provides for the terms
insurance on his life for the sum of P40,000 and hereditary estate the property or securities he may of the contract, the relations of the parties and the
made it payable to the defendant ANDRES DEL VAL have received from the deceased during the life of destination of the proceeds of the policy.
as sole beneficiary. After his death the defendant the same, by way of dowry, gift, or for any good - Assuming that the proceeds of the life-insurance
collected the face of the policy. From said policy he consideration, in order to compute it in fixing the policy being the exclusive property of the defendant
paid the sum of P18,365.20 to redeem certain real legal portions and in the account of the division." and he having used a portion thereof in the
estate which the decedent had sold to third persons - Counsel also claims that the proceeds of the repurchase of the real estate sold by the decedent
with a right to repurchase. insurance policy were a donation or gift made by the prior to his death with right to repurchase, and such
- The redemption of said premises was made by the father during his lifetime to the defendant and that, repurchase having been made and the conveyance
attorney of defendant ANDRES in the name of the as such, its ultimate destination is determined by taken in the names of all of the heirs instead of the
plaintiffs and the defendant as heirs of the deceased those provisions of the Civil Code which relate to defendant alone, plaintiffs claim that the property
vendor. It further appears from the pleadings that donations, especially article 819. This article provides belongs to the heirs in common and not to the
the defendant, on the death of the deceased, took that "gifts made to children which are not defendant alone.
possession of most of his personal property, which he betterments shall be considered as part of their legal - The Court rejected this contention unless the fact
still has in his possession, and that he has also the portion." appear or be shown that the defendant acted as he
balance on said insurance policy amounting to did with the intention that the other heirs should
P21,634.80. ISSUES enjoy with him the ownership of the estate ---- in
- Plaintiffs contend that the amount of the insurance 1. WON the insurance belongs to the defendant and other words, that he proposed, in effect, to make a
policy belonged to the estate of the deceased and not to the decedent’s estate gift of the real estate to the other heirs. If it is
not to the defendant personally; that, therefore, they 2. WON the Civil code provisions on succession established by the evidence that was his intention
are entitled to a partition not only of the real and prevail over any other law with respect to the and that the real estate was delivered to the
personal property, but also of the P40,000 life insurance plaintiffs with that understanding, then it is probable
insurance. The complaint prays a partition of all the that their contention is correct and that they are
property, both real and personal, left by the HELD entitled to share equally with the defendant therein.
deceased; that the defendant account for 1. YES If, however, it appears from the evidence in the case
P21,634.80, and that the sum be divided equally - The SC agreed with the finding of the trial court that the conveyances were taken in the name of the
among the plaintiffs and defendant along with the that the proceeds of the life-insurance policy belong plaintiffs without his knowledge or consent, or that it
other property of deceased. exclusively to the defendant as his individual and was not his intention to make a gift to them of the
- The defendant denies the material allegations of separate property, we agree. That the proceeds of an real estate, then it belongs to him. If the facts are as
the complaint and sets up as special defense and insurance policy belong exclusively to the beneficiary stated, he has two remedies. The one is to compel
counterclaim that the redemption of the real estate and not to the estate of the person whose life was the plaintiffs to reconvey to him and the other is to
sold by his father was made in the name of the insured, and that such proceeds are the separate and let the title stand with them and to recover from
plaintiffs and himself instead of in his name alone individual property of the beneficiary, and not of the them the sum he paid on their behalf.
without his knowledge or consent. Andres contends heirs of the person whose life was insured, is the - For the complete and proper determination of the
that it was not his intention to use the proceeds of doctrine in America. We believe that the same questions at issue in this case, the Court was of the
the insurance policy for the benefit of any person but doctrine obtains in these Islands by virtue of section opinion that the cause should be returned to the trial
himself, he alleging that he was and is the sole 428 of the Code of Commerce, which reads: court with instructions to permit the parties to frame
owner thereof and that it is his individual property. such issues as will permit the settlement of all the
INSURANCE Page 53

questions involved and to introduce such evidence as force, or the general principles of law, guide the surrender value, an automatic extension by
may be necessary for the full determination of the court in its decision premiums paid, and to an endowment policy, as well
issues framed. Upon such issues and evidence taken 2. WON the insured, the husband, has the power to as to an ordinary life insurance policy. If the husband
thereunder the court will decide the questions change the beneficiary, the former wife, and to name wishes to retain to himself the control and ownership
involved according to the evidence, subordinating his instead his actual wife, where the insured and the of the policy he may so provide in the policy. But if
conclusions of law to the rules laid down in this beneficiary have been divorced and where the policy the policy contains no provision authorizing a change
opinion. REMANDED. of insurance does not expressly reserve to the of beneficiary without the beneficiary's consent, the
insured the right to change the beneficiary insured cannot make such change. Accordingly, it is
GERCIO v. SUN LIFE ASSURANCE OF CANADA held that a life insurance policy of a husband made
48 PHIL 53 HELD payable to the wife as beneficiary, is the separate
MALCOLM; September 28, 1925 1. Whether the case be considered in the light of the property of the beneficiary and beyond the control of
Code of Commerce, the Civil Code, or the Insurance the husband.
NATURE Act, the deficiencies in the law will have to be - Unlike the statutes of a few jurisdictions, there is
Mandamus to compel Sun Life Assurance Co. of supplemented by the general principles prevailing on no provision in the Philippine Law permitting the
Canada to change the beneficiary in the policy issued the subject. To that end, we have gathered the rules beneficiary in a policy for the benefit of the wife of
by the defendant company on the life of the plaintiff which follow from the best considered American the husband to be changed after a divorce. It must
Hilario Gercio authorities. In adopting these rules, we do so with follow, therefore, in the absence of a statute to the
the purpose of having the Philippine Law of contrary, that if a policy is taken out upon a
FACTS Insurance conform as nearly as possible to the husband's life the wife is named as beneficiary
- On January 29, 1910, the Sun Life Assurance Co. of modern Law of Insurance as found in the United therein, a subsequent divorce does not destroy her
Canada issued an insurance policy on the life of States proper. rights under the policy.
Hilario Gercio. The policy was what is known as a 20- - Court’s first duty is to determine what law should Reasoning
year endowment policy. By its terms, the insurance be applied to the facts. The insurance policy was - Yore vs. Booth
company agreed to insure the life of Hilario Gercio taken out in 1910, that the Insurance Act. No. 2427, “. . . It seems to be the settled doctrine, with but
for the sum of P2,000, to be paid him on February 1, became effective in 1914, and that the effort to slight dissent in the courts of this country, that a
1930, or if the insured should die before said date, change the beneficiary was made in 1922. person who procures a policy upon his own life,
then to his wife, Mrs. Andrea Zialcita, should she - Code of Commerce- there can be found in it no payable to a designated beneficiary, although he
survive him; otherwise to the executors, provision either permitting or prohibiting the insured pays the premiums himself, and keeps the policy
administrators, or assigns of the insured. The policy to change the beneficiary. in his exclusive possession, has no power to
did not include any provision reserving to the insured - Civil Code- it would be most difficult, if indeed it is change the beneficiary, unless the policy itself, or
the right to change the beneficiary. practicable, to test a life insurance policy by its the charter of the insurance company, so provides.
- On the date the policy was issued, Andrea Zialcita provisions. In the case of Del Val vs. Del Val, it In policy, although he has parted with nothing,
was the lawful wife of Hilario Gercio. Towards the end declined to consider the proceeds of the insurance and is simply the object of another's bounty, has
of the year 1919, she was convicted of the crime of policy as a donation or gift, saying "the contract of acquired a vested and irrevocable interest in the
adultery. On September 4, 1920, a decree of divorce life insurance is a special contract and the destination policy, which he may keep alive for his own benefit
was issued in civil case no. 17955, which had the of the proceeds thereof is determined by special laws by paying the premiums or assessments if the
effect of completely dissolving their bonds of which deal exclusively with that subject. The Civil person who effected the insurance fails or refuses
matrimony Code has no provisions which relate directly and to do so.”
- On March 4, 1922, Hilario Gercio formally notified specifically to life-insurance contracts or to the - Connecticut Mutual Life Insurance Company
the Sun Life that he had revoked his donation in destination of life-insurance proceeds. . . ." vs Schaefer
favor of Andrea Zialcita, and that he had designated - Insurance Act- there is likewise no provision “We do not hesitate to say, however, that a policy
in her stead his present wife, Adela Garcia de Gercio, either permitting or prohibiting the insured to change taken out in good faith and valid at its inception, is
as the beneficiary of the policy. Gercio requested the the beneficiary. not avoided by the cessation of the insurable
insurance company to eliminate Andrea Zialcita as 2. NO interest, unless such be the necessary effect of the
beneficiary. This, the insurance company has refused Ratio The wife has an insurable interest in the life of provisions of the policy itself.. . . .In our judgment
and still refuses to do. her husband. The beneficiary has an absolute vested of life policy, originally valid, does not cease to be
interest in the policy from the date of its issuance so by the cessation of the assured party's interest
ISSUES and delivery. So when a policy of life insurance is in the life insured.”
1. (Preliminary) WON the provisions of the Code of taken out by the husband in which the wife is named - Central National Bank of Washington City vs.
Commerce and the Civil Code shall be in force in as beneficiary, she has a subsisting interest in the Hume
1910, or the provisions of the Insurance Act now in policy. And this applies to a policy to which there are “It is indeed the general rule that a policy, and the
attached the incidents of a loan value, cash money to become due under it, belong, the
INSURANCE Page 54

moment it is issued, to the person or persons Disposition The judgment appealed from will be petition. Petitioner then filed a MFR which was also
named in it as the beneficiary or beneficiaries, and reversed and the complaint ordered dismissed as to denied hence this petition.
that there is no power in the person procuring the the appellant.
insurance, by any act of his, by deed or by will, to ISSUE
transfer to any other person the interest of the SEPARATE OPINION 1. WON the designation of the irrevocable
person named.” beneficiaries could be changed or amended without
- In re Dreuil & Co. JOHNSON [concur] the consent of all the irrevocable beneficiaries
“In so far as the law of Louisiana is concerned, it - I agree with the majority of the court, that the 2. WON the irrevocable beneficiaries herein, one of
may also be considered settled that where a policy judgment of the lower court should be revoked, but whom is already deceased while the others are all
is of the semitontine variety, as in this case, the for a different reason. The purpose of the petition is minors could validly give consent to the change or
beneficiary has a vested right in the policy, of to have declared the rights of certain persons in an amendment in the designation of the irrevocable
which she cannot be deprived without her insurance policy which is not yet due and payable. It beneficiaries
consent” may never become due and payable. The premiums
- Wallace vs Mutual Benefit Life Insurance Co. may not be paid, thereby rendering the contract of HELD
“As soon as the policy was issued Mrs. Wallace insurance of non effect, and many other things may 1. NO
acquired a vested interest therein, of which she occur, before the policy becomes due, which would - Based on the provision of their contract and the law
could not be deprived without her consent, except render it non effective. The plaintiff and the other applicable, it is only with the consent of all the
under the terms of the contract with the insurance parties who are claiming an interest in said policy beneficiaries that any change or amendment in the
company. No right to change the beneficiary was should wait until there is something due them under policy concerning the irrevocable beneficiaries may
reserved. Her interest in the policy was her the same. For the courts to declare now who are the be legally and validly effected. Both the law and the
individual property, subject to be divested only by persons entitled to receive the amounts due, if they Policy do not provide for any other exception.
her death, the lapse of time, or by the failure of ever become due and payable, is impossible, for the Reasoning
the insured to pay the premiums. She could keep reason that nothing may ever become payable under - Since the policy was procured in 1968, the
the policy alive by paying the premiums, if the the contract of insurance, and for many reasons such applicable law in this case is the Insurance Act and
insured did not do so. It was contingent upon persons may never have a right to receive anything under that law, the beneficiary designated in a life
these events, but it was free from the control of when the policy does become due and payable. In insurance contract cannot be changed without the
her husband. He had no interest in her property in my judgment, the action is premature and should consent of the beneficiary because he has a vested
this policy, contingent or otherwise. Her interest have been dismissed. interest in the policy.
was free from any claim on the part of the insured - The Beneficiary Designation Indorsement in the
or his creditors. He could deprive her of her PHIL. AMERICAN LIFE INSURANCE v. PINEDA policy in the name of Dimayuga states that the
interest absolutely in but one way, by living more 175 SCRA 416 designation of the beneficiaries is irrevocable: “no
than twenty years.” PARAS; July 19, 1989 right or privilege under the Policy may be exercised,
- Filley vs. Illinois Life Insurance Company or agreement made with the Company to any
“The benefit accruing from a policy of life NATURE change in or amendment to the Policy, without the
insurance upon the life of a married man, payable Petition for review on certiorari the orders of CFI consent of the said beneficiary/beneficiaries.”
upon his death to his wife, naming her, is payable Judge Pineda - Contracts which are the private laws of the
to the surviving beneficiary named, although she contracting parties should be fulfilled according to
may have years thereafter secured a divorce from FACTS the literal sense of their stipulations, if their terms
her husband, and he was thereafter again married - In 1968, Private Respondent Rodolfo Dimayuga are clear and leave no room for doubt as to the
to one who sustained the relation of wife to him at procured an ordinary life insurance policy from the intention of the contracting parties, for contracts are
the time of his death. petitioner company and designated his wife and obligatory, no matter in what form they may be,
The rights of a beneficiary in an ordinary life children as irrevocable beneficiaries. On Feb. 22, whenever the essential requisites for their validity
insurance policy become vested upon the issuance 1980, Dimayuga filed with the CFI a petition to are present.
of the policy, and can thereafter, during the life of amend the designation of the beneficiaries in his life - Finally, the fact that the contract of insurance does
the beneficiary, be defeated only as provided by policy from irrevocable to revocable. Petitioner filed not contain a contingency when the change in the
the terms of the policy.” an Urgent Motion to reset hearing as well as its designation of beneficiaries could be validly effected
- On the admitted facts and the authorities comment and/or Opposition to the respondent’s means that it was never within the contemplation of
supporting the nearly universally accepted principles petition. the parties.
of insurance, we are irresistibly led to the conclusion - Respondent Judge denied petitioner’s Urgent 2. NO
that the question at issue must be answered in the Motion, thus allowing private respondent to adduce - The parent-insured cannot exercise rights and/or
negative evidence, the consequence of which was the privileges pertaining to the insurance contract, for
issuance of the questioned Order granting the otherwise, the vested rights of the irrevocable
INSURANCE Page 55

beneficiaries would be rendered inconsequential. The the proceeds of the policy. The lower court found On the applicability of the Insolvency Law
alleged acquiescence of the 6 children beneficiaries that Ingersoll had a better right and ordered Sun Life - Sec 32 of the Insolvency Law among other things,
cannot be considered an effective ratification to the to pay the insurance proceeds to him. declares that the assignment to be made by the clerk
change of the beneficiaries from irrevocable to of the court "shall operate to vest in the assignee all
revocable. They were minors at the time, and could ISSUE of the estate of the insolvent debtor not exempt by
not validly give consent. Neither could they act WON Ingersoll, as assignee, has a right to the law from execution." Moreover, by section 24, the
through their father-insured since their interests are proceeds of the insurance court is required, upon making an order adjudicating
quite divergent from one another. any person insolvent, to stay any civil proceedings
Disposition questioned Orders of respondent judge HELD pending against him; and it is declared in section 60
are nullified and set aside. NO that no creditor whose debt is provable under the Act
On the Philippine Insolvency Law (Act No. 1956) shall be allowed, after the commencement of
SUN LIFE ASSURANCE v. INGERSOLL - The property and interests of the insolvent which proceedings in insolvency, to prosecute to final
41 PHIL 331 become vested in the assignee of the insolvent are judgment any action therefor against the debtor. In
STREET; November 8, 1921 specified in section 32 of the Insolvency Law which connection with the foregoing may be mentioned
reads as follows: subsections 1 and 2 of section 36, as well as the
NATURE "SEC. 32. As soon as an assignee is elected opening words of section 33, to the effect that the
Action of interpleader or appointed and qualified, the clerk of the court assignee shall have the right and power to recover
shall, by an instrument under his hand and seal of and to take into his possession, all of the estate,
FACTS the court, assign and convey to the assignee all the assets, and claims belonging to the insolvent, except
- April 16, 1918, Sun Life Assurance Company of real and personal property, estate, and effects of the such as are exempt by law from execution.
Canada (Sun Life), in consideration of the payment of debtor with all his deeds, books, and papers relating - These provisions clearly evince an intention to vest
a stipulated annual premium during the period of the thereto, and such assignment shall relate back to the in the assignee, for the benefit of all the creditors of
policy, or until the premiums had been completely commencement of the proceedings in insolvency, and the insolvent, such elements of property and
paid for twenty years, issued a policy of insurance on shall relate back to the acts upon which the property right as could be reached and subjected by
the life of Dy Poco for US$12,500, payable to the said adjudication was founded, and by operation of law process of law by any single creditor suing alone.
assured or his assigns on the 21st day of February, shall vest the title to all such property, estate, and And this is exactly as it should be: for it cannot be
1938, and if he should die before that date then to effects in the assignee, although the same is then supposed that the Legislature would suppress the
his legal representatives. attached on mesne process, as the property of the right of action of every individual creditor upon the
- June 23, 1919, the assured, Dy Poco, was adjudged debtor. Such assignment shall operate to vest in the adjudication of insolvency, and at the same time
an involuntary insolvent by the CFI Manila, and Frank assignee all of the estate of the insolvent debtor not allow the insolvent debtor to retain anything subject
B. Ingersoll was appointed assignee of his estate. exempt by law from execution." to the payment of his debts in a normal state of
- July 10, 1919, Dy Poco died, and on August 21, - the Insolvency Law is in great part a copy of the solvency.
1919, Tan Sit, was duly appointed as the Insolvency Act of California, enacted in 1895, though - "leviable assets" and "assets in insolvency" are
administratrix of his intestate estate. it contains a few provisions from the American practically coextensive terms. Hence, in determining
- By the terms of the policy it was provided that after Bankruptcy Law of 1898 what elements of value constitute assets in
the payment of three full premiums, the assured - Under each of said laws the assignee acquires all insolvency, SC is at liberty to consider what elements
could surrender the policy to the company for a the real and personal property, estate, and effects of of value are subject to be taken upon execution, and
"cash surrender value," indicated in an annexed the debtor, not exempt by law from execution, with vice versa.
table; but inasmuch as no more than two premiums all deeds, books and papers relating thereto; and On whether a policy of insurance having no cash
had been paid upon the policy now in question up to while this language is broad, it nevertheless lacks the surrender value, but payable to insured or his legal
the time of the death of the assured, this provision comprehensiveness of section 70 (a) of the American representative, is property that may be taken upon
had not become effective; and it does not appear Bankruptcy Law of 1898 in at least two particulars; execution against him.
that the company would in accordance with its own for under subsection 3 of section 70 (a) of the last - Philippine laws declare no exemption with respect
usage or otherwise have made any concession to the mentioned law, the trustee in bankruptcy acquires to insurance policies; and this species of property is
assured in the event he had desired, before his the right to exercise any powers which the insolvent not enumerated, in section 48 of the Insolvency Law,
death, to surrender the policy. It must therefore be might have exercised for his own benefit, and under among items from the ownership of which the
accepted that this policy had no cash surrender subsection 5 the trustee acquires any property of the assignee is excluded. Moreover, all life insurance
value, at the time of the assured's death, either by insolvent which the latter could by any means have policies are declared by law to be assignable,
contract or by convention practice of the company in assigned to another. The Insolvency Law here in regardless of whether the assignee has an insurable
such cases. force, in common with the predecessor laws above- interest in the life of the insured or not (Insurance
- Both Ingersoll, as assignee, and Tan Sit, as mentioned, contains nothing similar to these Act No. 2427, sec. 166).
administratix of Dy Poco's estate, asserted claims to provisions.
INSURANCE Page 56

- SC has held that insurance policies having a practical, though not the legal, relation of the law from execution." Moreover, by section 24, the
present cash surrender value are subject to be taken company to this fund. "Upon the surrender of the court is required, upon making an order adjudicating
upon execution. (Misut Garcia vs. West Coast San policy before the death of the assured, the company, any person insolvent, to stay any civil proceedings
Francisco Life Ins. Co.) to be relieved from all responsibility for the increased pending against him; and it is declared in section 60
- a policy devoid of a cash surrender value cannot be risk, which is represented by this accumulating that no creditor whose debt is provable under the Act
either "leviable assets" or "assets in insolvency." reserve, could well afford to surrender a considerable shall be allowed, after the commencement of
- the assignee in insolvency acquired no beneficial part of it to the assured, or his representative. A proceedings in insolvency, to prosecute to final
interest in the policy of insurance in question; that return of a part in some form or other is now Usually judgment any action therefor against the debtor. In
its proceeds are not liable for any of the debts made." (In re McKinney) connection with the foregoing may be mentioned
provable against the insolvent in the pending - the stipulation providing for a cash surrender value subsections 1 and 2 of section 36, as well as the
proceedings, and that said proceeds should therefore is a comparatively recent innovation in life insurance. opening words of section 33, to the effect that the
be delivered to his administratrix. Formerly the contracts provided — as they still assignee shall have the right and power to recover
On applicable US case commonly do in the policies issued by fraternal and to take into his possession, all of the estate,
- In re McKinney: no beneficial interest in the policy organizations and benefit societies — for the assets, and claims belonging to the insolvent, except
had ever passed to the assignee over and beyond payment of a premium sufficient to keep the such as are exempt by law from execution.
what constituted the surrender value, and that the estimated risk covered; and in case of a lapse the - These provisions clearly evince an intention to vest
legal title to the policy was vested in the assignee policy-holder received nothing. Furthermore, the in the assignee, for the benefit of all the creditors of
merely in order to make the surrender value- practice is common among insurance companies the insolvent, such elements of property and
available to him. The assignee should surrender the even now to concede nothing in the character of cash property right as could be reached and subjected by
policy upon the payment to him of said value, as he surrender value, until three full premiums have been process of law by any single creditor suing alone.
was in fact directed to do. The assignee in paid, as in this case. And this is exactly as it should be: for it cannot be
bankruptcy had no right to keep the estate unsettled - CONLUSION (from this case and other English and supposed that the Legislature would suppress the
for an indefinite period, for the mere purpose of American cases cited following the same opinion): right of action of every individual creditor upon the
speculating upon the chances of the bankrupt's the assignee acquires no beneficial interest in adjudication of insolvency, and at the same time
death. As regards everything beyond the surrender insurance effected on the life of the insolvent, except allow the insolvent debtor to retain anything subject
value, the assignee in bankruptcy would, after the to the extent that such insurance contains assets to the payment of his debts in a normal state of
discharge of the bankrupt, have no insurable interest which can be realized upon as of the date when the solvency.
in the life of the bankrupt. petition of insolvency is filed. The explanation is to - "leviable assets" and "assets in insolvency" are
- surrender value of a policy "arises from the fact be found in the consideration that the destruction of practically coextensive terms. Hence, in determining
that the fixed annual premiums is much in excess of a contract of life insurance is not only highly what elements of value constitute assets in
the annual risk during the earlier years of the policy, prejudicial to the insured and those dependent upon insolvency, SC is at liberty to consider what elements
an excess made necessary in order to balance the him, but is inimical to the interests of society. of value are subject to be taken upon execution, and
deficiency of the same premium to meet the annual Insurance is a species of property that should be vice versa.
risk during the latter years of the policy. This excess conserved and not dissipated. As is well known, life On whether a policy of insurance having no cash
in the premium paid over the annual cost of insurance is increasingly difficult to obtain with surrender value, but payable to the insured or his
insurance, with accumulations of interest, constitutes advancing years, and even when procurable after the legal representative, is property that may be taken
the surrender value. Though this excess of premiums age of fifty, the cost is then so great as to be upon execution against him.
paid is legally the sole property of the company, still practically prohibitive to many. Insolvency is a - Philippine laws declare no exemption with respect
in practical effect, though not in law, it is moneys of disaster likely to overtake men in mature life; and to insurance policies; and this species of property is
the assured deposited with the company in advance one who has gone through the process of bankruptcy not enumerated, in section 48 of the Insolvency Law,
to make up the deficiency in later premiums to cover usually finds himself in his declining years with the among items from the ownership of which the
the annual cost of insurance, instead of being accumulated savings of years swept away and assignee is excluded. Moreover, all life insurance
retained by the assured and paid by him to the earning power diminished. The courts are therefore policies are declared by law to be assignable,
company in the shape of greatly-increased practically unanimous in refusing to permit the regardless of whether the assignee has an insurable
premiums, when the risk is greatest. It is the 'net assignee in insolvency to wrest from the insolvent a interest in the life of the insured or not (Insurance
reserve' required by law to be kept by the company policy of insurance which contains in it no present Act No. 2427, sec. 166).
for the benefit of the assured, and to be maintained realizable assets. - SC has held that insurance policies having a
to the credit of the policy. So long as the policy On the applicability of the Insolvency Law present cash surrender value are subject to be taken
remains in force the company has not practically any - Sec 32 of the Insolvency Law among other things, upon execution. (Misut Garcia vs. West Coast San
beneficial interest in it, except as its custodian, with declares that the assignment to be made by the clerk Francisco Life Ins. Co., 41 Phil., 258.)
the obligation to maintain it unimpaired and suitably of the court "shall operate to vest in the assignee all - a policy devoid of a cash surrender value cannot be
invested for the benefit of the insured. This is the of the estate of the insolvent debtor not exempt by either "leviable assets" or "assets in insolvency."
INSURANCE Page 57

- the assignee in insolvency acquired no beneficial WON there was concealment (Was appellant, his ailment and operation. Indeed, such statement
interest in the policy of insurance in question; that because of insured's aforesaid representation, misled must be presumed to have been made by him
its proceeds are not liable for any of the debts or deceived into entering the contract or in accepting without knowledge of its incorrectness and without
provable against the insolvent in the pending the risk at the rate of premium agreed upon?) any deliberate intent on his part to mislead the
proceedings, and that said proceeds should therefore appellant.
be delivered to his administratrix. HELD 3) Waiver:
Disposition Judgment reversed. Sun Life is directed NO While it may be conceded that, from the viewpoint of
to pay the proceeds of the policy to Tan Sit. -"concealment exists where the assured had a medical expert, the information communicated was
knowledge of a fact material to the risk, and honesty, imperfect, the same was nevertheless sufficient to
CHAPTER VI – RESCISSION OF INSURANCE good faith, and fair dealing requires that he should have induced appellant to make further inquiries
CONTRACTS: CONCEALMENT, communicate it to the assurer, but he designedly and about the ailment and operation of the insured.
MISREPRESENTATION, & BREACH OF intentionally withholds the same." Section 32 of Insurance Law [Act No. 2427]
WARRANTIES - It has also been held "that the concealment must, provides:
in the absence of inquiries, be not only material, but “The right to information of material facts may be
NG v. ASIAN CRUSADER LIFE ASSURANCE CORP fraudulent, or the fact must have been intentionally waived either by the terms of insurance or by neglect
122 SCRA 461 withheld." to make inquiries as to such facts where they are
ESCOLIN; May 30, 1983 Reasoning distinctly implied in other facts of which information
1) The evidence shows that the Insular Life is communicated.”
FACTS Assurance Co., Ltd. approved Kwong Nam's request It has been held that where, "upon the face of the
- On May 12, 1962, Kwong Nam applied for a 20- for reinstatement and amendment of his lapsed application, a question appears to be not answered
year endowment insurance on his life for the sum of insurance policy on April 24, 1962…. It results, at all or to be imperfectly answered, and the insurers
P20,000, with his wife, Ng Gan Zee, as beneficiary. therefore, that when on May 12, 1962 Kwong Nam issue a policy without any further inquiry, they waive
- He died on Dec 1963 of cancer of the liver with answered `No' to the question whether any life the imperfection of the answer and render the
metastasis. All premiums had been paid at the time insurance company ever refused his application for omission to answer more fully immaterial.
of his death. reinstatement of a lapsed policy he did not Disposition the judgment appealed from is hereby
- Ng presented a claim for payment of the face value misrepresent any fact. affirmed, with costs against appellant
of the policy. Appellant (Asian Crusader) denied the 2) Assuming that the aforesaid answer given by the
claim on the ground that the answers given by the insured is false, Sec. 278 of the Insurance Law CANILANG v. CA (GREAT PACIFIC LIFE
insured to the questions appearing in his application nevertheless requires that fraudulent intent on the ASSURANCE CORP.)
for life insurance were untrue. part of the insured be established to entitle the 223 SCRA 443
-Appellant: the insured was guilty of insurer to rescind the contract. And as correctly FELICIANO; June 17, 1993
misrepresentation when observed by the lower court, "misrepresentation as a
1) he answered "No" to the question (in the defense of the insurer to avoid liability is an NATURE
application) of "Has any life insurance company ever `affirmative’ defense. The duty to establish such a Petition for review on certiorari of the decision of the
refused your application for insurance or for defense by satisfactory and convincing evidence Court of Appeals
reinstatement of a lapsed policy or offered you a rests upon the defendant. The evidence before the
policy different from that applied for?" when in fact, Court does not clearly and satisfactorily establish FACTS
Insular Life denied his application for reinstatement that defense." - June 18, 1982 – Jaime Canilang was diagnosed by
of his lapsed life insurance policy -Kwong Nam had informed the appellant's medical Dr. Claudio to have sinus tachycardia. He was
2) he gave the appellant's medical examiner false examiner that the tumor for which he was operated directed by the doctor to take a tranquilizer
and misleading information as to his ailment and on was ''associated with ulcer of the stomach." In the (Trazepam) and a beta-blocker drug (Aptin).
previous operation when he said he was absence of evidence that the insured had sufficient - August 3, 1982 – Jaime consulted Dr. Claudio again
“operated on for a Tumor [mayoma] of the medical knowledge as to enable him to distinguish and was diagnosed to have acute bronchitis.
stomach… associated with ulcer of stomach. Tumor between "peptic ulcer" and "a tumor", his statement - August 4, 1982 – Jaime applied for a nonmedical
taken out was hard and of a hen's egg size. that said tumor was "associated with ulcer of the insurance policy with Great Pacific Life Assurance
Operation was two years ago in Chinese General stomach" should be construed as an expression Company. He named his wife Thelma as his
Hospital by Dr. Yap. Claims he is completely made in good faith of his belief as to the nature of beneficiary. He was issue the policy with a face
recovered.” Medical report show that insured was value of P19,700 effective August 9, 1982.
operated on for "peptic ulcer", involving the excision 8 - August 5, 1983 – Jaime died of congestive heart
of a portion of the stomach, not tumor. "Sec. 27. Such party to a contract of insurance must communicate failure, anemia and chronic anemia. Thelma filed her
to the other, in good faith, all facts within his knowledge which are
material to the contract, and which the other has not the means of
claim but the insurance company refused to grant it
ISSUE ascertaining, and as to which he makes no warranty." on the ground that Jaime had concealed information.
INSURANCE Page 58

- Thelma filed a complaint against Great Pacific to Ratio Section 27 of the Insurance Code of 1978 is the very least, required a higher premium for the
recover the insurance proceeds. She testified that properly read as referring to "any concealment same coverage.
she was not aware of her husband’s ailments and without regard to whether such concealment is - As held in the case of Saturnino vs. Philippine-
that she thought he had died from a kidney disorder. intentional or unintentional. The restoration in 1985 American Life Insurance, “the waiver of medical
- Great Pacific presented as witness Dr. Quismorio by B.P. Blg. 874 of the phrase "whether intentional or examination in a non-medical insurance contract
who testified that Jaime’s insurance application was unintentional" merely underscored the fact that all renders even more material the information inquired
the basis of his medical declaration and she throughout (from 1914 to 1985), the statute did not of the applicant concerning previous condition of
explained that an applicant was required to undergo require proof that concealment must be "intentional" health and diseases suffered, for such information
medical examination only if the applicant had in order to authorize rescission by the injured party. necessarily constitutes an important factor which the
disclosed that he had previously been consulted with Reasoning insurer takes into consideration in deciding whether
a doctor and had been hospitalized. - Art. 27 of the 1978 Insurance Code reads that “a to issue the policy or not.”
- The Insurance Commissioner ordered Great Pacific concealment entitles the injured party to rescind a Disposition the Petition for Review is DENIED for
to pay Thelma the insurance proceeds, including contract of insurance,” which does not include the lack of merit and the Decision of the Court of Appeals
attorney’s fees, holding that Jaime’s illness was not words “whether intentional or unintentional” from the dated 16 October 1989 in C.A.-G.R. SP No. 08696 is
that serious as to Great Pacific’s decision to insure previous statutes. The Insurance Commissioner hereby AFFIRMED.
him and that there was no concealment on the part relied on this deletion in arguing that the statute
of Jaime with regard to his illness. intended to limit the kinds of concealment which YU PANG CHENG v. CA
Petitioners’ Claim: generate a right to rescind on the part of the injured 105 PHIL 930
> Thelma argues that the non-disclosure of Jaime party to "intentional concealments." BAUTISTA ANGELO; May 29, 1959
did not amount to fraud. - In the case at bar, the nature of the facts not
> She also argues that the CA erred in not holding conveyed to the insurer was such that the failure to FACTS
that the issue in the case agreed upon between the communicate must have been intentional rather than - September 5, 1950: Yu Pang Eng submitted parts
parties before the Insurance Commission is whether merely inadvertent. II and III of his application for insurance consisting
or not Jaime 'intentionally' made material > Jaime could not have been unaware that his of the medical declaration made by him to the
concealment in stating his state of health; heart beat would at times rise to high and medical examiner of defendant and the medical
Respondents’ Comments: alarming levels and that he had consulted a doctor examiner's report
> The CA reversed the Insurance Commissioner’s twice two months before applying for non-medical - September 7: he submitted part I of his application
decision, holding that the use of the word insurance. which is the declaration made by him to an agent of
'intentionally" by the Insurance Commissioner in > The last medical consultation took place just the defendant
defining and resolving the issue agreed upon by the day before the insurance application was filed. - September 8: defendant issued to the insured
parties at pre-trial before the Insurance 2. YES Policy No. 812858
Commissioner was not supported by the evidence Ratio Materiality relates rather to the "probable and - December 27, 1950: the insured entered St.
and that the issue agreed upon by the parties had reasonable influence of the facts" upon the party to Luke's Hospital for medical treatment but he died on
been whether Jaime made a material concealment as whom the communication should have been made, in February 27, 1951.
to the state of his health at the time of the filing of assessing the risk involved in making or omitting to - According to the death certificate, he died of
insurance application, justifying the denial of the make further inquiries and in accepting the "infiltrating medullary carcinoma, Grade 4, advanced
claim. application for insurance; that "probable and cardiac and of lesser curvature, stomach metastases
> It also found that the failure of Jaime to disclose reasonable influence of the farts" concealed must, of spleen."
previous medical consultation and treatment course, be determined objectively, by the judge - Plaintiff, brother and beneficiary of the insured,
constituted material information which should have ultimately. demanded from defendant the payment of the
been communicated to Great Pacific to enable the Reasoning proceeds of the insurance policy and when the
latter to make proper inquiries. - The information which Jaime failed to disclose was demand was refused, he brought the present action.
material to the ability of Great Pacific to estimate the - The insured, in his application for insurance,
ISSUES probable risk he presented as a subject of life particularly in his declarations to the examining
1. WON Jaime intentionally withheld information insurance. physician, stated the following in answering the
from Great Pacific - Had Canilang disclosed his visits to his doctor, the questions propounded to him:
2. WON the information withheld would have been diagnosis made and the medicines prescribed by 14. Have you ever had any of the following diseases
material to Great Pacific’s decision to grant Jaime the such doctor, in the insurance application, it may be or symtoms? Each question must be read and
insurance policy reasonably assumed that Great Pacific would have answered "Yes" or "No.".
made further inquiries and would have probably "Gastritis, Ulcer of the Stomach or any disease of
HELD refused to issue a non-medical insurance policy or, at that organ? No.
1. YES
INSURANCE Page 59

"Vertigo, Dizziness, Fainting-spells or In fact, according to the death certificate, the insured Petition for review on certiorari of the CA decision,
Unconsciouness? No. died of "infiltrating medullary carcinoma, Grade, 4, which set aside the decision of CFI Manila, which had
"Cancer, Tumors or Ulcers of any kind? No. advanced cardiac and of lesser curvature, stomach in turn granted the complaint for a sum of money in
- 15. Have you ever consulted any physician riot metastases spleen", which may have a direct civil case filed by Pacific Banking against Oriental
included in any of the above answers? Give names connection with his previous illness. Assurance.
and address or physicians list ailments or accidents - Our Insurance Law provides that "A neglect to
and date. No." communicate that which a party knows and ought to FACTS
- It appears that the insured entered the Chinese communicate, is called concealment" (Section 25, - October 21,1963: an open Fire Policy was issued to
General Hospital for medical treatment on January Act No. 2427). Whether intentional or unintentional, the Paramount Shirt Manufacturing Co. (insured), by
29, 1950 having stayed there up to February 11, the concealment entitles the insurer to rescind the which Oriental Assurance Corporation bound itself to
1950. contract of insurance (Section 26). indemnify the insured for any loss or damage, not
- An X-ray picture of his stomach was taken and the - Our law even requires the insured to communicate exceeding P61,000.00, caused by fire to its property
diagnosis made of him by his doctors showed that to the insurer all facts within his knowledge which consisting of stocks, materials and supplies usual to
his illness was "peptic ulcer, bleeding." are material to the contract and which the other a shirt factory, including furniture, fixtures,
party has not the means of ascertaining (Section machinery and equipment while contained in the
ISSUE 27), and the materiality is to be determined not by ground, second and third floors of the building
WON the insured is guilty of concealment of some the event but solely by the probable and reasonable situated at number 256 Jaboneros St., San Nicolas,
facts material to the risk insured against which has influence of the facts upon the party to whom the Manila, for a period of one year commencing from
the effect of avoiding the policy as found by communication is due (Section 30). that date to October 21, 1964.
respondent court. - Argente vs. West Coast Life Insurance Co.: "One - Insured was at the time of the issuance of the
ground for the rescission of a contract of insurance policy and is up to this time, a debtor of Pacific
HELD under the Insurance Act is 'a concealment', which in Banking in the amount of not less P800,000.00 and
- It should be noted that the insured's confinement section 25 is defined 'A neglect to communicate that the goods described in the policy were held in trust
in the Chinese General Hospital took place from which a party knows and ought to communicate.' by the insured for the Pacific Banking under thrust
January 29, 1950 to February 11, 1950, whereas his Appellant argues that the concealment was receipts.
application for insurance wherein he stated his immaterial and insufficient to avoid the policy. We - Said policy was duly endorsed to Pacific Banking as
answers to the questions propounded to him by the cannot agree. In an action on a life insurance policy mortgagee/trustor of the properties insured, with the
examining physician of defendant was submitted to where the evidence conclusively shows that the knowledge and consent of Oriental Assurance to the
defendant on September 5, 1950. answers to questions concerning diseases were effect that "loss if any under this policy is payable to
- It is apparent that when the insured gave his untrue, the truth or falsity of the answers become the Pacific Banking Corporation".
answers regarding his previous ailment, particularly the determining factor. If the policy was procured by - While the aforesaid policy was in full force and
with regard to "Gastritis, Ulcer of the Stomach or any fraudulent representations, the contract of insurance effect, a fire broke out on the subject premises
disease of that organ" and "Vertigo, Dizziness, apparently set forth therein was never legally destroying the goods contained in its ground and
Fainting-spells or Unconsciousness", he concealed existent. It can fairly be assumed that had the true second floors. Counsel for the Pacific Banking sent a
the ailment of which he was treated in the Chinese facts been disclosed by the assured, the insurance letter of demand to Oriental Assurance for indemnity
General Hospital which precisely has direct would never have been granted." due to the loss of property by fire. Oriental
connection with the subject of the questions Disposition Decision affirmed. Assurance informed counsel that it was not yet ready
propounded. to accede to the latter's demand as the former is
- The negative answers given by the insured awaiting the final report of the insurance adjuster,
regarding his previous ailment, or his concealment of H.H. Bayne Adjustment Company.
the fact that he was hospitalized and treated for - Said insurance adjuster notified counsel for the
sometime of peptic ulcer and had suffered from Pacific Banking that the insured under the policy had
"dizziness, anemia, abdominal pains and tarry not filed any claim with it, nor submitted proof of
stools", deprived defendant of the opportunity to GREAT PACIFIC LIFE v. CA (supra p.34) loss which is a clear violation of Policy Condition
make the necessary inquiry as to the nature of his No.11, and for which reason, determination of the
past illness so that it may form its estimate relative PACIFIC BANKING CORP v. CA (ORIENTAL liability of Oriental Assurance could not be had.
to the approval of his application. ASSURANCE CORPORATION) Pacific Banking's counsel replied asking the insurance
- Had defendant been given such opportunity, 168 SCRA 1 adjuster to verify from the records of the Bureau of
considering the previous illness of the insured as PARAS; November 28, 1988 Customs the entries of merchandise taken into the
disclosed by the records of the Chinese General customs bonded warehouse razed by fire as a
Hospital, defendant would probably had never NATURE reliable proof of loss.
consented to the issuance of the policy in question.
INSURANCE Page 60

- For failure of the insurance company to pay the loss - Policy Condition No. 3 explicitly provides: “The 2. NO
as demanded, Pacific Banking field before CFI an Insured shall give notice to the Company of any - Subject mortgage clause pecifically provides: “Loss,
action for a sum of money against the Oriental insurance already effected, or which may if any, under this policy, shall be payable to the
Assurance, in the principal sum of P61,000.00 issued subsequently be effected, covering any of the PACIFIC BANKING CORPORATION Manila
in favor of Paramount Shirt Manufacturing Co. property hereby insured, and unless such notice be mortgagee/trustor as its interest may appear, it
Oriental Assurance defenses given and the particulars of such insurance or being hereby understood and agreed that this
(a) lack of formal claim by insured over the loss and insurances be stated in or endorsed on this Policy by insurance as to the interest of the mortgagee/trustor
(b) premature filing of the suit as neither plaintiff nor or on behalf of the Company before the occurrence only herein, shall not be invalidated by any act or
insured had submitted any proof of loss on the basis of any loss or damage, all benefit under this policy neglect except fraud or misrepresentation, or arson
of which defendant would determine its liability and shall be forfeited.” of the mortgagor or owner/trustee of the property
the amount thereof, either to the Oriental Assurance - It is not disputed that the insured failed to reveal insured; provided, that in case the mortgagor or
or its adjuster H.H. Bayne Adjustment Co. before the loss three other insurances. By reason of owner/ trustee neglects or refuses to pay any
Pacific Banking said unrevealed insurances, the insured had been premium, the mortgagee/ trustor shall, on demand
> presented evidence that insured has undeclared guilty of a false declaration; a clear pay the same.”
co-insurances with the following: P30,000.00 with misrepresentation and a vital one because where the - The paragraph clearly states the exceptions to the
Wellington Insurance; P25,000. 00 with Empire insured had been asked to reveal but did not, that general rule that insurance as to the interest of the
Surety and P250,000.00 with Asian Surety; was deception. Otherwise stated, had the insurer mortgagee, cannot be invalidated; namely: fraud, or
undertaken by insured Paramount on the same known that there were many co-insurances, it could misrepresentation or arson.
property covered by its policy with Oriental have hesitated or plainly desisted from entering into - Concealment of the aforecited co-insurances can
Assurance whereas the only co-insurances declared such contract. Hence, the insured was guilty of clear easily be fraud, or in the very least,
in the subject policy are those of P30,000.00 with fraud. misrepresentation. It is but fair and just that where
Malayan, P50,000.00 with South Sea, and - Pacific Banking's contention that the allegation of the insured who is primarily entitled to receive the
P25.000.00 with Victory fraud is but a mere inference or suspicion is proceeds of the policy has by its fraud and/or
- NOTE: the defense of fraud and/or violation of non- untenable. Concrete evidence of fraud or false misrepresentation, forfeited said right, with more
declaration of co-insurances was not pleaded in the declaration by the insured was furnished by the reason Pacific Banking which is merely claiming as
answer, also not pleaded in the Motion to Dismiss. Pacific Banking itself when the facts alleged in the indorsee of said insured, cannot be entitled to such
- CFI denied Oriental Assurance's motion on the policy under clauses "Co-Insurances Declared" and proceeds.
ground that since the defense was raised for the first "Other Insurance Clause" are materially different - The fact of fraud was tried by express or at least
time, it must be deemed to have waived the from the actual number of co-insurances taken over implied consent of the parties. Pacific Banking did not
requirement of proof of loss. Case was submitted for the subject property. Consequently, the whole only object to the introduction of evidence but on the
decision. But upon MR, Oriental Asurance was foundation of the contract fails, the risk does not contrary, presented the very evidence that proved its
allowed to present additional evidence, "in order to attach and the policy never becomes a contract existence.
prove that 'insured has committed a violation of between the parties. Representations of facts are the - Be that as it may, SC has ample authority to give
condition No. 3 of the policy in relation to the other foundation of the contract and if the foundation does beyond the pleadings where in the interest of justice
Insurance Clause.' " CFI eventually adjudged Oriental not exist, the superstructure does not arise. and the promotion of public policy, there is a need to
Assurance liable to the Pacific Banking under the said Falsehood in such representations is not shown to make its own finding to support its conclusion.
contract of insurance. vary or add to the contract, or to terminate a Otherwise stated, the Court can consider a fact which
- Court of Appeals reversed. Pacific Banking's MR contract which has once been made, but to show surfaced only after trial proper.
denied. that no contract has ever existed (Tolentino). A void - Generally, the cause of action on the policy accrues
or inexistent contract is one which has no force and when the loss occurs, but when the policy provides
ISSUES effect from the very beginning, as if it had never that no action shall be brought unless the claim is
1. WON insured is guilty of fraud been entered into, and which cannot be validated first presented extrajudicially in the manner provided
2. WON mortgagee/assignee can still claim from the either by time or by ratification. in the policy, the cause of action will accrue from the
insurance - As the insurance policy against fire expressly time the insurer finally rejects the claim for payment.
required that notice should be given by the insured - In the case at bar, policy condition No. 11
HELD of other insurance upon the same property, the total specifically provides that the insured shall on the
1. YES absence of such notice nullifies the policy. happening of any loss or damage give notice to the
- The crux of the controversy centers on two points: - Argument that notice of co-insurances may be company and shall within fifteen (15) days after such
(a) unrevealed co-insurances which violated policy made orally is preposterous and negates policy loss or damage deliver to the Oriental Assurance (a)
conditions No. 3; and (b) failure of the insured to file condition No. 20 which requires every notice and a claim in writing giving particular account as to the
the required proof of loss prior to court action. other communications to the insurer to be written or articles or goods destroyed and the amount of the
printed. loss or damage and (b) particulars of all other
INSURANCE Page 61

insurances, if any. Likewise, insured was required "at - Contracts of insurance are contracts of indemnity the plea on the ground that that the facts concealed
his own expense to produce, procure and give to the upon the terms and conditions specified in the policy. by the insured were made in good faith and under
company all such further particulars, plans, The parties have a right to impose such reasonable the belief that they need not be disclosed, and that
specifications, books, vouchers, invoices, duplicates conditions at the time of the making of the contract the disclosure was not material since the policy was
or copies thereof, documents, proofs and information as they may deem wise and necessary. The non-medical.
with respect to the claim". agreement has the force of law between the parties. - Sunlife appealed to the CA, but the latter denied
- Evidence adduced shows that 24 days after the fire, The terms of the policy constitute the measure of the the appeal on the ground that the cause of death
Pacific Banking merely wrote letters to Oriental insurer's liability, and in order to recover, the insured was unrelated to the facts concealed by the insured.
Assurance to serve as a notice of loss, thereafter, the must show himself within those terms. The
former did not furnish the latter whatever pertinent compliance of the insured with the terms of the Petitioner’s Claim
documents were necessary to prove and estimate its policy is a condition precedent to the light of > The insured did not disclose facts relevant to the
loss. Instead, Pacific Banking shifted upon Oriental recovery. issuance of the policy, thus rescission of the contract
Assurance the burden of fishing out the necessary - It appearing that insured has violated or failed to may be invoked by the insurance company.
information to ascertain the particular account of the perform the conditions under No. 3 and 11 of the Respondents’ Comments
articles destroyed by fire as well as the amount of contract, and such violation or want of performance > The actual cause of death was not relevant to the
loss. has not been waived by the insurer, the insured concealed information, and the policy was entered
- Oriental Assurance and its adjuster notified Pacific cannot recover, much less the herein Pacific Banking. into by the insured in good faith.
Banking that insured had not yet filed a written claim Courts are not permitted to make contracts for the
nor submitted the supporting documents in parties; the function and duty of the courts is simply ISSUE
compliance with the requirements set forth in the to enforce and carry out the contracts actually made. WON the concealment renders the insurance policy
policy. Despite the notice, the latter remained Disposition Petition dismissed. CA affirmed. rescissible
unheedful. Since the required claim by insured,
together with the preliminary submittal of relevant SUNLIFE ASSURANCE COMPANY v. CA (SPS. HELD
documents had not been complied with, it follows BACANI) YES
that Oriental Assurance could not be deemed to have 245 SCRA 268 Ratio The terms of the contract are clear. The
finally rejected Pacific Banking's claim and therefore QUIASON; June 22, 1995 insured is specifically required to disclose to the
the latter's cause of action had not yet arisen. insurer matters relating to his health.
Compliance with condition No. 11 is a requirement NATURE Reasoning
sine qua non to the right to maintain an action as A petition for review on certiorari. SEC. 26 (IC)
prior thereto no violation of Pacific Banking's right A neglect to communicate that which a party
can be attributable to Oriental Assurance. As before FACTS knows and ought to communicate, is called a
such final rejection, there was no real necessity for - April 15, 1986: Robert John B. Bacani procured a concealment.
bringing suit. Pacific Banking should have life insurance contract for himself from SUNLIFE SEC. 31 (IC)
endeavored to file the formal claim and procure all (petitioner) valued at P100K. The designated Materiality is to be determined not by the event,
the documents, papers, inventory needed by Oriental beneficiary was his mother, Bernarda Bacani but solely by the probable and reasonable influence
Assurance or its adjuster to ascertain the amount of (respondent). of the facts upon the party to whom communication
loss and after compliance await the final rejection of - June 26, 1987: the insured died in a plane crash. is due, in forming his estimate of the disadvantages
its claim. Indeed, the law does not encourage Bernarda Bacani filed a claim with Sunlife, seeking of the proposed contract or in making his inquiries
unnecessary litigation. the benefits of the insurance policy taken by her son. - The information which the insured failed to disclose
- Pacific Banking prematurely filed the civil case and Petitioner conducted an investigation and its findings was material and relevant to the approval and the
dismissal thereof was warranted under the prompted it to reject the claim on the ground that issuance of the insurance policy. The matters
circumstances. While it is a cardinal principle of the insured did not disclose facts material to the concealed would have definitely affected petitioner's
insurance law that a policy or contract of insurance is issuance of the policy. The insured gave false action on his application, either by approving it with
to be construed liberally in favor of the insured and statements in the application when he answered in the corresponding adjustment for a higher premium
strictly as against the insurer company yet, contracts the negative to the question “have you ever had or or rejecting the same.
of insurance, like other contracts, are to be sought advice for urine, kidney, bladder - Good faith is no defense in concealment. It appears
construed according to the sense and meaning of the disorder?” that such concealment was deliberate on the part of
terms which the parties themselves have used. If - Sunlife discovered that two weeks prior to the the insured.
such terms are clear and unambiguous, they must be issuance, insured was diagnosed with renal failure, - The waiver of a medical examination [in a non-
taken and understood in their plain, ordinary and was confined, and underwent tests. medical insurance contract] renders even more
popular sense. - November 17, 1988: Bacani and her husband filed material the information required of the applicant
for specific performance against Sunlife. RTC granted concerning previous condition of health and diseases
INSURANCE Page 62

suffered, for such information necessarily constitutes WON the life insurance obtained by Dominador Albay extensive investigation. Qua Chee Gan submitted the
an important factor which the insurer takes into was issued through fraud and deceit corresponding fire claims, totaling P398,562.81 (but
consideration in deciding whether to issue the policy reduced to the full amount of the insurance,
or not. HELD P370,000), the Insurance Company resisted
- Anent the finding that the facts concealed had no YES payment, claiming violation of warranties and
bearing to the cause of death of the insured, it is well Ratio In a contract where one of the contracting conditions, filing of fraudulent claims, and that the
settled that the insured need not die of the disease parties may have given his consent through error, fire had been deliberately caused by the insured or
he had failed to disclose to the insurer. It is sufficient violence, intimidation, or deceit, and in any of such by other persons in connivance with him.
that his non-disclosure misled the insurer in forming cases the contract is void, even though, despite this - Qua Chee Gan, his brother and his employees were
his estimates of the risks of the proposed insurance nullity, no crime was committed. There may not have tried for arson, where counsel of the insurance
policy or in making inquiries been estafa in the case at bar, but it was conclusively company acted as a private prosecutor. They were
Disposition Petition is granted and the decision of demonstrated by the trial that deceit entered into the acquitted.
CA is reversed and set aside. insurance contract, fulfillment whereof is claimed, - This civil suit was then instituted to claim against
and therefore the conclusions reached by the court in the insurance company. The CFI ruled in favor of Qua
EGUARAS v. GREAT EASTERN the judgment it rendered in the criminal proceedings Chee Gan and ordered Law Union Rock Co. to pay.
33 PHIL. 263 for estafa do not affect this suit, nor can they
TORRES.; January 24, 1916 produce in the present suit the force of res ISSUES
adjudicata. 1. WON there was a breach of the fire hydrant
NATURE Reasoning warranty
Appeal filed through bill of exceptions from the - It is proven that the signatures on the insurance 2. WON the insured violated the Hemp warranty
judgment of the CFI applications reading "Dominado Albay" are false and 3. WON Qua Chee Gan is guilty of overvaluation
forged; that the person who presented himself to Dr. 4. WON Qua Chee Gan caused the fire
FACTS Vidal to be examined was not the real Dominador 5. WON there was an error in the amount of copra
- Francisca Eguaras filed a written complaint in court, Albay, but Castor Garcia who was positively and hemp lost
alleging as a cause of action that her son-in-law identified by Dr. Vidal; that at the time of the 6. WON the claims contained false and fraudulent
Dominador Albay had applied in writing to the application for insurance and the issuance of the statements
defendant insurance company to insure his life for policy which is the subject matter of this suit the real
the sum of P5,000, naming as the beneficiary in case Dominador Albay was informed of all those HELD
of his death the plaintiff Francisca Eguaras; that after machinations, wherefore it is plain that the insurance 1. NO
compliance with the requisites and the investigation contract between the defendant and Dominador - It is argued that he should have 11 fire hydrants in
carried on by the defendant company, it accepted the Albay is null and void because it is false, fraudulent the compound, but he only had 2. We are in
application for insurance and issued the policy; that, and illegal. agreement with the trial Court that the appellant is
said policy being in force, the insured died, and Disposition The judgment appealed from is barred by waiver (or rather estoppel) to claim
despite the fact that the beneficiary submitted reversed and the defendant absolved from the violation of the so-called fire hydrants warranty, for
satisfactory proofs of his death and that the complaint without special finding as to the costs. the reason that knowing fully all that the number of
defendant company investigated the event, still it hydrants demanded therein never existed from the
refused and continues to refuse to pay to the plaintiff QUA CHEE GAN v. LAW UNION AND ROCK very beginning, the appellant nevertheless issued the
the value of the policy. 98 PHIL 85 policies in question subject to such warranty, and
- Defendant set forth in special defense that the REYES; December 17, 1955 received the corresponding premiums.
insurance policy issued in the name of Dominador 2. NO
Albay had been obtained through fraud and deceit FACTS - The insurance company avers that the insured
known and consented to by the interested parties - Qua Chee Gan insured 4 of his bodegas with Law violated the hemp warranty when it admitted that it
and is therefore completely illegal, void, and Union & Rock Insurance Co in 1937. These bodegas had 36 cans of gasoline in the building. It is well to
ineffective. were used for the storage of stocks of copra and of note that gasoline is not specifically mentioned
- A criminal case for frustrated estafa was filed by hemp, baled and loose. among the prohibited articles listed in the so-called
defendant against Ponciano Remigio, Castor Garcia - Fire of undetermined origin that broke out in the "hemp warranty." The cause relied upon by the
and Francisca Eguaras. They were acquitted, and early morning of July 21, 1940, and lasted almost insurer speaks of "oils (animal and/or vegetable
claim that the judgment produces the effect of res one week, gutted and completely destroyed Bodegas and/or mineral and/or their liquid products having a
judicata in the present suit. Nos. 1, 2 and 4, with the merchandise stored flash point below 300o Fahrenheit", and is decidedly
therein. ambiguous and uncertain; for in ordinary parlance,
ISSUE - Qua Chee Gan informed the insurance company of "Oils" mean "lubricants" and not gasoline or
the fire. Fire adjusters of the company conducted an kerosene. And how many insured, it may well be
INSURANCE Page 63

wondered, are in a position to understand or materially less convincing than that required in order excess in the case of the insured is not by itself
determine "flash point below 003o Fahrenheit. Here, to convict the insured of the crime of arson. sufficient to establish fraudulent intent. Certainly, the
again, by reason of the exclusive control of the 5. NO insured's overclaim of 20 per cent in the case at bar,
insurance company over the terms and phraseology - As to the defense that the burned bodegas could duly explained by him to the Court a quo, appears
of the contract, the ambiguity must be held strictly not possibly have contained the quantities of copra puny by comparison (compared to other cases cited
against the insurer and liberally in favor of the and hemp stated in the fire claims, the insurer's case by the court), and can not be regarded as "more
insured, especially to avoid a forfeiture rests almost exclusively on the estimates, inferences than misstatement, more than inadvertence of
- Another point that is in favor of the insured is that and conclusions of its adjuster investigator, mistake, more than a mere error in opinion, more
the gasoline kept in Bodega No. 2 was only incidental Alexander D. Stewart, who examined the premises than a slight exaggeration" that would entitle the
to his business, being no more than a customary 2 during and after the fire. His testimony, however, insurer to avoid the policy. It is well to note that the
day's supply for the five or six motor vehicles used was based on inferences from the photographs and overcharge of 20 per cent was claimed only on a part
for transporting of the stored merchandise). "It is traces found after the fire, and must yield to the (70 per cent) of the hemp stock; had the insured
well settled that the keeping of inflammable oils on contradictory testimony of engineer Andres Bolinas, acted with fraudulent intent, nothing prevented him
the premises though prohibited by the policy does and specially of the then Chief of the Loan from increasing the value of all of his copra, hemp
not void it if such keeping is incidental to the Department of the National Bank's Legaspi branch, and buildings in the same proportion. This also
business." (Bachrach vs. British American Ass. Co., Porfirio Barrios, and of Bank Appraiser Loreto applies to the alleged fraudulent claim for burned
17 Phil. 555, 560) Samson, who actually saw the contents of the empty sacks, that was likewise explained to our
3. NO bodegas shortly before the fire, while inspecting satisfaction and that of the trial Court. The rule is
- The charge that the insured failed or refused to them for the mortgagee Bank that to avoid a policy, the false swearing must be
submit to the examiners of the insurer the books, 6. NO willful and with intent to defraud which was not the
vouchers, etc. demanded by them was found - Appellant insurance company also contends that cause. Of course, the lack of fraudulent intent would
unsubstantiated by the trial Court, and no reason the claims filed by the insured contained false and not authorize the collection of the expected profit
has been shown to alter this finding. fraudulent statements that avoided the insurance under the terms of the polices, and the trial Court
- In view of the discrepancy in the valuations policy. But the trial Court found that the correctly deducted the same from its award.
between the insured and the adjuster Stewart for the discrepancies were a result of the insured's Disposition Decision affirmed
insurer, the Court referred the controversy to a erroneous interpretation of the provisions of the
government auditor, Apolonio Ramos; but the latter insurance policies and claim forms, caused by his ARGENTE v. WEST COAST LIFE
reached a different result from the other two. Not imperfect knowledge of English, and that the 51 PHIL 725
only that, but Ramos reported two different misstatements were innocently made and without MALCOLM; March 19, 1928
valuations that could be reached according to the intent to defraud. The trial court’s ruling must be
methods employed. Clearly then, the charge of upheld. FACTS
fraudulent overvaluation cannot be seriously - For example, the occurrence of previous fires in the - This is an action upon a joint life insurance policy
entertained. premises insured in 1939, altho omitted in the for P15,000 issued by the West Coast Life Insurance
4. NO claims, Exhibits EE and FF, were nevertheless Co., on May 15, 1925, in favor of Bernardo Argente,
- This defense is predicted on the assumption that revealed by the insured in his claims Exhibits Q (filed and his wife, Vicenta de Ocampo, the latter having
the insured was in financial difficulties and set the simultaneously with them), KK and WW. Considering died on November 18, 1925. Fraud in obtaining the
fire to defraud the insurance company, presumably in that all these claims were submitted to the smae policy was pleaded by way of special defense. On the
order to pay off the Philippine National Bank, to agent, and that this same agent had paid the loss issue thus suggested, the court adopted the theory
which most of the insured hemp and copra was caused by the 1939 fire, we find no error in the trial of the defendant, and held the insurance policy null
pledged. This defense is fatally undermined by the Court's acceptance of the insured's explanation that and void, with the result that the complaint was
established fact that, notwithstanding the insurer's the omission in Exhibits EE and FF was due to dismissed, with costs.
refusal to pay the value of the policies the extensive inadvertance, for the insured could hardly expect -Bernardo Argente signed an application for joint
resources of the insured enabled him to pay off the under such circumstances, that the 1939 would pass insurance with his wife in the sum of P2,000. The
National Bank in a short time; and if he was able to unnoticed by the insurance agents. Similarly, the 20 wife, Vicenta de Ocampo, signed a like application for
do so, no motive appears for attempt to defraud the per cent overclaim on 70 per cent of the hemo stock, the same policy.
insurer. While the acquittal of the insured in the was explained by the insured as caused by his belief - Bernardo Argente and his wife was examined by Dr.
arson case is not res judicata on the present civil that he was entitled to include in the claim his Cesareo Sta. Ana, a medical examiner for the West
action, the insurer's evidence, to judge from the expected profit on the 70 per cent of the hemp, Coast Life Insurance Co. which did not show previous
decision in the criminal case, is practically identical in because the same was already contracted for and and existing health problems.
both cases and must lead to the same result, since sold to other parties before the fire occurred. - A temporary policy for P15,000 was issued to
the proof to establish the defense of connivance at Compared with other cases of over-valuation Bernardo Argente and his wife as of May 15, 1925.
the fire in order to defraud the insurer "cannot be recorded in our judicial annals, the 20 per cent In view of the fact that more than thirty days had
INSURANCE Page 64

elapsed since the applicants were examined by the WON the contract of insurance may be rescinded be known by him. And a concealment of such facts
company's physician, each of them was required to vitiates the policy. 'It does not seem to be
file a certificate of health before the policy was necessary . . . that the . . . suppression of the
delivered to them. truth should have been willful.' If it were but an
- On November 18, 1925, Vicenta de Ocampo died of HELD inadvertent omission, yet if it were material to the
cerebral apoplexy. Thereafter Bernardo Argente YES risk and such as the plaintiff should have known to
presented a claim. Following investigation conducted - Bernardo Argente and his wife applications’ were be so, it would render the policy void. But it is held
by the Manager of the Manila office of the insurance false with respect to their state of health during the that if untrue or false answers are given in
company, it was apparently disclosed that the period of five years preceding the date of such response to inquiries and they relate to material
answers given by the insured in their medical applications and that they knew the representations facts the policy is avoided without regard to the
examinations with regard to their health and made by them in their applications were false. The knowledge or fraud of assured, although under the
previous illnesses and medical attendance were question arises as to the state of the law in relation statute statements are representations which must
untrue. West Coast Life Insurance Co. refused to pay thereto. be fraudulent to avoid the policy. So under certain
the claim of Bernardo Argente, and wrote him to the - One ground for the rescission of a contract of codes the important inquiries are whether the
effect that the claim was rejected because the insurance under the Insurance Act is "a concealment was willful and related to a matter
insurance was obtained through fraud and concealment," which in section 25 is defined as "A material to the risk.
misrepresentation. neglect to communicate that which a party knows xxx xxx xxx
- It is admitted that it appears in the Medical and ought to communicate." In an action on a life "If the assured has exclusive knowledge of
Examiner's Report that Bernardo Argente gave false insurance policy where the evidence conclusively material facts, he should fully and fairly disclose
responses. As well as with the Medical Examiner's shows that the answers to questions concerning the same, whether he believes them material or
Report that Vicenta de Ocampo. It is, however, not diseases were untrue, the truth or falsity of the not. But notwithstanding this general rule it will
disputed that Vicenta de Ocampo was taken by a answers become the determining factor. If the policy not infrequently happen, especially in life risks,
patrolman, at the request of her husband, Bernardo was procured by fraudulent representations, the that the assured may have a knowledge actual or
Argente, on May 19, 1924, to the Meisic police contract of insurance apparently set forth therein presumed of material facts, and yet entertain an
station, and from there was transferred to the San was never legally existent. It can fairly be assumed honest belief that they are not material. . . . The
Lazaro Hospital. In San Lazaro Hospital, her case that had the true facts been disclosed by the determination of the point whether there has or
was diagnosed by the admitting physician as assured, the insurance would never have been has not been a material concealment must rest
"alcoholism," but later Doctor Domingo made a granted. largely in all cases upon the form of the questions
diagnosis of probable "manic-depressive psychosis," - In Joyce, The Law of Insurance, second edition, propounded and the exact terms of the contract.
and still, later in Mary Chiles Hospital, made a final volume 3, Chapter LV, is found the following: Thus, where in addition to specifically named
diagnosis of "phycho-neurosis." "The basis of the rule vitiating the contract in diseases the insured was asked whether he had
- Bernardo Argente, while readily conceding most of cases of concealment is that it misleads or had any sickness within ten years, to which he
the facts herein narrated, yet alleges that both he deceives the insurer into accepting the risk, or answered 'No,' and it was proven that within that
and his wife revealed to the company's physician, accepting it at the rate of premium agreed upon; period he had had a slight attack of pharyngitis, it
Doctor Sta. Ana, all the facts concerning their The insurer, relying upon the belief that the was held a question properly for the jury whether
previous illnesses and medical attendance, but that assured will disclose every material fact within his such an inflammation of the throat was a 'sickness'
Doctor Sta. Ana, presumably acting in collusion with actual or presumed knowledge, is misled into a within the intent of the inquiry, and the court
the insurance agent, Jose Geronimo del Rosario, belief that the circumstance withheld does not remarked on the appeal decision that if it could be
failed to record them in the medical reports. The exist, and he is thereby induced to estimate the held as a matter of law that the policy was thereby
evidence on these points consists of the testimony of risk upon a false basis that it does not exist. The avoided, then it was a mere device on the part of
the plaintiff and his subordinate clerk, Apolonio principal question, therefore, must be, Was the insurance companies to obtain money without
Espiritu, on the one hand, and of the testimony of assurer misled or deceived into entering a contract rendering themselves liable under the policy. . . .
Doctor Sta. Ana and Jose Geronimo del Rosario on obligation or in fixing the premium of insurance by " . . . The question should be left to the jury
the other. This was rejected by the Trial Court. Trial a withholding of material information or facts whether the assured truly represented the state of
judge found with the insurance company with regard within the assured's knowledge or presumed his health so as not to mislead or deceive the
to the question of fact. SC agrees. There appears no knowledge? insurer; and if he did not deal in good faith with
motive whatever on the part of Doctor Sta. Ana to "It therefore follows that the assurer in assuming a the insurer in that matter, then the inquiry should
falsify the Medical Examiner's Reports and thereby risk is entitled to know every material fact of which be made, Did he know the state of his health so as
not only jeopardize his career as a physician, but the assured has exclusive or peculiar knowledge, to be able to furnish a proper answer to such
also gravely implicate himself criminally. as well as all material facts which directly tend to questions as are propounded? A Massachusetts
increase the hazard or risk which are known by the case, if construed as it is frequently cited, would
ISSUE assured, or which ought to be or are presumed to be opposed to the above conclusion; but, on the
INSURANCE Page 65

contrary, it sustains it, for the reason that (Rankin vs. Amazon Insurance Co. [1891], 89 Cal., NO
symptoms of consumption had so far developed 203.) - The binding deposit receipt is merely a provisional
themselves within a few months prior to effecting Disposition Judgment affirmed, with the costs of contract and only upon compliance with the ff
the insurance as to induce a reasonable belief that this instance against the appellant. conditions: (1) that the company be satisfied that
the applicant had that fatal disease, and we should the applicant was insurable on standard rates (2)
further construe this case as establishing the rule GREAT PACIFIC LIFE v. CA (NGO HING) that if the company does not accept the application
that such a matter cannot rest alone upon the 89 SCRA 543 and offers a different policy, the insurance contract
assured's belief irrespective of what is a DE CASTRO, J; April 30, 1979 shall not be binding until the applicant accepts the
reasonable belief, but that it ought to be judged by new policy (3) that if the applicant is not found to be
the criterion whether the belief is one fairly NATURE insurable on standard rates and the application is
warranted by the circumstances. A case in Indiana, Petition for certiorari disapproved, the insurance shall not be in force at
however, holds that if the assured has some any time and the premium be returned to the
affection or ailment of one or more of the organs FACTS applicant.
inquired about so well defined and marked as to - On March 14, 1957, private respondent Ngo Hing -This implies the receipt is merely an
materially derange for a time the functions of such filed an application with the Great Pacific Life acknowledgement, on behalf of the company, that
organ, as in the case of Bright's disease, the policy Assurance Co. (Pacific Life) for a 20 year endowment the Cebu branch of Pacific Life had received the
will be avoided by a nondisclosure, irrespective of policy of P50k on the life of his 1 year old daughter, premium and had accepted the application subject to
the fact whether the assured knew of such ailment Helen. Ngo Hing supplied the essetntial data which processing by the insurance company, which will
or not. . . ." petitioner Mondragon, branch manager of the Pacific approve or reject it depending on whether the
- Lastly, appellant contends that even if the Life in Cebu, wrote on the corresponding form in his applicant is insurable on standard rates. As such, the
insurance company had a right to rescind the own handwriting, later typing the data on an receipt was never in force—it does not insure
contract, such right cannot now be enforced in view application form signed by Ngo Hing. The latter paid outright. No liability attaches until the principal
of the provisions of section 47 of the Insurance Act the P1077.75 annual premium but retained P1,317 approves the risk and a receipt is given by the
providing "Whenever a right to rescind a contract of as commission as he was also a duly authorized agent; because private respondent failed to accept
insurance is given to the insurer by any provision of agent of Pacific Life. The binding deposit receipt was Pacific Life’s offer for the Juvenile Triple Action plan,
this chapter, such right must be exercised previous then issued to Ngo Hing; Mondragon handwrote his there was no meeting of the minds and thus no
to the commencement of an action on the contract." strong recommendation for the approval of the contract. Also, being an authorized agent of Pacific
This section was derived from section 2583 of the application on the back of the form. Life, Ngo Hing must have known the company did
California Civil Code, but in contrast thereto, makes - On April 30, Mondragon received a letter from not offer the insurance applied for and merely took a
use of the imperative "must" instead of the Pacific Life which stated that the 20 year endowment chance on Mondragon’s recommendation.
permissive "may." Nevertheless, there are two plan was not available for minors below 7, but that Disposition the decision appealed from is set aside,
answers to the problem as propounded. The first is Pacific Life could consider the same under the absolving Pacific Life from their civil liabilities
that the California law as construed by the code Juvenile Triple Action Plan, advising that if the offer
examiners, at whose recommendation it was was acceptable, the Juvenile Non-Medical Declaration EDILLON v. MANILA BANKERS LIFE
adopted, conceded that "A failure to exercise the be sent to the company. 117 SCRA 187
right (of rescission), cannot, of course, prejudice any -Mondragon allegedly failed to inform Ngo Hing of VASQUEZ; September 30, 1982
defense to the action which the concealment may the non-acceptance of the insurance plan, instead
furnish." (Codes of California Annotated; Tan Chay writing Pacific Life again, recommending the approval NATURE
Heng vs. West Coast Life Insurance Company of the endowment plan to children since customers Appeal from a decision of the CFI
[1927], p. 80, ante.) The second answer is that the had been asking for such coverage since 1954.
insurance company more than one month previous to -On May 28, 1957, Helen died of influenza. Ngo Hing FACTS
the commencement of the present action wrote the sought the payment of the proceeds of the - Sometime in April 1969, Carmen O, Lapuz applied
plaintiff and informed him that the insurance contract insurance, but having failed to do so, filed an action with respondent insurance corporation for insurance
was void because it had been procured through for recovery with the CFI of Cebu. The Court ordered coverage against accident and injuries. In the
fraudulent representations, and offered to refund to Pacific Life to pay P50k with 6% interest, hence this application form which was dated April 15, 1969, she
the plaintiff the premium which the latter had paid petition. gave the date of her birth as July 11, 1904. On the
upon the return of the policy for cancellation. As held same date, she paid the sum of P20.00 representing
in California as to a fire insurance policy, where any ISSUE the premium for which she was issued the
of the material representations are false, the WON the binding deposit receipt constituted a corresponding receipt signed by an authorized agent
insurer's tender of the premium and notice that the temporary contract of the life insurance in question of the respondent insurance corporation. Upon the
policy is canceled, before the commencement of suit filing of said application and the payment of the
thereon, operate to rescind the contract of insurance. HELD premium on the policy applied for, the respondent
INSURANCE Page 66

insurance corporation issued to Carmen O. Lapuz its or FORTY-FIVE (45) DAYS after the insurance
Certificate of Insurance. The policy was to be coverage was applied for. There was sufficient time ISSUE
effective for a period of 90 days. for the private respondent to process the application WON according to Sec. 48 of the Insurance Code,
- On May 31, 1969 or during the effectivity of the and to notice that the applicant was over 60 years of insurance company is barred from rescinding
Insurance, Carmen O. Lapuz died in a vehicular age and thereby cancel the policy on that ground if it contract
accident. was minded to do so. If the private respondent failed
- On June 7, 1969, petitioner Regina L. Edillon, a to act, it is either because it was willing to waive HELD
sister of the insured and who was the named such disqualification; or, through the negligence or - Section 48. Whenever a right to rescind a contract
beneficiary in the policy, filed her claim for the incompetence of its employees for which it has only of insurance is given to the insurer by any provision
proceeds of the insurance, submitting all the itself to blame, it simply overlooked such fact. Under of this chapter, such right must be exercised previous
necessary papers and other requisites with the the circumstances, the insurance corporation is to the commencement of an action on the contract.
private respondent. Her claim having been denied, already deemed in estoppel. Its inaction to revoke After a policy of life insurance made payable on the
Regina L. Edillon instituted this action in the Court of the policy despite a departure from the exclusionary death of the insured shall have been in force during
First Instance of Rizal. condition contained in the said policy constituted a the lifetime of the insured for a period of two years
- In resisting the claim of the petitioner, the waiver of such condition. from the date of its issue or of its last reinstatement,
respondent insurance corporation relies on a Disposition Judgment appealed from is REVERSED the insurer cannot prove that the policy is void ab
provision contained in the Certificate of Insurance, and SET ASIDE and respondent insurance initio or is rescindable by reason of the fraudulent
excluding its liability to pay claims under the policy in corporation is ordered to pay to the petitioner the concealment or misrepresentation of the insured or
behalf of "persons who are under the age of sixteen proceeds of Insurance his agent.
(16) years of age or over the age of sixty (60) years - According to the petitioners, the Insurance Law was
..." It is pointed out that the insured being over sixty HARDING v. COMMERCIAL UNION (supra p.36) amended and the second paragraph of Section 48
(60) years of age when she applied for the insurance added to prevent the insurance company from
coverage, the policy was null and void, and no risk TAN v. CA (PHILIPPINE AMERICAN LIFE exercising a right to rescind after the death of the
on the part of the respondent insurance corporation INSURANCE COMPANY) insured
had arisen therefrom. 174 SCRA 403 - The so-called "incontestability clause" precludes the
- RTC dismissed the complaint. GUTIERREZ; June 29, 1989 insurer from raising the defenses of false
representations or concealment of material facts
ISSUE NATURE insofar as health and previous diseases are
WON the acceptance by the private respondent Review on certiorari of the decision of the Court of concerned if the insurance has been in force for at
insurance corporation of the premium and the Appeals affirming the decision of the Insurance least two years during the insured's lifetime. The
issuance of the corresponding certificate of insurance Commissioner phrase "during the lifetime" found in Section 48
should be deemed a waiver of the exclusionary simply means that the policy is no longer considered
condition of overage stated in the said certificate of FACTS in force after the insured has died. The key phrase in
insurance - On September 23,1973, Tan Lee Siong, father of the second paragraph of Section 48 is "for a period
herein petitioners, applied for life insurance in the of two years."
HELD amount of P 80,000.00 with respondent company. - The policy was issued on November 6,1973 and the
YES Said application was approved and was issued insured died on April 26,1975. The policy was thus in
- The age of the insured Carmen 0. Lapuz was not effective November 6, 1973 force for a period of only one year and five months.
concealed to the insurance company. Her application - On April 26,1975, Tan Lee Siong died of hepatoma Considering that the insured died before the two-
for insurance coverage which was on a printed form (Exhibit B). Petitioners then filed with respondent year period had lapsed, respondent company is not,
furnished by private respondent and which contained company their claim for the proceeds of the life therefore, barred from proving that the policy is void
very few items of information clearly indicated her insurance policy ab initio by reason of the insured's fraudulent
age of the time of filing the same to be almost 65 -respondent company denied petitioners' claim and concealment or misrepresentation.
years of age. Despite such information which could rescinded the policy by reason of the alleged - The petitioners contend that there could have been
hardly be overlooked in the application form, misrepresentation and concealment of material facts no concealment or misrepresentation by their late
considering its prominence thereon and its made by the deceased Tan Lee Siong in his father because Tan Lee Siong did not have to buy
materiality to the coverage applied for, the application for insurance. The premiums paid on the insurance. He was only pressured by insistent
respondent insurance corporation received her policy were thereupon refunded salesmen to do so
payment of premium and issued the corresponding - Petitioners filed on November 27, 1975, a -The legislative answer to the arguments posed by
certificate of insurance without question. The complaint against the former with the Office of the the petitioners is the "incontestability clause" added
accident which resulted in the death of the insured, a Insurance Commissioner. Commissioner denied by the second paragraph of Section 48. The insurer
risk covered by the policy, occurred on May 31, 1969 petition. CA affirmed Commissioners decision has two years from the date of issuance of the
INSURANCE Page 67

insurance contract or of its last reinstatement within well known to the plaintiff and his said regulations of the defendant insurance company,
which to contest the policy, whether or not, the coconspirators. plaintiff and coconspirators caused Tan Ceang to
insured still lives within such period. After two years, 3. Locsin, in his capacity as medical examiner for sign a health certificate for reinstatement; that
the defenses of concealment or misrepresentation, the defendant, prepared and falsified the the said temporary policy was delivered by
no matter how patent or well founded, no longer lie necessary medical certificate, in which it was defendant to the insured on April 10, 1925, in
made to appear, among other things, that Tan the belief that said statements and
TAN CHAY HENG v. WEST COAST LIFE Ceang had never used morphine, cocaine or any representations were true and in reliance
INSURANCE other drug; that he was then in good health and thereon.
51 PHIL 80 had never consulted any physician; that he had 7. 2 ½ months after the supposed medical
JOHNS; November 21, 1927 never spit blood; and that there was no sign of examination above referred to, and exactly 1
either present or past disease of his lungs; month after the date of the health certificate for
FACTS whereas in truth and in fact, plaintiff and reinstatement above set forth, Tan Ceang died in
- Plaintiff alleges that defendant accepted and coconspirators well knew, Tan Ceang was Valladolid, Occidental Negros, of pulmonary
approved a life insurance policy of for the sum of addicted to morphine, cocaine, and opium and tuberculosis, the same illness from which
P10,000 in which the plaintiff was the sole had been convicted and imprisoned therefor, and suffering at the time it is supposed he was
beneficiary; that the policy was issued upon the for about three year prior thereto had been examined by Dr. V. S. Locsin, but that the
payment by the said Tan Ceang of the first year's suffering from pulmonary tuberculosis. plaintiff coconspirators, pursuant to their
premium amounting to P936; that in and by its 4. Plaintiff caused a confidential report to the conspiracy, caused the said Dr. V. S. Locsin to
terms, the defendant agreed to pay the plaintiff as defendant insurance company to be signed by state falsely in the certificate of death that the
beneficiary the amount of the policy upon the receipt one V. Sy Yock Kian, who was an employee of Go said Tan Ceang had died of cerebral
of the proofs of the death of the insured while the Chulian, in which it was falsely represented that hemorrhage.
policy was in force; that without any premium being Tan Ceang was worth about P40,000, had an - Defendant also alleges that plaintiff was, like V. Sy
due or unpaid, Tan Ceang died on May 10, 1925; annual income of from eight to ten thousand Yock Kian, an employee of Go Chulian; that the latter
that in June, 1925, plaintiff submitted the proofs of pesos net, had the appearance of good health, was the ringleader of a gang of malefactors, who,
the death of Tan Ceang with a claim for the payment and never had tuberculosis. during, and for some years previous to the dates
of the policy which the defendant refused to pay, for 5. After said application for insurance, medical above mentioned, were engaged in the illicit
which he prays for a corresponding judgment, with certificate and confidential report had been enterprise of procuring fraudulent life insurances
legal interest from the date of the policy, and costs. prepared and falsified, plaintiff and from the present defendant, similar to the one in
- Defendant alleges that the insurance policy on the coconspirators caused the same to be forwarded question, and which enterprise was capitalized by
life of Tan Ceang, upon which plaintiff's action is to the defendant at its office in Manila, the him by furnishing the funds with which to pay the
based, was obtained by the plaintiff in confabulation medical certificate thru the said Dr. V. S. Locsin premium on said fraudulent insurance; that the said
with one Go Chulian, of Bacolod, Negros Occidental; as medical examiner, and said application for Go Chulian was the one who furnished the money
Francisco Sanchez of the same place; and Dr. V. S. insurance and confidential report thru the said with which to pay the first and only annual premium
Locsin, of La Carlota, Negros Occidental, thru fraud Francisco Sanchez in his capacity as one of the on the insurance here in question, amounting to
and deceit perpetrated against this defendant in the agents of the defendant insurance company in P936.50; that the said Go Chulian, on August 28,
following manner, to wit: the Province of Occidental Negros; that the 1926, was convicted by the Court of First Instance of
1. Go, Sanchez and Locsin, caused Tan Caeng to defendant, believing that the representations the City of Manila, in criminal case No. 31425 of that
sign an application for insurance with the made in said document were true, and relying court, of the crime of falsification of private
defendant in the sum of P10,000, in which it was thereon, provisionally accepted the said documents in connection with an fraudulent
said that Tan Ceang was single and was a application for insurance on the life of Tan Ceang insurance, similar to the present, committed against
merchant, and that the plaintiff Tan Chai Heng, in the sum of P10,000 and issued a temporary this defendant in the month of September, 1924;
the beneficiary, was his nephew, whereas in policy pending the final approval or disapproval that in the same case the said Francisco Sanchez
truth and in fact and as the plaintiff and his said of said application by defendant's home-office in was one of the coaccused of the said Go Chulian but
coconspirators well knew, the said Tan Ceang San Francisco, California, where in case of was discharged from the complaint, because he
was not single but was married and had several approval a permanent policy was to be issued; offered himself and was utilized as a state's witness;
children; and was not a merchant but a mere that such permanent policy was never delivered that there is another civil action now pending against
employee of Tan Quina from whom he received to the plaintiff because defendant discovered the Go Chulian and Sanchez in the Court of First
only a meager salary, and that plaintiff was not a fraud before its delivery. Instance of Manila (civil case No. 28680), in which
nephew of the said Tan Ceang. 6. That the first agreed annual premium on the the present defendant is the plaintiff, for the
2. Tan Ceang was seriously ill, suffering from insurance in question of P936.50 not having recovery of the amounts of two insurance policies
pulmonary tuberculosis of about three years' been paid within 60 days after medical aggregating P19,000, fraudulently obtained by the
duration, which illness was incurable and was examination of the applicant as required by the said Go Chulian and Sanchez.
INSURANCE Page 68

- To this, plaintiff filed a demurrer which was insurance policy for P20,000 covering the same
granted. PIONEER INSURANCE AND SURETY properties, from Federal Insurance Company. This
CORPORATION v. YAP policy was procured without notice to and the written
ISSUE 61 SCRA 426 consent of Pioneer, and was therefore not noted as a
WON defense is barred by Art. 47 FERNANDEZ; December 19, 1974 co-insurance in Policy 4219.
- December 19, 1962: Fire burned Yap’s store
HELD NATURE
NO Appeal by certiorari from CA decision affirming a CFI ISSUE
Ratio The word "rescind" has a well defined legal decision which declared plaintiff Yap entitled to WON petitioner should be absolved from liability on
meaning, and as applied to contracts, it presupposes recover from defendant Pioneer Insurance and Fire insurance Policy No. 4219 on account of any
the existence of a contract to rescind. Surety Corp, the full amount of the damage inquired violation by respondent Yap of the co-insurance
Reasoning in Policy No. 4219 clause therein
- Plaintiff vigorously contends that section 47 of the
Insurance Act should be applied, and that when so FACTS HELD
applied, defendant is barred and estopped to plead - Yap owned a store in a 2 storey building, where she YES
and set forth the matters alleged in its special sold shopping bags and footwear. Her son-in-law was - The petitioner should be absolved.
defense. That section is as follows: in charge of the store Reasoning
Whenever a right to rescind a contract of insurance - April 19, 1962- Yap took out Fire Insurance Policy - There was a violation by Yap of the co-insurance
is given to the insurer by any provision of this No. 4216 from Pioneer with a face value of P25,000 clause contained in Policy No. 4219 which resulted in
chapter, such right must be exercised previous to covering her stocks, office furniture, fixtures, etc. the avoidance of the petitioner’s liability.
the commencement of an action on the contract. - among the conditions set forth: - By the plain terms of the policy, other insurance
- It will be noted that defendant does not seek to The Insured shall give notice to the without the consent of petitioner would ipso facto
have the alleged insurance contract rescinded. It Company of any insurance or insurances already avoid the contract. It required no affirmative act of
denies that it ever made any contract of insurance on effected, or which may subsequently be election on the part of the company to make
the life of Tan Ceang or that any such a contract ever effected, covering any of the property hereby operative the clause avoiding the contract, wherever
existed, and that is the question which it seeks to insured, and unless such notice be given and the specified conditions should occur. Its obligations
have litigated by its special defense. In the very the particulars of such insurance or insurances ceased, unless, being informed of the fact, it
nature of things, if the defendant never made or be stated in, or endorsed on this Policy by or on consented to the additional insurance.
entered into the contract in question, there is no behalf of the Company before the occurrence of - The obvious purpose of the aforesaid requirement
contract to rescind, and, hence, section 47 upon any loss or damage, all benefits under this in the policy is to prevent over-insurance and thus
which the lower based its decision in sustaining the Policy shall be forfeited. (emphasis supplied) avert the perpetration of fraud. The public, as well as
demurrer does not apply. As stated, an action to It is understood that, except as may be the insurer, is interested in preventing the situation
rescind a contract is founded upon and presupposes stated on the face of this policy there is no other in which a fire would be profitable to the insured.
the existence of the contract which is sought to be insurance on the property hereby covered and According to Justice Story: "The insured has no right
rescinded. If all of the material matters set forth and no other insurance is allowed except by the to complain, for he assents to comply with all the
alleged in the defendant's special plea are true, there consent of the Company endorsed hereon. Any stipulation on his side, in order to entitle himself to
was no valid contract of insurance, for the simple false declaration or breach or this condition will the benefit of the contract, which, upon reason or
reason that the minds of the parties never met and render this policy null and void. principle, he has no right to ask the court to dispense
never agreed upon the terms and conditions of the - At the time of insurance of Policy 4219(April 19, with the performance of his own part of the
contract. We are clearly of the opinion that, if such 1962), an insurance policy for P20,000 issued by the agreement, and yet to bind the other party to
matters are known to exist by a preponderance of Great American Insurance Company covering the obligations, which, but for those stipulation would
the evidence, they would constitute a valid defense same properties was noted on said policy as co- not have been entered into."
to plaintiff's cause of action. Upon the question as to insurance. Disposition the appealed judgment of the Court of
whether or not they or are not true, we do not at this - August 29, 1962 : parties executed an Appeals is reversed and set aside, and the petitioner
time have or express any opinion, but we are clear endorsement on Policy 4219 stating: absolved from all liability under the policy.
that section 47 does not apply to the allegations It is hereby declared and agreed that the co-
made in the answer, and that the trial court erred in insurance existing at present under this policy is as NEW LIFE ENTERPRISES v. CA
sustaining the demurrer. follows: P20,000.00 � Northwest Ins., and not as 207 SCRA 609
Disposition The judgment of the lower court is originally stated. (emphasis supplied) REGALADO; March 31, 1992
reversed and the case is remanded for such other Except as varied by this endorsement, all other
and further proceedings as are not inconsistent with terms and conditions remain unchanged. FACTS
this opinion, with costs against the plaintiff. - September 26, 1962: Yap took out another fire
INSURANCE Page 69

- Julian Sy and Jose Sy Bang are partners engaged in benefits thereunder if we are to follow the express have hesitated or plainly desisted from entering into
the business of selling construction materials under stipulation in Policy Condition No. 3. such contract. Hence, the insured was guilty of clear
the business name “New Life Enterprises.” Julian Sy - The terms of the contract are clear and fraud.
insured against fire the stocks in trade of New Life unambiguous. The insured is specifically required to - As the insurance policy against fire expressly
Enterprises with Western Guaranty Corporation, disclose to the insurer any other insurance and its required that notice should be given by the insured
Reliance Surety and Insurance Co. Inc., and particulars which he may have effected on the same of other insurance upon the same property, the total
Equitable Insurance Corporation in the aggregate subject matter. The knowledge of such insurance by absence of such notice nullifies the policy.
amount of P1,550,000.00. When the building where the insurer's agents, even assuming the acquisition - Additionally, insofar as the liability of respondent
New Life Enterprises was located, along with the thereof by the former, is not the "notice" that would Reliance is concerned, it is not denied that the
stocks in trade therein, were gutted by fire, stop the insurers from denying the claim. Besides, complaint for recovery was filed in court by
petitioners filed an insurance claim against the three the so-called theory of imputed knowledge, that is, petitioners only on January 31, 1984, or after more
companies. The insurance companies all denied knowledge of the agent is knowledge of the principal, than one (1) year had elapsed from petitioners'
Julian Sy’s claim on the ground of “breach of policy aside from being of dubious applicability here has receipt of the insurers' letter of denial on November
condition,” (i.e., the “other insurance” clause which likewise been roundly refuted by respondent court 29, 1982.
required New Life Enterprises to inform each of the whose factual findings we find acceptable. The mere - The condition contained in an insurance policy that
insurance companies in case the former insures with fact that Yap Kam Chuan was an agent for both claims must be presented within one year after
another company the same property already insured Reliance and Equitable does not justify the allegation rejection is not merely a procedural requirement but
by each of the insurance companies). that the two are sister companies. Availment of the an important matter essential to a prompt settlement
- Because of the denial of their claims for payment services of the same agents and adjusters by of claims against insurance companies as it demands
by the 3 insurance companies, petitioners filed different companies is a common practice in the that insurance suits be brought by the insured while
separate civil actions against the former before the insurance business and such facts do not warrant the the evidence as to the origin and cause of
Regional Trial Court of Lucena City, which cases were speculative conclusion of the trial court. destruction have not yet disappeared.
consolidated for trial. The trial court ruled in favor of - Considering the terms of the policy which required
petitioner. However, the Court of Appeals reversed the insured to declare other insurances, the QUA CHEE GAN v. LAW UNION (supra p.48)
the trial court’s decision, found petitioner to have statement in question must be deemed to be a
violated Clauses 3 and 27 of the separate insurance statement (warranty) binding on both insurer and
policies issued by the 3 companies, and exonerated insured, that there were no other insurance on the
the insurance companies from liability. property. The annotation then, must be deemed to
be a warranty that the property was not insured by
ISSUE any other policy. Violation thereof entitled the insurer
WON petitioners violated the “Other Insurance to rescind.
Clause” of the insurance policies - The obvious purpose of the aforesaid requirement YOUNG v. MIDLAND TEXTILE INSURANCE CO.
in the policy is to prevent over-insurance and thus 30 PHIL 617
HELD avert the perpetration of fraud. The public, as well as JOHNSON; March 31, 1915
YES the insurer, is interested in preventing the situation
- Petitioners admit that the respective insurance in which a fire would be profitable to the insured. The FACTS
policies issued by private respondents did not state insured has no right to complain, for he assents to - K.S. Young had a candy and fruit store on the
or endorse thereon the other insurance coverage comply with all the stipulations on his side, in order Escolta, Manila, and occupied a building at 321 Calle
obtained or subsequently effected on the same to entitle himself to the benefit of the contract, Claveria, as a residence and bodega. The Midland
stocks in trade for the loss of which compensation is which, upon reason or principle, he has no right to Textile Insurance Co. in consideration of the payment
claimed by petitioners. It is further admitted by ask the court to dispense with the performance of his of a premium of P60, entered into a contract of
petitioners that Equitable's policy stated "nil" in the own part of the agreement, and yet to bind the other insurance with Young by the terms of which the
space thereon requiring indication of any co- party to obligations, which, but for those stipulations, company, upon certain conditions, promised to pay
insurance although there were 3 policies subsisting would not nave been entered into. Young the sum of P3,000 in case said residence and
on the same stocks in trade at the time of the loss, - It is not disputed that the insured failed to reveal bodega and contents should be destroyed by fire.
namely, that of Western in the amount of before the loss three other insurances. By reason of - One of the conditions of the contract is: "Warranty
P350,000.00 and two 2 policies of Reliance in the said unrevealed insurances, the insured had been B – It is hereby declared and agreed that during the
total amount of P1,000,000.00. guilty of a false declaration; a clear pendency of this policy no hazardous goods be
- The coverage by other insurance or co-insurance misrepresentation and a vital one because where the stored or kept for sale, and no hazardous trade or
effected or subsequently arranged by petitioners insured had been asked to reveal but did not, that process be carried on, in the building to which this
were neither stated nor endorsed in the policies of was deception. Otherwise stated, had the insurer insurance applies, or in any building connected
the 3 private respondents, warranting forfeiture of all known that there were many co-insurances, it could therewith."
INSURANCE Page 70

- Young placed in the residence and bodega three - In the present case no claim is made that the policy upon the theory of a different risk. He was
boxes filled with fireworks. Said residence and "hazardous goods" were placed in the bodega for enjoying, if his contention may be allowed, the
bodega and the contents thereof were partially present or daily use. It is admitted that they were benefits of an insurance policy upon one risk,
destroyed by fire. placed in the bodega "for future use," or for future whereas, as a matter of fact, it was issued upon an
- The fireworks had been given to Young by the consumption, or for safe keeping. It seems clear to entirely different risk. The defendant had neither
former owner of the Luneta Candy Store. He us that the "hazardous goods" in question were been paid nor had issued a policy to cover the
intended to use them in the celebration of the "stored" in the bodega, as that word is generally increased risk. An increase of risk which is
Chinese New Year. However, the authorities of the defined. That being true, suppose the defendant had substantial and which is continued for a considerable
city of Manila had prohibited the use of fireworks on made an examination of the premises, even in the period of time, is a direct and certain injury to the
said occasion, so Young then placed them in the absence of a fire, and had found the "hazardous insurer, and changes the basis upon which the
bodega where they remained from the 4th or 5th of goods" there, would it not have been justified in contract of insurance rests.
February, 1913 until after the fire of March 18, 1913. declaring the policy null and of no effect by reason of Disposition Decision of the lower court is
- Both parties agree that the fireworks come within a violation of its terms? If it might, then may it not REVERSED.
the phrase "hazardous goods," mentioned in repudiate its liability, even after the fire? If the
"Warranty B" of the policy; that the fireworks were "warranty" is a term of the contract, will not its TAN v. CA (supra p.51)
found in a part of the building not destroyed by the violation cause a breach and justify noncompliance
fire and that they in no way contributed to the fire, or repudiation? AREOLA v. CA (supra p.26)
or to the loss that resulted. - Contracts of insurance are contracts of indemnity,
- The lower court rendered a judgment in favor of upon the terms and conditions specified therein. TAN CHAY v. WEST COAST (supra p.51)
Young for the sum of P2,708.78, and costs. Parties have a right to impose such reasonable
conditions at the time of the making of the contract FILIPINAS LIFE ASSURANCE v. NAVA
ISSUE as they deem wise and necessary. The rate of 17 SCRA 210
1. WON the placing of the fireworks in the building premium is measured by the character of the risk BAUTISTA ANGELO; May 20, 1966
insured, they being "hazardous goods," was a assumed. The insurer, for a comparatively small
violation of the terms of the contract of insurance consideration, undertakes to guarantee the insured NATURE
and especially of "Warranty B." against loss or damage, upon the terms and Petition for review of a decision of the Court of
conditions agreed upon, and upon no other. When Appeals
HELD the insurer is called upon to pay, in case of loss, he
1. YES. may justly insist upon a fulfillment of the terms of FACTS
Reasoning It is admitted by both parties that the the contract. If the insured, cannot bring himself - Before the war, Nava entered into a contract of
fireworks are hazardous goods. The defendant within the terms and conditions of the contract, he is insurance with Insular Life Assurance Co., Ltd. (face
alleged that they were "stored." The plaintiff not entitled to recover for any loss suffered. The value of P5k), and 17 separate contracts of life
contends that under all the facts and circumstances terms of the contract constitute the measure of the insurance with Filipinas Life Assurance Co. (total face
of the case, they were not “stored” in said building, insurer's liability. If the contract has been value of P90k). Each and everyone of the 18 policies
and that the placing of them in the building was not terminated, by a violation of its terms on the part of issued by defendants to plaintiff contains a loan
a violation of the terms of the contract. the insured, there can be no recovery. Compliance clause of the following tenor:
- Whether a particular article is "stored" or not must, with the terms of the contract is a condition Policy loans. After three full years'
in some degree, depend upon the intention of the precedent to the right of recovery. premiums have been paid upon this Policy, if no
parties. Nearly all of the cases cited by the lower - Young argues that since the "storing" of the premium payment is in default, the Company,
court are cases where the article was being put to fireworks on the premises did not contribute in any subject to its then existing rules, will advance on
some reasonable and actual use, which might easily way to the damage occasioned by the fire, he should proper assignment and delivery of this Policy and
have been permitted by the terms of the policy, and be permitted to recover. That argument, however, is on the sole security thereof a sum equal to, or at
within the intention of the parties, and excepted from beside the question, if the "storing" was a violation the option of the owner less than, the cash value
the operation of the warranty, like the present. of the terms of the contract. The violation of the specified in the Schedule of Policy Values, less any
- (1) Where merchants have had or kept the terms of the contract, by virtue of the provisions of existing indebtedness on or secured by this Policy
"hazardous" articles in small quantities, and for the policy itself, terminated, at the election of either and any unpaid balance of the premium for the
actual daily use, for sale, such as gasoline, party, the contractual relations. current policy-year; provided interest at six per
gunpowder, etc.; (2) Where such articles have been - Young paid a premium based upon the risk at the centum per annum on the whole amount of the
brought on the premises for actual use thereon, and time the policy was issued. Certainly, the placing of loan is paid in advance to the end of the current
in small quantities, such as oil, paints, etc; and (3) the firecrackers in the building insured increased the policy-year. At the end of the current policy-year
Where such articles or goods were used for lighting risk. Young had not paid a premium based upon the interest at the same rate for one year in advance
purposes, and in small quantities. increased risk, neither had the defendant issued a will be due and payable, and annually thereafter,
INSURANCE Page 71

and if not so paid will be added to the principal - CFI: (1) rescinded the insurance contracts; (2) concerned for whether he likes it or not that
and bear the same rate of interest. Failure to ordered defendant Filipinas Life Assurance Co. to pay premium has to be paid lest he allows the lapse of
repay any such loan or interest shall not avoid this plaintiff the amount of P32,072.60; and (3) ordered his policy. Consequently, the payment of premiums
Policy unless the total indebtedness shall equal or defendant Insular Life Assurance Co., Ltd. to pay on the life insurance policies made by Nava before
exceed the full amount of the loan value available plaintiff the amount of P2,574.00 and during the war up to the time he applied for the
hereunder. - CA affirmed. loan in question with petitioners should be
Any indebtedness on this Policy shall first be considered likewise as valid payments upon the
deducted from any money payable or in any ISSUES theory that such insurance policies are in the nature
settlement under this Policy. 1. WON CA erred in ruling that as a consequence of of a contractual obligation within the meaning of the
- Nava had so far paid to Insular a total of P2,574; the decision in the Haw Pia case petitioners violated civil law. In effect, therefore, those payments were
and to Filipinas Life, a total of P32,072.60. the loan clause contained in the insurance policies made by a debtor to a creditor within the meaning of
- April 28, 1948: Nava applied to the companies for a thereby entitling respondent to their rescission the requirement of the regulations of the Insurance
P5k loan in line with the loan clause, but they 2. WON CA erred in ruling that by virtue of Article Commissioner and as such they can offer no excuse
refused to grant it because certain regulations issued 1295 of the old Civil Code petitioners should refund to petitioners for refusing to grant the loan as
by the Insurance Commissioner required the to defendant all the premiums paid on his insurance contemplated in the loan clause embodied in the
insurance companies to withhold the payments on policies as a consequence of their rescission policies in question.
premiums made during the Japanese occupation 3. WON CA erred in not ruling that, even if - It is clear from the foregoing that the petitioners
because the same shall be subject to future respondent is entitled to the rescission of said violated the loan clause embodied in each of the 18
adjustments " as soon as debtor-creditor relationship insurance policies, he can only recover their cash life insurance policies issued to respondent to rescind
is established" and because of such process of surrender value at the time the complaint was filed all said policies under Section 69 of the Insurance
"withholding" plaintiff was not entitled to borrow any Act, which provides: "The violation of a material
amount until such adjustment has been made. HELD warranty, or other material provision of a policy, on
- Sept 30, 1948: Nava called the attention of the 1. NO. the part of either party thereto, entitles the other to
insurance companies to the SC decision (Haw Pia v. - Even assuming the validity of the Insurance rescind."
China Banking Corporation) establishing and Commissioner’s regulations, the fact however is that - "The general rule is that a breach of the agreement
recognizing the relationship of debtor and creditor such requirement has already lost its legal effect and to make the loan does not entitle the insured to
with respect to payments in fiat currency made value when our Supreme Court rendered its decision rescind the contract," is not controlling in this
during the Japanese occupation on pre-war in the Haw Pia case wherein it was declared, among jurisdiction. Firstly, it was not shown that the
obligations. others, that all payments made in fiat currency insurance laws in the states where said ruling
- Companies still refused saying that the SC decision during the Japanese occupation in relation with any prevails contain a provision identical to Section 69 of
was not applicable to transactions undertaken during contractual obligation executed before the war were our Insurance Law we quoted above, and secondly,
Japanese occupation when they relate to life valid to all intents and purposes, and yet petitioners the rule cited by Vance is not a rule uniformly
insurance policies. apparently did not give any importance to such followed by all states in the US, for on this matter
- Feb 4, 1949: Nava was again refused even if the decision for in their opinion it does not have any there is a marked divergence of opinion.
total amount of the cash surrender values of the 18 application to transactions which have any relation to 2. NO
policies reached the sum of P9,468.29. payment of premiums on life insurance policies. - Considering that our Insurance Law does not
- Feb 10, 1949: Nava brought case to the CFI Manila - It cannot be denied that a life insurance policy contain an express provision as to what the court
praying for the rescission of the abovementioned 18 involves a contractual obligation wherein the insured should do in cases of rescission of an insurance
policies and for the refund to him of all the premiums becomes duty bound to pay the premiums agreed policy under Section 69, the provision that should
so far paid by him to defendants in the amount of upon, lest he runs the risk of having his insurance apply is that embodied in Article 1225 of the old Civil
P31,633.80, plus 6% interest thereon as damages policy lapse if he fails to pay such premiums. Code, as postulated in Article 16 of the same Code,
- Nov 28, 1951: companies passed a resolution - The fact that if the insured had paid in full the which provides that on matters which are not
which was approved by the Insurance Commissioner, premiums corresponding to the first 3 years of the governed by special laws the provisions of said Code
giving full credit to all premium payments made by life of his policy he cannot be considered delinquent shall supplement its deficiency. And said Article 1295
their policyholders in fiat currency during the that would cause the lapse of his policy if the same provides:
Japanese occupation on account of pre-war policies contains an automatic premium payment clause ART. 1295. Rescission makes necessary the return
for which reason they filed an amended answer cannot divest such policy of its contractual nature, of the things which were the subject-matter of the
offering to pay plaintiff the amount of P9,468.29 for the result of such failure would only be for him to contract, with their fruits, and of the price paid,
which represents the aggregate cash surrender pay later the premium plus the corresponding with interest thereon. ...xxx
values of all the policies in question as of February interest depending upon the condition of the policy. - Said the petitioners: "Recovery of the full amount
10, 1949, but apparently this offer was refused. But certainly it does not cease to be a contractual of the premium after the insurer has sustained for
liability insofar as the payment of that premium is sometime the risk of the insurance and the insured
INSURANCE Page 72

has enjoyed the benefit of protection is obviously still alive and damages were awarded, not for his Certiorari
unjust and is so recognized by the better death, but for physical injuries suffered.
authorities." The ruling above quoted merely FACTS
represents the minority rule in the US, the majority ISSUE/S - Oct. 22, 1986: Carlie Surposa was insured with
rule being that the insured can recover all premiums WON the proximate cause is the overturning of the Finman General Assurance Corporation under Finman
paid, in some cases with interest in case of wrongful bus or the fire General Teachers Protection Plan Master Policy No.
cancellation, repudiation, termination or rescission of 2005 and Individual Policy No. 08924 with his
the contract of life insurance. HELD parents, spouses Julia and Carlos Surposa, and
- Contention that because respondent cannot restore - The proximate cause is the overturning of the bus. brothers Christopher, Charles, Chester and Clifton, all
to petitioners the "value of the benefit of protection" - Ordinarily, when a bus overturns and pins down surnamed, Surposa, as beneficiaries.
which he might have received under the 18 life passenger, merely causing him injuries. If through - While said insurance policy was in full force and
insurance policies in question he is not entitled to some event, unexpected and extraordinary, the bus effect, the insured, Carlie Surposa, died on October
rescind them under the provision of Article 1295 of is set on fire, and passenger is burned to death, one 18, 1988 as a result of a stab wound inflicted by one
the old Civil Code, is untenable because said article might contend that the proximate cause was the fire of the 3 unidentified men without provocation and
only contemplates a transaction whether material and not the overturning of the vehicle. warning on the part of the former as he and his
things are involved, and do not refer to intangible - But here, the proximate cause of Bataclan’s death cousin, Winston Surposa, were waiting for a ride on
ones which cannot be the subject of restoration, for is the overturning of the bus, this for the reason that their way home after attending the celebration of the
to interpret it otherwise would be to defeat the law when the vehicle turned not only on its side but "Maskarra Annual Festival."
itself with the result that rescission can never be had completely on its back, leaking of gasoline from the - Thereafter, Julia Surposa and the other
under Section 69 of our Insurance Law. tank was not unnatural or unexpected. beneficiaries of said insurance policy filed a written
- It cannot be denied that petitioners had in turn - The coming of the men with the torch was in notice of claim with the FINMAN Corp which denied
already derived material benefits from the use of response to the call for help, made only not by the said claim contending that murder and assault are
premiums paid to them by respondent before, during passengers but even the driver and conductor, and not within the scope of the coverage of the insurance
and after the last war from which they must have because it was very dark, about 2:30 am, rescuers policy.
realized huge profits, and in this light alone had to carry a light with them. Coming as they did - Feb. 24, 1989: Surposa filed a complaint with the
petitioners cannot claim prejudice or unfairness if from rural area where lanterns and flashlights were Insurance Commission which subsequently ordered
they are ordered to refund the premiums paid by not available, they had to use a torch. What was FINMAN to pay Surposa the proceeds of the policy
respondents. more natural than that said rescuers should with interest.
3. NO. innocently approach the overturned vehicle to extend - CA affirmed said decision.
- Issue is corollary to preceding issue. No need to aid.
refute. - The coming of the men with the torch was to be ISSUE
Disposition Decision appealed from is AFFIRMED. expected, and was a natural sequence of the WON CA committed GAD in applying the principle of
Costs against petitioners overturning of the bus, the trapping of some of its "expresso unius exclusio alterius" in a personal
passengers and the call for outside help. accident insurance policy (since death resulting from
CHAPTER VII. RISKS AND COVERAGES - The burning of bus can also in part be attributed to murder and/or assault are impliedly excluded in said
negligence of carrier, through its driver and insurance policy considering that the cause of death
VDA. DE BATACLAN v. MEDINA conductor. They, or at least the driver, should have of the insured was not accidental but rather a
102 PHIL 181 known that in the position in which the overturned deliberate and intentional act of the assailant in
MONTEMAYOR; October 22, 1957 bus was, gasoline could and must have leaked from killing the former as indicated by the location of the
the gasoline tank and soaked the area in and around lone stab wound on the insured) [TF they cannot be
FACTS the bus. Gasoline can be smelt and detected even made to indemnify the Surposa heirs]
- Juan Bataclan rode Bus 30 of Medina Transport, from a distance, and yet neither the driver nor the
driven by Saylon, shortly after midnight. The bus conductor would appear to have cautioned or taken HELD
was running very fast. One of the front tires burst. steps to warn rescuers not to bring the lighted torch NO
Bus fell into canal and turned turtle. 4 passengers too near the bus. - The record is barren of any circumstance showing
couldn’t get out, including Bataclan. Gasoline began how the stab wound was inflicted. While the act may
to leak from the overturned bus. 10 men came to FINMAN GENERAL ASSURANCE CORPORATION not exempt the unknown perpetrator from criminal
help. 1 carried a torch and when he approached the v. CA (SURPOSA) liability, the fact remains that the happening was a
bus, fire started, killing the trapped passengers. 213 SCRA 493 pure accident on the part of the victim. The insured
- TC opined that proximate cause of Bataclan’s NOCON; September 2, 1992 died from an event that took place without his
death was not the overturning of bus but the fire. At foresight or expectation, an event that proceeded
the time fire started, Bataclan, though injured, was NATURE
INSURANCE Page 73

from an unusual effect of a known cause and, - NPC vs. CA [1986]~ It is well settled that which caused the latter to ask for help. While at first
therefore, not expected. contracts of insurance are to be construed liberally in he declined the invitation of Atty. Ojeda to go with
Reasoning favor of the insured and strictly against the insurer. him to his residence to inquire into what was going
- De la Cruz vs. Capital Insurance & Surety Co., Inc Thus ambiguity in the words of an insurance contract on because he was not a regular policeman, he later
(1966)~ The terms "accident" and "accidental" as should be interpreted in favor of its beneficiary. agreed to come along when prompted by the traffic
used in insurance contracts have not acquired any Disposition DENIED for lack of merit. policeman, and upon approaching the gate of the
technical meaning, and are construed by the courts residence he was shot and died. The circumstance
in their ordinary and common acceptation. Thus, the CALANOC v. CA that he was a mere watchman and had no duty to
terms have been taken to mean that which happen 98 PHIL 79 heed the call of Atty. Ojeda should not be taken as a
by chance or fortuitously, without intention and BAUTISTA; December 16, 1955 capricious desire on his part to expose his life to
design, and which is unexpected, unusual, and danger considering the fact that the place he was in
unforeseen. An accident is an event that takes place FACTS duty-bound to guard was only a block away. In
without one's foresight or expectation an event that - Basilio was a watchman of the Manila Auto Supply volunteering to extend help under the situation, he
proceeds from an unknown cause, or is an unusual located at the corner of Avenida Rizal and Zurbaran. might have thought, rightly or wrongly, that to know
effect of a known cause and, therefore, not He secured a life insurance policy from the Philippine the truth was in the interest of his employer it being
expected. American Life Insurance Company in the amount of a matter that affects the security of the
Ratio The generally accepted rule is that, death or P2,000 to which was attached a supplementary neighborhood. No doubt there was some risk coming
injury does not result from accident or accidental contract covering death by accident. On January 25, to him in pursuing that errand, but that risk always
means within the terms of an accident-policy if it is 1951, he died of a gunshot wound on the occasion of existed it being inherent in the position he was
the natural result of the insured's voluntary act, a robbery committed in the house of Atty. Ojeda at holding. He cannot therefore be blamed solely for
unaccompanied by anything unforeseen except the the corner of Oroquieta and Zurbaran streets. doing what he believed was in keeping with his duty
death or injury. There is no accident when a Calanoc, the widow, was paid the sum of P2,000, as a watchman and as a citizen. And he cannot be
deliberate act is performed unless some additional, face value of the policy, but when she demanded the considered as making an arrest as an officer of the
unexpected, independent, and unforeseen happening payment of the additional sum of P2,000 law, as contended, simply because he went with the
occurs which produces or brings about the result of representing the value of the supplemental policy, traffic policeman, for certainly he did not go there for
injury or death. In other words, where the death or the company refused alleging, as main defense, that that purpose nor was he asked to do so by the
injury is not the natural or probable result of the the deceased died because he was murdered by a policeman.
insured's voluntary act, or if something unforeseen person who took part in the commission of the - Much less can it be pretended that Basilio died in
occurs in the doing of the act which produces the robbery and while making an arrest as an officer of the course of an assault or murder considering the
injury, the resulting death is within the protection of the law which contingencies were expressly excluded very nature of these crimes. In the first place, there
the policies insuring against death or injury from in the contract and have the effect of exempting the is no proof that the death of Basilio is the result of
accident. company from liability. either crime for the record is barren of any
- The personal accident insurance policy involved - It is contended in behalf of the company that circumstance showing how the fatal shot was fired.
herein specifically enumerated only 10 circumstances Basilio was killed which "making an arrest as an Perhaps this may be clarified in the criminal case
wherein no liability attaches to FINMAN for any officer of the law" or as a result of an "assault or now pending in court as regards the incident but
injury, disability or loss suffered by the insured as a murder" committed in the place and therefore his before that is done anything that might be said on
result of any of the stimulated causes. death was caused by one of the risks excluded by the the point would be a mere conjecture. Nor can it be
-The principle of " expresso unius exclusio alterius" supplementary contract which exempts the company said that the killing was intentional for there is the
the mention of one thing implies the exclusion of from liability. This contention was upheld by the possibility that the malefactor had fired the shot
another thing is therefore applicable in the instant Court of Appeals. Hence, this petition. merely to scare away the people around for his own
case since murder and assault, not having been protection and not necessarily to kill or hit the victim.
expressly included in the enumeration of the ISSUE In any event, while the act may not exempt the
circumstances that would negate liability in said WON the death of the victim comes within the triggerman from liability for the damage done, the
insurance policy: the failure of the FINMAN to include purview of the exception clause of the supplementary fact remains that the happening was a pure accident
death resulting from murder or assault among the policy and, hence, exempts the company from on the part of the victim. The victim could have been
prohibited risks leads inevitably to the conclusion liability either the policeman or Atty. Ojeda for it cannot be
that it did not intend to limit or exempt itself from pretended that the malefactor aimed at the deceased
liability for such death. HELD precisely because he wanted to take his life.
- A1377 NCC: The interpretation of obscure words NO Disposition Decision set aside
or stipulations in a contract shall not favor the party - Basilio was a watchman of the Manila Auto Supply
who caused the obscurity. which was a block away from the house of Atty. BIAGTAN v. THE INSULAR LIFE ASSURANCE
Ojeda where something suspicious was happening COMPANY, LTD.
INSURANCE Page 74

44 SCRA 58 - In Calanoc vs. CA: Where a shot was fired and it claims, would seemingly except also all other
MAKALINTAL; March 29, 1972 turned out afterwards that the watchman was hit in injuries, intentionally inflicted by a third party,
the abdomen, the wound causing his death, the regardless of any violation of law or provocation by
NATURE Court held that it could not be said that the killing the insured, and defeat the very purpose of the
Appeal from decision of CFI Pangasinan. was intentional for there was the possibility that the policy of giving the insured double indemnity in case
malefactor had fired the shot to scare the people of accidental death by "external and violent means"
FACTS around for his own protection and not necessarily to — in the very language of the policy.'
- Juan Biagtan was insured with Insular for P5k and kill or hit the victim. A similar possibility is clearly - It is obvious from the very classification of the
a supplementary contract “Accidental Death Benefit” ruled out by the facts in this case. For while a single exceptions and applying the rule of noscitus a sociis,
clause for another P5k if "the death of the Insured shot fired from a distance, and by a person who was that the double-indemnity policy covers the insured
resulted directly from bodily injury effected solely not even seen aiming at the victim, could indeed against accidental death, whether caused by fault,
through external and violent means sustained in an have been fired without intent to kill or injure, nine negligence or intent of a third party which is
accident . . . and independently of all other causes." wounds inflicted with bladed weapons at close range unforeseen and unexpected by the insured. All the
The clause, however, expressly provided that it would cannot conceivably be considered as innocent insofar associated words and concepts in the policy plainly
not apply where death resulted from an injury as such intent is concerned. exclude the accidental death from the coverage of
"intentionally inflicted by a third party." - In Hucthcraft's Ex'r vs. Travelers' Ins. Co. (US the policy only where the injuries are self-inflicted or
- One night, a band of robbers entered their house. case): where the insured was waylaid and attended by some proscribed act of the insured or
Juan went out of his room and he was met with 9 assassinated for the purpose of robbery, the court are incurred in some expressly excluded calamity
knife stabs. He died. The robbers were convicted of rendered judgment for the insurance company and such as riot, war or atomic explosion.
robbery with homicide. held that while the assassination of the insured was - The untenability of insurer's claim that the
- The family was claiming the additional P5k from as to him an unforeseen event and therefore insured's death fell within the exception is further
Insular, under the Accidental Death Benefit clause. accidental, "the clause of the proviso that excludes heightened by the stipulated fact that two other
Insular refused on the ground that the death resulted the (insurer's) liability, in case death or injury is insurance companies which likewise covered the
from injuries intentionally inflicted by 3rd parties and intentionally inflicted by any other person, applies to insured for much larger sums under similar
was therefore not covered. Biagtans filed against this case." accidental death benefit clauses promptly paid the
Insular. CFI ruled in favor of Biagtans. Disposition CFI decision reversed. benefits thereof to plaintiffs beneficiaries.

ISSUE SEPARATE OPINION SUN INSURANCE v. CA (LIM)


WON the injuries were intentionally inflicted 211 SCRA 554
TEEHANKEE [dissent] CRUZ; July 17, 1992
HELD - Calanoc v. CA is controlling in this case because
YES the insurance company wasn’t able to prove that the FACTS
- Whether the robbers had the intent to kill or merely killing was intentional. (Burden of proof is with the - The petitioner issued Personal Accident Policy to
to scare the victim or to ward off any defense he insurance company) Felix Lim, Jr. with a face value of P200,000.00. Two
might offer, it cannot be denied that the act itself of - Insurance, being contracts of adhesion, must be months later, he was dead with a bullet wound in his
inflicting the injuries was intentional. construed strictly against insurance company in head. As beneficiary, his wife Nerissa Lim sought
- The exception in the accidental benefit clause cases of ambiguity. payment on the policy but her claim was rejected.
invoked by the appellant does not speak of the - The supplementary contract enumerated The petitioner agreed that there was no suicide. It
purpose — whether homicidal or not — of a third exceptions. The only exception which is not argued, however, that there was no accident either.
party in causing the injuries, but only of the fact that susceptible of classification is that provided in Pilar Nalagon, Lim's secretary, was the only
such injuries have been "intentionally" inflicted — paragraph 5(e), the very exception herein involved, eyewitness to his death. According to Nalagon, Lim
this obviously to distinguish them from injuries which would also except injuries "inflicted was in a happy mood (but not drunk) and was
which, although received at the hands of a third intentionally by a third party, either with or without playing with his handgun, from which he had
party, are purely accidental. provocation on the part of the insured, and whether previously removed the magazine. As she watched
- Examples of unintentional: or not the attack or the defense by the third party the television, he stood in front of her and pointed
>> A gun which discharges while being cleaned and was caused by a violation of the law by the insured." the gun at her. She pushed it aside and said it might
kills a bystander; - This ambiguous clause conflicts with all the other be loaded. He assured her it was not and then
>> a hunter who shoots at his prey and hits a four exceptions in the same paragraph 5 particularly pointed it to his temple. The next moment there was
person instead; that immediately preceding it in item (d) which an explosion and Lim slumped to the floor. He was
>> an athlete in a competitive game involving excepts injuries received where the insured has dead before he fell.
physical effort who collides with an opponent and violated the law or provoked the injury, while this - The term "accident" has been defined as follows:
fatally injures him as a result. clause, construed as the insurance company now
INSURANCE Page 75

The words "accident" and "accidental" have never himself to needless peril when he pointed the gun to and which is unexpected, unusual, and unforeseen.
acquired any technical signification in law, and his temple because the fact is that he thought it was An accident is an event that takes place without
when used in an insurance contract are to be not unsafe to do so. The act was precisely intended one's foresight or expectation, an event that
construed and considered according to the to assure Nalagon that the gun was indeed harmless. proceeds from an unknown cause, or is an unusual
ordinary understanding and common usage and Disposition CA Affirmed effect of a known cause and, therefore, not
speech of people generally. In substance, the expected.
courts are practically agreed that the words DE LA CRUZ v. CAPITAL INSURANCE - The generally accepted rule is that, death or injury
"accident" and "accidental" mean that which 17 SCRA 554 does not result from accident or accidental means
happens by change or fortuitously, without BARRERA; June 30, 1966 within the terms of an accident-policy if it is the
intention or design, and which is unexpected, natural result of the insured's voluntary act,
unusual, and unforeseen. The definition that has NATURE unaccompanied by anything unforeseen except the
usually been adopted by the courts is that an Appeal from the decision of the CFI of Pangasinan death or injury. There is no accident when a
accident is an event that takes place without one's deliberate act is performed unless some additional,
foresight or expectation — an event that proceeds FACTS unexpected, independent, and unforeseen happening
from an unknown cause, or is an unusual effect of - Eduardo de la Cruz, employed in the Itogon-Suyoc occurs which produces or brings about the result of
a known case, and therefore not expected. Mines, Inc., was the holder of an accident insurance injury or death. In other words, where the death or
- An accident is an event which happens without any policy underwritten by the Capital Insurance & injury is not the natural or probable result of the
human agency or, if happening through human Surety Co., Inc., for the period beginning November insured's voluntary act, or if something unforeseen
agency, an event which, under the circumstances, is 13, 1956 to November 12, 1957. occurs in the doing of the act which produces the
unusual to and not expected by the person to whom - On January 1, 1957, the Itogon-Suyoc Mines, Inc. injury, the resulting death is within the protection of
it happens. It has also been defined as an injury sponsored a boxing contest wherein the insured policies insuring against death or injury from
which happens by reason of some violence or Eduardo de la Cruz participated. accident.
casualty to the insured without his design, consent, - In the course of his bout, Eduardo slipped and was - In the present case, while the participation of the
or voluntary co-operation. hit by his opponent on the left part of the back of the insured in the boxing contest is voluntary, the injury
head, causing Eduardo to fall, with his head hitting was sustained when he slid, giving occasion to the
ISSUE the rope of the ring. infliction by his opponent of the blow that threw him
WON what happened was an accident - He was brought to the Baguio General Hospital, but to the ropes of the ring.
he died as a result of hemorrhage, intracranial, left. - The fact that boxing is attended with some risks of
HELD - Simon de la Cruz, the father and named beneficiary external injuries does not make any injuries received
YES of the insured, filed a claim with the insurance in the course of the game not accidental. In boxing
- The petitioner, however, cites one of the four company for payment of the indemnity, but it was as in other equally physically rigorous sports, such as
exceptions provided for in the insurance contract and denied. basketball or baseball, death is not ordinarily
contends that the private petitioner's claim is barred - He instituted the action in the CFI of Pangasinan for anticipated to result. If, therefore, it ever does, the
by such provision. It is there stated: specific performance. injury or death can only be accidental or produced by
Exceptions —The company shall not be liable in - Defendant insurer set up the defense that the some unforeseen happening or event as what
respect of. death of the insured, caused by his participation in a occurred in this case.
1. Bodily injury. boxing contest, was not accidental and, therefore, - Furthermore, the policy involved herein specifically
xxx xxx xxx not covered by insurance excluded from its coverage:
b. consequent upon. - The court rendered the decision in favor of the (e) Death or disablement consequent upon the
i) The insured persons attempting to commit plaintiff, hence, the present appeal. Insured engaging in football, hunting, pigsticking,
suicide or wilfully exposing himself to needless peril steeplechasing, polo-playing, racing of any kind,
except in an attempt to save human life. ISSUE mountaineering, or motorcycling.
- To repeat, the parties agree that Lim did not WON the death of the insured was not accidental - Death or disablement resulting from engagement in
commit suicide. Nevertheless, the petitioner and, therefore, not covered by insurance boxing contests was not declared outside of the
contends that the insured willfully exposed himself to HELD protection of the insurance contract. Failure of the
needless peril and thus removed himself from the NO defendant insurance company to include death
coverage of the insurance policy. That posture is - The terms "accident" and "accidental", as used in resulting from a boxing match or other sports among
arguable. But what is not is that, as the secretary insurance contracts, have not acquired any technical the prohibitive risks leads inevitably to the conclusion
testified, Lim had removed the magazine from the meaning, and are construed by the courts in their that it did not intend to limit or exempt itself from
gun and believed it was no longer dangerous. He ordinary and common acceptation. Thus, the terms liability for such death.
expressed assured her that the gun was not loaded. have been taken to mean that which happen by Disposition The decision appealed from is affirmed
It is submitted that Lim did not willfully expose chance or fortuitously, without intention and design,
INSURANCE Page 76

FORTUNE INSURANCE v. CA (supra p.7) - RTC: dismissed PHAC's complaint and ruled in caused by lightning or a natural disaster or casualty
favor of ESLI. not attributable to human agency.
PHIL HOME ASSURANCE CORP v. CA (EASTERN - The burning of the vessel was not the fault or Reasoning
SHIPPING) negligence of defendant but a natural disaster or - There was no showing, and none was alleged by
257 SCRA 468 calamity. Salvage operations conducted by Fukuda the parties, that the fire was caused by a natural
KAPUNAN; June 20, 1996 Salvage Company was perfectly a legal operation and disaster. On the contrary, there is strong evidence
charges made on the goods recovered were indicating that the acetylene cylinder caught fire
NATURE legitimate charges. Section 19 of Act No. 2616, the because of the fault and negligence of respondent
- Eastern Shipping Lines, Inc. loaded on board SS Salvage Law is applicable. With respect to the ESLI, its captain and its crew:
Eastern Explorer in Kobe, Japan, the following additional freight charged by defendant from the (1) The acetylene cylinder which was fully loaded
shipment for carriage to Manila and Cebu, freight consignees of the goods, the same are also validly should not have been stored near the engine room
pre-paid and in good order and condition: (a) 2 demandable. where the heat generated therefrom could cause the
boxes internal combustion engine parts, consigned to - The burning of "EASTERN EXPLORER" while off acetylene cylinder to explode by reason of
William Lines, Inc.; (b) 10 metric tons (334 bags) Okinawa rendered it physically impossible for spontaneous combustion. ESLI should have easily
ammonium chloride, consigned to Orca's Company; defendant to comply with its obligation of delivering foreseen that the acetylene cylinder, containing
(c) 200 bags Glue 300, consigned to Pan Oriental the goods to their port of destination pursuant to the highly inflammable material, was in a real danger of
Match Company; and (d) garments, consigned to contract of carriage. Under Article 1266 of the Civil exploding.
Ding Velayo. All consignations were made by virtue Code, the physical impossibility of the prestation (2) ESLI should have known that by storing the
of a Bill of Lading. extinguished defendant's obligation. acetylene cylinder in the accommodation area
- While the vessel was off Okinawa, a small flame - Note: The goods subject of the present supposed to be reserved for passengers, it
was detected on the acetylene cylinder located in the controversy were neither lost nor damaged in transit unnecessarily exposed its passengers to grave
accommodation area near the engine room. As the by the fire that razed the carrier. In fact, these were danger and injury.
crew was trying to extinguish the fire, the cylinder all delivered to the consignees, even if the (3) The fact that the acetylene cylinder was checked,
suddenly exploded, thus causing death and severe transshipment took longer than necessary. What is tested and examined and subsequently certified as
injuries to the crew and instantly setting fire to the at issue therefore is not whether or not the carrier is having complied with the safety measures and
whole vessel. liable for the loss, damage, or deterioration of the standards by qualified experts before it was loaded in
- SS Eastern Explorer was then found to be a goods transported by them but who, among the the vessel only shows to a great extent that
constructive total loss and its voyage was declared carrier, consignee or insurer of the goods, is liable for negligence was present in the handling of the
abandoned. the additional charges or expenses incurred by the acetylene cylinder after it was loaded and while it
- After the fire was extinguished, the cargoes which owner of the ship in the salvage operations and in was on board the ship.
were saved were loaded to another vessel for the transshipment of the goods via a different carrier. - From the foregoing premises, it indubitably follows
delivery to their original ports of destination. ESLI In absolving respondent carrier of any liability, CA that the cargo consignees cannot be made liable to
charged the consignees several amounts sustained the trial court's finding that the fire that respondent carrier for additional freight and salvage
corresponding to additional freight and salvage gutted the ship was a natural disaster or calamity. charges.
charges. Disposition Judgment appealed from is REVERSED
- The charges were all paid by Philippine Home ISSUE and SET ASIDE. Respondent Eastern Shipping Lines,
Assurance Corporation (PHAC) under protest for and WON the burning of the SS Eastern Explorer Inc. is ORDERED to return to petitioner Philippine
in behalf of the consignees. PHAC, as subrogee of rendering it a constructive total loss was a natural Home Assurance Corporation the amount it paid
the consignees, then filed a complaint before the RTC disaster or calamity under protest in behalf of the consignees herein.
of Manila, against ESLI to recover the sum paid
under protest on the ground that the same were HELD
actually damages directly brought about by the fault, NO
negligence, illegal act and/or breach of contract of Ratio In our jurisprudence, fire may not be
ESLI. considered a natural disaster or calamity since it
- ESLI contended that it exercised the diligence almost always arises from some act of man or by
required by law in the handling, custody and carriage human means. It cannot be an act of God unless
of the shipment; that the fire was caused by an
unforeseen event; that the additional freight charges 9
Section 1. When in case of shipwreck, the vessel or its cargo shall
PHILIPPINE AMERICAN LIFE INSURANCE
are due and demandable pursuant to the Bill of be beyond the control of the crew, or shall have been abandoned by COMPANY v. THE AUDITOR GENERAL
Lading; and that salvage charges are properly them, and picked up and conveyed to a safe place by other persons, 22 SCRA 135
the latter shall be entitled to a reward for the salvage.
collectible under Act No. 2616, known as the Salvage Those who, not being included in the above paragraph, assist in saving
SANCHEZ, JANUARY 18, 1968
Law. a vessel or its cargo from shipwreck, shall be entitled to like reward.
INSURANCE Page 77

NATURE . . In contradistinction, a reinsurance treaty is international reserves; to provide the Central Bank
PETITION FOR REVIEW of a ruling of the Auditor merely an agreement between two insurance with an additional instrument for effectively coping
General. companies whereby one agrees to cede and the with the problem and achieving domestic and
other to accept reinsurance business pursuant to international stability of our currency; to reduce the
FACTS provisions specified in the treaty. The practice of excessive demand-for foreign exchange.
- Philamlife, a domestic life insurance corp., and issuing policies by insurance companies includes, - implementation of Margin Law in accordance with
American International Reinsurance Company among other things, the issuance of reinsurance police power
(Airco), a corporation organized under the laws of policies on standard risks and also on substandard 3. NO
the Republic of Panama, entered into a risks under special arrangements. The lumping of the Reasoning
REINSURANCE TREATY wherein Philamlife agrees to different agreements under a contract has resulted in - First, there is no concrete evidence that such
reinsure with Airco on January 1950. Philamlife the term known to the insurance world as 'treaties.' imposition of the 25% margin fee is unreasonable,
agreed to pay premiums for all reinsurances on an Such a treaty is, in fact, an agreement between Second, if really continuance of the existing
annual premium basis. insurance companies to cover the different situations reinsurance treaty becomes unbearable, that
- In July 16, 1959, the Margin Law was approved and described. Reinsurance treaties and reinsurance contract itself provides that petitioner may
became effective, which exempts certain “obligations policies are not synonymous. Treaties are potestatively write finis thereto on ninety days'
from payment of margin fees, particularly contractual contracts for insurance; reinsurance policies or written notice. Petitioner is not forced to continue
obligations calling for payment of foreign exchange cessions are contracts of insurance. its reinsurance treaty indefinitely with Airco.
issued, approved and outstanding as of the date this Reasoning Disposition For the reasons given, the petition for
Act takes place”. - Even if reinsurance treaty preceded the Margin Law review is hereby denied, and the ruling of the Auditor
- Central Bank of the Philippines collected by over nine years, nothing in the treaty obligates General of October 24, 1961 denying refund is
P268,747.48 as foreign exchange margin on Philamlife to remit to Airco a fixed, certain, and hereby affirmed.
Philamlife remittances to Airco purportedly totalling obligatory sum by way of reinsurance premiums. The Costs against petitioner. So ordered.
$610,998.63 and made subsequent to July 16, 1959. reinsurance treaty per se cannot give rise to a
Philamlife filed a claim for refund on the ground that contractual obligation for the payment of foreign FIELDMEN'S INSURANCE CO INC v. ASIAN
the reinsurance premiums remitted were paid in exchange. Philamlife’s obligation to remit reinsurance SURETY & INSURANCE CO INC
pursuant to the January 1950 reinsurance treaty, and premiums becomes fixed and definite upon the 34 SCRA 36
therefore exempted. execution of the reinsurance cession. It is only MAKALINTAL; July 31, 1970
- Monetary Board exempted Philamlife from payment after a reinsurance cession is made that payment of
of margin fee. However, Auditor of CB refused to reinsurance premium may be exacted, as it is only FACTS
pass in audit Philamlife’s claim for refund. Philamlife after Philamlife seeks to remit that reinsurance - On various dates between April 11, 1960 and Jan.
sought reconsideration but was denied, saying premium that the obligation to pay the margin fee 9, 1961 the Asian Surety & Insurance Company, Inc.
reinsurance treaty NOT EXEMPTED. arises. and the Fieldmen's Insurance Company, Inc. entered
2. NO into 7 reinsurance agreements under which the
ISSUES Ratio. Existing laws form part of the contract "as the former, as the ceding company undertook to cede to
1. WON the premia remitted were in pursuance of measure of the obligation to perform them by the the latter, as the reinsuring company, a specified
the reinsurance treaty between Philamlife and Airco one party and the right acquired by the other. If the portion of the amount of insurance underwritten by
of January 1959, a contract antedating the Margin obligation does not inhere and subsist in the ASIAN upon payment to FIELDMEN'S of a
Law, and therefore, Philamlife exempted from paying contract itself, propio vigore, but in the law proportionate share of the gross rate of the premium
margin fee applicable to the contract. applicable with respect to each cession after
2. WON Margin Law impairs the obligation of contract deducting a commission. Said agreements were to
3. WON reinsurance contracts abroad would be made take effect from certain specific dates and were to be
impractical by the imposition of the 25% margin fee in force until cancelled by either party upon previous
Reasoning notice of at least 3 months by registered mail to the
HELD - . When petitioner entered into the reinsurance other party, the cancellation to take effect as of Dec.
1. NO treaty of January 1, 1950 with Airco, it did so with 31 of the year in which the notice was given.
- For an exemption to come into play, there must be the understanding that the municipal laws of the - On Sep. 19, 1961 FIELDMEN'S, by means of
a reinsurance policy or, as in the reinsurance treaty Philippines at the time said treaty was executed, registered mail, served notice to ASIAN of the
provided, a "reinsurance cession" which may be became an unwritten condition thereof. Such former's desire to be relieved from all participation in
automatic or facultative. municipal laws constitute part of the obligation of its various agreements with the latter effective Dec.
Ratio A reinsurance policy is thus a contract of contract. 31, 1961. This communication, although admittedly
indemnity one insurer makes with another to protect -Rationale of Margin Law: to reduce the excessive received by ASIAN on Sep. 25, 1961, did not elicit
the first insurer from a risk it has already assumed. . demand on and prevent further decline of our any reply from ASIAN.
INSURANCE Page 78

- On Dec. 7, 1961 FIELDMEN'S sent another letter to Dec 31, 1961. At the same time, it upheld ASIAN'S had been made, FIELDMEN'S cannot avoid liability
ASIAN expressing regrets at alleged violations position that all cessions of reinsurance made by it to which arose by reason of the burning of the insured
committed by the latter with respect to the various FIELDMEN'S prior to the cancellation of the property.
agreements between them and reiterated its position reinsurance treaties continued in full force and effect - With respect to the other 4 agreements, it would
that it would consider itself "no longer at risk for any until expiry dates and ordered FIELDMEN'S to make seem that the petition for declaratory relief is moot,
reinsurance and/or cession" given by ASIAN which an accounting of its business transactions with ASIAN and that no useful purpose would be served by
might be in force on Dec. 31, 1961. Not having within 30 days. defining the respective rights and obligations of the
received any formal reply from ASIAN, FIELDMEN'S - On appeal to the CA, the decision of the trial court parties thereunder. The said agreements have been
sent a new a letter on Feb. 17, 1962 reminding was substantially affirmed, with the slight cancelled, and it does not appear that any claim by
ASIAN of the cancellation of all the reinsurance modification that the order for accounting was or liability in favor of the insured has actually arisen
treaties and cessions as of Dec. 31, 1961 and eliminated, without prejudice to the filing of a proper under any of the reinsurance cessions made prior to
requested ASIAN to submit its final accounting of all action between the parties for that purpose. such cancellation. Future conflicts of the same nature
cessions made to the former for the preceding as those which have motivated the present action
months when the reinsurance agreements were in ISSUE can of course be obviated by using more precise and
force. WON the cancellation as of Dec. 31, 1961 of the definite terminology in the reinsurance agreements
- Meanwhile one of the risks reinsured with reinsurance treaties had the effect of terminating which the parties may enter into henceforth.
FIELDMEN'S issued in favor of the GSIS became a also the liability of FIELDMEN'S as reinsurer with
liability when the insured property was burned on respect to policies or cessions issued prior to the EQUITABLE INSURANCE v. RURAL INSURANCE
February 16, 1962. Since the policy was issued on termination of the principal reinsurance contracts or 4 SCRA 343
July 1, 1961, it was supposed to expire on July 1, treaties BARRERA; January 31, 1962
1962. 2 The next day, Feb. 17, ASIAN immediately
notified FIELDMEN'S of said fire loss. HELD FACTS
- FIELDMEN'S, relying on the sufficiency of its notice NO to the 2 reinsurance contracts - Plaintiff Equitable Insurance file a complaint with
of termination dated September 19, 1961 and - Of the 6 reinsurance contracts, 2 contain the CFI of Manila against defendant Rural Insurance
obviously bent on avoiding its liability under the provisions, which clearly and expressly recognize the alleging, as first cause of action, that they entered
reinsurance agreements with ASIAN, filed a petition continuing effectivity of policies ceded under them into a reciprocal facultative reinsurance agreement,
for declaratory relief with the CFI of Manila to seek a for reinsurance notwithstanding the cancellation of wherein they agreed to cede to each other. Pursuant
declaration that all the reinsurance contracts entered the contracts themselves. The said treaties provide to said agreement, plaintiff reinsured for P2k with
into between them had terminated as of December "that in the event of termination of this Agreement . defendant the stock covered by fire insurance Policy
31, 1961 and to obtain an order directing ASIAN to . ., the liability of the Fieldmen's under current No. 5880 issued by plaintiff which was later burned;
render final accounting of the transactions between cessions shall continue in full force and effect until the share of the loss of defendant as per insurance
them with respect to said reinsurance treaties as of their natural expiry . . .;" and the 4th paragraph of agreement was computed at P2,024 for which
the cut-off date. Article VI of the Personal Accident Reinsurance Treaty plaintiff sent to defendant a statement of account for
- In its answer below ASIAN denied having received states: payment by the latter. Despite repeated demands by
FIELDMEN'S letter dated Sep 19, 1961, and argued "4. On the termination of this Agreement from plaintiff, defendant refused to pay.
that even assuming it did, FIELDMEN'S could not any cause whatever, the liability of the REINSURER - On the second cause of action, plaintiff reinsured
have terminated the reinsurance treaties as of Dec (Fieldmen's) under any current cession including for P2k with defendant stock covered by fire
31, 1961 because the letter was merely an any amounts due to be ceded under the terms of insurance Policy No. 6062 which also burned. Again,
expression of FIELDMEN'S desire to cancel the this Agreement and which are not cancelled in the defendant refused to pay its share of the loss of
treaties and not a formal notice of cancellation as ordinary course of business shall continue in full P1,334 hence said complaint.
contemplated in their reinsurance agreements. By force until their expiry unless the COMPANY - Defendant filed a motion to dismiss on the ground
way of special defense Asian contended that even if (Asian) shall, prior to the thirty-first December that it states no cause of action, as pursuant to Art
the Sep. 19 letter were considered sufficient notice of next following such notice, elect to withdraw the VIII of the Reinsurance Agreement between the
cancellation — thereby rendering the reinsurance existing cessions . . ." parties, before a court action can be brought, the
agreements terminated as of December 31, 1961 — - Thus, insofar as the two reinsurance agreements parties agreed to submit all disputes to a board of
the liability of FIELDMEN'S with respect to policies or are concerned, there is clearly no merit in arbitrators. The Court denied the motion and
cessions issued under two of the said agreements FIELDMEN'S claim that their cancellation carried with required defendant to answer.
prior to their cancellation continued to have full force it ipso facto the termination of all reinsurance - Defendant filed its answer, alleging that the nature
and effect until the stated expiry dates of such cessions thereunder. Such cessions continued to be of the agreement is “self-liquidating between the
policies or cessions. in force until their respective dates of expiration. parties”, the reinsurer becoming the reinsured and
- On Dec. 4, 1962, the trial court declared 6 of the 7 Since it was under one of said agreements that the vice versa; and that said agreement has not yet
reinsurance agreements in question cancelled as of reinsurance cession corresponding to the GSIS policy been abrogated so the liability of either to the other
INSURANCE Page 79

is not yet known. Defendant prayed that the NO


complaint be dismissed and plaintiff filed a motion ARTEX DEVELOPMENT CO INC v. WELLINGTON - Unless there is a specific grant in, or assignment of,
for judgment on the pleadings which the court INSURANCE CO INC the reinsurance contract in favor of the insured or a
denied. 51 SCRA 352 manifest intention of the contracting parties to the
- Instead of going into a formal hearing, the parties TEEHANKEE; June 27, 1973 reinsurance contract to grant such benefit or favor to
submitted their case for decision stipulating the ff the insured, the insured, not being privy to the
facts: defendant admits the allegations of the FACTS reinsurance contract, has no cause of action against
complaint and that plaintiff admits that the issues of - Wellington Insurance Co. Inc. insured for the reinsurer. It is expressly provided in Section 91
the complaint were not submitted to a Board of P24,346,509.00 the buildings, stocks and machinery the Insurance Act 1 that "(T)he original insured has
Arbitrators as provided in par VIII of the complaint, of plaintiff Artex Development Co. Inc. against loss no interest in a contract of insurance."
but instead referred it to the Insurance or damage by fire or lighting upon payment of the
Commissioner. The CFI rendered judgment in favor plaintiff of the corresponding premiums; that said PERLA COMPANIA DE SEGUROS v. CA(LIM)
of plaintiff. Hence this appeal. properties were insured for an additional sum of 208 SCRA 487
P883,034.00; that defendant insured plaintiff against NOCON; May 7, 1992
ISSUES business interruption (use and occupancy) for
1. WON Equitable had no cause of action as the P5,200,000.00; Wellington entered into a contract of NATURE
matter was not referred to the decision of arbitrators reinsurance with Alexander and Alexander, Inc. of Petition for certiorari by Perla Compania de Seguros
2. WON in a facultative obligation the right to choose New York. USA. and FOC Credit Corporation seeking to annul and set
an alternative remedy lies only with the debtor (here - The buildings, stocks and machineries of plaintiffs aside CA decision revering the RTC decision for
the defendant) under Art 1206 spinning department were burned. replevin and damages.
- Notice of the loss and damage was given the
HELD defendant; that as per report of the adjusters, the FACTS
1. NO total property loss of the plaintiff was the sum of - Private respondents spouses Herminio and Evelyn
- The requirement of submission for decision to 2 P10,106,554.40 and the total business interruption Lim executed a promissory note in favor of
arbitrators or an umpire the matter of losses by fire loss was P3,000,000.00; Supercars, Inc. in the sum of P77,940.00, payable in
or the liability of the parties thereto under Art VIII of - That defendant has paid to the plaintiff the sum of monthly installments according to the schedule of
the agreement arises only if the same is disputed by P6,481,870.07 of the property loss suffered by payment indicated in said note, and secured by a
one of the parties. In the instant case, there is no plaintiff and P1,864,134.08 on its business chattel mortgage over a brand new red Ford Laser,
dispute between the parties; in the stipulation of interruption loss, leaving a balance of P3,624,683.43 which is registered under the name of private
facts defendant admitted that plaintiff had paid its and P1,748,460.00, respectively. respondent Herminio Lim and insured with the
liability and defendant likewise admitted that it - The counsel for Artex filed a Manifestation saying petitioner Perla Compania de Seguros, Inc. (Perla for
ignored plaintiff’s demands for reimbursement for that in view of the Deeds of Discharge and Collateral brevity) for comprehensive coverage.
defendant’s failure to pay its share as reinsurer. As Agreement, the only remaining liability subject of - On the same date, Supercars, Inc., with notice to
held in Maligad v United Assurance Co., if in the litigation shall be the proportion of the loss reinsured private respondents spouses, assigned to petitioner
course of the settlement of a loss, the action of the with or through Alexander and Alexander, Inc. of FCP Credit Corporation (FCP for brevity) its rights,
company or its agents amounts to refusal to pay, the New York, USA, namely, P397,813.00. title and interest on said promissory note and chattel
company will be deemed to have waived the - The document recited further that Artex mortgage as shown by the Deed of Assignment.
condition precedent with reference to arbitration and acknowledges receipt of the sum of P3.6M paid by - At around 2:30pm Nov9, 1982, said vehicle was
a suit upon the policy will lie. the insurer in full and final settlement of all or any carnapped while parked at the back of Broadway
2. NO claims of Artex against its insurer. It discharges its Centrum. Evelyn Lim, who was driving said car
- There is no connection between Art 1206 NCC and insurer from all actions, proceedings, claims, before it was carnapped, immediately called up the
the agreement of this action. The term “facultative” demands, costs and expenses in respect thereof. Anti-Carnapping Unit of the Philippine Constabulary
is used in reinsurance contracts, and it is so used in - With regard the balance unpaid, Wellington to report said incident and thereafter, went to the
this particular case, merely to define the right of the contends that Artex should have been directed nearest police substation to make a police report
reinsurer to accept or not to accept participation in against the reinsurers to cover the liability and not regarding said incident.
the risk insured. But once the share is accepted, as it against Wellington. - On Nov10, 1982, Evelyn Lim reported said incident
was in the case at bar, the obligation is absolute and to the LTO in compliance with the insurance
the liability assumed thereunder can be discharged ISSUE requirement. She also filed a complaint with the
by only one way—the payment of the share of the WON the insured (Artex) has a cause of action Headquarters. Constabulary Highway Patrol Group.
losses. against the reinsurer - On Nov11, 1982, private respondent filed a claim
Disposition judgment appealed from the TC is for loss with the petitioner Perla but said claim was
affirmed HELD denied on Nov18, 1982 on the ground that Evelyn
INSURANCE Page 80

Lim, who was using the vehicle before it was by whatever befalls the subject matter of the
carnapped, was in possession of an expired driver's HELD accessory contract.
license at the time of the loss of said vehicle which is 1. NO - The unpaid balance on the promissory note should
in violation of the authorized driver clause of the - The comprehensive insurance policy issued by be paid, and not just the installments due and
insurance policy, which states, to wit: petitioner Perla undertook to indemnify the private payable before the automobile was carnapped, as
"AUTHORIZED DRIVER: respondents against loss or damages to the car (a) erronously held by the CA
Any of the following: (a) The Insured (b) Any by accidental collision or overturning, or collision or - However, this does not mean that private
person driving on the Insured's order, or with his overturning consequent upon mechanical breakdown respondents are bound to pay the interest, litigation
permission. Provided that the person driving is or consequent upon wear and tear; (b) by fire, expenses and attorney's fees stipulated in the
permitted, in accordance with the licensing or external explosion, self-ignition or lightning or promissory note. Because of the peculiar relationship
other laws or regulations, to drive the Scheduled burglary, housebreaking or theft; and (c) by between the three contracts in this case, i. e., the
Vehicle, or has been permitted and is not malicious act. promissory note, the chattel mortgage contract and
disqualified by order of a Court of Law or by - Where a car is unlawfully and wrongfully taken the insurance policy, the Court is compelled to
reason of any enactment or regulation in that without the owner's consent or knowledge, such construe all three contracts as intimately interrelated
behalf." taking constitutes theft, and, therefore, it is the to each other, despite the fact that at first glance
- On Nov17, 1982, private respondents requested "THEFT" clause, and not the "AUTHORIZED DRIVER" there is no relationship whatsoever between the
from petitioner FCP for a suspension of payment on clause, that should apply.The risk against accident is parties thereto.
the monthly amortization agreed upon due to the distinct from the risk against theft. The 'authorized - Under the promissory note, Lim spouses are
loss of the vehicle and, since the carnapped vehicle driver clause' in a typical insurance policy as in obliged to pay Supercars, Inc. the amount stated
was insured with petitioner Perla, said insurance contemplation or anticipation of accident in the legal therein in accordance with the schedule provided for.
company should be made to pay the remaining sense in which it should be understood, and not in To secure said promissory note, private respondents
balance of the promissory note and the chattel contemplation or anticipation of an event such as constituted a chattel mortgage in favor of Supercars,
mortgage contract. theft. The distinction often seized upon by insurance Inc. over the automobile the former purchased from
- Perla, however, denied private respondents' claim. companies in resisting claims from their assureds the latter. The chattel mortgage, in turn, required
Consequently, petitioner FCP demanded that private between death occurring as a result of accident and private respondents to insure the automobile and to
respondents pay the whole balance of the promissory death occurring as a result of intent may apply to the make the proceeds thereof payable to Supercars,
note or to return the vehicle but the latter refused. case at bar. Inc. The promissory note and chattel mortgage were
- On July25, 1983, petitioner FCP filed a complaint - If the insured vehicle had figured in an accident at assigned by Supercars, Inc. to petitioner FCP, with
against private respondents, who in turn filed an the time she drove it with an expired license, then, the knowledge of private respondents. Private
amended third party complaint against petitioner appellee Perla Compania could properly resist respondents were able to secure an insurance policy
Perla on Dec8, 1983. After trial on the merits, TC appellants' claim for indemnification for the loss or from petitioner Perla, and the same was made
ordered sps Lim to pay jointly and severally, plaintiff destruction of the vehicle resulting from the accident. specifically payable to petitioner FCP.
the sum of P55,055.93 plus interest thereon at the But in the present case, the loss of the insured - From the abovementioned provision that upon the
rate of 24% per annum from July 2, 1983 until fully vehicle did not result from an accident where intent loss of the insured vehicle, the insurance company
paid; as well as the cost of suit. It also ordered the was involved; the loss in the present case was Perla undertakes to pay directly to the mortgagor or
dismissal of the Third party complaint against Third- caused by theft, the commission of which was to their assignee, FCP, the outstanding balance of the
Party Defendant. attended by intent." mortgage at the time of said loss under the
- Upon appeal, CA reversed said decision - There is no causal connection between the mortgage contract. If the claim on the insurance
- After petitioners' separate MFRs were denied by CA, possession of a valid driver's license and the loss of a policy had been approved by petitioner Perla, it
petitioners filed these separate petitions for review vehicle. To rule otherwise would render car insurance would have paid the proceeds thereof directly to
on certiorari. practically a sham since an insurance company can petitioner FCP, and this would have had the effect of
ISSUE easily escape liability by citing restrictions which are extinguishing private respondents' obligation to
1. WON there was grave abuse of discretion on the not applicable or germane to the claim, thereby petitioner FCP. Therefore, private respondents were
part of the appellate court in holding that private reducing indemnity to a shadow. justified in asking petitioner FCP to demand the
respondents did not violate the insurance contract 2. The court agrees with FCP that Lim spouses are unpaid installments from petitioner Perla.
because the authorized driver clause is not applicable not relieved of their obligation to pay the former the - Because petitioner Perla had unreasonably denied
to the "Theft" clause of said Contract installments due on the promissory note on account their valid claim, private respondents should not be
2. WON the loss of the collateral exempted the of the loss of the automobile. The chattel mortgage made to pay the interest, liquidated damages and
debtor from his admitted obligations under the constituted over the automobile is merely an attorney's fees as stipulated in the promissory note.
promissory note particularly the payment of interest, accessory contract to the promissory note. Being the As mentioned above, the contract of indemnity was
litigation expenses and attorney's fees principal contract, the promissory note is unaffected procured to insure the return of the money loaned
from petitioner FCP, and the unjustified refusal of
INSURANCE Page 81

petitioner Perla to recognize the valid claim of the Poblete, Sr. who was on board of the said alleged bodily injuries caused to said third party,
private respondents should not in any way prejudice Volkswagen car sustained physical injuries which arose from the offense charged in the criminal case,
the latter. injuries causing deformity on the face. The owner of from which the injured (Jovencio Poblete, Sr.) has
- Private respondents can not be said to have unduly the damaged Volkswagen car filed a separate civil sought to recover civil damages. Hence, such claim
enriched themselves at the expense of FCP since action against petitioner for damages, while Jovencio of petitioner against the insurance company cannot
they will be required to pay the latter the unpaid Poblete, Sr., who was a passenger in the Volkswagen be regarded as not related to the criminal action.
balance of its obligation under the promissory note. car, did not reserve his right to file a separate civil - A third party complaint is a device allowed by the
- In view of the foregoing discussion, We hold that action for damages. Instead, in the course of the trial rules of procedure by which the defendant can bring
the Court of Appeals did not err in requiring in the criminal case, Poblete, Sr. testified on his claim into the original suit a party against whom he will
petitioner Perla to indemnify private respondents for for damages for the serious physical injuries which have a claim for indemnity or remuneration as a
the loss of their insured vehicle. However, the latter he claimed to have sustained as a result of the result of a liability established against him in the
should be ordered to pay petitioner FCP the amount accident. original suit. 13 Third party complaints are allowed to
of P55,055.93, representing the unpaid installments - The court issued an order dismissing the third party minimize the number of lawsuits and avoid the
from December 30, 1982 up to July 1, 1983, as complaint on the ground that it was premature, necessity of bringing two (2) or more actions
shown in the statement of account prepared by based on the premise that unless the accused involving the same subject matter. They are
petitioner FCP, 18 plus legal interest from July 2, (herein petitioner) is found guilty and sentenced to predicated on the need for expediency and the
1983 until fully paid. pay the offended party (Poblete Sr.) indemnity or avoidance of unnecessary lawsuits. If it appears
- As to the award of moral damages, exemplary damages, the third party complaint is without cause probable that a second action will result if the
damages and attorney's fees, private respondents of action. The court further stated that the better plaintiff prevails, and that this result can be avoided
are legally entitled to the same since Perla had acted procedure is for the accused (petitioner) to wait for by allowing the third party complaint to remain, then
in bad faith by unreasonably refusing to honor the the outcome of the criminal aspect of the case to the motion to dismiss the third party complaint
insurance claim of the private respondents. Besides, determine whether or not the accused, also the third should be denied.
awards for moral and exemplary damages, as well as party plaintiff, has a cause of action against the third - Compulsory Motor Vehicle Liability Insurance (third
attorney's fees are left to the sound discretion of the party defendant for the enforcement of its third party party liability, or TPL) is primarily intended to provide
Court. Such discretion, if well exercised, will not be liability (TPL) under the insurance contract. 6 compensation for the death or bodily injuries
disturbed on appeal. Petitioner moved for reconsideration of said order, suffered by innocent third parties or passengers as a
Disposition the assailed decision of the CA is but the motion was denied; hence, this petition. result of a negligent operation and use of motor
hereby MODIFIED to require private respondents to vehicles. The victims and/or their dependents are
pay petitioner FCP the amount of P55,055.93, with ISSUE assured of immediate financial assistance, regardless
legal interest from July 2, 1983 until fully paid. The WON the court a quo erred in dismissing petitioner's of the financial capacity of motor vehicle owners.
decision appealed from is hereby affirmed as to all third party complaint on the ground that petitioner - The liability of the insurance company under the
other respects. No pronouncement as to costs. had no cause of action yet against the insurance Compulsory Motor Vehicle Liability Insurance is for
company loss or damage. Where an insurance policy insures
SHAFER v. JUDGE directly against liability, the insurer's liability accrues
167 SCRA 386 HELD immediately upon the occurrence of the injury or
PADILLA; November 14, 1988 YES event upon which the liability depends, and does not
- There is no need on the part of the insured to wait depend on the recovery of judgment by the injured
NATURE for the decision of the trial court finding him guilty of party against the insured.
Petition for review on certiorari reckless imprudence. The occurrence of the injury to - The injured for whom the contract of insurance is
the third party immediately gave rise to the liability intended can sue directly the insurer. The general
FACTS of the insurer under its policy. Respondent insurance purpose of statutes enabling an injured person to
- Sherman Shafer obtained a private car policy over company's contention that the third party complaint proceed directly against the insurer is to protect
his Ford Laser from Makati Insurance Company, Inc., involves extraneous matter which will only clutter, injured persons against the insolvency of the insured
for third party liability. During the effectivity of the complicate and delay the criminal case is without who causes such injury, and to give such injured
policy, an information for reckless imprudence merit. The civil aspect of the offense charged, i.e., person a certain beneficial interest in the proceeds of
resulting in damage to property and serious physical serious physical injuries allegedly suffered by the policy, and statutes are to be liberally construed
injuries was filed against shafer. The information Jovencio Poblete, Sr., was impliedly instituted with so that their intended purpose may be accomplished.
said that on or about the 17th day of May 1985, in the criminal case. Petitioner may thus raise all It has even been held that such a provision creates a
the City of Olongapo. Shafer hit and bumped a defenses available to him insofar as the criminal and contractual relation which inures to the benefit of any
Volkswagen car owned and driven by Felino llano y civil aspects of the case are concerned. The claim of and every person who may be negligently injured by
Legaspi, thereby causing damage in the total amount petitioner for payment of indemnity to the injured the named insured as if such injured person were
of P12,345.00 and as a result thereof one Jovencio third party, under the insurance policy, for the specifically named in the policy.
INSURANCE Page 82

- In the event that the injured fails or refuses to imprisonment in case of insolvency, plus five does not depend on the recovery of judgment by the
include the insurer as party defendant in his claim for thousand pesos in the concept of moral and injured party against the insured."
indemnity against the insured, the latter is not exemplary damages with costs. No appeal was - The underlying reason behind the third party
prevented by law to avail of the procedural rules interposed by accused who later applied for liability (TPL) of the Compulsory Motor Vehicle
intended to avoid multiplicity of suits. Not even a "no probation. Liability Insurance is "to protect injured persons
action" clause under the policy-which requires that a - The lower court rendered a decision finding that against the insolvency of the insured who causes
final judgment be first obtained against the insured Destrajo had not exercised sufficient diligence as the such injury, and to give such injured person a certain
and that only thereafter can the person insured operator of the jeepney ordering him to pay plaintiffs beneficial interest in the proceeds of the policy . . ."
recover on the policy can prevail over the Rules of the sum for loss of income; funeral and burial Since petitioners had received from AFISCO the sum
Court provisions aimed at avoiding multiplicity of expenses of the deceased; moral damages, and of P5,000.00 under the no-fault clause, AFISCO's
suits. attorney's fees and costs of suit. The defendant liability is now limited to P15,000.00.
Disposition instant petition is GRANTED. The insurance company is ordered to reimburse - However, we cannot agree that AFISCO is likewise
questioned order dated 24 April 1987 is SET ASIDE defendant Destrajo whatever amounts the latter shall solidarily liable with Destrajo. In Malayan Insurance
and a new one entered admitting petitioner's third have paid only up to the extent of its insurance Co., Inc. v. Court of Appeals, this Court had the
party complaint against the private respondent coverage. opportunity to resolve the issue as to the nature of
Makati Insurance Company, Inc. - Petitioners filed a motion for the reconsideration of the liability of the insurer and the insured vis-a-vis
the second paragraph of the decision contending that the third party injured in an accident. We
VDA DE MAGLANA v. CONSOLACION AFISCO should not merely be held secondarily liable categorically ruled thus: While it is true that where
212 SCRA 268 because the Insurance Code provides that the the insurance contract provides for indemnity against
ROMERO; August 6, 1992 insurer's liability is "direct and primary and/or jointly liability to third persons, such third persons can
and severally with the operator of the vehicle, directly sue the insurer, however, the direct liability
NATURE although only up to the extent of the insurance of the insurer under indemnity contracts against
Petition for certiorari coverage." Hence, they argued that the P20,000.00 third party liability does not mean that the insurer
coverage of the insurance policy issued by AFISCO, can be held solidarily liable with the insured and/or
FACTS should have been awarded in their favor. the other parties found at fault. The liability of the
- Lope Maglana was an employee of the Bureau of - AFISCO argued that since the Insurance Code does insurer is based on contract; that of the insured is
Customs whose work station was at Lasa, here in not expressly provide for a solidary obligation, the based on tort. In the case at bar, petitioner as
Davao City. One day, when he was on his way to his presumption is that the obligation is joint. insurer of Sio Choy, is liable to respondent Vallejos
work, he met an accident that resulted in his death. - The lower court denied the motion for (the injured third party), but it cannot, as incorrectly
He died on the spot. reconsideration ruling that since the insurance held by the trial court, be made "solidarily" liable
- The PUJ jeep that bumped the deceased was driven contract "is in the nature of suretyship, then the with the two principal tortfeasors, namely
by Pepito Into, operated and owned by defendant liability of the insurer is secondary only up to the respondents Sio Choy and San Leon Rice Mill, Inc.
Destrajo. From the investigation conducted by the extent of the insurance coverage." For if petitioner-insurer were solidarily liable with
traffic investigator, the PUJ jeep was overtaking - Petitioners filed a second motion for reconsideration said, two (2) respondents by reason of the indemnity
another passenger jeep that was going towards the reiterating that the liability of the insurer is direct, contract against third party liability under which an
city poblacion. While overtaking, the PUJ jeep of primary and solidary with the jeepney operator insurer can be directly sued by a third party this will
defendant Destrajo running abreast with the because the petitioners became direct beneficiaries result in a violation of the principles underlying
overtaken jeep, bumped the motorcycle driven by under the provision of the policy which, in effect, is a solidary obligation and insurance contracts.
the deceased. The point of impact was on the lane of stipulation pour autrui. This motion was likewise - While in solidary obligations, the creditor may
the motorcycle and the deceased was thrown from denied for lack of merit. enforce the entire obligation against one of the
the road and met his untimely death. solidary debtors, in an insurance contract, the
- Heirs of Lope Maglana, Sr. filed an action for ISSUE insurer undertakes for a consideration to indemnify
damages and attorney's fees against operator WON AFISCO can be held directly liable the insured against loss, damage or liability arising
Patricio Destrajo and the Afisco Insurance from an unknown or contingent event.
Corporation (AFISCO). An information for homicide HELD - Similarly, petitioners herein cannot validly claim
thru reckless imprudence was also filed against YES that AFISCO, whose liability under the insurance
Pepito Into. - As this Court ruled in Shafer vs. Judge, RTC of policy is also P20,000.00, can be held solidarily liable
- During the pendency of the civil case, Into was Olongapo City, Br. 75, "[w]here an insurance policy with Destrajo for the total amount of P53,901.70 in
sentenced to suffer an indeterminate penalty, with all insures directly against liability, the insurer's liability accordance with the decision of the lower court.
the accessory penalties provided by law, and to accrues immediately upon the occurrence of the Since under both the law and the insurance policy,
indemnify the heirs of Lope Maglana, Sr. in the injury or even upon which the liability depends, and AFISCO's liability is only up to P20,000.00, the
amount of twelve thousand pesos with subsidiary second paragraph of the dispositive portion of the
INSURANCE Page 83

decision in question may have unwittingly sown involved in accidents caused by, or arising out of, the - vehicular accident with 2 children running across
confusion among the petitioners and their counsel. use of the motor vehicle. A motion for the path of a Chevrolet "Carry-All", belonging to a
What should have been clearly stressed as to leave reconsideration was filed in and dismissed by the CA. partnership known as Diman & Company driven by
no room for doubt was the liability of AFISCO under its driver, Perfecto Amar, as it was passing a national
the explicit terms of the insurance contract. ISSUE highway at barrio Makiling Calamba, Laguna. They
Disposition present petition is hereby GRANTED. WON Far Eastern Surety is liable to the insured on its were killed. It was insured with the Empire Insurance
The award of P28,800.00 representing loss of income insurance policy Co., Inc. under a so-called 'comprehensive coverage"
is INCREASED to P192,000.00 and the death policy, loss by theft excluded. The policy was in force
indemnity of P12,000.00 to P50,000.00. HELD at the time of the accident.
NO - Placida Peza, the managing partner of Diman & Co.
- The award for damages made to the passengers filed a claim with Empire, for payment of
was exclusively predicated on the representation compensation to the family of the 2 children who
made by La Mallorca that its passengers were died as a result of the accident. Empire refused to
insured against accidents and not because it was at pay on the ground that the driver had no authority to
fault in causing the accident. operate the vehicle, a fact which it expressly
FAR EASTERN SURETY v. MISA Reasoning excepted from liability under the policy. What Peza
25 SCRA 663 - In this case, the findings of the CA and the trial did was to negotiate directly with the deceased
REYES; October 26, 1968 court that the causative factor of the mishap was the children father for an out-of-court settlement. The
negligence of the gravel and truck driver would have father agreed to accept P6,200.00 in fun settlement
NATURE been sufficient to relieve the taxi company of any of the liability of the vehicles owner and driver, and
Appeal by petition for review from a CA judgment liability arising from the accident. However, in view of Peza paid him this sum.
the sticker in all of its taxicabs, La Mallorca has - Peza thereafter sued Empire to recover this sum of
FACTS insured its passengers against accidents, whether it P6,200.00 as actual damages, as well as P20,000.00
- Socorro Dancel Vda.de Misa and Araceli Pinto hired was at fault or not. In other words, La Mallorca as moral damages, P10,000.00 as exemplary
a taxi cab operated by La Mallorca on September 3, accepted the responsibility for damages or injuries to damages, and P10,000.00 as attorney's fees. She
1957. The taxi they were riding in collided with a passengers even if it had no fault at all. amended her complaint shortly thereafter to include
gravel and sand truck resulting to injuries to both - In the case of the insurance company, the SC ruled Diman & Co. as alternative party plaintiff.
Misa and Pinto. that it neither authorized nor consented to the - Empire's basic defense to the suit was anchored
- The two passengers instituted a suit for damages representations made by the taxi company to its on the explicit requirement in the policy limiting the
against La Mallorca who, while denying responsibility, passengers. As such, the liability of the said operation of the insured vehicle to the "authorized
instituted a third party complaint against Far Eastern insurance company based on its insurance contract driver" therein defined, namely, (a) the insured, or
Surety to recoup from the latter any award for is limited to the recovery by the insured of all sums, (b) any person driving on the insured order or with
damages that might be recovered by the passengers. cost and expenses which the insured shall become his permission, provided that-
- It would appear from the case that a sticker was legally liable. The insurance company therefore ... that the person driving is permited in
placed in all the taxis of La Mallorca stating that cannot be held liable for the award. accordance with the licensing or other laws or
passengers of the taxis were insured against - The taxi company is adjudged to be the sole party regulations to drive the Motor vehicle or has been
accidents. This was done to entice the public into responsible for the award. so permitted and is not disqualified by order of the
patronizing La Mallorca. Disposition The decision of the CA is modified by Court of Law of by reason of any enactment or
- The trial court awarded to Misa and Pinto actual, eliminating the award against Far Eastern. regulation in that behalf from driving such Motor
moral and exemplary damages, and attorney’s fees Vehicle.-
payable by La Mallorca and sentenced Far Eastern to PEZA v. ALIKPALA - driver Perfecto Amar, only having a temporary
pay La Mallorca P10,000. on its third party liability 160 SCRA 31 operator's permit (TVR) [already expired] his driver’s
insurance. NARVASA; April 15, 1988 license having earlier been confiscated by an agent
- On appeal, the CA, while holding that the collision of the Land Transportation Commission for an alleged
was due to the fault of the driver of the gravel and NATURE violation of Land Transportation and Traffic Rules,
sand truck, found the taxi company liable for Motion praying that Judge Alikpala be declared guilty was not permitted by law and was in truth
damages to the passengers on the strength of its of contempt of court for having decided the case on disqualified to operate any motor vehicle; Peza
representation contained in the sticker above noted the merits despite the pendency in this Court of the attempted to neutralize that fact by(1) the issuance
that the passengers were insured against accidents. certiorari action instituted by the plaintiffs of the TVR by the LTC officer to Amar; in proof of the
In so ruling, the CA overruled the defense of the proposition that there was no reason for confiscation
insurance company to the effect that it was FACTS of Amar's license (2) Amar's license had not expired,
responsible only if the insured, La Mallorca, was but had been renewed.
INSURANCE Page 84

- Judge Alikpala did not admit such evidence


ISSUE
ISSUES WON Perla is the insurer liable to indemnify under
1. WON Judge Alikapala committed grave abuse of Sec. 378
discretion in not admitting evidence PERLA COMPANIA DE SEGUROS v. ANCHETA
2. WON confiscation of license and expiration of TVR 164 SCRA 144 HELD
of the driver would serve as bar for Peza in CORTES; August 8, 1988 NO
recovering from Empire Ratio The law is very clear – the claim shall lie
NATURE against the insurer of the vehicle in which the
HELD Petition for certiorari and prohibition with prelim “occupant xxx is riding,” and no other. The claimant
1. NO injunction to review orders of CFI Camarines Norte is not free to choose from which insurer he will claim
- Even positing error in the Judge's analysis of the the “no fault indemnity,” as the law, by using the
evidence attempted to be introduced and his FACTS word “shall”, makes it mandatory that the claim be
rejection thereof, it is clear that it was at most an - Perla was the insurer of a Superlines bus which made against the insurer of the vehicle in which the
error of judgment, not such an error as may be figured in a collision with a III Scout (it’s a kind of occupant is riding, mounting or dismounting from.
branded a grave abuse of discretion, i.e., such vehicle). Injured passengers of the latter (and Reasoning
capricious and whimsical exercise of judgment as is respondents in this case) filed a complaint for - the rules on claims under the “no fault indemnity”
equivalent to lack of jurisdiction, against which the damages against Superlines, the bus driver, and provision, where proof of fault or negligence is not
writ of certiorari will lie. In any event, the established Perla (as insurer of the bus). CFI Judge Ancheta necessary for payment of any claim for death or
principle is "that ruling of the trial court on ordered that Perla should pay the respondents injury to a passenger or to a 3 rd party, are
procedural questions and on admissibility of evidence immediately the P5000 under the “no fault clause” as established:
during the course of the trial are interlocutory in provided in Sec. 378. 1. A claim may be made against one motor vehicle
nature and may not be the subject of separate Sec. 378: Any claim for death or injury to any only.
appeal or review on certiorari, but are to be assigned passenger or 3rd party pursuant to the provisions of 2. If the victim is an occupant of a vehicle, the claim
as errors and reviewed in the appeal properly taken this chapter shall be paid without the necessity of shall lie against the insurer of the vehicle in which he
from the decision rendered by the trial court on the proving fault or negligence of any kind. Provided, is riding, mounting or dismounting from.
merits of the case. That for purposes of this section 3. In any other case (i.e. if the victim is not an
- In the meantime, Judge Alikpala rendered (i) The indemnity in respect of any one person shall occupant of a vehicle), the claim shall lie against the
judgment on the merits, since the case was then not exceed P5,000; insurer of the directly offending vehicle.
already ripe for adjudication. The judgment ordered (ii) The following proofs of loss, when submitted 4. In all cases, the right of the party paying the claim
dismissal of the case for failure on the part of the under oath, shall be sufficient evidence to to recover against the owner of the vehicle
plaintiff to prove their cause of action against substantiate the claim: responsible for the accident shall be maintained.
Empire. Notice of the judgment was served on the (a) Police report of accident, and -That the vehicle ridden might not be the one that
parties in due course. (b) Death certificate and evidence sufficient to caused the accident is of no moment since the law
2. YES establish, the proper payee, or itself provides that the party paying the claim under
- It would seem fairly obvious that whether the LTC (c) Medical report and evidence of medical or Sec. 378 may recover against the owner of the
agent was correct or not in his opinion that driver hospital disbursement in respect of which refund is vehicle responsible for the accident. This is precisely
Amar had violated some traffic regulation warranting claimed; the essence of “no fault indemnity” insurance which
confiscation of his license and issuance of a TVR in (iii) Claim may be made against one motor vehicle was introduced to and made part of our laws in order
lieu thereof, this would not alter the undisputed fact only. In the case of an occupant of a vehicle, claim to provide victims of vehicular accidents or their
that Amar's licence had indeed been confiscated and shall lie against the insurer of the vehicle in which heirs immediate compensation, although in a limited
a TVR issued to him, and the TVR had already the occupant is riding, mounting or dismounting amount, pending final determination of who is
expired at the time that the vehicle being operated from. In any other case, claim shall lie against the responsible for the accident and liable for the victims'
by him killed two children by accident. Neither would insurer of the directly offending vehicle. In all cases, injuries or death. In turn, the “no fault indemnity”
proof of the renewal of Amar's license change the the right of the party paying the claim to recover provision is part and parcel of the Code provisions on
fact that it had really been earlier confiscated by the against the owner of the vehicle responsible for the compulsory motor vehicle liability insurance and
LTC agent. accident shall be maintained. should be read together with the requirement for
Disposition petition is DISMISSED for lack of merit Perla denied its liability under the above provision compulsory passenger and/or 3rd party liability
and said that the insurer of the vehicle that the insurance (Sec. 377) which was mandated in order
respondents were riding (Malayan Insurance in this to ensure ready compensation for victims of
case) should be liable. Its 2 MFRs denied, Perla filed vehicular accidents.
this action
INSURANCE Page 85

-Irrespective of whether or not fault or negligence for in this policy excluding the cost of additional - The Schedule of Indemnities does not purport to
lies with the driver of the Superlines bus, as medicines, and such other burial and funeral restrict the kinds of damages that may be awarded
respondents were not occupants of the bus, they expenses that might have been incurred. against Western once liability has arisen. Section 1,
cannot claim the “no fault indemnity” provided in - Respondent Priscilla Rodriguez filed a complaint for quoted above, does refer to certain "Limits of
Sec. 378 from Perla. The claim should be made damages before the Regional Trial Court of Makati Liability" which in the case of the third party liability
against the insurer of the vehicle they were riding. against De Dios Transportation Co. and Walter A. section of the Master Policy, is apparently P50,000.00
Disposition Petition GRANTED. Orders of CFI Saga Respondent De Dios Transportation Co., in turn, per person per accident. Within this over-all
ordering Perla to pay respondents immediately filed a third-party complaint against its insurance quantitative limit, all kinds of damages allowable by
P5000 ANNULLED and SET ASIDE carrier, petitioner Western. On 6 August 1985, the law" actual or compensatory damages"; "moral
trial court rendered a decision in favor of respondent damages'; "nominal damages"; "temperate or
WESTERN GUARANTY CORPORATION v. CA Priscilla E. Rodriguez, awarding moral damages, moderate damages"; "liquidated damages"; and
(RODRIGUEZ & DE DIOS TRANS) lossof earning and attorney's fees among others. "exemplary damages" ? may be awarded by a
185 SCRA 652 - On appeal, the Court of Appeals affirmed in toto competent court against the insurer once liability is
FELICIANO; July 20, 1990 the decision of the trial court. shown to have arisen, and the essential requisites or
- Petitioner contends that it cannot be held liable for conditions for grant of each species of damages are
NATURE loss of earnings, moral damages and attorney's fees present. It appears to us self-evident that the
Petition for review the decision of CA affirming in toto because these items are not among those included in Schedule of Indemnities was not intended to be an
the damages awarded to private respondent by the the Schedule of Indemnities set forth in the enumeration, much less a closed enumeration, of the
trial court. insurance policy. specific kinds of damages which may be awarded
under the Master Policy Western has issued.
FACTS ISSUE - The reading urged by Western of the Schedule of
- Respondent Priscilla E. Rodriguez was struck by a WON petitioner can be held liable for loss of Indemnities comes too close to working fraud upon
De Dios passenger bus owned by respondent De Dios earnings, moral damages and attorney's fees both the insured and the third party beneficiary of
Transportation Co., Inc. Priscilla was thrown to the Section 1, quoted above. For Western's reading
ground, hitting her forehead. She was treated at the HELD would drastically and without warning limit the
Protacio Emergency Hospital and later on YES otherwise unlimited and comprehensive scope of
hospitalized at the San Juan De Dios Hospital. Her - The Schedule of Indemnities does not purport to liability assumed by the insurer Western under
face was permanently disfigured, causing her serious restrict the kinds of damages that may be awarded Section 1: "all sums necessary to discharge liability
anxiety and moral distress. Respondent bus company against Western once liability has arisen. It was of the insured in respect of [bodily injury to a third
was insured with petitioner Western Guaranty merely meant to set limits to the amounts the party]". This result- which is not essentially different
Corporation ("Western") under its Master Policy movant would be liable for in cases of claims for from taking away with the left hand what had been
which provided, among other things, for protection death, bodily injuries of, professional services and given with the right hand we must avoid as obviously
against third party liability, the relevant section hospital charges, for services rendered to traffic repugnant to public policy. If what Western now
reading as follows: accident victims,' and not necessarily exclude claims urges is what Western intended to achieve by its
Section 1. Liability to the Public ? Company will, against the insurance policy for other kinds of Schedule of Indemnities, it was incumbent upon
subject to the Limits of Liability, pay all sums damages, such as those in question. Western to use language far more specific and
necessary to discharge liability of the insured in - It will be seen that the above quoted Schedule of precise than that used in fact by Western, so that the
respect of ? Indemnities establishes monetary limits which insured, and potential purchasers of its Master Policy,
(a) death of or bodily injury to or damage to Western may invoke in case of occurrence of the and the Office of the Insurance Commissioner, may
property of any passenger as defined herein. particular kinds of physical injury there listed. be properly informed and act accordingly.
(b) death of or bodily injury or damage to property - It must be stressed, however, that the Schedule of - Moreover, an insurance contract is a contract of
of any THIRD PARTY as defined herein in any Indemnities does not purport to limit, or to adhesion. The rule is well entrenched in our
accident caused by or arising out of the use of the enumerate exhaustively, the species of bodily injury jurisprudence that the terms of such contract are to
Schedule Vehicle, provided that the liability shall occurrence of which generate liability for petitioner be construed strictly against the party which
have first been determined. In no case, however, Western. A car accident may, for instance, result in prepared the contract, which in this case happens to
shall the Company's total payment under both injury to internal organs of a passenger or third be petitioner Western.
Section I and Section 11 combined exceed the party, without any accompanying amputation or loss Disposition Petition denied.
Limits of Liability set forth herein. With respect to of an external member (e.g., a foot or an arm or an
death of or bodily injury to any third party or eye). But such internal injuries are surely covered by SUMMIT GUARANTY & INSURANCE COMPANY v.
passenger, the company's payment per victim in Section I of the Master Policy, since they certainly ARNALDO
any one accident shall not exceed the limits constitute bodily injuries. 158 SCRA 332
indicated in the Schedule of indemnities provided GANCAYCO; February 29, 1988
INSURANCE Page 86

setting forth the amount of his loss, and/or the ready to pay. In the same case, petitioner company
NATURE nature, extent and duration of the injuries admits that it took no final action or adjudication of
PETITION to review the order of the Insurance sustained as certified by a duly licensed physician. the claim. Worse still, in G.R. No. L-48679,
Commissioner. Notice of claim must be filed within six months assurances of payment were constantly given and
from date of the accident, otherwise, the claim petitioner company even said that a check was ready
FACTS shall be deemed waived. Action or suit for recovery for release. This Court has made the observation that
- On Nov. 26, 1976, a Ford Pick-up truck owned by of damage due to loss or injury must be brought, some insurance companies have been inventing
Marcos Olasco was bumped by a cargo truck owned in proper cases, with the Commissioner or the excuses to avoid their just obligations and it is only
by Floralde. Courts within one year from date of accident, the State that can give the protection which the
FGU Insurance Corporation (FG U) by reason of otherwise, the claimant's right of action shall insuring public needs from possible abuses of the
Motor Vehicle Insurance Policy No. IC-VF-07185 paid prescribe. insurers. In view of the foregoing,
Olaso the sum of P2,817.50 as its share in the repair - It is very clear that the one-year period is only - It is not denied that an extrajudicial demand for
cost of the said Ford Pick-up. Having been required In proper cases. Had the lawmakers payment was made by respondent FGU on petitioner
subrogated to the rights and causes of action of intended it to be the way Petitioner Company but petitioner failed to respond to the same.
Olaso in the said amount FGU formally demanded assumes it to be, then the phrase 'in proper cases' Nevertheless the complaint was filed even before a
payment of said amount from Floralde and would not have been inserted. denial of the claim was made by petitioner. For all
attempted to verify Floralde's insurance carrier but - in Aisporna. vs. Court of Appeals: legal purposes, the one-year prescriptive period
failed to do so. In 1978 FGU was able to ascertain 'Legislative intent must be ascertained from a provided for in Section 384 of the Insurance Code
the identity of Floralde's insurance carrier to be the consideration of the statute as a whole. The has not begun to run.The cause of action arises only
Summit Guaranty and Insurance Company, Inc. particular words, clauses and phrases should not and starts to run upon the denial of the claim by the
(Summit) and thus requested the insurance be studied as detached and isolated expressions, insurance company.The court takes note of the
commissioner for a conference with Summit and but the whole and every part of the statute must dilatory tactics employed by petitioner in this as in
demanded from Summit through counsel on be considered in fixing the meaning of any of its the several cases aforecited to avoid payment of its
February 28, 1978 the payment of the damages parts and in order to produce a harmonious whole. liabilities.
sustained by the car of Olaso but to no avail. A statute must be so construed as to harmonize
- Hence on May 22, 1978 FGU filed a case in the and give effect to all its provisions whenever VILLACORTA v. THE INSURANCE COMMISSION
Insurance Commissioner's Office against Summit for possible.' 100 SCRA 467
recovery of said amount. - Petitioner company is trying to use Section 384 of TEEHANKEE; October 30, 1980
- Summit filed a motion to dismiss on the ground of the Insurance Code as a cloak to hide itself from its
prescription under Section 384 of PD No. 612. liabilities. The facts of these cases evidently reflect FACTS
Averring that the accident happened on November the deliberate efforts of petitioner company to - JEWEL VILLACORTA was the owner of a Colt Lancer,
26, 1976 while the complaint was filed on May 22, prevent the filing of a formal action against it. Model 1976, insured with respondent company for
1978 beyond the one-year period from the time of Bearing in mind that if it succeeds in doing so until P35,000.00 - Own Damage; P30,000.00 - Theft; and
the accident provided for by the said provision. one year lapses from the date of the accident it could P30,000.00 - Third Party Liability, effective May 16,
set up the defense of prescription, petitioner 1977 to May 16, 1978.
ISSUE company made private respondents believe that their - On May 9, 1978, the vehicle was brought to the
WON the action must be dismissed on the ground of claims would be settled in order that the latter will Sunday Machine Works, Inc., for general check-up
prescription under Section 384 of PD No. 612 not find it necessary to immediately bring suit. In and repairs. On May 11, 1978, while it was in the
violation of its duties to adopt and implement custody of the Sunday Machine Works, the car was
HELD reasonable standards for the prompt investigation of allegedly taken by six (6) persons and driven out to
NO claims and to effectuate prompt, fair and equitable Montalban, Rizal. While travelling along Mabini St.,
- The case do not fall within the meaning of proper settlement of claims, and with manifest bad faith, Sitio Palyasan, Barrio Burgos, going North at
cases' as contemplated in Section 384 of the petitioner company devised means and ways of Montalban, Rizal, the car figured in an accident,
Insurance Code. stalling the settlement proceedings. In G.R. No. L- hitting and bumping a gravel and sand truck parked
50997, no steps were taken to process the claim and at the right side of the road going south. As a
no rejection of said claim was ever made even if consequence, the gravel and sand truck veered to
Reasoning private respondent had already complied with all the the right side of the pavement going south and the
- Section 384 of PD 612 (Insurance Code) requirements. car veered to the right side of the pavement going
Any person having any claim upon the policy - In G.R. No. L-48758-petitioner company even north. The driver, Benito Mabasa, and one of the
issued pursuant to this chapter shall, without any provided legal assistance to one of the private passengers died and the other four sustained
unnecessary delay, present to the insurance respondents in the criminal case filed against him physical injuries. The car, as well, suffered extensive
company concerned a written notice of claim leading Private respondents to believe that it was damage. Complainant, thereafter, filed a claim for
INSURANCE Page 87

total loss with the respondent company but claim participation of the other party is the signing of his
was denied. Hence, complainant was compelled to signature or his "adhesion" thereto, "obviously call
institute the present action." for greater strictness and vigilance on the part of
- The comprehensive motor car insurance policy for courts of justice with a view of protecting the weaker
P35,000.00 issued by respondent Empire Insurance party from abuse and imposition, and prevent their
Company admittedly undertook to indemnify the becoming traps for the unwary."
petitioner-insured against loss or damage to the car - The main purpose of the "authorized driver" clause, CHAPTER VIII. MARINE INSURANCE
(a) by accidental collision or overturning, or collision as may be seen from its text, supra, is that a person
or overturning consequent upon mechanical other than the insured owner, who drives the car on MAGSAYSAY INC v. AGAN
breakdown or consequent upon wear and tear; (b) the insured's order, such as his regular driver, or with 96 PHIL 504
by fire, external explosion, self-ignition or lightning his permission, such as a friend or member of the REYES; January 31, 1955
or burglary, housebreaking or theft; and (c) by family or the employees of a car service or repair
malicious act. shop must be duly licensed drivers and have no FACTS
- Respondent insurance commission, however, disqualification to drive a motor vehicle. A car owner - The S S "San Antonio", vessel owned and operated
dismissed petitioner's complaint for recovery of the who entrusts his car to an established car service by plaintiff, left Manila on October 6, 1949, bound for
total loss of the vehicle against private respondent, and repair shop necessarily entrusts his car key to Basco, Batanes, vis Aparri, Cagayan, with general
sustaining respondent insurer's contention that the the shop owner and employees who are presumed to cargo belonging to different shippers, among them
accident did not fall within the provisions of the have the insured's permission to drive the car for the defendant. The vessel reached Aparri, but while
policy either for the Own Damage or Theft coverage, legitimate purposes of checking or road-testing the still in the port, it ran aground at the mouth of the
invoking the policy provision on "Authorized Driver" car. The mere happenstance that the employee(s) of Cagayan river, and, attempts to refloat it under its
clause, which clause limits the use of the insured the shop owner diverts the use of the car to his own own power having failed, plaintiff had it refloated by
vehicle to two (2) persons only, namely: the insured illicit or unauthorized purpose in violation of the trust the Luzon Stevedoring Co. at an agreed
himself or any person on his (insured's) permission. reposed in the shop by the insured car owner does compensation. Once afloat the vessel returned to
Apparently, the Insurance commission sees the not mean that the "authorized driver" clause has Manila to refuel and then proceeded to Basco, the
unauthorized taking of the vehicle for a joyride as a been violated such as to bar recovery, provided that port of destination. There the cargoes were delivered
violation of the 'Authorized Driver' clause of the such employee is duly qualified to drive under a valid to their respective owners or consignees, who, with
policy." driver's license. the exception of defendant, made a deposit or signed
- Respondent commission likewise upheld private - Secondly, and independently of the foregoing (since a bond to answer for their contribution to the
respondent's assertion that the car was not stolen when a car is unlawfully taken, it is the theft clause, average.
and therefore not covered by the Theft clause, ruling not the "authorized driver" clause, that applies), - On the theory that the expenses incurred in floating
that "(T)he element of 'taking' in Article 308 of the where a car is admittedly as in this case unlawfully the vessel constitute general average to which both
Revised Penal Code means that the act of depriving and wrongfully taken by some people, be they ship and cargo should contribute, plaintiff brought
another of the possession and dominion of a movable employees of the car shop or not to whom it had the present action in the CFI of Manila to make
thing is coupled . . . with the intention, at the time of been entrusted, and taken on a long trip to defendant pay his contribution, which, as determined
the 'taking', of withholding it with the character of Montalban without the owner's consent or by the average adjuster, amounts to P841.40.
permanency knowledge, such taking constitutes or partakes of - Defendant denies liability to his amount, alleging,
the nature of theft as defined in Article 308 of the among other things, that the stranding of the vessel
ISSUE Revised Penal Code. was due to the fault, negligence and lack of skill of
WON the Insurance commission’s findings are in - The Court rejects respondent commission's its master, that the expenses incurred in putting it
accord with law premise that there must be an intent on the part of afloat did not constitute general average, and that
the taker of the car "permanently to deprive the the liquidation of the average was not made in
HELD insured of his car" and that since the taking here was accordance with law.
NO for a "joy ride" and "merely temporary in nature," a - The lower court found for plaintiff
- First, respondent commission's ruling that the "temporary taking is held not a taking insured
person who drove the vehicle in the person of Benito against." ISSUE
Mabasa, who, according to its own finding, was one - The insurer must therefore indemnify the petitioner WON the expenses incurred in floating a vessel so
of the residents of the Sunday Machine Works, Inc. owner for the total loss of the insured car in the sum stranded should be considered general average and
to whom the car had been entrusted for general of P35,000.00 under the theft clause of the policy, shared by the cargo owners
check-up and repairs was not an "authorized driver" subject to the filing of such claim for reimbursement
of petitioner-complainant is too restrictive and or payment as it may have as subrogee against the HELD
contrary to the established principle that insurance Sunday Machine Works, Inc. NO
contracts, being contracts of adhesion where the only
INSURANCE Page 88

Ratio The law on averages is contained in the Code the common safety of vessel and cargo, since they, Sea. Arrests, Restraints and Detainments, of all
of Commerce. Under that law, averages are classified or at least the cargo, were not in imminent peril. Kings, Princes and People of what Nation,
into simple or particular and general or gross. - With respect to the third requisite, the salvage Condition or Quality soever; Barratry of the Master
Generally speaking, simple or particular averages operation, it is true, was a success. But as the and Marines, and of all other Perils, Losses and
include all expenses and damages caused to the sacrifice was for the benefit of the vessel to enable it Misfortunes, that have or shall come to the Hurt,
vessel or cargo which have not inured to the to proceed to destination and not for the purpose of Detriment, or Damage of the said Vessel or any
common benefit (Art. 809), and are, therefore, to be saving the cargo, the cargo owners are not in law part thereof; and in case of any Loss or
borne only by the owner of the property gave rise to bound to contribute to the expenses. Misfortunes, it shall be lawful for the Assured, his
same (Art. 810); while general or gross averages - The final requisite has not been proved, for it does or their Factors, Servants, or assigns, to sue,
include "all the damages and expenses which are not appear that the expenses here in question were labour and travel for, in and about the Defence.
deliberately caused in order to save the vessel, its incurred after following the procedure laid down in Safeguard, and recovery of the said Vessel or any
cargo, or both at the same time, from a real and article 813. part thereof, without Prejudice to this Insurance;
known risk" (Art. 811). Being for the common Disposition Wherefore, the decision appealed from to the Charges whereof the said Company, will
benefit, gross averages are to be borne by the is reversed. contribute, according to the rate and quantity of
owners of the articles saved (Art. 812). the sum herein assured...”
Reasoning JARQUE v. SMITH, BELL & CO. - Attached to the policy over and above the said
- the stranding of plaintiff's vessel was due to the 56 PHIL 758 clause is a “rider” containing typewritten provisions,
sudden shifting of the sandbars at the mouth of the OSTRAND; November11, 1932 among which appears in capitalized type the
river which the port pilot did not anticipate. The following clause:
standing may, therefore, be regarded as accidental. NATURE “AGAINST THE ABSOLUTE TOTAL LOSS OF THE
- Tolentino, in his commentaries on the Code of Appeal from judgment of the lower court VESSEL ONLY, AND TO PAY PROPORTIONATE
Commerce, gives the following requisites for SALVAGE CHARGES OF THE DECLARED VALUE.”
general average: FACTS
First, there must be a common danger. This means, - Plaintiff’s motorboat, “Pandan” was insured on a ISSUES
that both the ship and the cargo, after has been marine insurance policy with National Union Fire 1. WON the lower court erred in disregarding the
loaded, are subject to the same danger, whether Insurance Company (NUFIC) for P45K. According to typewritten clause endorsed upon the policy,
during the voyage, or in the port of loading or the provisions of a “rider” attached to the policy, the expressly limiting insurer's liability thereunder of the
unloading; that the danger arises from the accidents insurance was against the “absolute total loss of the total loss of the wooden vessel Pandan and to
of the sea, dispositions of the authority, or faults of vessel only.” On Oct. 31, 1928, the ship ran into very proportionate salvage charges
men, provided that the circumstances producing the heavy sea and it became necessary to jettison a 2. WON lower court erred in concluding that
peril should be ascertained and imminent or may portion of the cargo. As a result of the jettison, the defendant and appellant, NUFIC is liable to
rationally be said to be certain and imminent. This NUFIC was assessed P2,610.86 as its contribution to contribute to the general average resulting from the
last requirement exclude measures undertaken the general average. jettison of a part of said vessel's cargo
against a distant peril. - The insurance company, insisting that its obligation
Second, that for the common safety part of the did not extend beyond the insurance of the “absolute HELD
vessel or of the cargo or both is sacrificed total loss of the vessel only, and to pay proportionate 1. NO
deliberately. salvage of the declared value,” refused to contribute Ratio In case repugnance exists between written
Third, that from the expenses or damages caused to the settlement of the gen. ave. The present action and printed portions of a policy, the written portion
follows the successful saving of the vessel and cargo. was thereupon instituted, and after trial the court prevails.
Fourth, that the expenses or damages should have below rendered judgment in favor of the plaintiff and Reasoning
been incurred or inflicted after taking proper legal ordered the defendant to pay the plaintiff P2,610.86 - Section 291 of the Code of Civil Procedure provides
steps and authority. as its part of the indemnity for the gen. ave. brought that “when an instrument consists partly of written
- With respect to the first requisite, the evidence about by the jettison of cargo. The insurance words and partly of a printed form and the two are
does not disclose that the expenses sought to be company then appealed to the SC. inconsistent, the former controls the latter.”
recovered from defendant were incurred to save - The insurance contract is printed in the English
vessel and cargo from a common danger...it is the common form of marine policies. One of the clauses
safety of the property, and not of the voyage, which of the document originally read as follows: 2. NO
constitutes the true foundation of the general “Touching the Adventures and Perils which the Ratio The liability for contribution in general average
average. said NUFIC is content to bear, and to take upon is not based on the express terms of the policy, but
- As to the second requisite, we need only repeat them in this Voyage; they are of the Seas, Men-of- rests upon the theory that from the relation of the
that the expenses in question were not incurred for War, Fire, Pirates, Thieves, Jettison, Letters of parties and for their benefit, a quasi contract is
Mart and Countermart, Surprisals, and Takings at implied by law.
INSURANCE Page 89

Reasoning - On discharging the rice from one of the ISSUE


- In the absence of positive legislation to the compartments in the after hold, upon arrival at WON Union Insurance is liable for the loss of the Go
contrary, the liability of the defendant insurance Cebu, it was discovered that 1473 sacks had been Tiaco Brothers
company on its policy would, perhaps, be limited to damaged by sea water. The loss was P3,875.25.
“absolute loss of the vessel only, and to pay - The trial court found that the inflow of the sea HELD
proportionate salvage of the declared value.” But the water during the voyage was due to a defect in one NO
policy was executed in this jurisdiction and of the drain pipes of the ship and concluded that the - the words "all other perils, losses, and misfortunes"
“warranted to trade within the waters of the loss was not covered by the policy of insurance. The are to be interpreted as covering risks which are of
Philippine Archipelago only.” Here, Art. 859 of the trial court made the ff findings: like kind (ejusdem generis) with the particular risks
Code of Commerce is still in force: The drain pipe which served as a discharge which are enumerated in the preceding part of the
“ART. 859. The underwriters of the vessel, of the from the water closet passed down through same clause of the contract. ''According to the
freight, and of the cargo shall be obliged to pay for the compartment where the rice in question ordinary rules of construction, these words must be
the indemnity of the gross average in so far as is was stowed and thence out to sea through the interpreted with reference to the words which
required of each one of these objects wall of the compartment, which was a part of immediately precede them. They were no doubt
respectively.” the wall of the ship. The joint or elbow where inserted in order to prevent disputes founded on nice
- The article is mandatory in its terms, and the the pipe changed its direction was of cast distinctions. X x x For example, if the expression
insurers (whether for the vessel or for the freight or iron; and in course of time it had become 'perils of the seas' is given its widest sense the
for the cargo) are bound to contribute to the corroded and abraded until a longitudinal general words have little or no effect as applied to
indemnity of the general average. The provision opening had appeared in the pipe about one that case. If on the other hand that expression is to
simply places the insurer on the same footing as inch in length. This hole had been in existence receive a limited construction, as apparently it did in
other persons who have an interest in the vessel, or before the voyage was begun, and an attempt Cullen vs. Butler (5 M. & S., 461), and loss by perils
the cargo therein, at the time of the occurrence of had been made to repair it by filling with of the seas is to be confined to loss ex marine
the general average and who are compelled to cement and bolting over it a strip of iron. The tempestatis discrimine, the general words become
contribute (Art. 812, Code of Commerce). effect of loading the boat was to submerge most important. X x x" (Thames and Mersey Marine
- In the present case it is not disputed that the ship the vent, or orifice, of the pipe until it was Insurance Co. vs. Hamilton, Fraser & Co.)
was in grave peril and that the jettison of part of the about 18 inches or 2 feet below the level of - a loss which, in the ordinary course of events,
cargo was necessary. If the cargo was in peril to the the sea. As a consequence the sea water rose results from the natural and inevitable action of the
extent of call for general average, the ship must also in the pipe. Navigation under these conditions sea, from the ordinary wear and tear of the ship, or
have been in great danger, possibly sufficient to resulted in the washing out of the cement- from the negligent failure of the ship's owner to
cause its absolute loss. The jettison was therefore as filling from the action of the sea water, thus provide the vessel with proper equipment to convey
much to the benefit of the underwriter as to the permitting the continued flow of the salt water the cargo under ordinary conditions, is not a peril of
owner of the cargo. The latter was compelled to into the compartment of rice. the sea. Such a loss is rather due to what has been
contribute to the indemnity; why should not the - The court found in effect that the opening above aptly called the "peril of the ship." The insurer
insurer be required to do likewise? If no jettison had described had resulted in course of time from undertakes to insure against perils of the sea and
taken place and if the ship by reason thereof had ordinary wear and tear and not from the straining of similar perils, not against perils of the ship. There
foundered, the underwriter's loss would have been the ship in rough weather on that voyage. The court must, in order to make the insurer liable, be "some
many times as large as the contribution now also found that the repairs that had been made on casualty, something which could not be foreseen as
demanded. the pipe were slovenly and defective and that, by one of the necessary incidents of the adventure. The
Disposition Appealed judgment is affirmed reason of the condition of this pipe, the ship was not purpose of the policy is to secure an indemnity
properly equipped to receive the rice at the time the against accidents which may happen, not against
GO TIACO v. UNION INSURANCE voyage was begun. For this reason the court held events which must happen." (Wilson, Sons & Co. vs.
40 PHIL 40 that the ship was unseaworthy. Owners of Cargo per the Xantho)
STREET; September 1, 1919 - The policy purports to insure the cargo from the - In the present case the entrance of the sea water
following among other risks: "Perils . . . of the seas, into the ship's hold through the defective pipe
FACTS men, of war, fire, enemies, pirates, rovers, thieves, already described was not due to any accident which
- Union Insurance Society of Canton, Ltd., issued a .jettisons, . . . barratry of the master and mariners, happened during the voyage, but to the failure of the
marine insurance policy upon a cargo of rice and of all other perils, losses, and misfortunes that ship's owner properly to repair a defect of the
belonging to the Go Tiaoco Brothers, which was have or shall come to the hurt, detriment, or damage existence of which he was apprised. The loss was
transported in the early days of May, 1915, on the of the said goods and merchandise or any part therefore more analogous to that which directly
steamship Hondagua from the port of Saigon to thereof." results from simple unseaworthiness than to that
Cebu. which results from perils of the sea.
INSURANCE Page 90

- there is no room to doubt the liability of the shipment is not covered by a "square provision of the that Manila Bay would carry 422.18 cu. meters of
shipowner for such a loss as occurred in this case. By contract," but private respondent claims implied logs on its vessel Mable 10 from Malampaya Sound,
parity of reasoning the insurer is not liable; for, coverage from the phrase "perils of the sea" Palawan to Manila North Harbor. Roque insured the
generally speaking, the shipowner excepts the perils mentioned in the opening sentence of the policy; 2. logs with Pioneer Insurance for P100,000.
of the sea from his engagement under the bill of The insistence of private respondent that rusting is a - February 29, 1972 – 811 logs were loaded in
lading, while this is the very peril against which the peril of the sea is erroneous; 3. Rusting is not a Malampaya but en route to Manila, Mable 10 sank.
insurer intends to give protection. As applied to the risk insured against, since a risk to be insured - March 8,1972 – Roque and Chiong wrote a letter to
present case it results that the owners of the against should be a casualty or some casualty, Manila Bay, demanding payment of P150,000.00 for
damaged rice must look to the shipowner for redress something which could not be foreseen as one of the the loss of the shipment plus P100,000.00 as
and not to the insurer. necessary incidents of adventure; 4. A fact capable unrealized profits but the latter ignored the demand.
The same conclusion must be reached if the question of unquestionable demonstration or of public - A letter was also sent to Pioneer, claiming the full
be discussed with reference to the seaworthiness of knowledge needs no evidence. This fact of amount of P100,000.00 under the insurance policy
the ship. It is universally accepted that in every unquestionable demonstration or of public knowledge but Pioneer refused to pay on the ground that its
contract of insurance upon anything which is the is that heavy rusting of steel or iron pipes cannot liability depended upon the "Total Loss by Total Loss
subject of marine insurance, a warranty is implied occur within a period of a seven (7) day voyage. of Vessel only".
that the ship shall be seaworthy at the time of the Besides, petitioner had introduced the clear cargo - After hearing, the trial court favored Roque.
inception of the voyage. This rule is accepted in our receipts or tally sheets indicating that there was no Pioneer and Manila Bay were ordered to pay Roque
own Insurance Law (Act No. 2427, sec. 106). It is damage on the steel pipes during the voyage. P100,000. Pioneer appealed the decision.
also well settled that a ship which is seaworthy for - January 30, 1984 – Pioneer was absolved from
the purpose of insurance upon the ship may yet be ISSUE liability after finding that there was a breach of
unseaworthy for the purpose of insurance upon the WON rusting is a “peril of the sea” implied warranty of seaworthiness on the part of the
cargo (Act No. 2427, sec. 106). petitioners and that the loss of the insured cargo was
Disposition Decision of trial court is affirmed HELD caused by the "perils of the ship" and not by the
YES "perils of the sea". It ruled that the loss is not
CATHAY INSURANCE CO. v. CA (REMINGTON - There is no question that the rusting of steel pipes covered by the marine insurance policy.
INDUSTRIAL SALES CORP.) in the course of a voyage is a "peril of the sea" in - It was alleged that Mable 10 was not seaworthy
151 SCRA 710 view of the toll on the cargo of wind, water, and salt and that it developed a leak
PARAS; June 30 1987 conditions. At any rate if the insurer cannot be held - The IAC found that one of the hatches was left
accountable therefor, We would fail to observe a open, causing water to enter the barge and because
FACTS cardinal rule in the interpretation of contracts, the barge was not provided with the necessary cover
- Remington Industrial Sales Corp insured its namely, that any ambiguity therein should be or tarpaulin, the splash of sea waves brought more
shipment of seamless steel pipes. It incurred losses construed against the maker/issuer/drafter thereof, water inside the barge.
and damages (I gather the steel pipes rusted during namely, the insurer. Besides the precise purpose of - Petitioners contend that the implied warranty of
the voyage from Japan to the Phils. on board vessel insuring cargo during a voyage would be rendered seaworthiness provided for in the Insurance Code
SS "Eastern Mariner”) and filed complaint against fruitless. refers only to the responsibility of the shipowner who
Cathay Insurance Co seeking collection of the sum of Disposition WHEREFORE, this petition is hereby must see to it that his ship is reasonably fit to make
P868,339.15 DENIED, and the assailed decision of the Court of in safety the contemplated voyage.
- TC decided for Remington. Cathay filed MR, which Appeals is hereby AFFIRMED. - The petitioners state that a mere shipper of cargo,
was denied. CA affirmed. having no control over the ship, has nothing to do
- CA said (among other things): 1. Coverage of ROQUE v. IAC (PIONEER INSURANCE AND with its seaworthiness. They argue that a cargo
private respondent's loss under the insurance policy SURETY CORP.) owner has no control over the structure of the ship,
issued by petitioner is unmistakable; 139 SCRA 596 its cables, anchors, fuel and provisions, the manner
2. Alleged contractual limitations contained in GUTIERREZ; November 11, 1985 of loading his cargo and the cargo of other shippers,
insurance policies are regarded with extreme caution and the hiring of a sufficient number of competent
by courts and are to be strictly construed against the NATURE officers and seamen.
insurer; obscure phrases and exceptions should not Petition for certiorari to review the decision of the ISSUE
be allowed to defeat the very purpose for which the IAC WON the loss should have been covered by the
policy was procured; marine insurance policy
3. Rust is not an inherent vice of the seamless steel FACTS
pipes without interference of external factors - February 19, 1972 – Common carrier Manila Bay HELD
- Cathay contend (among other things): 1. private Lighterage Corp. entered into a contract with Roque NO
respondent has admitted that the questioned Timber Enterprises and Chiong. The contract stated
INSURANCE Page 91

Ratio It is universally accepted that in every contract - The words "all other perils, losses, and
of insurance upon anything which is the subject of FACTS misfortunes" are to be interpreted as covering risks
marine insurance, a warranty is implied that the ship - This is an action on a policy of marine insurance which are of like kind (ejusdem generis) with the
shall be seaworthy at the time of the inception of the issued by the Union Insurance Society of Canton, particular risks which are enumerated in the
voyage. In marine insurance, the risks insured Ltd., upon a cargo of rice belonging to the plaintiffs, preceding part of the same clause of the contract.
against are classified as 'perils of the sea,’ which Go Tiaoco Brothers, which was transported on the - A loss which, in the ordinary course of events,
includes such losses that are of extraordinary nature, steamship Hondagua from the port of Saigon to results from the natural and inevitable action of the
or arise from some overwhelming power, which Cebu. sea, from the ordinary wear and tear of the ship, or
cannot be guarded against by the ordinary exertion - On discharging the rice from one of the from the negligent failure of the ship's owner to
of human skill and prudence. compartments in the after hold, upon arrival at provide the vessel with proper equipment to convey
Reasoning Cebu, it was discovered that 1,473 sacks had been the cargo under ordinary conditions, is not a peril of
- Based on Sec. 113 and Sec. 99 of the Insurance damaged by sea water. the sea. Such a loss is rather due to what has been
Code, the term "cargo" can be the subject of marine - The loss so resulting to the owners of rice, after aptly called the "peril of the ship." The insurer
insurance and that once it is so made, the implied proper deduction had been made for the portion undertakes to insure against perils of the sea and
warranty of seaworthiness immediately attaches to saved, was P3,875.25. similar perils, not against perils of the ship.
whoever is insuring the cargo whether he be the - The trial court found that the inflow of the sea - As was said by Lord Herschell in Wilson, Sons & Co.
shipowner or not. water during the voyage was due to a defect in one vs. Owners of Cargo per the Xantho, there must, in
- The fact that the un-seaworthiness of the ship was of the drain pipes of the ship and concluded that the order to make the insurer liable, be "some casualty,
unknown to the insured is immaterial in ordinary loss was not covered by the policy of insurance. something which could not be foreseen as one of the
marine insurance and may not be used by him as a Judgment was accordingly entered in favor of the necessary incidents of the adventure. The purpose of
defense in order to recover on the marine insurance defendant and the plaintiffs appealed. the policy is to secure an indemnity against accidents
policy. - The court found in effect that the opening above which may happen, not against events which must
- Since the law provides for an implied warranty of described had resulted in course of time from happen."
seaworthiness in every contract of ordinary marine ordinary wear and tear and not from the straining of - In the present case the entrance of the sea water
insurance, it becomes the obligation of a cargo the ship in rough weather on that voyage. The court into the ship's hold through the defective pipe
owner to look for a reliable common carrier which also found that the repairs that had been made on already described was not due to any accident which
keeps its vessels in seaworthy condition. The shipper the pipe were slovenly and defective and that, by happened during the voyage, but to the failure of the
of cargo my have no control over the vessel but he reason of the condition of this pipe, the ship was not ship's owner properly to repair a defect of the
has full control in the choice of the common carrier properly equipped to receive the rice at the time the existence of which, he was apprised. The loss was
that will transport his goods. voyage was begun. For this reason the court held therefore more analogous to that which directly
- In marine cases, the risks insured against are that the ship was unseaworthy. results from simple unseaworthiness than to that
'perils of the sea.’ The term extends only to losses which results from perils of the sea.
caused by sea damage, or by the violence of the ISSUE - It is universally accepted that in every contract of
elements, and does not embrace all losses happening WON the insurer is liable insurance upon anything which is the subject of
at sea. marine insurance, a warranty is implied that the ship
- It is quite unmistakable that the loss of the cargo HELD shall be seaworthy at the time of the inception of the
was due to the perils of the ship rather than the - The question whether the insurer is liable on this voyage. This rule is accepted in our own Insurance
perils of the sea. policy for the loss caused in the manner above stated Law (Act No. 2427, see. 106).
- Loss which, in the ordinary course of events, presents two phases which are in a manner involved - It is also well settled that a ship which is
results from the natural and inevitable action of the with each other. One has reference to the meaning of seaworthy for the purpose of insurance upon the ship
sea, from the ordinary wear and tear of the ship, or the expression "perils of the seas and all other perils, may yet be unseaworthy for the purpose of insurance
from the negligent failure of the ship's owner to losses, and misfortunes," as used in the policy; the upon the cargo (Act No. 2427, see. 106).
provide the vessel with proper equipment to convey other has reference to the implied warranty, on the Disposition Jjudgment affirmed.
the cargo under ordinary conditions, is not a ‘peril of part of the insured, as to the seaworthiness of the
the sea’ but is called ‘peril of the ship.’ ship. MALAYAN INSURANCE v. CA (supra p.10)
Disposition Decision appealed from is affirmed. - The meaning of the expression "perils * * * of the
seas * * * and all other perils, losses, and FILIPINO MERCHANTS INS. CO. v. CA (supra
LA RAZON v. UNION INSURANCE SOCIETY OF misfortunes," used in describing the risks covered by p.19)
CANTON, LTD. policies of marine insurance, has been the subject of
40 PHIL 40 frequent discussion; and certain propositions relative COASTWISE LIGHTERAGE CORP v. CA
STREET; September 1, 1919 thereto are now so generally accepted as to be (PHILIPPINE GENERAL INSURANCE COMPANY)
considered definitely settled. 245 SCRA 796
INSURANCE Page 92

FRANCISCO; July 12, 1995 the same goods at the place of destination in bad
HELD order makes for a prima facie case against the
NATURE 1. NO carrier. The presumption of negligence that attaches
Petition for review of CA Decision affirming decision - The distinction between the two kinds of charter to common carriers, once the goods it transports are
of RTC Manila holding that Coastwise is liable to pay parties (i.e. bareboat or demise and contract of lost, destroyed or deteriorated, applies to Coastwise.
PhilGen Insurance the amount of P700thou plus legal affreightment) is more clearly set out in the case of This presumption, which is overcome only by proof of
interest thereon, another sum of P100thou as Puromines, Inc. vs. Court of Appeals, wherein SC the exercise of extraordinary diligence, remained
attorney's fees and the cost of the suit. ruled: unrebutted in this case.
“Under the demise or bareboat charter of the - The damage to the barge which carried the cargo of
FACTS vessel, the charterer will generally be regarded molasses was caused by its hitting an unknown
- Pag-asa Sales, Inc. entered into a contract to as the owner for the voyage or service sunken object as it was heading for Pier 18. The
transport molasses from the province of Negros to stipulated. The charterer mans the vessel with object turned out to be a submerged derelict vessel.
Manila with Coastwise, using the latter's dumb his own people and becomes the owner pro hac The evidence on record appeared that far from
barges. The barges were towed in tandem by the vice, subject to liability to others for damages having rendered service with the greatest skill and
tugboat MT Marica, also owned by Coastwise. Upon caused by negligence. To create a demise, the utmost foresight, and being free from fault, the
reaching Manila Bay, while approaching Pier 18, one owner of a vessel must completely and carrier was culpably remiss in the observance of its
of the barges struck an unknown sunken object. The exclusively relinquish possession, command and duties.
forward buoyancy compartment was damaged, and navigation thereof to the charterer, anything - Jesus R. Constantino, the patron of the vessel
water gushed in through a hole "two inches wide and short of such a complete transfer is a contract of "Coastwise 9" admitted that he was not licensed. The
twenty-two inches long." affreightment (time or voyage charter party) or Code of Commerce, which subsidiarily governs
- As a consequence, the molasses at the cargo tanks not a charter party at all. common carriers (which are primarily governed by
were contaminated and rendered unfit for the use it On the other hand a contract of affreightment is the provisions of the Civil Code) provides: “Art. 609.
was intended. This prompted consignee Pag-asa one in which the owner of the vessel leases part Captains, masters, or patrons of vessels must be
Sales to reject the shipment of molasses as a total or all of its space to haul goods for others. It is a Filipinos, have legal capacity to contract in
loss. Thereafter, Pag-asa Sales filed a formal claim contract for special service to be rendered by the accordance with this code, and prove the skill
with the insurer of its lost cargo (PhilGen) and owner of the vessel and under such contract the capacity and qualifications necessary to command
against the carrier (Coastwise). Coastwise denied the general owner retains the possession, command and direct the vessel, as established by marine and
claim and it was PhilGen which paid Pag-asa Sales and navigation of the ship, the charterer or navigation laws, ordinances or regulations, and must
the amount of P700k representing the value of the freighter merely having use of the space in the not be disqualified according to the same for the
damaged cargo of molasses. vessel in return for his payment of the charter discharge of the duties of the position.”
- PhilGen then filed an action against Coastwise hire... - Clearly, Coastwise Lighterage's embarking on a
before the RTC Manila, seeking to recover the P700k An owner who retains possession of the ship voyage with an unlicensed patron violates this rule.
which it paid to Pag-asa Sales for the latter's lost though the hold is the property of the charterer, It cannot safely claim to have exercised
cargo. PhilGen now claims to be subrogated to all the remains liable as carrier and must answer for extraordinary diligence, by placing a person whose
contractual rights and claims which the consignee any breach of duty as to the care, loading and navigational skills are questionable, at the helm of
may have against the carrier, which is presumed to unloading of the cargo.” the vessel which eventually met the fateful accident.
have violated the contract of carriage. - Although a charter party may transform a common It may also logically, follow that a person without
- RTC awarded the amount prayed for by PhilGen. CA carrier into a private one, the same however is not license to navigate, lacks not just the skill to do so,
affirmed. Hence, this petition. true in a contract of affreightment on account of the but also the utmost familiarity with the usual and
aforementioned distinctions between the two. safe routes taken by seasoned and legally authorized
ISSUES - SC agrees with Coastwise's admission that the ones. Had the patron been licensed, he could be
1. WON Coastwise Lighterage was transformed into a contract it entered into with the consignee was one presumed to have both the skill and the knowledge
private carrier, by virtue of the contract of of affreightment. Pag-asa Sales, Inc. only leased that would have prevented the vessel's hitting the
affreightment which it entered into with the three of petitioner's vessels, in order to carry cargo sunken derelict ship that lay on their way to Pier 18.
consignee, Pag-asa Sales, Inc. (Corollarily, if it were from one point to another, but the possession, - As a common carrier, Coastwise is liable for breach
in fact transformed into a private carrier, did it command and navigation of the vessels remained of the contract of carriage, having failed to overcome
exercise the ordinary diligence to which a private with Coastwise. As such, Coastwise, by the contract the presumption of negligence with the loss and
carrier is in turn bound?) of affreightment, was not converted into a private destruction of goods it transported, by proof of its
2. WON the insurer was subrogated into the rights of carrier, but remained a common carrier and was still exercise of extraordinary diligence.
the consignee against the carrier, upon payment by liable as such. 2. YES
the insurer of the value of the consignee's goods lost - Therefore, the mere proof of delivery of goods in - Coastwise is liable for breach of the contract of
while on board one of the carrier's vessels good order to a carrier and the subsequent arrival of carriage it entered into with Pag-asa Sales, Inc.
INSURANCE Page 93

However, for the damage sustained by the loss of the Policy. softdrink bottles were improperly stowed on deck.
cargo which petitioner-carrier was transporting, it - The vessel sank in the waters of Zamboanga del Nonetheless, the appellate court denied the claim of
was not the carrier which paid the value thereof to Norte bringing down her entire cargo with her PHILAMGEN on the ground that the assured’s implied
Pag-asa Sales, Inc. but the latter's insurer, herein including the subject 7,500 cases of 1-liter Coca-Cola warranty of seaworthiness was not complied with.
private respondent PhilGen. softdrink bottles. Perfunctorily, PHILAMGEN was not properly
- Article 2207 of the Civil Code: If the plaintiffs - The consignee filed a claim with respondent subrogated to the rights and interests of the shipper.
property has been insured, and he has received FELMAN for recovery of damages it sustained as a Furthermore, respondent court held that the filing of
indemnity from the insurance company for the injury result of the loss of its softdrink bottles that sank notice of abandonment had absolved the
or loss arising out of the wrong or breach of contract with “MV Asilda.” Respondent denied the claim shipowner/agent from liability under the limited
complained of, the insurance company shall be thus prompting the consignee to file an insurance liability rule.
subrogated to the rights of the insured against the claim with PHILAMGEN which paid its claim of
wrongdoer or the person who violated the contract.” P755,250.00. ISSUES
- This legal provision is founded on the well-settled - Claiming its right of subrogation PHILAMGEN 1. WON “MV Asilda” was seaworthy when it left the
principle of subrogation. If the insured property is sought recourse against respondent FELMAN which port of Zamboanga
destroyed or damaged through the fault or disclaimed any liability for the loss. Consequently, 2. WON the limited liability under Art. 587 of the
negligence of a party other than the assured, then PHILAMGEN sued the shipowner for sum of money Code of Commerce should apply
the insurer, upon payment to the assured will be and damages. 3. WON PHILAMGEN was properly subrogated to the
subrogated to the rights of the assured to recover - PHILAMGEN alleged that the sinking and total loss rights and legal actions which the shipper had
from the wrongdoer to the extent that the insurer of “MV Asilda” and its cargo were due to the vessel’s against FELMAN, the shipowner
has been obligated to pay. Payment by the insurer to unseaworthiness as she was put to sea in an
the assured operated as an equitable assignment to unstable condition. It further alleged that the HELD
the former of all remedies which the latter may have vessel was improperly manned and that its 1. YES
against the third party whose negligence or wrongful officers were grossly negligent in failing to take - “MV Asilda” was unseaworthy when it left the port
act caused the loss. The right of subrogation is not appropriate measures to proceed to a nearby port or of Zamboanga. We subscribe to the findings of the
dependent upon, nor does it grow out of, any privity beach after the vessel started to list. Elite Adjusters, Inc., and the Court of Appeals that
of contract or upon written assignment of claim. It - FELMAN filed a motion to dismiss based on the the proximate cause of the sinking of “MV Asilda”
accrues simply upon payment of the insurance claim affirmative defense that no right of subrogation in was its being top-heavy. Contrary to the ship
by the insurer. favor of PHILAMGEN was transmitted by the shipper, captain’s allegations, evidence shows that
- Undoubtedly, upon payment by respondent insurer and that, in any event, FELMAN had abandoned all approximately 2,500 cases of softdrink bottles were
PhilGen of the amount of P700,000.00 to Pag-asa its rights, interests and ownership over “MV Asilda” stowed on deck. Several days after “MV Asilda”
Sales, Inc., the consignee of the cargo of molasses together with her freight and appurtenances for the sank, an estimated 2,500 empty Coca-Cola plastic
totally damaged while being transported by purpose of limiting and extinguishing its liability cases were recovered near the vicinity of the sinking.
petitioner Coastwise Lighterage, the former was under Art. 587 of the Code of Commerce. Considering that the ship’s hatches were properly
subrogated into all the rights which Pag-asa Sales, - Trial court dismissed the complaint of PHILAMGEN. secured, the empty Coca-Cola cases recovered could
Inc. may have had against the carrier, herein On appeal the Court of Appeals set aside the have come only from the vessel’s deck cargo. It is
petitioner Coastwise Lighterage. dismissal and remanded the case to the lower court settled that carrying a deck cargo raises the
Disposition Petition denied. CA affrimed. for trial on the merits. FELMAN filed a petition for presumption of unseaworthiness unless it can be
certiorari with this Court but it was subsequently shown that the deck cargo will not interfere with the
THE PHILIPPINE AMERICAN GENERAL denied on 13 February 1989. proper management of the ship. However, in this
INSURANCE COMPANY INC v. CA (FELMAN - Trial court rendered judgment in favor of FELMAN. case it was established that “MV Asilda” was not
SHIPPING LINES) It ruled that “MV Asilda” was seaworthy when it left designed to carry substantial amount of cargo on
273 SCRA 226 the port of Zamboanga as confirmed by certificates deck. The inordinate loading of cargo deck resulted
BELLOSILLO; June 11, 1997 issued by the Philippine Coast Guard and the in the decrease of the vessel’s metacentric height
shipowner’s surveyor attesting to its seaworthiness. thus making it unstable. The strong winds and
FACTS Thus the loss of the vessel and its entire shipment waves encountered by the vessel are but the
- Coca-Cola Bottlers Philippines, Inc., loaded on could only be attributed to either a fortuitous event, ordinary vicissitudes of a sea voyage and as such
board “MV Asilda,” a vessel owned and operated by in which case, no liability should attach unless there merely contributed to its already unstable and
Felman 7,500 cases of 1-liter Coca-Cola softdrink was a stipulation to the contrary, or to the negligence unseaworthy condition.
bottles to be transported from Zamboanga City to of the captain and his crew, in which case, Art. 587 2. NO
Cebu for consignee Coca-Cola Bottlers Philippines, of the Code of Commerce should apply. - The ship agent is liable for the negligent acts of the
Inc., Cebu. The shipment was insured with petitioner - CA ruled that “MV Asilda” was unseaworthy for captain in the care of goods loaded on the vessel.
Philippine American General under Marine Open being top- heavy as 2,500 cases of Coca-Cola This liability however can be limited through
INSURANCE Page 94

abandonment of the vessel, its equipment and the insurance company for the injury or loss - The ship was sunk in July 1, 1918. After several
freightage as provided in Art. 587. Nonetheless, arising out of the wrong or breach of contract futile attempts, it was finally raised on Sept. 20,
there are exceptional circumstances wherein the complained of, the insurance company shall 1918. It is faitr to assume that in its then condition
ship agent could still be held answerable despite the be subrogated to the rights of the insured much further time would be required to make the
abandonment, as where the loss or injury was due to against the wrongdoer or the person who has necessary repairs and install the new machinery
the fault of the shipowner and the captain. The violated the contract. If the amount paid by before it could again be placed in commission.
international rule is to the effect that the right of the insurance company does not fully cover During that time the owner would be deprived of the
abandonment of vessels, as a legal limitation of a the injury or loss, the aggrieved party shall use of its vessel or the interest on its investment.
shipowner’s liability, does not apply to cases where be entitled to recover the deficiency from the When those questions are considered the testimony
the injury or average was occasioned by the person causing the loss or injury. is conclusive that the cost of salvage, repair and
shipowner’s own fault. Disposition Petition is GRANTED. Respondent reconstruction was more than the original cost of the
3. YES FELMAN SHIPPING LINES is ordered to pay petitioner ship at the time the policy was issued. As found by
- The doctrine of subrogation has its roots in equity. PHILIPPINE AMERICAN GENERAL INSURANCE CO., the trial court, “t is difficult to see how there could
It is designed to promote and to accomplish justice INC., Seven Hundred Fifty-five Thousand Two have been a more complete loss of the vessel than
and is the mode which equity adopts to compel the Hundred and Fifty Pesos (P755,250.00) plus legal that which actually occurred”. Upon the facts shown
ultimate payment of a debt by one who in justice, interest thereon counted from 29 November 1983, here, any other construction would nullify the statute
equity and good conscience ought to pay. Therefore, the date of judicial demand, pursuant to Arts. 2212 and as applied to the conditions existing in the
the payment made by PHILAMGEN to Coca-Cola and 2213 of the Civil Code. Manila Bay, this kind of policy would be worthless,
Bottlers Philippines, Inc., gave the former the right to and there would not be any consideration for the
bring an action as subrogee against FELMAN. Having PHILIPPINE MFTG. CO. v. UNION INSURANCE premium.
failed to rebut the presumption of fault, the liability SOCIETY OF CANTON 2. NO
of FELMAN for the loss of the 7,500 cases of 1-liter 42 PHIL 378 - The defendant argues that the policy contains the
Coca-Cola softdrink bottles is inevitable. JOHNS; November 22, 1921 provision that it “shall be of as force and effect as
- Sec. 113 of the Insurance Code provides that “(i)n the surest writing or policy of insurance made in
every marine insurance upon a ship or freight, or FACTS London”. However, for such law to apply to our courts
freightage, or upon anything which is the subject of - The plaintiff’s steel tank lighter was insured by the existence of such law must be proven. It cannot
marine insurance, a warranty is implied that the ship defendant company for absolute total loss. As a apply when such proof is lacking. Nevertheless, in
is seaworthy.” Under Sec. 114, a ship is “seaworthy result of a typhoon, the lighter sunk in Manila Bay. the English practice, a ship is a total loss when she
when reasonably fit to perform the service, and to The plaintiff demanded payment from the defendant has sustained such extensive damages that it would
encounter the ordinary perils of the voyage, insurance company but the latter refused. The not be reasonably practical to repair her.
contemplated by the parties to the policy.” Thus it company asked the plaintiff to salvage the ship, Disposition Decision reversed
becomes the obligation of the cargo owner to look for which it was able to do so.
a reliable common carrier which keeps its vessels in - With the plaintiff able to raise the lighter,
seaworthy condition. He may have no control over reconstruct it and placed it in commission, the
the vessel but he has full control in the selection of defendant insurance company claims that it was only
the common carrier that will transport his goods. He liable for a total absolute loss and that there was no
also has full discretion in the choice of assurer that total destruction of the lighter.
will underwrite a particular venture. - The trial court decided in favor of the defendant,
- In policies where the law will generally imply a saying that the policy only covered an actual total
warranty of seaworthiness, it can only be excluded loss, not a constructive total loss.
by terms in writing in the policy in the clearest
language. And where the policy stipulates that the ISSUES CHOA TIEK SENG v. CA (FILIPINO MERCHANTS
seaworthiness of the vessel as between the assured 1. WON there was an absolute total loss that can be INSURANCE)
and the assurer is admitted, the question of covered by the policy 183 SCRA 223
seaworthiness cannot be raised by the assurer 2. WON the Marine Law of Great Britain applies GANCAYO; March 15, 1990
without showing concealment or misrepresentation
by the assured. HELD NATURE
- PHILAMGEN’s action against FELMAN is squarely 1. YES Appeal from a decision of the Court of Appeals
sanctioned by Art. 2207 of the Civil Code which - At the time that the lighter was at the bottom of
provides: the bay, it was of no value to the owner, thus there FACTS
Art. 2207. If the plaintiff’s property has been was an actual total loss. - Petitioner imported some lactose crystals from
insured, and he has received indemnity from Holland.
INSURANCE Page 95

- The importation involved fifteen (15) metric tons HELD


packed in 600 6-ply paper bags with polythelene YES ABOITIZ SHIPPING v. PHILAMGEN INSURANCE
inner bags, each bag at 25 kilos net. The goods were - In Gloren Inc. vs. Filipinas Cia. de Seguros, 12 it 179 SCRA 357
loaded at the port at Rotterdam in sea vans on board was held that an all risk insurance policy insures GANCAYCO; October 5, 1989
the vessel "MS Benalder' as the mother vessel, and against all causes of conceivable loss or damage,
thereafter aboard the feeder vessel "Wesser Broker except as otherwise excluded in the policy or due to NATURE
V-25" of respondent Ben Lines Container, Ltd. (Ben fraud or intentional misconduct on the part of the Petition for review on certiorari
Lines for short). The goods were insured by the insured. It covers all losses during the voyage
respondent Filipino Merchants' Insurance Co., Inc. whether arising from a marine peril or not, including FACTS
(insurance company for short) for the sum of pilferage losses during the war. - Marinduque Mining Industrial Corporation
P98,882.35, the equivalent of US$8,765.00 plus 50% - In the present case, the "all risks" clause of the (Marinduque) shipped on board SS Arthur Maersk
mark-up or US $13,147.50, against all risks under policy sued upon reads as follows: from Boston, U.S.A. a shipment of 1 skid carton
the terms of the insurance cargo policy. Upon arrival "5. This insurance is against all risks of loss or parts for valves. The shipment was ordered from
at the port of Manila, the cargo was discharged into damage to the subject matter insured but shall in Jamesbury, Singapore PTE, LTD., which issued the
the custody of the arrastre operator respondent E. no case be deemed to extend to cover loss, cargo's packing list and Invoice number showing the
Razon, Inc. (broker for short), prior to the delivery to damage, or expense proximately caused by delay contents of the carton. The Philippine Consulate in
petitioner through his broker. Of the 600 bags or inherent vice or nature of the subject matter Singapore issued invoice for the shipment showing
delivered to petitioner, 403 were in bad order. The insured. Claims recoverable hereunder shall be the contents and its total price of $39,419.60 and
surveys showed that the bad order bags suffered payable irrespective of percentage." the freight and other charges of $2,791.73. When
spillage and loss later valued at P33,117.63. - The terms of the policy are so clear and require no the cargo arrived in Manila, it was received and
Petitioner filed a claim for said loss dated February interpretation. The insurance policy covers all loss or deposited in the office of Aboitiz Shipping
16, 1977 against respondent insurance company in damage to the cargo except those caused by delay Corporation (Aboitiz) for transhipment to Nonoc
the amount of P33,117.63 as the insured value of or inherent vice or nature of the cargo insured. It is Island.
the loss. the duty of the respondent insurance company to - In July 1980, Marinduque, as consignee of the
- Respondent insurance company rejected the claim establish that said loss or damage falls within the cargo, made a report that said cargo was pilfered on
alleging that assuming that spillage took place while exceptions provided for by law, otherwise it is liable July 3, 1980 due to heavy rain at the Aboitiz
the goods were in transit, petitioner and his agent therefor. terminal and that of the total value of the cargo of
failed to avert or minimize the loss by failing to - An "all risks" provision of a marine policy creates a $42,209.33, only $7,412.00 worth remains of the
recover spillage from the sea van, thus violating the special type of insurance which extends coverage to cargo with the recommendation that the claim be
terms of the insurance policy sued upon; and that risks not usually contemplated and avoids putting made against Aboitiz.
assuming that the spillage did not occur while the upon the insured the burden of establishing that the - The services of the Manila Adjusters and Surveyors
cargo was in transit, the said 400 bags were loaded loss was due to peril falling within the policy's Co. (Manila Adjusters) were engaged by the Phil-
in bad order, and that in any case, the van did not coverage. The insurer can avoid coverage upon American General Insurance Co., Inc. (Phil Am)
carry any evidence of spillage. demonstrating that a specific provision expressly which came out with the report that the cargo in
- Petitioner filed a complaint in the RTC against the excludes the loss from coverage. question, when inspected, showed that it was
insurance company seeking payment of the sum of - In this case, the damage caused to the cargo has pilfered. A confirmatory report was submitted by the
P33,117.63 as damages plus attorney's fees and not been attributed to any of the exceptions provided Manila Adjusters.
expenses of litigation. Insurance company denied all for nor is there any pretension to this effect. Thus, - On August 11, 1980 Marinduque then filed a claim
the material allegations of the complaint and raised the liability of respondent insurance company is against Aboitiz in the amount of P246,430.80
several special defenses as well as a compulsory clear. representing the value of the pilfered cargo. On the
counterclaim. Insurance company filed a third-party Disposition the decision appealed from is hereby same day Marinduque filed a claim for the same
complaint against respondents Ben Lines and broker. REVERSED AND SET ASIDE and another judgment is amount against the Phil-Am on the latter's policy.
- RTC dismissed the complaint, the counterclaim and hereby rendered ordering the respondent Filipinas Phil-Am paid Marinduque the sum of P246,430.80 as
the third-party complaint with costs against the Merchants Insurance Company, Inc. to pay the sum insurer of the cargo.
petitioner. Appealed in CA but denied. MFR was of P33,117.63 as damages to petitioner with legal - Phil-Am then filed a complaint in RTC Manila
denied as well. interest from the filing of the complaint, plus against Aboitiz for recovery of same amount alleging
attorney's fees and expenses of litigation in the that it has been subrogated to the rights of
ISSUE amount of P10,000.00 as well as the costs of the Marinduque. Complaint dismissed and MFR denied.
WON insurance company should be held liable even if suit. CA reversed. MFR thereof was denied. Hence, this
the technical meaning in marine insurance of an petition.
“insurance against all risk" is applied FILIPINO MERCHANTS INS. CO. v. CA (supra
p.19) ISSUE
INSURANCE Page 96

WON petitioner Aboitiz was properly held liable to the Petition for review on certiorari consideration. The insurance contract must,
private respondent Phil-Am by the appellate court therefore, be considered indivisible.
FACTS - More importantly, the insurer's liability was for
HELD - Sometime in January 1986, private respondent "total loss only." A total loss may be either actual or
YES Panama Sawmill Co., Inc. (Panama) bought, in constructive (Sec. 129, Insurance Code). An actual
- The questioned shipment is covered by a continuing Palawan, 1,208 pieces of apitong logs, with a total total loss is caused by:
open insurance coverage (which took effect after volume of 2,000 cubic meters. It hired Transpacific (a) A total destruction of the thing insured;
Sept. 1, 1975, as contained in Marine Open Policy Towage, Inc., to transport the logs by sea to Manila (b) The irretrievable loss of the thing by sinking, or
No. 100184) from the time it was loaded aboard the and insured it against loss for P1-M with petitioner by being broken up;
SS Arthur Maersk in Boston, U.S.A. to the time it was Oriental Assurance Corporation (Oriental Assurance). (c) Any damage to the thing which renders it
delivered to the possession of petitioner at its offices - While the logs were being transported, rough seas valueless to the owner for the purpose for which
at Pier 4 in Manila until it was pilfered when the great and strong winds caused damage to one of the two he held it; or
majority of the cargo was lost on July 3, 1980. barges resulting in the loss of 497 pieces of logs out (d) Any other event which effectively deprives the
Hence, petitioner Aboitiz was properly held liable to of the 598 pieces loaded thereon. owner of the possession, at the port of destination,
Phil-Am. - Panama demanded payment for the loss but of the thing insured. (Section 130, Insurance
Reasoning Oriental Assurance refuse on the ground that its Code).
[a] Records of the case show that Phil-Am executed contracted liability was for "TOTAL LOSS ONLY." - A constructive total loss is one which gives to a
a continuous and open insurance coverage covering - Unable to convince Oriental Assurance to pay its person insured a right to abandon, under Section
goods of Marinduque imported into and exported claim, Panama filed a Complaint for Damages against 139 of the Insurance Code. This provision reads:
from the Philippines which took effect after Sept. 1, Oriental Assurance before the Regional Trial Court. SECTION 139. A person insured by a contract of
1975, as contained in Marine Open Policy No. - RTC ordered Oriental Assurance to pay Panama marine insurance may abandon the thing insured,
100184. A similar insurance coverage was also with the view that the insurance contract should be or any particular portion thereof separately valued
executed by petitioner in favor of Marinduque for all liberally construed in order to avoid a denial of by the policy, or otherwise separately insured, and
its goods shipped or moved within the territorial substantial justice; and that the logs loaded in the recover for a total loss thereof, when the cause of
limits of the Philippines also effective after Sept. 1, two barges should be treated separately such that the loss is a peril injured against,
1975 and contained in Marine Open Policy No. the loss sustained by the shipment in one of them (a) If more than three-fourths thereof in value is
100185. may be considered as "constructive total loss" and actually lost, or would have to be expended to
[b] TC in dismissing the complaint apparently relied correspondingly compensable. CA affirmed in toto. recover it from the peril;
on Marine Risk Note No. 017545 issued by private (b) If it is injured to such an extent as to reduce
respondent Phil-Am only on July 28, 1980 after the ISSUE its value more than three-fourths;
shipment in question was already pilfered. Obviously WON Oriental Assurance can be held liable under its xxx xxx xxx
TC mistook said Marine Risk Note as an insurance marine insurance policy based on the theory of a - The requirements for the application of Section 139
policy when it is NOT. It is only an acknowledgment divisible contract of insurance and, consequently, a of the Insurance Code, quoted above, have not been
or declaration of the private respondent confirming constructive total loss met. The logs involved, although placed in two
the specific shipment covered by its Marine Open barges, were not separately valued by the policy, nor
Policy, the evaluation of the cargo and the HELD separately insured. Resultantly, the logs lost in the
chargeable premium. NO damaged barge in relation to the total number of
[c] The contention of the Aboitiz that it could not be - The terms of the contract constitute the measure of logs loaded on the same barge cannot be made the
liable for the pilferage of the cargo as it was stolen the insurer liability and compliance therewith is a basis for determining constructive total loss. The logs
even before it was loaded on its vessel is untenable. condition precedent to the insured's right to recovery having been insured as one inseparable unit, the
Aboitiz received cargo when it arrived in Manila at its from the insurer. Whether a contract is entire or correct basis for determining the existence of
offices, and it was while in its possession and before severable is a question of intention to be determined constructive total loss is the totality of the shipment
loading it in its vessel that the cargo was pilfered. Its by the language employed by the parties. The policy of logs. Of the entirety of 1,208, pieces of logs, only
liability is clear. in question shows that the subject matter insured 497 pieces thereof were lost or 41.45% of the entire
Disposition Petition DISMISSED. was the entire shipment of 2,000 cubic meters of shipment. Since the cost of those 497 pieces does
apitong logs. The fact that the logs were loaded on not exceed 75% of the value of all 1,208 pieces of
ORIENTAL ASSURANCE v. CA (PANAMA SAW two different barges did not make the contract logs, the shipment cannot be said to have sustained
MILL) several and divisible as to the items insured. The a constructive total loss under Section 139(a) of the
200 SCRA 459 logs on the two barges were not separately valued or Insurance Code.
MELENCIO-HERRERA; August 9, 1991 separately insured. Only one premium was paid for Disposition judgment under review is SET ASIDE
the entire shipment, making for only one cause or
NATURE
INSURANCE Page 97

PAN MALAYAN INSURANCE v. CA (THE FOOD ISSUE Ed. 216), as where the cargo by the process of
AND AGRICULTURAL ORGANIZATION OF THE 1. WON respondent court committed a reversible decomposition or other chemical agency no longer
UNITED NATIONS) error in holding that the trial court is correct in remains the same kind of thing as before (Williams
201 SCRA 382 holding that there is a total loss of the shipment vs. Cole, 16 Me. 207).
REGALADO; September 5, 1991 - It is thus clear that FAO suffered actual total loss
HELD under Section 130 of the Insurance Code, specifically
FACTS 1. NO under paragraphs (c) and (d) thereof, recompense
- The Food and Agricultural Organization of the - The law classifies loss into either total or partial. for which it has been denied up to the present
United Nations (hereinafter referred to as FAO), Total loss may be actual or absolute, or it may -Section 135 of the Insurance Code explicitly
ntended and made arrangements to send to otherwise be constructive or technical. Petitioner provides that "(u)pon an actual total loss, a person
Kampuchea 1,500 metric petitions of IR-36 certified submits that respondent court erred in ruling that insured is entitled to payment without notice of
rice seeds to be distributed to the people for seedling there was total loss of the shipment despite the fact abandonment." This is a statutory adoption of a long
purposes that only 27,922 bags of rice seeds out of 34,122 standing doctrine in maritime insurance law that in
- LUZTEVECO was to ship the cargo amounting to bags were rendered valueless to FAO and the case of actual total loss, the right of the insured to
US$83,325.92 in respect of one lot of 1,500 metric shipment sustained only a loss of 78%. - FAO, claim the whole insurance is absolute, without need
petitions winch is the subject of the present action. however, claims that, for all intents and purposes, it of a notice of abandonment
The cargo was loaded on board LUZTEVECO Barge has practically lost its total or entire shipment in this
No. LC-3000 and consisted of 34,122 bags of IR-36 case, inclusive of expenses, premium fees, and so PHILIPPINE AMERICAN LIFE INSURANCE
certified rice seeds purchased by FAO from the forth, despite the alleged recovery by defendant COMPANY v. CA (ELIZA PULIDO)
Bureau of Plant Industry for P4,602,270.00 LUZTEVECO. As found by the court below and 344 SCRA 260
- FAO secured insurance coverage in the amount of reproduced with approval by respondent court, FAO GONZAGA-REYES; November 15, 2000
P5,250,000.00 from petitioner, Pan Malayan "has never been compensated for this total loss or
Insurance Corporation damage, a fact which is not denied nor controverted NATURE
- On June 16, 1980, FAO gave instructions to - If there were some cargoes saved, by LUZTEVECO, This petition for review on certiorari seeks to reverse
LUZTEVECO to leave for Vaung Tau, Vietnam to private respondent abandoned it and the same was the Decision of the Special Second Division of the
deliver the cargo which, by its nature, could not sold or used for the benefit of LUZTEVECO or Pan Court of Appeals
withstand delay because of the inherent risks of Malayan Corporation. Under Sections 129 and 130 of
termination and/or spoilage. On the same date, the the New Insurance Code, a total loss may either be FACTS
insurance premiums on the shipment was paid by actual or constructive. In case of total loss in Marine - On January 9, 1989, petitioner received from one
FAO petitioner Insurance, the assured is entitled to recover from Florence Pulido an application for life insurance,
- On June 26, 1980, FAO was advised of the sinking the underwriter the whole amount of his subscription dated December 16, 1988, in the amount of
of the barge in the China Sea, hence it informed - SEC. 130. An actual total loss is caused by: (c) Any P100,000.00 which designated her sister, herein
petitioner thereof and, later, formally filed its claim damage to the thing which renders it valueless to the private respondent, as its principal beneficiary.
under the marine insurance policy. On July 29, 1980, owner for the purpose for which he held it; or Because the insurance applied for was non-medical,
FAO was informed by LUSTEVECO of the recovery of (d) Any other event which effectively deprives the petitioner did not require a medical examination and
the lost shipment, for which reason FAO formally owner of the possession, at the port of destination of issued a policy on the sole basis of the application on
filed its claim with LUZTEVECO for compensation of the thing insured. February 11, 1989. On April 1992, petitioner
damage to its cargo -as said and proven, the seeds were of fragile received private respondent’s claim, which declared
- LUZTEVECO failed and refused to pay. Pan Malayan nature. And the wetting of said seeds affected the that the insured, Florence Pulido, died of acute
likewise failed to pay for the losses and damages state of seeds. Thus rendering them useless for FAO. pneumonia on September 10, 1991.
sustained by FAO by reason of its inability to recover Although there were bags which were recovered, - Petitioner withheld payment on the ground that the
the value of the shipment from LUZTEVECO these were “stained” and not in the same condition it policy claimed under was void from the start for
- Pan Malayan claims that part of the cargo was was brought in. in addition to this, FAO did not having been procured in fraud. It is petitioner’s
recovered and thus the claim by FAO was receive any compensation for said recovered bags as contention that even before they received private
unwarranted. This is evidenced by two surveys upon the same were distributed by LUZVETECO without respondent’s claim for death benefits, their
the cargo wherein it was found that only around 78% authorization of FAO investigation concerning the subject policy yielded
was lost. - the complete physical destruction of the subject the information that the insured, Florence Pulido,
- FAO filed a civil case against both LUZTEVECO and matter is not essential to constitute an actual total died in 1988, before the application for insurance on
Pan Malayan. Trial court found in favor of FAO and loss. Such a loss may exist where the form and her life was made. While this was communicated to
ordered both to pay jointly and severally the full specie of the thing is destroyed, although the private respondent in a letter dated April 29, 1992,
amount of the claim. This was affirmed by CA materials of which it consisted still exist (Great private respondent had already filed her claim earlier
Western Ins. Co. vs. Fogarty, N.Y., 19 Wall 640, 22 L. that month. In another letter dated July 27, 1992,
INSURANCE Page 98

however, petitioner confirmed to private respondent - Death certificates, and notes by a municipal health - On April 14, 1943, the National Life Insurance
receipt of the claim papers and assured her that her officer prepared in the regular performance of his Company of the Philippines issued a policy on the life
case was “being given preferential attention and duties, are prima facie evidence of facts therein of Jose C. Londres whereby it undertook to pay its
prompt action”. stated. A duly-registered death certificate is beneficiary upon his death the sum of P3,000. All the
- Following the filing by private respondent of her considered a public document and the entries found premiums due under the policy were actually paid on
claim, petitioner caused another investigation therein are presumed correct, unless the party who their dates of maturity and the policy was in force
respecting the subject policy. Pursuant to the contests its accuracy can produce positive evidence when the insured died on February 7, 1945.
findings of this second investigation, petitioner stood establishing otherwise. Petitioner’s contention that Salvacion V. Londres, as beneficiary, demanded from
by its initial decision to treat the policy as void and the death certificate is suspect because Dr. Gutierrez the company the payment of the proceeds of the
not to honor the claim. On November 9, 1992, was not present when Florence Pulido died, and policy, and her demand having been refused, she
private respondent enlisted the services of counsel in knew of Florence’s death only through Ramon instituted the present action against the company in
reiterating her claim for death benefits. Petitioner Piganto, does not merit a conclusion of fraud. No the Court of First Instance of Manila.
still refused to make payment and thus, this action. motive was imputed to Dr. Gutierrez for seeking to - Defendant in its answer denied, for lack of
- Petitioner: the results of its investigations having perpetuate a falsity in public records. Petitioner was sufficient proof, the allegation that the insured died
indicated that the insured was already dead at the likewise unable to make out any clear motive as to on February 7, 1945, and set up the following special
time the policy was applied for. It also why Ramon Piganto would purposely lie. Mere defenses: (a) that plaintiff's claim is covered by the
counterclaimed for attorney’s fees. The first report, allegations of fraud could not substitute for the full Moratorium Law; (b) that the policy having been
prepared by one Dr. Benedicto Briones, was dated and convincing evidence that is required to prove it. issued during the Japanese occupation, it is
April 1, 1992, and had attached to it a questionnaire, A failure to do so would leave intact the presumption presumed that its face value should be paid in
responded to by one Ramon Piganto, who of good faith and regularity in the performance of Japanese currency, there being no provision in the
represented to be the brother-in-law of the insured public duties, which was the basis of both respondent policy from which can be inferred that the parties
and the barangay chairman of Cardiz, Bagulin, La court and the trial court in finding the date of contemplated payment in any other currency; (c)
Union. To the question “Where does [Florence Florence Pulido’s death to be as plaintiff-private that the money paid by the insured as premiums,
Pulido] reside now?”, Piganto had replied that respondent maintained. together with the money received from other policy-
Florence Pulido used to live in Cardiz, but was dead - We cannot likewise give credence to petitioner’s holders, was all deposited by the defendant in the
since 1988. Piganto’s statement was signed by him, submission that the inconsistencies in the Philippine National Bank and said deposit was
and witnessed by his wife, Nenita Piganto. This testimonies of the witnesses for plaintiff-private declared without value by Executive Order No. 49 of
report was petitioner’s basis for treating the disputed respondent are in themselves evidence of fraud. the President of the Philippines; and (d) that the
policy as void since April 1992, even before receipt of Such alleged inconsistencies are matters of credibility policy having been issued under abnormal
private respondent’s claim. which had been ably passed upon by the lower court. circumstances, it should be considered in the light of
- The absence of fraud, as a factual finding of the equity which does not permit anyone to enrich
ISSUE lower court adopted by the Court of Appeals, entirely himself at the expense of another. Defendant,
WON there was fraud (whether the insured, Florence consistent with the evidence on record, will not be however, as a proof of good faith, offered to pay the
Pulido, was in fact dead before the application for reversed and, hence, is final and conclusive upon this value of the policy in accordance with the Ballantyne
insurance on her life was made) Court. scale of values, or the sum of P2,400, Philippine
Disposition The instant petition is DENIED currency.
HELD - It appears that the deceased took up the policy
NO CHAPTER IX. CLAIMS SETTLEMENT & under consideration on April 15, 1943 for the sum of
- This the lower courts had effected ruled on, upon a SUBROGATION P3,000. All the premiums due under the policy were
preponderance of the evidence duly received from actually paid on their dates of maturity and the
both parties. We see no reversible error in the LONDRES v. NATIONAL LIFE INSURANCE policy was in force when the insured died on
finding of both respondent court and the trial court in 94 PHIL 627 February 7, 1945. On said date, the battle of the
favor of the correctness of the entries in Certificate BAUTISTA ANGELO; March 29, 1954 liberation of the City of Manila was still raging. While
of Death, duly registered with the Local Civil the northern part may have been liberated, not so
Registrar of Bagulin, La Union, which declared that NATURE the southern part, as shown from the very affidavits
Florence Pulido died of acute pneumonia on Appeal from a decision of the Court of First Instance submitted by appellee wherein it was stated that on
September 10, 1991. Dr. Irineo Gutierrez, the of Manila ordering defendant to pay to plaintiff the the aforesaid date, the insured, Jose Londres, and
Municipal Health Officer of Bagulin, La Union whose sum of P3,000, Philippine currency, plus legal his two sons were taken by the Japanese soldiers
signature appeared in the death certificate, testified interest thereon from the time of the filing of the from their house at Singalong Street and were
in addition that he ministered to the ailing Florence complaint until its full payment. massacred by their captors. It may therefore be said
Pulido for two days immediately prior to her death. that the policy became due when the City of Manila
This fact is likewise noted in the death certificate. FACTS was still under the yoke of the enemy and became
INSURANCE Page 99

payable only after liberation which took place on Commerce which provides: "When the deposits are matured upon the insured’s death in November,
March 10, 1945 when President Osmena issued of cash, with a specification of the coins constituting 1944, they should compute the value of their claim
Proclamation No. 6 following the restoration of the them, . . . the increase or reduction which their value under the Ballantyne scale of values (which would
civil government by General Douglas Mac Arthur. And may suffer shall be for the account of the depositor." amount only to P500)
we say that the policy became payable only after Appellant, by entering into an insurance - beneficiaries commenced suit, and the lower court
liberation even if it matured sometime before, contract, cannot claim, if it suffers loss, that sustained the stand of the company, dismissed the
because before that eventuality the insurance the beneficiary cannot enrich herself at its complaint.
company, appellant herein, was not yet in a position expense. This is a risk attendant to any wagering - beneficiaries maintain that the obligation of the
to pay the value of the policy for the simple reason contract. One who gambles and loses cannot be company to pay accrued not upon the death of
that it had not yet reopened. heard to complain of his loss. To appellant, we can Fernandez, but only upon the receipt and approval
only repeat the following admonition: by the company, on proof of death of the insured,
ISSUE "The parties herein gambled and speculated on the which was in 1954. The policy reads:
WON the amount of P3,000 which appellant bound date of the termination of the war and the National Life Insurance Company of the Philippine
itself to pay to the insured under the policy upon his liberation of the Philippines by the Americans. This hereby agrees to pay at its Home Office, Manila,
death should be paid in accordance with the present can be gleaned from the stipulation about Ten Thousand Pesos to Juan D. Fernandez
currency or should be adjusted under the Ballantyne redemption, particularly that portion to the effect (hereinafter called the insured) on the 15th day of
scale of values that redemption could be effected not before the July, 1964, if the Insured is living and this Policy is
expiration of one year from June 24, 1944. This in force, or upon receipt and approved at its Office
HELD kind of agreement is permitted by law. We find of due proofs of the title of the claimant and of the
YES, present currency. nothing immoral or unlawful in it." (Gomez vs. prior death of the Insured while this Policy is in
Reasoning Tabia) force to Teresa Duat Vda. De Fernandez, Maria T.
- In the case of Rutter vs. Esteban, 93 Phil., 68, the Disposition Wherefore, the decision appealed from and Manuela Fernandez, mother and sisters
Moratorium Law was declared invalid and is affirmed, with costs against appellant. respectively of the Insured (Hereinafter called the
unconstitutional. Beneficiary) subject to the right of the Insured to
- During those days of liberation, while the people change the beneficiary as stated on the second
were rejoicing because of the happy event, the page of this Policy.
banks, the insurance companies, and for that matter - The above stipulation is apparently based on Sec.
other commercial and business firms, were still 91-A of the Insurance Law which provides as follows:
feeling the adverse effects of the sudden fall of VDA. DE FERNANDEZ v. NATIONAL LIFE The proceeds of a life insurance policy shall be paid
values and were uncertain and apprehensive as to INSURANCE CO OF THE PHILS immediately upon maturity of the policy, unless such
the manner the readjustment would be made by the 105 PHIL 59 proceeds are made payable in installments or a as an
new Government. It is for this reason that the ENDENCIA; January 27, 1959 annuity, in which case the installments or annuities
beneficiary, after realizing the truth about the death shall be paid as they become due: Provided,
of her husband, and after gathering evidence to NATURE however, That in case of a policy maturing by the
substantiate his death, had difficulty in effecting the Appeal from CFI decision applying the Ballantyne death of the insured, the proceeds thereof shall be
collection of her claim from the insurance company scale of values upon the proceeds of life insurance paid within sixty days after presentation of the claim
because at that time it had not yet reopened for taken and maturing during the Japanese occupation and filing of the proof of the death of the insured.
business purposes. Although the record does not but claimed after liberation Refused to pay the claim within the time prescribed
disclose the exact date on which the insurance herein will entitle the beneficiary to collect interest
company reopened for this purpose, this Court can FACTS on the proceeds of the policy for the duration of the
take judicial notice that it only did so after liberation. - National Life Insurance Company (NLIC) insured J. delay at the rate of six per centum per annum,
At that time the legal tender was already the Fernandez’s life for P10,000 upon his payment of unless such failure or refusal to pay is based on the
present currency. P444 from July 15, 1944 to July 14, 1945 ground that the claim is fraudulent . . . .
- As final plea, appellant invokes equity in its favor in - The insured died on November 2, 1944, while the - Based on the foregoing provision of law and the
view of the nullification of the deposits made by it policy was in force aforequoted stipulation as well as on the allegation
with the Philippine National Bank of all fiat money - After more than 7 years, in 1952, Atty de la Torre, that the filing of proof of death by the beneficiaries is
received from its policyholders, which money was representing the benficiaries of the policy, informed a condition precedent of the demandability of the
declared without value by Executive Order No. 49 of the company that Fernandez had died in 1944, and obligation of the insurer to pay the proceeds,
the President of the Philippines. Appellant claims claimed the proceeds of the policy. The company said appellants claim that they should be paid P10,000 in
that, considering the unexpected circumstances that that the status of the policies issued during the Philippine currency and not under the Ballantyne
developed, the indemnity to be paid by it should be Japanese occupation was still pending consideration scale of values.
suffered by it under Article 307 of the Code of before the courts. NLIC said that because the policy
INSURANCE Page
100

ISSUE refused to pay. Whereupon, petitioner sued them properly pegged by the Appellate Court at 6% per
WON the policy matured upon the death of the before the then Court of First Instance of Cebu for cent.
insured damages. EASCO, as the insurer, filed a counterclaim
against the petitioner for the recovery of the unpaid CATHAY INSURANCE v. CA(LUGAY)
HELD insurance premiums. 174 SCRA 11
YES - The trial court rendered judgment in favor of GRINO-AQUINO; June 5, 1989
Ratio In life insurance, the policy matures either petitioner. The judgment became final as to EASCO
upon the expiration of the term set forth therein, or but the shipping company appealed to the Court of FACTS
upon his death occuring at any time prior to the Appeals and was absolved from liability by the said - Petitioners are 6 insurance companies that issued
expiration of such stipulated term, in which case, the court. fire insurance policies for the total sum of
proceeds are payable to his beneficiaries within sixty - The trial court, upon motion by petitioner, issued a P4,000,000 to the Cebu Filipina Press owned by
days after their filing of proof of death. writ of execution against EASCO. The sheriff Emilia Chan Lugay. The fire policies described the
Reasoning enforcing the writ reportedly fixed the legal rate of insured property as "stocks of Printing materials,
- The sixty day period fixed by law within which to interest at 12%. Respondent EASCO moved to quash papers and general merchandise usual to the
pay the proceeds after presentation of proof of death the writ alleging that the legal interest to be Assured's trade" stored in a one-storey building of
Is merely procedural in nature, evidently to computed should be 6% per cent per annum in strong materials housing the Cebu Filipina Press
determine the exact amount to be paid and the accordance with Article 2209 of the Civil Code. The located at UNNO Pres. Quirino cor. Don V. Sotto Sts.,
interest thereon to which the beneficiaries may be trial court denied EASCO's motion. On appeal, the Mabolo, Cebu City. The co-insurers were indicated in
entitled to collect in case of unwarranted refusal of Court of Appeals reversed the trial court’s denial of each of the policies. All, except one policy
the company to pay, and also to enable the insurer EASCO’s motion and ruled that the applicable (Paramount's), were renewals of earlier policies
to verify or check on the fact of death which it may interest is 6% per annum. Hence, this petition. issued for the same property.
even validly waive. It is the happening of the - On December 18, 1981, the Cebu Filipina Press was
suspensive condition of death that renders a life ISSUE razed by electrical fire together with all the stocks
policy matured, and not ht efiling of proof of death WON the applicable rate of interest is 12% per and merchandise stored in the premises. On January
which, as above stated, is merely procedural. The annum 15, 1982, Lugay submitted sworn Statements of Loss
insured having died during the Japanese occupation, and Formal Claims to the insurers, through their
the proceeds of his policy should be adjusted HELD adjusters. She claimed a total loss of P4,595,000.
accordingly, for “The rule is already settled that NO - After nearly 10 months of waiting, she sued to
where a debtor could have paid his obligation at any - Sections 243 and 244 of the Insurance Code apply collect on December 15, 1982. The insurance
time during the Japanese occupation, payment after only when there is an unjustified refusal or companies denied liability, alleging violation of
liberation must be adjusted in accordance with the withholding of payment on the insured’s claim. In certain conditions of the policy, misdeclaration, and
Ballantyne schedule (De Asis vs. Agdamag, among this case, EASCO's refusal to settle the claim to Tio even arson which was not seriously pressed for,
other cases). (Collaboration is defined as the acts Khe Chio was based on some ground which, while come the pre-trial, the petitioners offered to pay
of working together in a joint project. not sufficient to free it from liability under its policy, 50% of her claim, but she insisted on full recovery.
Disposition Judgment affirmed nevertheless is sufficient to negate any assertion that - Trial court rendered judgment in her favor ordering
in refusing to pay, it acted unjustifiably. Simply put, the insurers to pay her a total of P4,000,000 as
TIO KHE CHIO v. CA (EASTERN ASSURANCE & the said provisions of the Insurance Code are not indemnity, P48,000 representing expenses of the
SURETY) pertinent to the instant case. They apply only when plaintiff, a separate amount of 20% of the
202 SCRA 119 the court finds an unreasonable delay or refusal in P4,000,000 representing fees of counsel, interests at
FERNAN; September 30, 1991 the payment of the claims. the rate of twice the ceiling being prescribed by the
- Circular No. 416 of the Central Bank, which raised Monetary Board starting from the time when the
FACTS the legal rate of interest from 6% to 12% per annum case was filed, and finally, with costs. CA affirmed.
- Petitioner Tio Khe Chio imported 1,000 bags of refers only to loans or forbearances of money, goods
fishmeal valued at $36,000.30 from Agro Impex, or credits and court judgments thereon but not to ISSUES
S.A. Dallas, Texas, U.S.A. The goods were insured court judgments for damages arising from injury to 1. WON the insured's cause of action had already
with respondent EASCO and shipped on board the persons and loss of property which does not involve accrued before she filed her complaint
M/V Peskov, a vessel owned by Far Eastern Shipping a loan. Clearly, the applicable law is Article 2209 of 2. WON sufficient proofs of loss had been presented
Company. When the goods reached Manila, they the Civil Code. by the insured
were found to have been damaged by sea water - And in the light of the fact that the contending 3. WON the private respondents claim for loss was
which rendered the fishmeal useless. Petitioner filed parties did not allege the rate of interest stipulated in inflated
a claim with EASCO and Far Eastern Shipping. Both the insurance contract, the legal interest was 4. WON lower court erred in awarding damages to
INSURANCE Page
101

the private respondent in the form of interest be adjudged to pay damages which shall consist of a two-storey building at 039 Barreda St., Mangagoy,
equivalent to double the interest ceiling set by the attorney's fees and other expenses incurred by the Bislig, Surigao del Sur.
Monetary Board insured person by reason of such unreasonable - While both policies were in force, fire destroyed
5. WON attorney's fees awarded were exorbitant denial or withholding of payment plus interest of petitioner's insured properties at the market site on
twice the ceiling prescribed by the Monetary Board September 5, 1977 and at Barreda St. on November
HELD of the amount of claim due the insured. 9, 1977.
1. YES - The petitioners' contention that the charging of - When petitioner failed to obtain indemnity on his
- As the fire which destroyed the Cebu Filipina Press double interest was improper because no claims from Zenith, he filed a complaint with the
occurred on December 19, 1981 and the proofs of unreasonable delay in the processing of the fire claim Insurance Commission praying that Zenith be
loss were submitted from January 15, 1982 through was proven is refuted by the trial court's explicit ordered to pay him P130kj representing the value of
June 21, 1982 in compliance with the adjusters' finding that "there was a delay that was not the 2 policies insured by respondent with interest at
numerous requests for various documents, payment reasonable in processing the claim and doing 12% per annum, plus damages, attorney's fees and
should have been made within 90 days thereafter payments". Under Section 244, a prima facie other expenses of litigation. ...
(Sec 243), or on or before September 21, 1982. evidence of unreasonable delay in payment of the - Zenith interposed that petitioner had no cause of
Hence, when the assured filed her complaint on claim is created by the failure of the insurer to pay action; that Policy No. F-03724 was not in full force
December 15, 1982, her cause of action had already the claim within the time fixed in both Sec. 242 and and effect at the time of the fire because the
accrued. 243 of the IC. premium on the policy was not paid; that Zenith's
2. YES - In view of the not insubstantial value of the private liability under Policy No. F-03734, if any, was limited
- There is no merit in the petitioners' contention that respondent's claims and the considerable time and to P15,472.50 in view of the co-insurance; and that
the proofs of loss were insufficient because Lugay effort expended by them and their counsel in petitioner failed to substantiate his claim as to the
failed to comply with the adjuster's request for the prosecuting these claims for the past 8 years, value of the goods reputedly destroyed by fire.
submission of her bank statements. Condition No. 13 attorney's fees were properly awarded to the private - While the case was pending, Zenith settled
of the policy does not require the insured to produce respondents. petitioner’s fire loss claim under Item 1 of Policy No.
her bank statements. Therefore, the insured was not 03734 in the amount of P15,472.50.
obligated to produce them and the insurers had no 5. YES - Insurance Commissioner allowed petitioner to
right to ask for them. Condition No. 13 was prepared - An award equivalent to 10% of the proceeds of the recover under said policy and ordered Zenith to pay
by the insurers themselves, hence, it should be policies would be more reasonable than the 20% him the amount of P20k with legal interest from the
taken most strongly against them. awarded by the trial court and the CA. date the complaint was filed, including P1k as
3. NO Disposition Decision of the CA AFFIRMED with attorney's fees but excluding the actual, moral and
- Both the trial court and the CA noted that the MODIFICATION. exemplary damages prayed for. As for petitioner's
proofs were ample and more than enough for claim under Policy No. F-03734, she held that in view
defendant insurers to do a just assessment NODA v. CRUZ of the payment of P15,472.50 to petitioner, Zenith
supporting the 1981 fire claim for an amount 151 SCRA 227 had fully discharged its liability under said policy
exceeding four million pesos. FERNAN; June 22, 1987 which covered furniture, fixtures, fittings and other
4. NO personal belongings of petitioner.
- The award of double interest on the claim is lawful NATURE - In allowing recovery under Policy No. F-03734,
and justified under Sections 243 and 244 of the Petition to review decision of the Insurance Commissioner placed much weight on the final report
Insurance Code which provide: Commissioner prepared by Dela Merced Adjustment Corporation, an
Sec. 243 Refusal or failure to pay the loss or independent fire, marine and casualty adjuster
damage within the time prescribed herein will FACTS contracted by Zenith to investigate the claims of its
entitle the assured to collect interest on the - In 1977, Noda obtained from Zenith Insurance various policyholders. Said report concluded that
proceeds of the policy for the duration of the delay Corporation 2 fire insurance policies: [1] No. F-03724 "the sound value of P26,666.67 represented the
at the rate of twice the ceiling prescribed by the with a face value of P30k covering the goods and whole loss and damage" incurred by petitioner, but
Monetary Board. stocks in trade in his business establishment at the with the application of the three-fourths loss clause,
Sec. 244 In case of any litigation for the market site in Mangagoy, Bislig, Surigao del Sur and Zenith's liability was reduced to P20k.
enforcement of any policy or contract of insurance, [2] No. F-03734 with a face value in the aggregate
it shall be the duty of the Commissioner or the amount of P100k and consisting of Item 1 for P40k ISSUES
Court, as the case may be, to make a finding as to on household furniture, fixtures, fittings and other 1. WON Insurance Commissioner erred in denying
whether the payment of the claim of the insured personal effects, and Item 2 for P60k on stocks in petitioner's demand for P60k under Item 2 of Policy
has been unreasonably denied or withheld; and in trade usual to petitioner's retail business situated in No. F-03734
the affirmative case, the insurance company shall
INSURANCE Page
102

2. WON Insurance Commissioner erred in not the greatest certainty of the facts in dispute. incident was caused by an unexpected inclement
awarding in favor of petitioner exemplary damages Respondent Commissioner should not have weather condition or force majeure, thus exempting
for Zenith's unjustified and wanton refusal to pay perfunctorily dismissed that particular evidence as a the common carrier from liability for the loss of its
petitioner's claim under the said two insurance worthless piece of paper. cargo.
contracts 2. NO - CA reversed RTC decision on the basis of evidence
- There is no showing that Zenith, in contesting from PAG-ASA that there were no 20 ft. waves in the
HELD payment, had acted in a wanton, oppressive or area. CA ruled that the petitioner is liable on its
1. YES malevolent manner to warrant the imposition of obligation as common carrier to respondent
- To prove the existence of the stocks in trade corrective damages. insurance company as subrogee of Caltex.
covered by Policy No. F-03734, petitioner offered his Disposition Zenith Insurance Corporation ordered to Petitioner’s Claim
testimony and that of his wife as well as pay petitioner Norman R. Noda the sum of > In every marine insurance upon a ship or freight,
documentary exhibits. The foregoing evidence for P60,592.10 with legal interest from the filing of the or freightage, or upon any thing which is the subject
petitioner preponderantly showed the presence of complaint until full payment, but deducting of marine insurance there is an implied warranty by
some P590k worth of goods in his retail store during therefrom the amount of P15,472.50 which it had the shipper that the ship is seaworthy. 10 When
the fire of November 9, 1977. earlier paid to petitioner. private respondent paid Caltex the value of its lost
- While the insurer, and the Insurance Commissioner cargo, the act of the private respondent is equivalent
for that matter, have the right to reject proofs of loss DELSAN TRANSPORT, INC. v. CA (AMERICAN to a tacit recognition that the ill-fated vessel was
if they are unsatisfactory, they may not set up for HOME ASSURANCE) seaworthy.
themselves an arbitrary standard of satisfaction. 369 SCRA 24
Substantial compliance with the requirements will DE LEON, JR; November 15, 2001 Respondent’s Comment
always be deemed sufficient. > American Home Assurance is entitled to payment
- Zenith introduced in evidence the final report on NATURE by its right of subrogation.
Policy No. F-03734 submitted by its own adjuster, A petition for review on certiorari of the decision of
Dela Merced Adjustment Corporation. Respondent CA. ISSUES
Commissioner however ignored such report, 1. WON payment made by American Home to Caltex
reasoning that with regard to Item 2 of Policy No. F- FACTS for the insured value of the lost cargo amounted to
03734 the claim for loss of the stocks in trade was - Caltex entered into a contract of affreightment with an admission that the vessel was seaworthy, thus
not successfully proven in view of petitioner's failure the petitioner, Delsan Transport Lines, Inc. precluding any action for recovery against the
to present evidence; that the adjuster's report (petitioner), for a period of one year whereby the petitioner
deserved scant consideration since the allegations said common carrier agreed to transport Caltex’s 2. WON MT Maysun was seaworthy at the time of the
therein were not substantiated, and that said report industrial fuel oil from the Batangas-Bataan Refinery voyage (outline topic)
did not even make a recommendation for payment. to different parts of the country. Delsan took on 3. WON non-presentation of the marine insurance
- A scrutiny of the abovementioned adjuster's report board its vessel, MT Maysun, 2,277.314 kiloliters of policy bars the complaint for recovery of sum of
reveals that together with the formal demand for full industrial fuel oil of Caltex to be delivered to the money for lack of cause of action
indemnity, petitioner submitted his income tax return Caltex Oil Terminal in Zamboanga City. The
for 1978, purchase invoices, certification from his shipment was insured by American Home Assurance HELD
suppliers as to his purchases, and other supporting Corporation (respondent). 1. NO
papers. The report even took into account the - August 14, 1986: MT Maysun set sail from Ratio The fact of payment grants American Home
appraisals of the other adjusters and concluded that Batangas for Zamboanga City. The vessel sank in the subrogatory right which enables it to exercise
the total loss sustained by petitioner in his household the early morning of August 16, 1986 near Panay legal remedies that would otherwise be available to
effects and stocks in trade reached P379,302.12. But Gulf in the Visayas taking with it the entire cargo of Caltex as owner of the lost cargo against the
after apportioning said amount among petitioner's fuel oil. petitioner common carrier.
six different insurers [the co-insurance being known - Respondent paid Caltex P5,096,635.57 Reasoning
to Zenith], the liability of Zenith was placed at representing the insured value of the lost cargo. Art. 2207. (Civil Code)
P60,592.10. It therefore recommended that Zenith Exercising its right of subrogation under Article 2207 If the plaintiff’s property has been insured, and he
pay the petitioner the amount of P60, 592.10. of the New Civil Code, the private respondent has received indemnity from the insurance
- Said document was offered as evidence by Zenith demanded of the petitioner the same amount it paid company for the injury or loss arising out of the
itself and could very well be considered as an to Caltex. Delsan refused to pay, forcing American wrong or breach of contract complained of, the
admission of its liability up to the amount home to file a case for collection in the RTC. insurance company shall be subrogated to the
recommended. Being in the nature of an admission - RTC found that the vessel, MT Maysun, was
against interest, it is the best evidence which affords seaworthy to undertake the voyage, and that the 10
Sec. 113 Insurance Code
INSURANCE Page
103

rights of the insured against the wrongdoer or the unseaworthy, for the cargo owner has no obligation ISSUE
person who has violated the contract. If the in relation to seaworthiness. WON Finman can be held liable for complainants’
amount paid by the insurance company does not 3. NO claims against Pan Pacific
fully cover the injury or loss, the aggrieved party Ratio The presentation in evidence of the marine
shall be entitled to recover the deficiency from the insurance policy is not indispensable in this case HELD
person causing the loss or injury. before the insurer may recover from the common YES
- The right of subrogation is designed to promote carrier the insured value of the lost cargo in the - Under Insurance Code, liability of surety in a
and to accomplish justice and is the mode which exercise of its subrogatory right. The subrogation surety bond is joint and several with the principal
equity adopts to compel the ultimate payment of a receipt, by itself, is sufficient to establish not only the obligor.
debt by one who in justice and good conscience relationship of respondent as insurer and Caltex, as - Conditions of a bond specified and required in the
ought to pay. It is not dependent upon, nor does it the assured shipper of the lost cargo of industrial fuel provisions of a statute providing for submission of
grow out of, any privity of contract or upon written oil, but also the amount paid to settle the insurance the bond, are incorporated into all bonds tendered
assignment of claim. It accrues simply upon claim. The right of subrogation accrues simply upon under that statute even though not set out in
payment by the insurance company of the insurance payment by the insurance company of the insurance printer’s ink.
claim. claim. - POEA held and Secretary of Labor affirmed that
2. NO Disposition Petition is denied, and the decision of Pan Pacific had violated Labor Code, and at least one
Ratio Seaworthiness relates to a vessel’s actual the CA is affirmed. of the conditions for the grant and continued use of
condition. Neither the granting of classification or the recruitment license. POEA and Secretary of
the issuance of certificates establishes Labor can require Pan Pacific to refund the placement
seaworthiness. fees and to impose the fine.
Reasoning - If Pan Pacific is liable, and if Finman is solidarily
- Common carriers are bound to observe FINMAN GENERAL ASSURANCE CORP v. liable with Pan Pacific, then Finman is liable both to
extraordinary diligence in the vigilance over the INOCENCIO private respondents and to POEA.
goods and for the safety of passengers transported 179 SCRA 480 - Cash and surety bonds are required from
by them, according to all the circumstances of each FELICIANO; November 15, 1989 recruitment companies as means of ensuring prompt
case. There is no liability if the loss, destruction or and effective recourse against such companies when
deterioration is by force majeure. FACTS held liable. Public policy will be effectively negated if
- The tale of strong winds and big waves by the said - Pan Pacific is a recruitment and employment POEA and the DoLE were held powerless to compel a
officers of the petitioner however, was effectively agency. It posted surety bond issued by Finman surety company to make good on its solidary
rebutted and belied by the weather report from PAG- General Assurance and was granted license to undertaking.
ASA. MT Maysun sank with its entire cargo for the operate by POEA.
reason that it was not seaworthy. There was no - Inocencio, Palero, Cardones, Hernandez filed with EAGLE STAR INSURANCE CO LTD v. CHIA YU
squall or bad weather or extremely poor sea POEA complaints against Pan Pacific for violation of 96 PHIL 696
condition in the vicinity when the said vessel sank. Labor Code and for refund of placement fees. POEA REYES; March 31, 1955
- Petitioner may not escape liability by presenting in Administrator motu propio impleaded Finman as
evidence certificates that tend to show that at the surety for Pan Pacific. NATURE
time of dry-docking and inspection by the Philippine - Pan Pacific moved out and no notice of transfer was Certiorari
Coast Guard MT Maysun, was fit for voyage. These furnished to POEA as required. POEA considered that
pieces of evidence do not necessarily take into constructive service of complaints had been effected. FACTS
account the actual condition of the vessel at the time - Finman denied liability and said that - Atkin, Kroll & Co., loaded on the S. S. Roeph
of the commencement of the voyage. At the time of - POEA had no jurisdiction over surety bonds; Silverlight owned and operated by Leigh Hoegh &
dry-docking and inspection, the ship may have jurisdiction is vested in Insurance Commission Co., A/S, of San Francisco California, 14 bales of
appeared fit. The certificates issued, however, do - Finman had not violated Labor Code assorted underwear valued at P8,085.23 consigned
not negate the presumption of unseaworthiness - Complainants have no cause of action against to Chia Yu in the City of Manila.
triggered by an unexplained sinking. Finman - The shipment was insured against all risks by Eagle
- Authorities are clear that diligence in securing - Amounts claimed were paid as deposits and not as Star Ins. Co. of San Francisco, California, under a
certificates of seaworthiness does not satisfy the placement fees. policy issued to the shipper and by the latter
vessel owner’s obligation. Also securing the approval - POEA Administrator issued Order that respondents assigned to the consignee.
of the shipper of the cargo, or his surveyor, of the should pay. Finman appealed to Secretary of Labor. - The vessel arrived in Manila but of the 14 bales
condition of the vessel or her stowage does not Secretary upheld the POEA order. (a.k.a. freights =p) consigned to Chia Yu only 10
establish due diligence if the vessel was in fact were delivered to him as the remaining 3 could not
INSURANCE Page
104

be found.3 of those delivered were also found - Insular Government vs. Frank(13 Phil. 236)~
damaged to the extent of 50 per cent. "matters respecting a remedy, such as the bringing
-Chia Yu claimed indemnity for the missing and of suit, admissibility of evidence, and statute of ACCFA v. ALPHA INSURANCE
damaged bales. But the claim was declined, first, by limitations, depend upon the law of the place where 24 SCRA 151
the carrier and afterward by the insurer, whereupon the suit is brought" TF any policy clause repugnant to REYES; July 29, 1968
Chia Yu brought the present action against both, this amendment to the Insurance Act cannot be
including their respective agents in the Philippines. given effect in an action in our courts. FACTS
- An action was filed at the CFI after more than 2 SEC. 61-A. (Insurance Code) ~ Any condition, - In order to guarantee the Asingan Farmers'
years after delivery of the damaged bales and the stipulation or agreement in any policy of insurance, Cooperative Marketing Association, Inc. (FACOMA)
date when the missing bales should have been limiting the time for commencing an action against loss on account of "personal dishonesty,
delivered, the action was resisted by the Atkins and thereunder to a period of less than one year from amounting to larceny or estafa of its Secretary-
Eagle Star principally on the ground of prescription. the time when the cause of action accrues, is void. Treasurer, Ladines, the appellee, Alpha Insurance &
-TC favored Chia Yu and CA affirmed. - The prescription clause could be harmonized with Surety Company had issued, on 14 February 1958,
*** CARRIER’s defense of prescription is made to section 61-A of the Insurance Act by taking it to its bond, No. P-FID-15-58, for the sum of P5,000
rest on the following stipulation of the bill of lading: mean that the time given the insured for bringing his with said Ladines as principal and the appellee as
In any event the carrier and the ship shall be suit is twelve months after the cause of action solidary surety. On the same date, the Asingan
discharged from all liability in respect of loss or accrues. FACOMA assigned its rights to the appellant,
damage unless suit is brought within one year - If so, when did the cause of action accrue? Chia Agricultural Credit Cooperative and Financing
after the delivery of the goods or the date when Yu’s action did not accrue until his claim was finally Administration (ACCFA for short), with approval of
the goods should have been delivered. (This rejected by the insurance company. This is because, the principal and the surety.
stipulation is but a repetition of a provision in the before such final rejection, there was no real - During the effectivity of the bond, Ladines
CA 65 which says that bills of lading covering necessity for bringing suit. converted and misappropriated, to his personal
shipments from the US to the Phils should be - As the policy provides that the insured should file benefit, some P11,513.22 of the FACOMA funds, of
brought w/in one year after the delivery of the his claim, first, with the carrier and then with the which P6,307.33 belonged to the ACCFA. Upon
goods or the date when the goods should have insurer, he had a right to wait for his claim to be discovery of the loss, ACCFA immediately notified in
been delivered to hold the carrier liable.) finally decided before going to court. writing the survey company on 10 October 1958, and
*** INSURER’s claim of prescription is founded upon - Furthermore, there is nothing in the record to show presented the proof of loss within the period fixed in
the terms of the policy and not upon the bill of that the claim was rejected in the year 1947, either the bond; but despite repeated demands the surety
lading. (But in our jurisdiction, as per A1144, by the insurance company in London or its settling company refused and failed to pay. Whereupon,
prescription is 10 years after action accrues.) agents in the Philippines. ACCFA filed suit against appellee on 30 May 1960.
No suit action on this Policy, for the recovery of - For the purpose of this action, Chia Yu's claim was - Defendant Alpha Insurance & Surety Co., Inc., (now
any claim, shall be sustainable in any Court of law considered to have been finally rejected by the appellee) moved to dismiss the complaint for failure
or equity unless the insured shall have fully insurer on April 22, 1948. Having been filed within to state a cause of action, giving as reason that (1)
complied with all the terms and conditions of this twelve months form that date, the action cannot be the same was filed more than one year after plaintiff
Policy nor unless commenced with twelve (12) deemed to have prescribed even on the supposition made claim for loss, contrary to the eighth condition
months next after the happening of the loss . . . that the period given the insured for bringing suit of the bond, providing as follows:
under the prescriptive clause of the policy is twelve EIGHT LIMITATION OF ACTION: No action, suit or
ISSUE months after the accrual of the cause of action. proceeding shall be had or maintained upon this
WON ATKIN’ s action has prescribed - Contractual limitations contained in insurance Bond unless the same be commenced within one
policies are regarded with extreme jealousy by year from the time of making claim for the loss upon
HELD courts and will be strictly construed against the which such action, suit or proceeding, is based, in
NO insurer and should not be permitted to prevent a accordance with the fourth section hereof.
- Being contrary to the law of the forum, the recovery when their just and honest application (2) the complaint failed to show that plaintiff had
stipulation in the policy cannot be given effect as it would not produce that result. (46 C. J. S. 273.) filed civil or criminal action against Ladines, as
would reduce the period allowed the insured for Disposition Judgment appealed from is REVERSED required by conditions 4 and 11 of the bond; and (3)
bringing his action to less than one year (because with respect to the carrier and its agents but that Ladines was a necessary and indispensable
the prescription period begins from the “happening AFFIRMED with respect to the insurance company party but had not been joined as such.
of the loss” and that before any suit could be and its agents. - At first, the Court of First Instance denied
sustained the insured shall have to comply with the dismissal; but, upon reconsideration, the court
terms and conditions of the policy first TF lessening reversed its original stand, and dismissed the
the period to less than a year. ) complaint on the ground that the action was filed
INSURANCE Page
105

beyond the contractual limitation period. Hence, this considering the unrelieved congestion in the courts. - The bringing of the action against the Paramount
appeal. As a consequence of the foregoing, action may be Surety & Insurance Company, the agent of the
brought within the statutory period of limitation for defendant company, cannot have any legal effect
ISSUE written contracts (New Civil Code, Article 1144). except that of notifying the agent of the claim.
WON the provision of a fidelity bond that no action Beyond such notification, the filing of the action can
shall be had or maintained thereon unless ANG v. FULTON FIRE INSURANCE CO. serve no other purpose. There is no law giving any
commenced within one year from the making of a 2 SCRA 945 effect to such action upon the principal. Besides,
claim for the loss upon which the action is based, is LABRADOR; July 31, 1961 there is no condition in the policy that the action
valid, in view of Section 61-A of the Insurance Act must be filed against the agent, and the Court can
invalidating stipulations limiting the time for NATURE not by interpretation extend the clear scope of the
commencing an action thereon to less than one year Appeal from judgment of the CFI ordering the agreement beyond what is agreed upon by the
from the time the cause of action accrues defendant Fulton Fire Insurance Co. to pay the parties.
plaintiffs the sum of P10,000.00, with interest, and - Their contract is the law between the parties, and
HELD an additional sum of P2,000.00 as attorney's fees, their agreement that an action on a claim denied by
NO and costs. the insurer must be brought within one year from the
- A fidelity bond is, in effect, in the nature of a denial, governs, not the rules on the prescription of
contract of insurance against loss from misconduct, FACTS actions.
and is governed by the same principles of - The stocks of general merchandise in the store of Disposition The judgment appealed from is hereby
interpretation. Consequently, the condition of the the Ang spouses are insured with Fulton. While the set aside and the case dismissed, with costs against
bond in question, limiting the period for bringing insurance was in force, fire destroyed the goods. The plaintiffs-appellees.
action thereon, is subject to the provisions of Section Angs filed their first claim immediately after the fire.
61-A of the Insurance Act (No. 2427), as amended - Their claim was denied on April 6, 1956. They TRAVELLERS INSURANCE & SURETY CORP. v.
by Act 4101 of the pre-Commonwealth Philippine received notice on April 19, 1956. CA (MENDOZA)
Legislature, prescribing that: - The Angs brought an action against the agent on 272 SCRA 536
SEC. 61-A: A condition, stipulation or agreement in May 11, 1956. The court denied the suit and the mfr HERMOSISIMA, JR; May 22, 1997
any policy of insurance, limiting the time for on Sept. 3 and 12, 1957.
commencing an action thereunder to a period of - The Angs filed against Fulton on May 26, 1958. NATURE
less than one year from the time when the cause - There was a clause in the policy: The petition herein seeks the review and reversal of
of action accrues is void. 13.If the claim be in any respect fraudulent, or if the decision of respondent Court of Appeals affirming
- Since a "cause of action" requires, as essential any false declaration is made or used in support in toto the judgment of the Regional Trial Court in an
elements, not only a legal right of the plaintiff and a thereof, or if any fraudulent means or devices are action for damages filed by private respondent
correlative obligation of the defendant but also "an used by the Insured or any one acting on his Vicente Mendoza, Jr. as heir of his mother who was
act or omission of the defendant in violation of said behalf to obtain any benefit under this Policy, or, if killed in a vehicular accident.
legal right," the cause of action does not accrue until the loss or damage be occasioned by the wilful act
the party obligated refuses, expressly or impliedly, to or with the connivance of the Insured, or, if the FACTS
comply with its duty (in this case, to pay the amount claim be made and rejected and an action or suit -an old lady was hit by a taxicab. The taxicab was
of the bond). The year for instituting action in court be not commenced within twelve months after later identified and a case was filed against the driver
must be reckoned, therefore, from the time of such rejection or (in case of arbitration taking and owner. Later, an amendment was filed to include
appellee's refusal to comply with its bond; it can not place in pursuance of the 18th condition of this the insurance company. RTC and CA ordered that the
be counted from the creditor's filing of the claim of Policy) within twelve months after the arbitrator or owner, driver as well as the insurance company be
loss, for that does not import that the surety arbitrators or umpire shall have made their award held solidarily liable.
company will refuse to pay. In so far, therefore, as all benefit under this Policy shall be forfeited."
condition eight of the bond requires action to be filed ISSUE
within one year from the filing of the claim for loss, ISSUE WON RTC and CA erred
such stipulation contradicts the public policy WON the suit against the agent tolled the HELD
expressed in Section 61-A of the Philippine Insurance prescription period, such that the filing against Fulton YES
Act. was only 9 months after the claim was rejected - Where the contract provides for indemnity against
- Condition eight of the bond, therefore, is null and liability to third persons, then third persons to whom
void, and the appellant is not bound to comply with HELD the insured is liable can sue the insurer. Where the
its provisions. The discouraging of unnecessary NO contract is for indemnity against actual loss or
litigation must be deemed a rule of public policy, payment, then third persons cannot proceed against
INSURANCE Page
106

the insurer, the contract being solely to reimburse - On 4 May 1970, the insured vehicle was bumped petitioner, the contents having expressed all intents
the insured for liability actually discharged by him and damaged by a truck owned by San Miguel and purposes of the parties.
thru payment to third persons, said third persons' Corporation. For the damage caused, ZENITH paid
recourse being thus limited to the insured alone. But MLA MAHOGANY P5,000 in amicable settlement. MLA ISSUE
in the case at bar, there was no contract shown. MAHOGANY's general manager executed a Release of WON the insurer may recover the sum of P5,000
What then was the basis of the RTC and the CA Claim, subrogating respondent company to all its
to say that the insurance contract was a third- right to action against San Miguel Corporation. HELD
party liability insurance policy? Consequently, the - On 11 Dec 1972, ZENITH wrote Insurance YES
trial court was confused as it did not distinguish Adjusters, Inc. to demand reimbursement from San Ratio Since the insurer can be subrogated to only
between the private respondent's cause of action Miguel. Insurance Adjusters, Inc. refused such rights as the insured may have, should the
against the owner and the driver of the Lady Love reimbursement, alleging that San Miguel had already insured, after receiving payment from the insurer,
taxicab and his cause of action against petitioner. paid petitioner P4,500, as evidenced by a cash release the wrongdoer who caused the loss, the
The former is based on torts and quasi-delicts while voucher and a Release of Claim executed by the insurer loses his rights against the latter. But in such
the latter is based on contract. General Manager of petitioner. a case, the insurer will be entitled to recover from
- Even assuming arguendo that there was such a - ZENITH thus demanded from petitioner the insured whatever it has paid to the latter, unless
contract, private respondent's cause of action can reimbursement of the sum of P4,500 paid by San the release was made with the consent of the
not prevail because he failed to file the written Miguel. insurer.
claim mandated by the Insurance Code (before - City Court ordered petitioner to pay respondent Reasoning
it was amended-action must be brought within P4,500. - Although petitioner’s right to file a deficiency claim
six months from date of the accident (this is - CFI affirmed the City Court's decision in toto. against San Miguel is with legal basis, without
what’s applicable here) ; after amendment- - CA affirned CFI, with the modification that prejudice to the insurer's right of subrogation,
"action or suit for recovery of damage due to loss or petitioner was to pay the total amount of P5,000 it nevertheless, when Manila Mahogany executed
injury must be brought in proper cases, with the had earlier received from ZENITH. another release claim discharging San Miguel from
Commissioner or the Courts within one year from Petitioner’s Claims "all actions, claims, demands and rights of action
denial of the claim, otherwise the claimant's right of > It is not bound to pay P4,500, and much more, that now exist or hereafter arising out of or as a
action shall prescribe" ). He is deemed, under this P5,000 to ZENITH as the subrogation in the Release consequence of the accident" after the insurer had
legal provision, to have waived his rights as against of Claim it executed in favor of respondent was paid the proceeds of the policy - the compromise
petitioner-insurer. conditioned on recovery of the total amount of agreement of P5,000 being based on the insurance
Disposition petition granted damages petitioner had sustained. Since total policy - the insurer is entitled to recover from the
damages were valued by petitioner at P9,486.43 and insured the amount of insurance money paid. Since
SUN INSURANCE v. CA (supra p.57) only P5,000 was received by petitioner, MLA petitioner by its own acts released San Miguel,
MAHOGANY argues that it was entitled to go after thereby defeating private respondent’s right of
COASTWISE v. CA (supra p.70) San Miguel to claim the additional P4,500. subrogation, the right of action of petitioner against
> It cites Art. 220711 and Art. 130412 of the Civil the insurer was also nullified.
CEBU SHIPYARD v. WILLIAM LINES (supra p.3) Code, and claims a preferred right to retain the - As held in Phil. Air Lines v. Heald Lumber Co.,
amount coming from San Miguel, despite the under Art. 2207, the real party in interest with
MANILA MAHOGANY MANUFACTURING CORP v. subrogation in favor of ZENITH. regard to the portion of the indemnity paid is the
CA (ZENITH INSURANCE CORP) Respondent’s Arguments insurer and not the insured.
154 SCRA 652 > There was no qualification to its right of SUBROGATION: The right of subrogation can only
PADILLA; October 12, 1987 subrogation under the Release of Claim executed by exist after the insurer has paid the insured,
otherwise the insured will be deprived of his right to
NATURE full indemnity. If the insurance proceeds are not
11
Petition to review CA decision ordering Manila Article 2207: “If the plaintiff's property has been insured, and he sufficient to cover the damages suffered by the
Mahogany Manufacturing Corporation to pay Zenith has received indemnity from the insurance company for the injury or insured, then he may sue the party responsible for
loss arising out of the wrong or breach of contract complained of the
Insurance Corporation P5,000 with 6% annual insurance company shall be subrogated to the rights of the insured the damage for the remainder. To the extent of the
interest, attorney's fees, and costs of suit against the wrongdoer or the person who has violated the contract. If amount he has already received from the insurer
the amount paid by the insurance company does not fully cover the
injury or loss the aggrieved party shall be entitled to recover the
enjoys the right of subrogation.
FACTS deficiency from the person causing the loss or injury.” Disposition Petition DENIED. Judgment appealed
- From 6 March 1970 to 6 March 1971, MLA 12
Article 1305: “A creditor, to whom partial payment has been made, from is AFFIRMED with costs against petitioner.
MAHOGANY insured its Mercedes Benz 4-door sedan may exercise his right for the remainder, and he shall be preferred to
with ZENITH. the person who has been subrogated in his place in virtue of the partial
payment of the same credit.”
INSURANCE Page
107

PIONEER INSURANCE v. CA (BORDER preliminary attachment against Lim and respondents, pays a loss (Universal Ins. Co. v. Old Time Molasses
MACHINERY & HEAVY EQUIPMENT INC) the Cervanteses, Bormaheco and Maglana. Co.).
175 SCRA 668 **Maglana, Bormaheco and the Cervanteses filed - The rules of practice in actions on original insurance
GUTIERREZ, JR.; July 28, 1989 cross-claims against Lim alleging that they were not policies are in general applicable to actions or
privies to the contracts signed by Lim and, by way of contracts of reinsurance (Delaware, Ins. Co. v.
NATURE counterclaim, sought for damages for being exposed Pennsylvania Fire Ins. Co.).
Petitions for review on certiorari of a decision of the to litigation and for recovery of the sums of money - Hence the applicable law is Article 2207 of the new
CA they advanced to Lim for the purchase of the Civil Code, to wit: Art. 2207. If the plaintiff’s
aircrafts in question. (this constitutes the second property has been insured, and he has received
FACTS petition but will no longer be discussed because it is indemnity from the insurance company for the injury
- In 1965, Jacob S. Lim was engaged in the airline not relevant to the topic) or loss arising out of the wrong or breach of contract
business as owner-operator of Southern Air Lines - After trial on the merits, a decision was rendered complained of, the insurance company shall be
(SAL), a single proprietorship. holding Lim liable to pay Pioneer but dismissed subrogated to the rights of the insured against the
-On May 17, 1965, at Tokyo, Japan, Japan Domestic Pioneer's complaint against all other wrongdoer or the person who has violated the
Airlines (JDA) and Lim entered into and executed a defendants. contract. If the amount paid by the insurance
sales contract for the sale and purchase of two (2) - CA modified the trial court's decision in that the company does not fully cover the injury or loss, the
DC-3A Type aircrafts and one (1) set of necessary plaintiff’s complaint against all the defendants aggrieved party shall be entitled to recover the
spare parts for the total agreed price of US (including Lim) was dismissed. deficiency from the person causing the loss or injury
$109,000.00 to be paid in installments. - If a property is insured and the owner receives the
- On May 22, 1965, Pioneer Insurance and Surety ISSUE indemnity from the insurer, it is provided in said
Corporation, as surety, executed and issued its WON the petition of Pioneer Insurance and Surety article that the insurer is deemed subrogated to the
Surety Bond No. 6639 in favor of JDA, in behalf of its Corporation against all defendants was rightly rights of the insured against the wrongdoer and if
principal, Lim, for the balance price of the aircrafts dismissed the amount paid by the insurer does not fully cover
and spare parts. the loss, then the aggrieved party is the one entitled
-Border Machinery and Heavy Equipment Company, HELD to recover the deficiency. Evidently, under this legal
Inc. (Bormaheco), Francisco and Modesto Cervantes YES provision, the real party in interest with regard to
(Cervanteses) and Constancio Maglana contributed - Both the TC and CA made the finding that Pioneer the portion of the indemnity paid is the insurer and
some funds used in the purchase of the above reinsured its risk of liability under the surety bond it not the insured (. Air Lines, Inc. v. Heald Lumber
aircrafts and spare parts. They executed two (2) had executed in favor of JDA, collected the proceeds Co., and Manila Mahogany Manufacturing
separate indemnity agreements in favor of Pioneer, of such reinsurance in the sum of P295,000, and paid Corporation v. Court of Appeals)
one signed by Maglana and the other jointly signed with the said amount the bulk of its alleged liability - It is clear from the records that Pioneer sued in its
by Lim for SAL, Bormaheco and the Cervanteses. to JDA under the said surety bond. The total amount own name and not as an attorney-in-fact of the
- On June 10, 1965, Lim doing business under the paid by Pioneer to JDA is P299,666.29. Since Pioneer reinsurer. Accordingly, the appellate court did not
name and style of SAL executed in favor of Pioneer has collected P295,000.00 from the reinsurers, the commit a reversible error in dismissing the
as deed of chattel mortgage as security for the uninsured portion of what it paid to JDA is the petitioner's complaint as against the respondents for
latter's suretyship in favor of the former. It was difference between the two amounts, or P3,666.28. the reason that the petitioner was not the real party
stipulated therein that Lim transfer and convey to This is the amount for which Pioneer may sue in interest in the complaint and, therefore, has no
the surety the two aircrafts. defendants, assuming that the indemnity agreement cause of action against the respondents.
- Lim defaulted on his subsequent installment is still valid and effective. But since the amount Disposition Petitions dismissed. Questioned decision
payments prompting JDA to request payments from realized from the sale of the mortgaged chattels are of CA affirmed.
the surety. P35,000.00 for one of the airplanes and P2,050.00
- Pioneer paid a total sum of P298,626.12. for a spare engine, or a total of P37,050.00, Pioneer
- Pioneer then filed a petition for the extrajudicial is still overpaid by P33,383.72. Therefore, Pioneer PAN MALAYAN INSURANCE CORPORATION v.
foreclosure of the said chattel mortgage before the has no more claim against defendants. CA (FABIE, HER UNKNOWN DRIVER)
Sheriff of Davao City. - The payment to the petitioner made by the 184 SCRA 54
- The Cervanteses and Maglana, however, filed a reinsurers was not disputed. Considering this CORTES, April 3, 1990
third party claim alleging that they are co-owners of admitted payment, the only question was the effect
the aircrafts, of payment made by the reinsurers to the petitioner NATURE
- On July 19, 1966, Pioneer filed an action for judicial - In general a reinsurer, on payment of a loss PETITION to review the decision of the Court of
foreclosure with an application for a writ of acquires the same rights by subrogation as are Appeals
acquired in similar cases where the original insurer
INSURANCE Page
108

FACTS assured to recover from the wrongdoer to the extent - Both Panmalay and Canlubang had the same
- Pan Malayan Insurance Company (Panmalay) that the insurer has been obligated to pay. Payment interpretation regarding the coverage of insured risk
insured the Mitsubishi Colt Lancer car registered in by the insurer to the assured operates as an regarding “accidental collision or overturning…” to
the name of Canlubang Automotive Resources equitable assignment to the former of all remedies include damages caused by 3rd party to Canlubang so
Corporation (Canlubang) under its motor vehicle which the latter may have against the third party it was improper for CA to ascribe meaning contrary
insurance policy. Among the provisions of the policy whose negligence or wrongful act caused the loss. to the clear intention and understanding of the
was a “own-damage” clause whereby Panmalay The right of subrogation is not dependent upon, nor parties.
agrees to indemnify Canlubang in cases of damage does it grow out of, any privity of contract or upon - Court on several occasions defined “accident” or
caused by “accidental collision or overturning, or written assignment of claim. It accrues simply upon “accidental” as taking place “without one’s foresight
collision or overturning consequent upon mechanical payment of the insurance claim by the insurer or expectation, an event that proceeds from an
breakdown or consequent upon wear and tear”. Exceptions unknown cause, or is an unusual effect of a known
- On 1985, the insured car was sideswept and (1) if the assured by his own act releases the cause and, therefore, not expected” [Dela Cruz v.
damaged by a car owned by Erlinda Fabie, driven by wrongdoer or third party liable for the loss or Capital Insurance & Surety Co.] The concept
an unknown driver who fled the scene. Panmalay, in damage, from liability, the insurer's right of "accident" is not necessarily synonymous with the
accordance with the policy, defrayed the cost of subrogation is defeated; concept of "no fault". It may be utilized simply to
repair of the insured car and was subrogated to the (2) where the insurer pays the assured the value of distinguish intentional or malicious acts from
rights of Canlubang against the driver and owner of the lost goods without notifying the carrier who has negligent or careless acts of man.
the pick-up. Panmalay then filed a complaint for in good faith settled the assured's claim for loss, the - damage/loss to insured vehicle due to negligence of
damages with RTC Makati against Erlinda Fabie and settlement is binding on both the assured and the 3rd parties not listed as exceptions to coverage in the
her driver on the grounds of subrogation, with the insurer, and the latter cannot bring an action against insurance policy
latter failing and refusing to pay their claim. Fabie the carrier on his right of subrogation; - Interpretation given by Panmalay is more in
filed a Motion for Bill of Particulars. (3) where the insurer pays the assured for a loss keeping with rationale behind rules on interpretation
- RTC: dismissed complaint for lack of cause of which is not a risk covered by the policy, thereby of insurance contracts in favor of assured or
action (payment by PANMALAY of CANLUBANG's effecting "voluntary payment", the former has no beneficiary: indemnity or payment
claim under the "own damage" clause of the right of subrogation against the third party liable for - EVEN if voluntarily indemnified Canlubang, as
insurance policy was an admission by the insurer the loss interpreted by TC: the insurer who may have no
that the damage was caused by the assured and/or Reasoning rights of subrogation due to "voluntary" payment
its representatives) – Panmalay appealed - Both TC and CA are incorrect. may never. theless recover from the third party
- CA: dismissed appeal, affirmed RTC (applying the ON TC: “Own damage” (not found in the insurance responsible for the damage to the insured property
ejusdem generis rule held that Section III-1 of the policy) simply meant that Panmalay had assumed to under Article 1236 of the Civil Code. [Sveriges
policy, which was the basis for settlement of reimburse the cost for repairing the damage to the Angfartygs Assurans Forening v. Qua Chee Gan]
CANLUBANG's claim, did not cover damage arising insured vehicle. It’s different from “Third Party Disposition the present petition is GRANTED.
from collision or overturning due to the negligence of Liability” coverage (liabilities arising from the death Petitioner's complaint for damages against private
third parties as one of the insurable risk) of or bodily injuries suffered by 3rd parties) and from respondents is hereby REINSTATED. Let the case be
“Property Damage” coverage (liabilities from damage remanded to the lower court for trial on the merits.
ISSUE caused by insured vehicle to properties of 3rd parties)
WON the insurer PANMALAY may institute an action ON CA: the terms of a contract are to be construed FIREMAN'S FUND INSURANCE COMPANY v.
to recover the amount it had paid its assured in according to the sense and meaning of the terms JAMILA & COMPANY, INC.
settlement of an insurance claim against private which the parties thereto have used. In the case of 70 SCRA 323
respondents as the parties allegedly responsible for property insurance policies, the evident intention of AQUINO; April 1976
the damage caused to the insured vehicle, in the contracting parties, i.e., the insurer and the
accordance with A2207, NCC assured, determine the import of the various terms FACTS
and provisions embodied in the policy. It is only - Jamila & Co., Inc. or the Veterans Philippine
HELD when the terms of the policy are ambiguous, Scouts Security Agency contracted to supply security
YES equivocal or uncertain, such that the parties guards to Firestone. Jamila assumed responsibility
Ratio Article 2207 of the Civil Code is founded on themselves disagree about the meaning of particular for the acts of its security guards. The First Quezon
the well-settled principle of subrogation. If the provisions, that the courts will intervene. In such an City Insurance Co., Inc. executed a bond in the sum
insured property is destroyed or damaged through event, the policy will be construed by the courts of P20,000 to guarantee Jamila's obligations under
the fault or negligence of a party other than the liberally in favor of the assured and strictly against that contract.
assured, then the insurer, upon payment to the the insurer - On May 18, 1963 properties of Firestone valued at
assured, will be subrogated to the rights of the P11,925 were lost allegedly due to the acts of its
INSURANCE Page
109

employees who connived with Jamila's security - The plaintiffs cited article 2207 of the Civil Code action which the insured may have against the third
guard. Fireman's Fund, as insurer, paid to Firestone which provides that "if the plaintiff's property has person whose negligence or wrongful act caused the
the amount of the loss. Fireman's Fund was been insured, and he has received indemnity from loss. The right of subrogation is of the highest equity.
subrogated to Firestone's right to get reimbursement the insurance company for the injury or loss arising The loss in the first instance is that of the insured
from Jamila. Jamila and its surety, First Quezon City out of the wrong or breach of contract complained of, but after reimbursement or compensation, it
failed to pay the amount of the loss in spite of the insurance company shall be subrogated to the becomes the loss of the insurer.
repeated demands. rights of the insured against the wrongdoer or the - Although many policies including policies in the
- Upon defendant's motions, the lower court person who has violated the contract". standard form, now provide for subrogation, and
dismissed the complaint as to Jamila on the ground - The lower court denied plaintiff's motion. They thus determine the rights of the insurer in this
that there was no allegation that it had consented to filed a second MR, calling the lower court's attention respect, the equitable right of subrogation as the
the subrogation and, therefore, Fireman's Fund had to the fact that the issue of subrogation was of no legal effect of payment inures to the insurer without
no cause of action against it. It also dismissed the moment because Firestone, the subrogor, is a party- any formal assignment or any express stipulation to
complaint as to First Quezon City on the ground of plaintiff and could sue directly Jamila in its own right. that effect in the policy. Stated otherwise, when the
res judicata. It appears that the same action was Without resolving that contention, the lower court insurance company pays for the loss, such payment
previously filed in a civil case which was dismissed denied plaintiffs' second MR. operates as an equitable assignment to the insurer of
because of the failure of the same plaintiffs and their the property and all remedies which the insured may
counsel to appear at the pre-trial. ISSUE have for the recovery thereof. That right is not
- Upon an MR, the lower court set aside its order of WON the complaint of Firestone and Fireman's Fund dependent upon, nor does it grow out of, any privity
dismissal and sustained plaintiff's contention that states a cause of action against Jamila of contract, or upon written assignment of claim, and
there was no res judicata as to First Quezon City payment to the insured makes the insurer an
because the civil case was dismissed without HELD assignee in equity.
prejudice. However, due to inadvertence, the lower YES - On the other hand, Firestone is really a nominal
court did not state in its order why it set aside its - Fireman's Fund's action against Jamila is squarely party in this case. It had already been indemnified
prior order dismissing the complaint with respect to sanctioned by article 2207. As the insurer, Fireman's for the loss which it had sustained. Obviously, it
Jamila. Jamilla had originally moved for the dismissal Fund is entitled to go after the person or entity that joined as a party-plaintiff in order to help Fireman's
of the complaint on the ground of lack of cause of violated its contractual commitment to answer for Fund to recover the amount of the loss from Jamila
action. Its basis for its contention were: (1) that the the loss insured against. and First Quezon City. Firestone had tacitly assigned
complaint did not allege that Firestone, pursuant to - The trial court erred in applying to this case the to Fireman's Fund its cause of action against Jamila
the contractual stipulation quoted in the complaint, rules on novation. The plaintiffs in alleging in their for breach of contract. Sufficient ultimate facts are
had investigated the loss and that Jamila was complaint that Fireman's Fund "became a party in alleged in the complaint to sustain that cause of
represented in the investigation and (2) that Jamila interest in this case by virtue of a subrogation right action.
did not consent to the subrogation of Fireman's Fund given in its favor by" Firestone, were not relying on
to Firestone's right to get reimbursement from Jamila the novation by change of creditors as contemplated TABACALERA v. NORTH FRONT SHIPPING
and its surety. The lower court in its order of in articles 1291 and 1300 to 1303 of the Civil Code 272 SCRA 527
dismissal had sustained the second ground. but rather on article 2207. BELLOSILLO; May 16, 1997
- Jamila in its MR invoked the first ground which - Article 2207 is a restatement of a settled principle
had never been passed upon by the lower court. But of American jurisprudence. Subrogation has been FACTS
the lower court in its order granting Jamila's motion referred to as the doctrine of substitution. It is an - 20,234 sacks of corn grains valued at P3.5M were
for reconsideration, completely ignored that first arm of equity that may guide or even force one to shipped on board North Front 777, defendant’s
ground. It reverted to the second ground which was pay a debt for which an obligation was incurred but vessel. The cargo was consigned to Republic Flour
relied upon in its order previous order. The lower which was in whole or in part paid by another. Mills Corp. under Bill of Lading No. 001 and insured
court reiterated its order, stating that Fireman's Fund - Subrogation is founded on principles of justice and with Tabacalera, Prudential Guarantee & Assurance,
had no cause of action against Jamila because Jamila equity, and its operation is governed by principles of and New Zealand Insurance.
did not consent to the subrogation. The court did not equity. It rests on the principle that substantial - Republic Flour was advised of the vessel’s arrival in
mention Firestone, the co-plaintiff of Fireman's Fund. justice should be attained regardless of form, that is, Manila, but did not immediately commence the
- Firestone and Fireman's Fund filed an MR on the its basis is the doing of complete, essential, and unloading operations. Unloading was sometimes
ground that Fireman's Fund was suing on the basis of perfect justice between all the parties without regard stopped due to varying weather and sometimes for
legal subrogation whereas the lower court to form. no apparent reason. Unloading was only completed
erroneously predicated its dismissal order on the - Subrogation is a normal incident of indemnity 20 days after the arrival of the barge; by then, the
theory that there was no conventional subrogation insurance. Upon payment of the loss, the insurer is cargo was short 26.333 metric tons and the rest was
because the debtor's consent was lacking. entitled to be subrogated pro tanto to any right of already moldy and deteriorating.
INSURANCE Page
110

- Analyses showed that the deterioration was caused - While petitioners presented evidence of the vessel’s letter, however, petitioner confirmed to private
by moisture content from salt water, which could be bad shape and a laboratory analysis revealing that respondent receipt of the claim papers and assured
arrested by drying. However, Republic Flour rejected the grains were contaminated with salt water, her that her case was “being given preferential
the entire cargo and demanded that defendant North defendants failed to rebut said arguments or even attention and prompt action”.
Front Shipping pay the damages suffered by it. The endeavor to establish that the loss, destruction or - Petitioner caused another investigation respecting
demands were unheeded and the insurance deterioration was due to a fortuitous event; an the subject policy. Pursuant to the findings of this
companies were obliged to pay Republic Flour act/omission of the owner of the goods; the second investigation, petitioner stood by its initial
P2,189,433 character of the goods or defects in their packing; or decision to treat the policy as void and not to honor
- By virtue of the insurance companies’ payment, an order or act of a competent public authority. the claim. On November 9, 1992, private
they were subrogated to the rights of Republic Flour. - However, Republic Flour is also found to be guilty of respondent enlisted the services of counsel in
Petitioners filed a complaint against North Front contributory negligence for not immediately staring reiterating her claim for death benefitsPetitioner still
Shipping, claiming the loss was exclusively the unloading operations and for providing no refused to make payment and thus, this action.
attributable to the latter’s fault and negligence. explanation for the delay. As such, it should share at
Having surveyed the vessel, it was found that the least 40% of the loss. ISSUE
barge had cracks in its bodega. The hatches on the Disposition The decision of the CA is REVERSED WON lower court erred in holding that there was no
crates of grain were not sealed and the tarpaulins and SET ASIDE fraud
used in covering them were not new, contrary to
North Front Shipping’s claims. North Front Shipping PHILIPPINE AMERICAN LIFE INSURANCE HELD
reiterated that the barge was inspected prior to COMPANY v. CA (ELIZA PULIDO) - The records bear out that since the onset of this
loading and found seaworthy and were issued a 344 SCRA 360 case, the main issue has always been whether there
permit to sail by the Coast Guard. They further GONZAGA-REYES; November 15, 2000 was fraud in the obtainment of the disputed policy, or
averred that the grains were farm wet and not put differently, whether the insured, Florence Pulido,
properly dried before loading. NATURE was in fact dead before the application for insurance
- The court dismissed the complaint, ruling that the This petition for review on certiorari seeks to reverse on her life was made. This the lower courts had
contract entered into was a charter-party the Decision of the Special Second Division of the effected ruled on, upon a preponderance of the
agreement; as such, only ordinary diligence in the Court of Appeals evidence duly received from both parties. We see no
care of the goods was required of North Front reversible error in the finding of both respondent
Shipping. FACTS court and the trial court in favor of the correctness of
- On January 9, 1989, petitioner received from one the entries in Certificate of Death, duly registered
ISSUE Florence Pulido an application for life insurance, with the Local Civil Registrar of Bagulin, La Union,
WON defendant is required to observe extraordinary dated December 16, 1988, in the amount of which declared that Florence Pulido died of acute
diligence in its vigilance over the goods it transports P100,000.00 which designated her sister, herein pneumonia on September 10, 1991. Dr. Irineo
private respondent, as its principal beneficiary. Gutierrez, the Municipal Health Officer of Bagulin, La
HELD Because the insurance applied for was non-medical, Union whose signature appeared in the death
YES petitioner did not require a medical examination and certificate, testified in addition that he ministered to
- As a corporation engaged in the business of issued a policy on the sole basis of the application on the ailing Florence Pulido for two days immediately
transporting cargo offering its services February 11, 1989. On April 1992, petitioner prior to her death. This fact is likewise noted in the
indiscriminately to the public, it is without a doubt a received private respondent’s claim, which declared death certificate.
common carrier. As such, it has the burden of that the insured, Florence Pulido, died of acute - Death certificates, and notes by a municipal health
proving that it observed extraordinary diligence to pneumonia on September 10, 1991. officer prepared in the regular performance of his
avoid responsibility for the lost cargo. The clean bill - Petitioner withheld payment on the ground that the duties, are prima facie evidence of facts therein
of lading it issued disprove the master of the vessel’s policy claimed under was void from the start for stated. A duly-registered death certificate is
claim that the grains were farm wet when loaded. If having been procured in fraud. It is petitioner’s considered a public document and the entries found
they were wet, the master of the vessel should have contention that even before they received private therein are presumed correct, unless the party who
known that the grains would eventually deteriorate respondent’s claim for death benefits, their contests its accuracy can produce positive evidence
when sealed in hot compartments in hatches of a investigation concerning the subject policy yielded establishing otherwise. Petitioner’s contention that
ship and should have undertaken precautionary the information that the insured, Florence Pulido, the death certificate is suspect because Dr. Gutierrez
measures to avoid this. The arrival of the goods at died in 1988, before the application for insurance on was not present when Florence Pulido died, and
the place of destination in bad order makes a prima her life was made. While this was communicated to knew of Florence’s death only through Ramon
facie case against the common carrier, which must private respondent in a letter, private respondent had Piganto, does not merit a conclusion of fraud. No
prove its non-liability. already filed her claim earlier that month. In another motive was imputed to Dr. Gutierrez for seeking to
INSURANCE Page
111

perpetuate a falsity in public records. Petitioner was & Marine Insurance Co., the insurance company, on appellees with respect to the lost or damaged
likewise unable to make out any clear motive as to the basis of such claim, paid to the consignee the shipments are expressly limited to the C.I.F. value of
why Ramon Piganto would purposely lie. Mere insured value of the lost and damagcd goods, the goods as per contract of sea carriage embodied
allegations of fraud could not substitute for the full including other expenses in connection therewith, in in the bill of lading.
and convincing evidence that is required to prove it. the total amount of $1,134.46 U.S. currency. - The plaintiff-appellant, as insurer, after paying the
A failure to do so would leave intact the presumption - As subrogee of the rights of' the shipper and/or claim of the insured for damages under the
of good faith and regularity in the performance of consignee, the insurer, St. Paul Fire & Marine insurance, is subrogated merely to the rights of the
public duties, which was the basis of both respondent Insurance Co., instituted an action against the assured. As subrogee, it can recover only the amount
court and the trial court in finding the date of defendants for the recovery of said amount of that is recoverable by the latter. Since the right of
Florence Pulido’s death to be as plaintiff-private $1,134.46, plus costs. the assured, in case of loss or damage to the goods,
respondent maintained. - The defendants resisted the action. However, for is limited or restricted by the provisions in the bill of
- We cannot likewise give credence to petitioner’s the purpose only of avoiding litigation without lading, a suit by the insurer as subrogee necessarily
submission that the inconsistencies in the admitting liability to the consignee, the defendants is subject to like limitations and restrictions.
testimonies of the witnesses for plaintiff-private offered to settle the latter’s claim in full by paying 2. On the date of the discharge of the cargo. The
respondent are in themselves evidence of fraud. the C.I.F. value of the damaged cargo, but this offer peso equivalent was based by the consignee on the
Such alleged inconsistencies are matters of credibility was declined by the plaintiff. exchange rate of P2.015 to $1.00 which was the rate
which had been ably passed upon by the lower court. - The LC rendered judgment ordering the existing at that time.
Disposition the instant petition is DENIED defendants to pay the plaintiff the sum of P300.00.
The plaintiff filed a MFR contending that it should PHILAM v. AUDITOR (supra p.59)
ST.PAUL FIRE & MARINE INSURANCE CO v. recover the amount of $1,134.46 or its equivalent in
MACONDRAY & CO INC pesos at the rate of P3.90, instead of P2.00, but this FIELDMEN’S v. ASIAN SURETY (supra p.60)
70 SCRA 122 was denied. Hence, this appeal.
ANTONIO; March 25, 1976 EQUITABLE v. RURAL INSURANCE (supra p.60)
ISSUES
FACTS 1. WON in case of loss or damage, the liability of the COQUIA v. FIELDMEN'S INSURANCE CO. INC.
- Winthrop Products, Inc. shipped aboard the SS "Tai carrier to the consignee is limited to the C.I.F. value 26 SCRA 178
Ping", owned and operated by Wilhelm Wilhelmsen, of the goods which were lost or damaged CONCEPCION; November 29, 1968
218 cartons and drums of drugs and medicine, with 2. WON the insurer who has paid the claim in dollars
the freight prepaid, which were consigned to to the consignee should be reimbursed in its peso NATURE
Winthrop-Stearns, Inc. Barber Steamship Lines, Inc., equivalent on the date of discharge of the cargo or Appeal from the decision of the CFI certified by CA
agent of Wilhelm Wilhelmsen issued Bill of Lading No. on the date of the decision
34, in the name of Winthrop Products, Inc. as FACTS
shipper, with arrival notice in-Manila to consignee HELD - December 1, 1961, appellant Fieldmen's Insurance
Winthrop-Stearns, Inc. The shipment was insured by 1. YES Company, Inc. issued, in favor of the Manila Yellow
the shipper against loss and/or damage with the St. Ratio The purpose of the bill of lading is to provide Taxicab Co., Inc. a common carrier accident
Paul Fire & Marine Insurance Company. for the rights and liabilities of the parties in reference insurance policy, covering the period from December
- The SS "Tai Ping" arrived at the Port of Manila and to the contract to carry. The stipulation in the bill of 1, 1961 to December ,1962. It was stipulated in said
discharged its aforesaid shipment into the custody of lading limiting the common carrier's liability to the policy that:
Manila Port Service, the arrastre contractor for the value of the goods appearing in the bill, unless the "The Company will, subject to the Limits of Liability
Port of Manila. The said shipment was discharged shipper or owner declares a greater value, is valid and under the Terms of this Policy, indemnify the
complete and in good order with the exception of one and binding. This limitation of the carrier's liability is Insured in the event of accident caused by or arising
(1) drum and several cartons which were in bad sanctioned by the freedom of the contracting parties out of the use of Motor Vehicle against all sums
order condition. Because consignee failed to receive to establish such stipulations, clauses, terms, or which the Insured will become legally liable to pay in
the whole shipment and as several cartons of conditions as they may deem convenient, provided respect of: Death or bodily injury to any fare-paying
medicine were received in bad order condition, the they are not contrary to law, morals, good customs passenger including the Driver, Conductor and/or
consignee filed the corresponding claim in the and public policy. A stipulation fixing or limiting the Inspector who is riding in the Motor Vehicle insured
amount of P1,109.67 representing the C.I.F. value of sum that may be recovered from the carrier on the at the time of accident or injury."
the damaged drum and cartons of medicine with the loss or deterioration of the goods is valid, provided it - While the policy was in force, or on February 10,
carrier and the Manila Port Service. However, both is (a) reasonable and just under the circumstances, 1962, a taxicab of the Insured, driven by Carlito
refused to pay such claim. Consequently, the and (b) has been fairly and freely agreed upon. In Coquia, met a vehicular accident to which he died.
consignee filed its claim with the insurer, St. Paul Fire the case at bar, the liabilities of the defendants- The Insured filed therefor a claim for P5,000.00 to
INSURANCE Page
112

which the Company replied with an offer to pay "3. In terms of and subject to the limitations of to any right of action or suit upon this Policy that
P2,000.00, by way of compromise. The Insured and for the purposes of this Section, the Company the award by such arbitrator, arbitrators or umpire
rejected it and made a counter-offer for P4,000.00, will indemnify any authorized Driver who is driving of the amount of the Company's liability hereunder
but the Company did not accept it. the Motor Vehicle . . . " if disputed shall be first obtained."
- On September 18, 1962, the Insured and Carlito's "Conditions - The record shows that none of the parties to the
parents filed a complaint against the Company to "7. In the event of death of any person entitled contract invoked this section, or made any reference
collect the proceeds of the policy. In its answer, the to indemnify under this Policy, the Company will, in to arbitration, during the negotiations preceding the
Company admitted the existence thereof, but respect of the liability incurred by such person, institution of the present case. In fact, counsel for
pleaded lack of cause of action on the part of the indemnify his personal representatives in terms of both parties stipulated, in the trial court, that none of
plaintiffs. and subject to the limitations of this Policy, provided, them had, at any time during said negotiations, even
- TC rendered a decision sentencing the Company to that such representatives shall, as though they were suggested the settlement of the issue between them
pay to the plaintiffs the sum of P4,000.00 and the the Insured, observe, fulfill and be subject to the by arbitration, as provided in said section. Their
costs. Hence, this appeal by the Company, which Terms of this Policy insofar as they can apply. aforementioned acts or omissions had the effect of a
contends that plaintiffs have no cause of action "8. The Company may, at its option, make waiver of their respective right to demand an
because: 1) the Coquias have no contractual relation indemnity payable directly to the claimants or heirs arbitration.
with the Company; and 2) the Insured has not of claimants, with or without securing the consent of Disposition The decision appealed from should be
complied with the provisions of the policy concerning or prior notification to the Insured, it being the true as it is hereby affirmed in toto, with costs against the
arbitration. intention of this Policy to protect, to the extent herein defendant-appellant, Fieldmen's Insurance
herein specified and subject always to the Terms of Co., Inc.
ISSUES this Policy, the liabilities of the Insured towards the
1. WON there was contractual relations between the passengers of the Motor Vehicle and the Public." COUNTRY BANKERS INSURANCE CORP v.
Coquias and the Company - Thus, the policy under consideration is typical of LIANGA BAY
2. WON the insured has not complied with the contracts pour autrui, this character being made DE LEON; January 25, 2002
provisions of the policy concerning arbitration more manifest by the fact that the deceased driver
paid fifty percent (50%) of the corresponding NATURE
HELD premiums, which were deducted from his weekly Petition for review on certiorari
1. Although, in general, only parties to a contract commissions. Under these conditions, it is clear that
may bring an action based thereon, this rule is the Coquias — who, admittedly, are the sole heirs of FACTS
subject to exceptions, one of which is found in the the deceased — have a direct cause of action against - Lianga Bay is a duly registered cooperative
Art 1311 CC, reading: the Company, and, since they could have maintained judicially declared insolvent and is here represented
"If a contract should contain some stipulation in this action by themselves, without the assistance of by, Cornelio Jamero. Country Bankers Insurance and
favor of a third person, he may demand its the Insured, it goes without saying that they could Lianga Bay entered into a contract of fire insurance.
fulfillment provided he communicated his acceptance and did properly join the latter in filing the complaint Country Bankers insured the respondent’s stocks-in-
of the obligor before its revocation. A mere incidental herein. trade against fire loss, damage or liability during the
benefit or interest of a person is not sufficient. The 2. Based upon Section 17 of the policy: period starting from June 20, 1989 at 4:00 p.m. to
contracting parties must have clearly and "If any difference or dispute shall arise with June 20, 1990 at 4:00 p.m., for the sum of
deliberately conferred a favor upon a third person." respect to the amount of the Company's liability P200,000.00.
- Does the policy in question belong to such class of under this Policy, the same shall be referred to the - On July 1, 1989, at or about 12:40 a.m., the
contracts pour autrui? decision of a single arbitrator to be agreed upon by respondent’s building at Barangay Diatagon, Lianga,
In this connection, said policy provides, inter alia: both parties or failing such agreement of a single Surigao del Sur was gutted by fire, resulting in the
"Section I — Liability to Passengers. 1. The arbitrator, to the decision of two arbitrators, one to total loss of the respondent’s stocks-in-trade, pieces
Company will, subject to the Limits of Liability and be appointed in writing by each of the parties of furnitures and fixtures, equipments and records.
under the Terms of this Policy, indemnify the Insured within one calendar month after having been - Due to the loss, the respondent filed an insurance
in the event of accident caused by or arising out of required in writing so to do by either of the parties claim with the petitioner under its Fire Insurance
the use of Motor Vehicle against all sums which the and in case of disagreement between the Policy, submitting: (a) the Spot Report of Pfc. Arturo
Insured will become legally liable to pay in respect arbitrators, to the decision of an umpire who shall V. Juarbal, INP Investigator, dated July 1, 1989; (b)
of: Death or bodily injury to any fare-paying have been appointed in writing by the arbitrators the Sworn Statement of Jose Lomocso; and (c) the
passenger including the Driver. . . who is riding in the before entering on the reference and the costs of Sworn Statement of Ernesto Urbiztondo.
Motor Vehicle insured at the time of accident or and incidental to the reference shall be dealt with - The petitioner, however, denied the insurance claim
injury. in the Award. And it is hereby expressly stipulated on the ground that, based on the submitted
"Section II. — Liability to the Public and declared that it shall be a condition precedent documents, the building was set on fire by 2 NPA
INSURANCE Page
113

rebels who wanted to obtain canned goods, rice and preponderance of evidence. But petitioner failed to (d) Mutiny, riot, military or popular uprising,
medicines as provisions for their comrades in the do so. insurrection, rebellion, revolution, military or usurped
forest, and that such loss was an excepted risk under - The petitioner relies on the Sworn Statements of power.
paragraph No. 6 of the policy conditions of Fire Jose Lomocso and Ernesto Urbiztondo as well as on - The insurers maintained that based on witnesses
Insurance Policy No. F-1397, which provides: the Spot Report of Pfc. Arturo V. Juarbal. A witness and evidence gathered at the site, the fire was
This insurance does not cover any loss or damage can testify only to those facts which he knows of his caused by the members of the Communist Party of
occasioned by or through or in consequence, personal knowledge, which means those facts which the Philippines/New People’s Army. Hence the refusal
directly or indirectly, of any of the following are derived from his perception. Consequently, a to honor their obligations.
occurrences, namely: witness may not testify as to what he merely learned - The trial court and the CA found in favor of the
(d) Mutiny, riot, military or popular uprising, from others either because he was told or read or respondent. In its findings, both courts mentioned
insurrection, rebellion, revolution, military or heard the same. Such testimony is considered the fact that there was no credible evidence
usurped power. hearsay and may not be received as proof of the presented that the CCP/NPA did in fact cause the fire
Any loss or damage happening during the truth of what he has learned. that gutted the radio station in Bacolod.
existence of abnormal conditions (whether Disposition the appealed Decision is MODIFIED.
physical or otherwise) which are occasioned by or The rate of interest on the adjudged principal ISSUE
through or in consequence, directly or indirectly, amount of Two Hundred Thousand Pesos WON the insurance companies are liable to pay Radio
of any of said occurrences shall be deemed to be (P200,000.00) shall be six percent (6%) per annum Mindanao Network under the insurance policies
loss or damage which is not covered by this computed from the date of filing of the Complaint in
insurance, except to the extent that the Insured the trial court. The awards in the amounts of Fifty HELD
shall prove that such loss or damage happened Thousand Pesos (P50,000.00) as actual damages, YES
independently of the existence of such abnormal Fifty Thousand Pesos (P50,000.00) as exemplary - The Court will not disturb the factual findings of the
conditions. damages, Five Thousand Pesos (P5,000.00) as appellant and trial courts absent compelling reason.
- Finding the denial of its claim unacceptable, Lianga litigation expenses, and Ten Thousand Pesos Under this mode of review, the jurisdiction of the
Bay then instituted in the trial court the complaint for (P10,000.00) as attorney?s fees are hereby court is limited to reviewing only errors of law.
recovery of "loss, damage or liability" against DELETED. - Particularly in cases of insurance disputes with
Country Bankers. regard to excepted risks, it is the insurance
- RTC ruled in favor of the cooperative. CA affirmed. DBP POOL OF ACCREDITED INSURANCE v. companies which have the burden to prove that the
RADIO MINDANAO NETWORK loss comes within the purview of the exception or
ISSUE 480 SCRA 314 limitation set up. It is sufficient for the insured to
WON the cause of the loss was an excepted risk MARTINEZ; January 27, 2006 prove the fact of damage or loss. Once the insured
under the terms of the fire insurance policy makes out a prima facie case in its favor, the duty or
NATURE burden of evidence shifts to the insurer to controvert
HELD Petition for certiorari said prima facie case.
- Where a risk is excepted by the terms of a policy Disposition Petition dismissed. Decision of the CA is
which insures against other perils or hazards, loss FACTS affirmed.
from such a risk constitutes a defense which the - In the evening of July 27, 1988, the radio station of
insurer may urge, since it has not assumed that risk, Radio Mindanao Network located at the SSS Building LEA MER INDUSTRIES v. MALAYAN INSURANCE
and from this it follows that an insurer seeking to in Bacolod City was burned down causing damage in 471 SCRA 698
defeat a claim because of an exception or limitation the amount of over one million pesos. Respondent PANGANIBAN; September 30, 2005
in the policy has the burden of proving that the loss sought to recover under two insurance policies but
comes within the purview of the exception or the claims were denied on the basis that the case of NATURE
limitation set up. If a proof is made of a loss the loss was an excepted risk under condition no. 6 Petition for Review
apparently within a contract of insurance, the burden (c) and (d), to wit:
is upon the insurer to prove that the loss arose from 6. This insurance does not cover any loss or damage FACTS
a cause of loss which is excepted or for which it is occasioned by or through or in consequence, directly - Ilian Silica Mining entered into a contract of
not liable, or from a cause which limits its liability. or indirectly, of any of the following consequences, carriage with Lea Mer Industries for the shipment of
Stated else wise, since Country bank here is namely: 900 metric tons of silica sand valued at P565,000.
defending on the ground of non-coverage and relying (c) War, invasion, act of foreign enemies, hostilities, Consigned to Vulcan Industrial and Mining
upon an exemption or exception clause in the fire or warlike operations (whether war be declared or Corporation, the cargo was to be transported from
insurance policy it has the burden of proving the not), civic war. Palawan to Manila. The silica sand was placed on
facts upon which such excepted risk is based, by a
INSURANCE Page
114

board Judy VII, a barge leased by Lea Mer, the vessel LOADSTAR SHIPPING CO INC v. PIONEER ASIA an ordinary peril of the voyage for which the M/V
sank, resulting in the loss of the cargo. INSURANCE CORP Weasel should have been normally able to cope with;
GR No. 157481 and (3) LOADSTAR was negligent in the selection and
ISSUE QUISUMBING; January 24, 2006 supervision of its agents and employees then
WON Lea Mer is liable for the loss of the cargo manning the M/V Weasel.
NATURE - LOADSTAR alleged that no fault nor negligence
HELD Review on certiorari (1) the Decision dated October could be attributed to it because it exercised due
YES 15, 2002 and (2) the Resolution dated February 27, diligence to make the ship seaworthy, as well as
- Common carriers are bound to observe 2003 of CA properly manned and equipped and failure to deliver
extraordinary diligence in their vigilance over the was due to force majeure.
goods and the safety of the passengers they FACTS - February 15, 1993 - RTC decided in favor of
transport, as required by the nature of their business - June 6, 1984 - Petitioner Loadstar Shipping Co., PIONEER and that LOADSTAR , as a common carrier,
and for reasons of public policy. Extraordinary Inc. (LOADSTAR), registered owner and operator of bears the burden of proving that it exercised
diligence requires rendering service with the greatest the vessel M/V Weasel, entered into a voyage- extraordinary diligence in its vigilance over the goods
skill and foresight to avoid damage and destruction charter with Northern Mindanao Transport Company, it transported. The trial court explained that in case
to the goods entrusted for carriage and delivery. Inc. for the carriage of 65,000 bags of cement from of loss or destruction of the goods, a statutory
- Common carriers are presumed to have been at Iligan City to Manila. The shipper was Iligan Cement presumption arises that the common carrier was
fault or to have acted negligently for loss or damage Corporation, while the consignee in Manila was negligent unless it could prove that it had observed
to the goods that they have transported. This Market Developers, Inc. (MARKET) extraordinary diligence. LOADSTAR’S defense of
presumption can be rebutted only by proof that they - June 24, 1984 - 67,500 bags of cement were force majeure was found bereft of factual basis as a
observed extraordinary diligence, or that the loss or loaded on board M/V Weasel and stowed in the cargo PAG-ASA report that at the time of the incident,
damage was occasioned by any of the following holds for delivery to the consignee. The shipment tropical storm “Asiang” had moved away from the
causes: was covered by petitioner’s Bill of Lading dated June Philippines was presented.
“(1) Flood, storm, earthquake, lightning, or other 23, 1984. - October 15, 2002 – CA affirmed RTC Decision with
natural disaster or calamity; - Prior to the voyage, the consignee insured the modification
“(2) Act of the public enemy in war, whether shipment of cement with respondent Pioneer Asia
international or civil; Insurance Corporation (PIONEER) for P1,400,000, for ISSUES
“(3) Act or omission of the shipper or owner of the which it issued Marine Open Policy No. MOP-006 1. WON LOADSTAR is a common carrier under
goods; dated September 17, 1980, covering all shipments Article 1732 CC
“(4) The character of the goods or defects in the made on or after September 30, 1980 2. Assuming it is a common carrier, WON proximate
packing or in the containers; - June 25, 1984 - Captain Montera of M/V Weasel cause of the loss of cargo was not a fortuitous event
“(5) Order or act of competent public authority.” ordered the vessel to be forced aground which but was allegedly due to the failure of petitioner to
- To excuse the common carrier fully of any liability, rendered the entire shipment of cement as good as exercise extraordinary diligence
the fortuitous event must have been the proximate gone due to exposure to sea water. LOASTAR thus
and only cause of the loss. It should have exercised failed to deliver the goods to MARKET in Manila. HELD
due diligence to prevent or minimize the loss before, - MARKET demanded from LOADSTAR full 1. YES
during and after the occurrence of the event. reimbursement of the cost of the lost shipment. - A1732 CC defines a “common carrier” as
- Petitioner bore the burden of proving that it had LOADSTAR refused to reimburse the MARKET despite follows:
exercised extraordinary diligence to avoid the loss, or repeated demands. Common carriers are persons, corporations, firms
that the loss had been occasioned by a fortuitous - March 11, 1985 – PIONEER paid the MARKET or associations engaged in the business of carrying
event -- an exempting circumstance. P1,400,000 plus an additional amount of P500,000, or transporting passengers or goods or both, by
- The evidence presented by petitioner in support of the value of the lost shipment of cement. In return, land, water, or air, for compensation, offering their
its defense of fortuitous event was sorely the MARKET executed a Loss and Subrogation services to the public.
insufficient. It was not enough for the common Receipt in favor of PIONEER concerning the latter’s - LOADSTAR is a corporation engaged in the business
carrier to show that there was an unforeseen or subrogation rights against LOADSTAR. of transporting cargo by water and for compensation,
unexpected occurrence. - October 15, 1986 – PIONEER filed a complaint offering its services indiscriminately to the public.
Disposition Petition is DENIED and the assailed against LOADSTAR with the RTC Manila alleging Thus, without doubt, it is a common carrier. Even if
that: (1) the M/V Weasel was not seaworthy at the it entered into a voyage-charter agreement with
Decision and Resolution are AFFIRMED. Costs commencement of the voyage; (2) the weather and Northern Mindanao Transport Company, Inc, it did
sea conditions then prevailing were usual and not in any way convert the common carrier into a
against petitioner. expected for that time of the year and as such, was private carrier.
INSURANCE Page
115

> Planters Products, Inc. v. CA - public carrier unexpected hazard. LOADSTAR has only itself to
shall remain as such, notwithstanding the charter of blame for its misjudgment.
the whole or portion of a vessel by one or more Disposition petition is DENIED
persons, provided the charter is limited to the ship
only, as in the case of a time-charter or voyage-
charter. It is only when the charter includes both the
vessel and its crew, as in a bareboat or demise that a
common carrier becomes private, at least insofar as
the particular voyage covering the charter-party is
concerned.
2. YES
- As a common carrier, LOADSTAR is required to
observe extraordinary diligence in the vigilance over
the goods it transports. When the goods placed in its
care are lost, LOADSTAR is presumed to have been
at fault or to have acted negligently. LOADSTAR has
the burden of proving that it observed extraordinary
diligence in order to avoid responsibility for the lost
cargo.
- Compania Maritima V CA - It requires common
carriers to render service with the greatest skill and
foresight and “to use all reasonable means to
ascertain the nature and characteristics of goods
tendered for shipment, and to exercise due care in
the handling and stowage, including such methods as
their nature requires.
- A1734 CC enumerates the instances when a carrier
might be exempt from liability for the loss of the
goods.
(1) Flood, storm, earthquake, lightning,
or other natural disaster or calamity;
(2) Act of the public enemy in war,
whether international or civil;
(3) Act or omission of the shipper or
owner of the goods;
(4) The character of the goods or defects
in the packing or in the containers; and
(5) Order or act of competent public
authority
- LOADSTAR claims that the loss of the goods was
due to a fortuitous event under paragraph 1. Yet, its
claim is not substantiated. It is supported by
evidence that the loss of the entire shipment of
cement was due to the gross negligence of
LOADSTAR
- Records show that in the evening of June 24, 1984,
the sea and weather conditions in the vicinity of
Negros Occidental were calm. The records reveal
that LOADSTAR took a shortcut route, instead of the
usual route, which exposed the voyage to
i

You might also like